Assistant Professor English Solved Papers [2 ed.]
 9415650134

  • 0 0 0
  • Like this paper and download? You can publish your own PDF file online for free in a few minutes! Sign Up
File loading please wait...
Citation preview

2024 ENGLISH

Youth Competition Times

SOLVED PAPERS

UPHESC, UPPSC, UKPSC, RPSC, MPPSC, CGPSC, HPSC

ENGLISH

SOLVED PAPERS

Asstt. Prof. Exam

Solution with detailed analysis and explanation UP Higher Education Service Commission ■ UPHESC Asstt. Prof. Exam. 2021 ■ UPHESC Asstt. Prof. Exam. 2016 [Exam. Date : 15 Dec 2018] ■ UPHESC Asstt. Prof. Exam. 2014 [Exam. Date : 1 1 Jan 2015] UPPSC Govt. Degree College ■ UPPSC Asstt. Prof. (GDC) Exam. 2021 [Exam. Date 15 March 2022] ■ UPPSC Asstt. Prof. (GDC) Exam. 2017 [Exam. Date 3 Nov. 2019] Uttrakhand Public Service Commission ■ UPSC Asstt. Prof. (GDC) Exam. 2017 [Exam. Date : 29 Oct 2017]

Rajasthan Public Service Commission ■ RPSC College Lecture Exam. 2020 PAPER-I ■ RPSC College Lecture Exam. 2020 PAPER-II ■ RPSC College Lecture Exam. 2014 PAPER-I [Exam. Date : 25 June, 2016] ■ RPSC College Lecture Exam. 2014 PAPER-II [Exam. Date : 25 June, 2016] Haryana Public Service Commission ■ HPSC Asstt. Prof. Re-Exam. 2019 [Exam. Date : 12 June 2019] ■ HPSC Asstt. Prof. Exam. 2019 [Exam. Date : 23 May 2019]

Chhattisgarh Public Service Commission ■ CPSC Asstt. Prof. Exam. 2019 [Exam. Date : 06 Nov 2020] ■ CPSC Asstt. Prof. Exam. 2017 [Exam. Date : 26 March 2017] ■ CPSC Asstt. Prof. Exam. 2016 [Exam. Date : 28 Sep. 2016] ■ CPSC Asstt. Prof. Exam. 2014 [Exam. Date : 21 May 2016] MP Public Service Commission ■ MPPSC Asstt. Prof. Exam. 2017 [Exam. Date : 26 March 2017]

Completion certified by the Revised Answer-Key of the Commission

U.P. HIGHER EDUCATION SERVICES COMMISSION, PRAYAGRAJ

U.P. HIGHER GOVT. DEGREE COLLEGE ASSISTANT PROFESSOR

ENGLISH Solved Papers Chief Editor A. K. Mahajan Written by Exam Expert Team Computer Graphics Balkrishna & Charan Singh Editorial Office 12, Church Lane Prayagraj-211002 © Mob. : 9415650134 Email : [email protected] website : www.yctbooks.com I www.yctfastbooks.com © All rights reserved with Publisher Publisher's Declaration Edited and Published by A.K. Mahajan and printed by printing press Laxmi Narayan Printing Press, Prayagraj for YCT Publications Pvt. Ltd. In order to Publish the book, full care has been taken by the Editor and the Publisher, still your cooperation and suggestions are needed. C 395/In the event of any dispute, the Judicial area will be Prayagraj.

(content ASSISTANT PROFESSOR SOLVED PAPER Assistant Professor Syllabus

3-6

UP Higher Education Service Commission UP Higher Education Service Commission Asstt. Prof. Exam. 2021 UP Higher Education Service Commission Asstt. Prof. Exam. 2016 [Exam. Date : 15 Dec 2018] UP Higher Education Service Commission Asstt. Prof. Exam. 2014 [Exam. Date : 11 Jan 2015]

7-14 15-23 24-31

UPPSC Govt Degree College UPPSC Govt. Degree College Asstt. Prof. (GDC) Exam. 2021 [Exam. Date 15 March 2022] UPPSC Govt. Degree College Asstt. Prof. (GDC) Exam. 2017 [Exam. Date 3 Nov. 2019]

32-43 44-55

Rajasthan Public Service Commission Rajasthan Public Service Commission College Lecture Exam. 2020 Rajasthan Public Service Commission College Lecture Exam. 2020 Rajasthan Public Service Commission College Lecture Exam. 2014 [Exam. Date : 25 June, 2016] Rajasthan Public Service Commission College Lecture Exam. 2014 [Exam. Date : 25 June, 2016]

PAPER-1 PAPER-II PAPER-I

56-71 72-85 86-102

PAPER-II

103-121

Uttrakhand Public Service Commission Uttrakhand Public Service Commission Asstt. Prof. (GDC) Exam. 2017 [Exam. Date : 29 Oct 2017]

122-132

MP Public Service Commission MP Public Service Commission Asstt. Prof. Exam. 2017 [Exam. Date : 26 March 2017]

133-150

Chhattisgarh Public Service Commission Chhattisgarh Public Service Commission [Exam. Date : 06 Nov 2020] Chhattisgarh Public Service Commission [Exam. Date : 26 March 2017] Chhattisgarh Public Service Commission [Exam. Date : 28 Sep. 2016] Chhattisgarh Public Service Commission [Exam. Date : 21 May 2016]

Asstt. Prof. Exam. 2019 151-160 Asstt. Prof. Exam. 2017 161-178 Asstt. Prof. Exam. 2016 179-189 Asstt. Prof. Exam. 2014 190-199

Haryana Public Service Commission Haryana Public Service Commission Asstt. Prof. Re-Exam. 2019 [Exam. Date : 12 June 2019] Haryana Public Service Commission Asstt. Prof. Exam. 2019 [Exam. Date : 23 May 2019] 2

200-209 210-224

UP Higher Education Services Commission, Prayagraj SYLLABUS An outline of the Course Content

Charles Lamb

: ’The South Sea House’, ’Dream Children’, ’Christ Hospital Five and Thirty Years Ago’, ’The Convalescent’, ’Poor Relations’, ’Imperfect Sympathies’

Thomas Carlyle

: ’The Hero as a Poet’

William Hazlitt

: ’Public Opinion’, ’On Reading Old Books’, ’On Reading New Books’

T.B. Macaulay

: ’Minutes on Education’ (1835)

Virginia Woolf

: ’A Room of One’s Own’

George Orwell

: ’Politics and the English Language’

Henry Fielding

: Joseph Andrews

Jane Austen

: Pride and Prejudice

Unit-I : Literature and Society in the following periods (a) Renaissance (b) Reformation (c) Restoration (d) Neo-classical Period (e) Romantic Period (f) Victorian Period (g) Modem Period (h) Post-Modem Period Unit-II : British Drama C. Marlowe

: Doctor Faustus, The Jew of Malta

Ben Jonson

: Everyman in His Humour

John Webster

Charles Dickens : Hamlet, King Lear, The Twelfth Night, As You Like It, The George Eliot Tempest, King Henry IV, Part I & II Thomas Hardy : All For Love D.H. Lawrence : The Way of the World Graham Greene : The Duchess of Malfi

John Galsworthy

: Strife, Escape

W. Shakespeare

John Dryden W. Congreve

William Golding

: Great Expectations, A Tale of Two Cities : Middle March : Tess of the D’Urbervilles : Sons and Lovers : The Power and the Glory : Lord of the Flies

: The Sea George Bernard Shaw: Candida, Saint Joan, Man and Iris Murdoch Unit-IV : British Poetry Superman John Synge

: The Playboy of the Western World

G. Chaucer

: ’The Prologue to the Canterbury Tales’

T.S. Eliot

: Murder in the Cathedral

E. Spenser

: ’The Faerie Queene Book I’

Sammuel Beckett

: Waiting for Godot

John Donne

John Osborme

: Look Back in Anger

Harold Pinter

: The Birthday Party

Arnold Wesker

: Roots

: ’The Canonization’, ’The Flea’, ’A Valediction Forbidding Mourning’, ’Goe and Catche a Falling Starre’, ’A Hymme to God the Father’

Unit-Ill : British Prose and Fiction Francis Bacon

John Milton

J. Milton : ’Of Truth’, ’Of Revenge’, ’Of Studies’, ’Of Marriage and Single J. Dryden A. Pope Life’, ’Of Regimen of Health’

: ’Areopagitica’

Addison and Steele : Essays dealings with Coverly Papers from ’The Spectator’. 3

: ’Paradise Lost’, Book I & II : ’Absalom & Achitophel’ : ’The Rape of the Lock’

William Blake

: ’London’

Thomas Gray

: ’Elegy Written in a Country Churchyard’

William Wordsworth: ’The Prelude’ Book I, ’Tintem

Matthew Arnold

: ’Study of Poetry’, ’Shelley’, ’Wordsworth’, ’Culture and

Abbey’ S.T. Coleridge

Anarchy’

: ’The Rime of the Ancient

P.B. Shelley

: ’Tradition and the Individual

T.S. Eliot

Mariner’, ’Kubla Khan’

Talent’, ’Hamlet’, ’The

: ’Ode to the West Wind’, ’Adonais’,

Metaphysical Poets’

’England in 1819’ John Keats

LA. Richards

: ’Ode on A Grecian Um’, ’Ode to a

: The Principles of Literary Criticism

Nightingale’, ’Ode to Autumn’ A. Tennyson

F.R. Leavis

: ’Lady of Shallot’, ’Ulysses’, ’The

: ’Johnson and Augustanism’, ’Literature and Society’, ’Literary

Princess’, ’The Lotos Eaters’ Robert Browning

Criticism and Philosophy’

: ’Rabbi Ben Ezra’, ’Prospice’, ’A

Practical Criticism

Grammarian’s Funeral’, ’The Last

Unit-VI : Literary Theory and Literary Criticism

Ride Together’, ’My Last Duchess’ Matthew Arnold

Sturcturalism,

: ’The Scholar Gypsy’, ’Thyrsis’,

Historicism,

’Dover Beach’

Deconstruction, Feminism,

Theory, Russian W.B. Yeats

: ’Easter 1916’, ’The Second Coming’, ’A Prayer for my Daughter’, ’Among School

Historicism,

Post-Colonialism,

New Cultural

Formalism, New Criticism,

Psycho-

analytical Criticism, Marxism, Reader-Response

Theory,

Technological

Dhvani

Criticism, Indian Poetics-Rasa,

Schools

Children’, ’Sailing to Byzantium’,

Unit-VII : American Literature

’Lapis Lazuli’, ’Byzantium’ T.S. Eliot

R.W. Emerson

: ’The Love Song of J. Alfred

’Brahma’, ’The Over Soul’

Prufrock’, ’The Waste Land’, Walt Whitman

’Burnt Norton’ W.H. Auden

Passage to India’, ’Crossing Brooklyn Ferry’

Achilles’, ’Partition’, ’The Emily Dickinson

Unknown Citizen’

: ’I never Lost as Much but Twice’, ’Success is Counted Sweetest’, ’I

: ’The Whitsun Weddings’,

felt a Funeral in my Brain’, ’If I

’Ambulances’, ’Church Going’ Ted Hughes

: ’One’s Self I Sing’, ’When Lilacs Last in the Dooryard Bloomed’, ’A

: ’Petition’, ’In Memory of W.B . Yeats’, ’Consider’, ’The Shield of

Philip Larkin

: ’Each and AH’, ’Hamatreya’,

should Die and you should Live’

: ’The Crow’, ’Hawk Roosting’, ’The

Robert Frost

Thought Fox’

: ’Birches’, ’Mending Wall’, ’Stopping by Woods on a Snowy

Unit-V: Literary Theory and Criticism

Evening’, ’The Road not Taken’, ’Mowing’, ’The Death of the Hired

Aristotle

: ’Poetics’

Longinus

: ’On the Sublime’

J. Dryden

: ’An Essay of Dramatic Poesy’

Dr Johnson

: ’Preface to Shakespeare’

Black Bird’, ’The World as

W. Wordsworth

: ’Preface to Lyrical Ballads’

Meditation’

S.T. Coleridge

: Biographia Literaria (Chaps XIII, Eugene O’ Neill

Man’ Wallace Stevens

: ’The Emperor of Ice Cream’, ’Thirteen Ways of Looking at a

XIV, XVIII)

: The Hairy Ape

Tennessee Williams: The Glass Manegerie

4

Arthur Miller

Death of a Salesman

6.

Consonant Clusters, Syllables and Word Stress

N. Hawthorne

The Scarlet Letter

7.

English Morphology and Syntax.

8.

Chomsky’s Transformational and Generative Grammar and Leech’s Communicative Grammar

9.

Stylistics and Literary Analysis, ELT & ESP

E. Hemingway

: The Old Man and the Sea

H. Melville

Moby Dick

Mark Twain

The Adventures of Huckleberry Finn

H.D. Thoreau

Civil Disobedience.

Sarojini Naidu

R.N. Tagore M.K. Gandhi

Discourse

10. Language Planning Unit-X : New Literatures in English

Unit-VIII : Indian Literature in English Torn Dutt

Criticism,

’Lakshman’, ’The Lotus’, ’Our Casuarina Tree’

Australian and New Zealand Literature Patrick White

: Voss

’Planquin Bearers’, ’Banglesellers’, ’Weavers’, ’The FlutePlayers’, ’Of Vrindavan’

Sally Morgan

: My Place

Peter Porter

: ’Your Attention Please’

Pry Lawler

: Summer of the Seventeenth Doll

Allen Curnow

: ’Time’, ’House and Land’

: Gitanjali Hind Swaraj Canadian Literature

Swami Vivekanand : ’Chicago Lecture’ M.G. Vassanji Sri Aurobindo

Nissim Ezekiel

Kamala Das

A.K. Ramanujan

Raja Rao R.K. Narayan Anita Desai

Savitri Canto I, ’The Future Poetry’, ’Is India Civilized’

: ’Am I a Canadian Writer?’

Marshall McLuhan : The Mechanical Bride : Folklore of Industrial Man

: ’A Time to Change’, ’Enterprise’, Margaret Atwood : Surfacing ’Poet, Lover, ’Birdwatcher’, ’Background, Casually’ A.M. Klein : ’Indian Reservation : Caughnwaga’ ’An Introduction’, ’My Grandmother’s House’, ’Summer P.K. Page : ’First Neighbours’ in Calcutta’ Earle Birney : ’Bushed’, ’The Bear on the Delhi ’Self-Portrait’, ’A River’, ’The Road’ Fall’, ’Fear No Fall’ African and Caribbean Literature Kanthapura Chinua Achebe : Things Fall Apart The Guide Dennis Brutus : ’A Common Hate Enriched our Cry the Peacock Love and US’

Shashi Deshpande : That Long Silence

Gabriel Okara

Unit-IX : Linguistics, Phonetics & Modern Grammar David Diop

: The Mystic Drum ’Africa’

1.

Properties of Human Language

Edward Braithwaite: ’Tizzic’

2.

Scope and Branches of Linguistics

Derek Walcott

: ’Ruins of a Great House’

3.

Language Varieties and Language Change

Mervyn Morris

: ’Judas’

4.

Saussure’s Concept of Linguistic Immutability and Mutability of Sign.

5.

Sign, Wole Soyinka Nadine Gordimer

Speech Mechanism and Phonemes in English 5

: ’Lion and the Jewel’ : ’English Language Literature and Politics in South Africa’

UP GDC English Literature (16th to 20th Century) Unit-I : Literary & Social Background Study of the main literary trends and socio-cultural background from 16th to 20th century. Unit-II : Poetry John Donne The Canonization A Valediction : Forbidding Mourning A Valediction : Of the Book A Valediction : Of My Name A Valediction : Of Weeping Alexander Pope The Rape of the Lock William Wordsworth: Lines Composed a Few Miles Above Tintem Abbey. Odeon Intimations of Immortality Sonnet on Sonnet S.T. Coleridge The Rime of the Ancient Mariner Ode to Dejection P.B. Shelley To A Skylark Ode to the West Wind The Indian Summer John Keats Ode on a Grecian Um Ode to Autumn on first looking into Chapman’s Homer Crossing the Bar Lord Tennyson Prologue (In Memorium) The Lotus Eaters. Robert Browning Rabbi Ben Ezra My Last Duchess Prospice T.S. Eliot The Waste Land W.B. Yeats The Second Coming Sailing to Byzantium Prayer for my Daughter W.H. Auden In Memory of W.B. Yeats The Unknown Citizen Petition Philip Larkin Church Going Poetry of Departures Reasons for Attendance. Unit-Ill : Drama William Shakespean The Tempest Macbeth Richard II Twelfth Night 6

John Webster Ben Jonson Goldsmith G.B. Shaw T.S. Eliot Harold Pinter S. Beckett John Osborne Unit-IV : Novel Henry Fielding Jane Austen Charles Dickens Thomas Hardy Virginia Woolf E.M. Forster Graham Greene William Golding D.H. Lawrence Unit-V: Prose Francis Bacon

: : : : : : : :

Duchess of Malfi Volpone She Stoops to Conquer Saint Joan The Family Reunion The Birthday Party Waiting for Godot Look Back in Anger

: : : : : : : : :

Tom Jones Pride and Prejudice Great Expectations Hard Times Mrs. Dalloway A Passage to India Power and the Glory Lord of the Flies The Rainbow

Of Marriage and Single Life Of Truth Addison and Steele Of The Club Sir Roger At Home Disappointment In Love Dream Children ’A Reverie’ Charles Lamb Poor Relations In Praise of Chimney Sweepers George Orwell Notes on Nationalism Walter Pater On Style Unpopular Essays- Ideas that have Bertrand Russell Helped Mankind, Ideas that have Harmed Mankind Unit-VI : Literary Criticism : The Poetics Aristotle Philip Sidney : An Apology for Poetry W. Wordsworth : Preface to the Lyrical Ballads. Matthew Arnold : Function of Criticism at Present Time : Tradition and The Individual T.S. Eliot Talent LA. Richards : Principles of Literary Criticism. F.R. Leavis : The Great Tradition

UPHESC Assistant Professor Exam. 2021

ENGLISH Solved Paper 1.

Maya in Cry, the Peacock is obsessed with the astrological predication that (a) her children will not survive (b) her husband will go insane in the fourth year of her marriage (c) her marriage will end in death in the fourth year (d) her house will crumble to dust Ans. (c) : Anita Desai had shown Maya in “Cry, the Peacock” is obsessed with the gloomy prophecy of the Albino astrologer, the prophecy she or her husband would die during the fourth year of their marriage. Anita Desai is an Indian novelist and she has been shortlisted for the Booker prize three time. 2. Who is the presiding goddess of the village of kanthapura in Raja Rao’s novel of the same name? (a) Durga (b) Laxmi _____(c) Kenchamma _______(d) Himavathy _______ Ans. (c) : Kanthapura's patron goddess, Kenchamma supposedly battled a demon on the red Kenchamma Hill near town “ages, ages ago” and has protected Kanthapura’s people. ‘Kanthapura’ novel is written by Raja Rao. It is the story of Gandhi’s struggle for independence from the British, come to village Kanthapura in South India. 3. Where did Raju in R.K Narayan’s The Guide go right after getting out of jail? (a) To the barber (b) To the school _____(c) To the railway station (d) To Rosie’s banker Ans. (a) : Raju, in R-K Narayan’s ’The Guide’ (1960) goes right after getting out of Jail to the barber. The talkative Barber says ’’you like a maharaja now”. He guesses rightly that Raju has been released from the Jail. Raju repeats ’’Not a bad place” R K- Narayan got Sahitya Akademi Award in 1960. 4. Which of the following is a not a statement from Macaulay’s ’’Minutes on Education? (a) We have to education a people who cannot at present be educated by means of their mother tongue. (b) In India. English is the language spoken by the ruling class. (c) In India, be good enough to consult their (the natives’) intellectual taste at expense of their intellectual health. (d) There is now in that country (Russia) a large educated class abounding with persons fit to serve the state in the highest of functions. UPHESC (Assistant Prof.) Exam 2021

7

[Exam Date: 28.11.2021

Ans. (c) : Thomas Babington Macaulay famous proposal of promoting the English language is called ’The Macaulay Minutes’ (1835). He suggested that English should be taught in place of Arabic, Sanskrit, and Persian in colonial schools in India. 1. To introduce English text books is schools. 2. To make English a medium of instruction in higher educational institutions. __________________________ 5. The persona in Sarojini Naidu’s ’’The Flute Player of Brindaban” Compares himself herself with: (a) and abandoned tree (b) a lost cow (c) a homeless bird (d) a distraught gopi Ans. (c) : 'The Flute Player of Brindaban' is one the most anthologized poems of Sarojini Naidu. This forms part of ’The Broken Wing’ . In this poem Sarojini Naidu compares herself with a homeless bird (staza-2) Note- 'The Flute player* by Ruskind Bond.___________ 6. In his ’’Chicago Lecture” Swami Vivekananda alludes to the fables a frog in a well: (a) to hint at the causes of religious differences (b) to explain the limitations of Christianity (c) to hint at the smallness of the Jews (d) to indicate the limited vision of some Hindu ________reformists_____________________________ Ans. (a) : In his "Chicago Lecture” Swami Vivekananda alludes to the fable of a frog in a well to hint at the causes of religious differences. This story told by Swami Vivekananda at the world's Parliament of Religions at Chicago on 15th September 1893. _ _ _ _ _ 7. In response to which of the following was Sri Aurobindo’s ”Is India Civilized”? (a) Sir John Woodroffe’s The Serpent Power (b) William Archer's India and the Future (c) Katherine Mayo’s The Mother India (d) Harry H Field’s After Mother India Ans. (b) : Sri Aurobindo's "Is India civilized” wrote it in answer to negative criticism of Indian culture by the English drama critic William Archer. ’India and the Future' by William Arches was published in the year 1917. It deals with concept of unity in India, Hindu spirituality, caste system in India, manners and customs, art and culture, and education. 8. Which of the following linguistic terms is associated with ’’Prescriptivism”? (a) Correctness (b) Description _____(c) Variation _________(d) Change__________ Ans. (a) : The linguistic term “Correctness” is associated with prescriptivism'. It is the attitude that tries to impost an unchanging standard of ’correct’ usage in language, especially in grammar, It is rejected as a misconceived dogma by most modem linguists. YCT

9.

In 'Rasa Theory' the external manifestation of a strongly felt emotion is called (a) Vibhava (b) Anubhava _____(c) Sthayibhava _______(d) Sattvikabhava Ans. (b) : In ’Rasa Theory’ the external Manifestation of strongly felt emotion is called Anubhava. Anubhava is the physical or bodily expressions of Bhavas or emotions. Since they are after effects of bhavas. They are called Anubhavas. ________________ 10. According to the Rasa school, Kavya and Natya are: (a) antonyms (b) cousins _____(c) synonyms _________(d) distantly related Ans. (c) : According to The Rasa School Kavya and Natya are synonyms. Kavaya:- It is highly artificial Sanskrit literary style employed in the court epics of India from the early centuries AD. Natya:- Natya refers to the theatrical dance art of India originating in the temple and still devoted largely to the enactment of divine epics and embracing an elaborate system of body postures, hand, gestures and foot movements. ___________________________________ 11. With reference to F.R. Leavis's ''Literary Criticism and Philosophy'' which of the following statement is/are correct? (1) Philosophy, we say, is "concrete” and poetry "abstract”. (2) Words in poetry invite us not to "think about" and judge but to "feel into" or "become" to realize a complex experience...... Select the correct answer using the codes below: (a) (1) only (b) (2) only (c) both (1) and (2) (d) Neither (1) nor (2) Ans. (b) : F-R- Leavis’s is ’Literary criticism and philosophy’ was first published in ‘Scrutiny’ in the year 1937 and it was written in response to the suggestion of wellek that Leavis should write about the theoretical basis of his criticism. F R- Leavis’ ’Literary criticism and Philosophy’ the correct statement is ’words in poetry invite us not to ’’think about” and judge but to ’’feel into” or become” —

12.

The concept of 'Arche writing' was developed by . (a) Jacques Derrida (b) Paul de Man (c) Michel Foucault (d) Stanley Fish Ans. (a) : ’’Arche-writing” is a term used by French Philosopher Jacques Derrida in his attempt to reorient the relationship between speech and writing. Derrida argued that as far as Plato’s, speech has been always given priority over writing. 13. Text is considered to be by New Critics. (a) A historical construct (b) A cultural construct

UPHESC (Assistant Prof.) Exam 2021

8

(c) Autotelic _____(d) A linguistic construct ___________________ Ans. (c) : Text is considered to be Autotelic by New criticism. The New critics emphasized ’’close reading” as a way to engage with a text, and paid close attention to the interactions between form and meaning. The term New Criticism coined by John Crowe Ransom in (1941). 14. According to Fredric Jameson, Post-Modern Culture is associated with capitalism. (a) market (b) Imperialist _____(c) Multi-national______(d) Monopolistic _____ Ans. (c) : According to Fredric James Post-Modern Culture is associated with Multi-national Capitalism. ’Postmodernism’ or ’cultural logic of Late capitalism is a 1991 book by Fredric Jameson, in which the author offers a critique of Modernism and Postmodernism from Marxist perspective. 15. Who has developed the term 'Strategic Essentialism'? (a) Homi Bhabha (b) Aijaz Ahmad (c) Gayatri Chakravorty Spivak _____(d) Edward Said __________________________ Ans. (c) : ’Strategic Essentialism’ is a term first coined by Gayatri Chakravorty Spivak, a postcolonial feminist philosopher and literary theorist. She employs the terms in her deconstructive reading of the work of the Subaltern Studies Group. _________________________ 16. "I sit in the top of the wood, my eyes closed". (Ted Hughes) In the above line, the pronoun 'I' has been used for a (a) fox (b) cow _____(c) hawk _____________(d) jaguar ___________ Ans. (c) : ”1 sit in the top of the wood, My eyes closed” In the line, The pronoun T has been used for a Hawk. Ted Hughes’ poem ’Hawk Roosting’ on its literal level of meaning is an expression of a bird of prey, the Hawk which is sitting on a tree and meditating about its power of destruction, its ability to suppress change and its conceited arrogance and superiority. ________________ 17. Who of the following was not a predecessor of William Shakespeare? (a) C. Marlowe (b) R. Greene (c) T. Lodge (d) Ben Jenison Ans. (c) : ’Thomas lodge’ was not predecessor of William Shakespeare. The Principal predecessors of Shakespeare in the ‘History of the English Drama’ were the ’’University wits as they are called the most notable of there are Christopher Marlowe, Robert Greene, Peele, Lyly, kyd and Ben Jonson. Thomas Lodge (1558-1625) was an English writer and author during Elizabethan and Jacobean Periods. ______ 18. The First Book of Common Prayer was issued in: (a) 1536 (b) 1540 (c) 1549 (d) 1600 YCT

23.

Which of the following texts of the Oxford Movement is not correctly matched? (a) (John Henry Newman) (Loss and Gain) (b) (Charlotte Mary Yonge) (Scenes of Critical Life) (c) (Gerard Manley Hopkins) (The Week of Deutschland) (d) (John Keble) (The Christian Year) Ans. (b) : Oxford movement, 19th century movement centered at the University of Oxford that sought a renewal of "Catholic" or 'Roman Catholic' thought and practice within the church of England in opposition to the protestant tendencies of the church. The member of Oxford movements are 1- John Henry Newman 2- Gerard Manley Hopkins 3- John Keble 4- Edward Pusey. Thus, correct option is (B). 24. The three sections of the play lion and Jewel are named ___________. (a) Sowing, Reaping and Gamering (b) Summer, Spring and Winter (c) Mosque, Temple and Caves (d) Morning, Noon and Night Ans. (d) : 'The Lion and the Jewel' is a play by Nigerian writer Wale Soyinka that was first performed in 1959 in Ibadan. The three Sections of the play 'Lion and Jewel’ are named Morning, Noon and Night. The play chronicles how Baraka, the lion fights with the modem Lakunle over the right to Marry Sidi, the titular Jewel. 25. Complete the following statement with the most appropriate choice: Chinna Achebe’s Things Fall Apart _____. (a) question Western understanding of Christianity (b) takes the reader to colonized Johannesburg (c) has a female protagonist (d) Challenges the western notions of historical truth Ans. (d) : 'Things Fall Apart’ is the debut novel by Nigerian author Chinua Achebe, first Published in 1958. The novel takes its title from a verse in the poem "The second coming" by W-B- Yeats an Irish poet. 'Things Fall Apart’ Challenges the western nations of historical truth. It is an intrusive religious presence and an insensitive government together cause the traditional Umufian world to fall apart. 26. Peter Porter’s ’’Your Attention Please” is a poem about __________. (a) apocalypse, government and society (b) family and society (c) school, collage and education (d) trees and forests

Ans. (c) : The First Book of Common Prayer was issued in 1549 in the reign of Edward VI. The First Book of Common Prayer was compiled originally to reform, Simplify and consolidate. The Latin Services of the medieval church and to produce a single, convenient and complete volume in English as an authoritative guide for the priests and people of the Church of England. 19. Which three Germanian tribes invaded Britons in the fifth century AD bringing them to the roots of modern English? (a) The Danes, Saxons and Celts (b) The Jutes, Angles and Saxons (c) The Celts, Jutes and Saxons (d) The Saxons, Danes and Angles Ans. (b) : The History of The English language really started with the arrival of three Germanic tribes who invaded Britain during The 15th century A D- These tribes The Angles, the Saxons and the Jutes, crossed the North sea from what today is Denmark and North Germany. 20. Make me thy lyre, even as the forest is: What if my leaves are falling like its own! The tumult of thy mighty harmonies will take form both a deep, autumnal tone, sweet thought in sadness. (P.B Shelley, "Ode to the West Wind") In the above quote, the word ’both’ in the fourth line refers to: (a) the forest and the persona (b) the autumn and the forest (c) the persona and the autumn (d) the West Wind and the persona Ans. (a) : The above given line has been taken from ’Ode to the West Wind’ by P-B- Shelley refers word ’both’ to the forest and the persona. The Speaker and the trees of the forest are both decaying the trees are losing their leaves and he has been bowed down by life. 21. In Browing’s poem "My Last Duchess", the name of the painter Fra Pandolf is used (a) once (b) twice (c) thrice (d) four times Ans. (c) : In Robert Browing’s poem ”My Last Duchess” the name of the painter Fra Pandolf is used thrice (three times) in the first 16 lines of the poem. He again implies that he values the painting because of its status as an object the shows off his (duke’s) wealth and clout. 22. Fill in the blank in the following sentence with right option from the list given below: The ____ display an aversion to "literal" language which might presume a one-to-one correspondence between words and things. (a) Elizabethans (b) Angustans (c) Victorians (d) Modems Ans. (d) : In the above sentence, the fill right option in blank is 'Modems' Thus correct sentence is ----’The Modems display on aversion to "literal” language which might presume a one to-one correspondence between words and things. UPHESC (Assistant Prof.) Exam 2021

9

YCT

Ans. (a) : ’Your Attention Please’ by Peter Porter is a sixty-three lines poem that contained within one Stanza of text. In this poem Peter Porter deals with themes of the apocalypse, government and society. 27. Which of the following novels is associated with Charistian? (a) The Vicar of Wakefield (b) Sense and Sensibility (c) Sybil (d) Vanity Fair Ans. (c) : Sybil or ’The Two Nations’ is an 1845 novel by Benjamin Disraeli. Sybil traces the plight of the working classes of England. Disraeli’s interest in the Chartist movement, a working class political reformist movement that sought Universal male Suffrage and other parliament reforms. 28. Which of the following is an example of Campus Novel? (a) Parents and Children (b) The Power and the Glory (c) Lucky Jim (d) Lord of Flies Ans. (c) : A Campus Novel also called the Academic Novels is a novel that’s story is around or about a campus of a University. The genre is said to have started in the 1950s. ’’The Groves of Academe ”by Mary McCarthy, written in 1952, is often called the first campus novel. ’Lucky Jim’ (1954) is a example of campus novel by Kinsley Amis. 29. Whose novels are called the ’Novels of Manners’? (a) N. Hawthorne’s (b) Jane Austen’s (c) Maria Edgeworth’s (d) Charles Lever’s Ans. (b) : The novel of Jane Austen is also Known as ’A Novel of Manners’ It is kind of fiction focused on everyday routine life and events. Her novels are based on the premise that these are vital relationship between manners, social behaviour and character. 30. In Margaret Atwood’s Surfacing, the fish is a symbol of . (a) suffering (b) consciousness (c) unity and wholeness (d) the unconscious Ans. (c) : In Margaret Atwood ’Surfacing’ the Fish begins to Symbolize wholeness, the unity between mind and body that She is seeking. Fish like frogs, snakes and worms, have heads that extend directly into the shoulders, thereby making it more difficult to see their heads and their bodies as separate. _________________ 31. The plays of which of the following EuroAmerican playwrights cannot be described as ’The Theatre of the Absurd’? (a) Samuel Beckett (b) Jean Gent (c) Edward Bond (d) Eugene Ionesco UPHESC (Assistant Prof.) Exam 2021

Ans. (c) : ’The Theatre of the Absurd’ term coined by Martin Esslin in his essay ’The Theatre of the Absurd (1960) which begins by focusing on the playwrights Samuel Beckett Jean Genet and Eugene Ionesco. ’Theatre of the Absurd’ agreed with the Existentialist philosopher Albert Camus’ essay ’’The Myth of Sisyphus” (1942) that the human situation is essentially absurd, devoid of purpose. ________________________ 32. Which of the following correctly describes India Queen? (a) an autobiography of an Indian princes (b) a fictional account of the life of Maharani Gayatri Devi (c) a heroic tragedy in rhymed couplet by John Dryden and Robert Howard (d) a long poem in free verse by Keki N. Daruwalla Ans. (c) : ’The Indian Queen’ is a heroic tragedy is rhymed couplet by Sir Robert Howard, written in Collaboration with John Dryden. It was first performed in 1664. The Play’s protagonists can be divided into the groups of two countries and their respective royal families. The Peruvians and the Mexicans. ___________ 33. T.S Eliot’s phrase ’’Dissociation of Sensibility” occurs in his essay entitled . (a) ’’Hamlet and His Problems” (b) ’’The Function of Criticism” (c) ’’Tradition and the Individual Talent” _____(d) ’’The Metaphysical Poets” _______________ Ans. (d) : TS- Eliot’s phrase ’’Dissociation of Sensibility” occurs in his essay entitled ’The Metaphysical poets’. The term ’Dissociation of Sensibility’ comprises two words which are dissociation and sensibility. The meaning of the word ‘dissociation’ is Separation or detachment and ‘sensibility’ is sensation or feeling, so this term stands for detachment of thought form sensation in the case of poetry writing. 34. In which of the following plays does Bobadill appear? (a) King Henry IV, Part II (b) Hamlet (c) Spanish Tragedy _____(d) Every Man in His Humour _______________ Ans. (d) : Bobadill is a character who appears in the play of Ben Jonson’s ’Every Man in His Humour’ (1598) Captain Bobadill, a braggart captain. He is fond of quoting Snatches of Elizabethan plays, particularly from ’The Spanish Tragedy’ he is foolish and cowardly but not Vicious. ___________________________________ 35. Who of the following English sovereigns was deeply intested in Witchcraft ? (a) Charles I (b) Charless II _____(c) James I ___________(d) Edward VI _______ Ans. (c) : James I had a longstanding fascination with witchcraft and he considered himself an expert in the subject. In 1597, he wrote a book about witchcraft ’Daemonologie’ which Shakespeare likely consulted when writing the Witches spells in ‘Macbeth’.

10

YCT

36.

In Beckett’s Waiting for Godot, who are waiting for Godot? (a) Pozzo and Lucky (b) Vladimir and Estragon (c) The little boy (d) The people in the audience Ans. (b) : ’Waiting for Godot’ is a play by Samuel Beckett in which two characters, Vladimir(Didi) and Estragon (Gogo), engage in a variety of discussions and encounters while awaiting the titular Godot who never arrives. 37. To which festival does the title of the play Twelfth Night refer? (a) Epiphany (b) Whitsun (c) Lent (d) Easter Ans. (a) : The title of ’Twelfth Night’ refers to the twelfth night of Christmas, also referred to as The Eve of Epiphany, a day that commemorates the visit of the of the Magi to the baby Jesus and is often celebrated with a temporary suspension of rules and social orders. 38. Which of the following is the right description ofMirabell? (a) He is not typical Restoration beau, a combination of the cynical and the gracious. (b) He follows Lady Wish fort because he loves her with all his heart. (c) He falls into the trap of trusting Fainall. (d) He has in him an attraction for undisguised ________materialism. ___________________________ Ans. (d) : ’Mirabell’ is a character of William Congreve’s play ’The Way of the World’ (1700). He is the ideal Restoration beau, a combination of the cynical and the gracious. He has the vices and the virtues of his kind. He has in him an attraction for undisguised materialism. 39. To which Greek Mythological character is Dr. Faustus compared in the Prologue to the play Dr. Faustus? (a) Hercules (b) Perseus _____(c) Icarus ____________(d) Theseus _________ Ans. (c) : In Christopher Marlowe’s play ’Doctor Faustus’ (1604) Prologue, Dr. Faustus is compare to Greek Mythological Icarus’. In the play Faustus ’desire to Icarus’ doomed attempt to fly. Icarus did not heed his father’s warning and flew too close the sun causing his wings melt and sending him plunging to his death. 40. T.S. Eliot’s Murder in the Cathedral drew on an eye witness to the event, a clerk named: (a) Christopher Higgins (b) Norman Laurence (c) Edward Grim (d) Sean Boman Ans. (c) : T.S. Eliot ‘Murder in the Cathedral’ first performed in 1935 that portrays the assassination of Archbishop Thomas Becket in Canterbury Cathedral during the reign of Henry II in 1170. Eliot drew heavily on the Writing of Edward Grim, a clerk who was an eyewitness to the event. UPHESC (Assistant Prof.) Exam 2021

41.

In Which of the following plays do the belowgiven lines appear? ”A Politician is the devil’s quilted anvil; He fashions all sins on him, and the blows Are never heard:” (a) The Tempest (b) Dr. Faustus (c) The Duchess of Malfi (d) The Way of the World Ans. (c) : "A Politician is the devil's---------- And never heard” lines are taken from John Webster's play "The Duchess of Malfi” (1623) in Act-3 Scence-2. 42. Which of Claudius and Laertes’ traps for Hamlet succeeds in Killing him? (a) The poisoned cup (b) The sharpened sword (c) The poisoned dagger _____(d) The poisoned sword ____________________ Ans. (c) : According to UPHSE commission correct option is (c) while correct option should be (d) because in The given play 'Hamlet' The poisoned Sword has been denoted in Act 4 scene v. Thus, option (c) is correct answer. _________________________________ 43. Which of the following is not a part of a Tragedy, according to Aristotle? (a) Fable (b) Spectacle _____(c) Imitation __________(d) Diction __________ Ans. (c) : According to Aristotle "Tragedy is an imitation of an action that is serious, complete, and of a certain magnitude-through pity and fear effecting the proper purgation of these emotions". These are six main elements of Tragedy- Plot, Character, diction thought, spectacle and Melody. Thus, option (c) is not appropriate answer ___________ 44. In which year does the play St. Joan open ? (a) 1492 (b) 1429 (c) 1560 _____________(d) 1776 ____________ Ans. (b) : 'Saint Joan’ is set in France in the period (1429-1431) with an epilogue set in 1456. Four of the scenes are set in castles in the northern part of the country, including the castle occupied by the court of the Dauphin in chin on and the castle in Rouen where Joan's trial takes places. 45. Which term has been used by Matthew Arnold in his Culture and Anarchy for the middle classes? (a) Barbarians (b) Philistines _____(c) Populace __________(d) Hellenic _________ Ans. (b) : Philistines term has been used by Matthew Arnold in his ’Culture and Anarchy’ (1869) for the middle classes. Culture is a study in perfection in making things better than they are, moved by the normal and social passion for doing good. Arnold divides human society into three main classes. 1- Barbarians - represents aristocrats 2- Philistines - represents middle class 3- The Populace- represents working class.

11

YCT

46.

How many phonemes are there in the word ’some’? (a) One (b) Two _____(c) Three ____________(d) Four ____________ Ans. (c) : ’Three’ Phonemes are there in the word ’some’, based on phonetics transcriptions. Phoneme A Minimal unit of Potentially meaningful sound within a given language’s system of recognized sound distinctions. Note:- Phoneme is a smallest unit of speech Sound. 47. In My Place, Sally Morgan tries to eliminate: (a) personal felling of bitterness (b) everything un-Australian (c) the white man’s burden _____(d) most features of bitterness________________ Ans. (a) : ’My Place’ is an autobiography written by artist Sally Morgan in 1987. It is about Morgan’s quest for knowledge of her family part and the fact that she has grown up under false pretence. In ’My place’ Sally Morgan tries to eliminate personal feelings of bitterness. 48. Formulating general hypothesis about language-change comes under the branch of linguistics called . (a) Computational Linguistics (b) Forensic Linguistics (c) Historical Linguistics (d) Pragmatics Ans. (c): Formulating general hypothesis about language-change comes under the branch of linguistics called ‘Historical Linguistics’. It is also called Diachronic linguistics the branch of linguistics concerned with the study of phonological grammatical and semantic changes the reconstruction of earlier stages of Languages. ____________________________ 49. Complete the statement given below with an appropriate option Saussure believed that a synchronic explanation of things demonstrates how forms and meanings are interrelated . (a) because of the space and time distinction of languages (b) because of signifiers (c) in a particular space that needs the imagination to comprehend (d) at a particular point of time in a particular _________language system _______________________ Ans. (d) : According to Saussure’s Synchronic’ Linguistics is the study of language at a particular point in time in a particular language system. Note- Saussure is also differenciate between Langue and Parole. ____________________________________ 50. Which of the following is not correct in respect of ’Wallace Stevens’ ’’Thirteen Ways of Looking at a Black Bird”? (a) The Poem has thirteen short, in equal but separate sections. (b) Stanzas II and XII consist of two lines each. UPHESC (Assistant Prof.) Exam 2021

(c) One stanza addresses the ’’thin men”. (d) The persona claims familiarity with base and uncouth accents. Ans. (d) : ’Thirteen Ways of Looking at a Blackbird’ is a poem from Wallace Stevens’ first book poetry Harmonium. The poem consists of thirteen short separate sections, each of which mentions black birds in some way. The Black Bird as a way to describe the relations between human kind, Nature and emotion. Thus, option (d) is not correct is respect of Wallace Stevens. 'Thirteen ways — ' ______________________ 51. A foot consisting of one stressed syllable followed by an unstressed one is called: (a) Spondee (b) Trochee (c) lambic (d) TerzaRima Ans. (b) : The term ’Trochee’ is from the French Trochee and from the Greek phrase ‘Trokhaiaspous’ which means ’’running foot”. A foot consisting of one stressed syllable followed by an unstressed one is called Trochee. Spondee :- It is two Successive Syllables with approximately equal strong stresses. 52. Milton in his Paradise Lost, Book I has written: ’’And justifie the ways of God to man. ’’What is the subject of the verb ’’justifie” in the text of the poem? (a) I (b) Eternal Providence (c) Heavenly Muse (d) Spirit Ans. (a) : Milton’s main purpose for ’Paradise Last’ book is to Justify God’s way to men. Man believed that was spiteful and these were put upon a person and that they needed to ask for forgiveness. In "And Justifie the ways of God to man” I is the subject of the Verb Justifie in the text of the poem. 53. Indentify the rheme in the following sentence: Yesterday, Paul went for a walk. (a) Paul went away, (b) Paul went for a walk. _____(c) Went for a walk, (d) Yesterday, Paul Ans. (b) : In the given Sentence the appropriate answer (b) is correct which other three option is not correct. In the context of ’Rheme’. The theme is the element of the message, it is that with which the clause is concerned. The rest of the message, the part in which the theme is developed is called the Rheme. Thus, correct Rheme is ----"Paul went for a walk.” 54. Indentify the word having wrong phonemic transcription against it : (a) abed/ d 'bed/ (b) abject /'aebjekt/ (c) ablution / d ’blu : j n /

12

(d) absurd / d ’ b ' z 3 : d / Ans. (b) : In the given Sentence, the wrong Phonemic transcription is option (B) 'object'/aebjekt/. The correct phonemic transcription is 'Abject' ae/aebd3ekt/. Phonetic Transcription :- It is the visual representation of speech sound by means of symbols. YCT

55.

Which of the following is an absolute characteristic of ESP. according to DudleyEvans? (a) ESP is defined to meet specific needs of the learners. (b) ESP makes use of underlying methodology and activities of the discipline it serves. (c) ESP is centred on the language appropriate to these activities in terms of grammar, lexis, register, study skills, discourse and genre. (d) Most ESP courses assume some basic knowledge of the language systems. Ans. (d) : An absolute characteristic of ESP, according to Dudley - Evans (1997) 1. ESP is defined to meet specific needs of the learners. 2. ESP makes use of underlying methodology and activities of the discipline it serves. 3. ESP is centered on the language appropriate to these activities in terms of grammar, lexis, register, study skills, discourse and genre. Thus correct option (d) part is not absolute characteristic. 56. Words ending with bound morpheme ’logy* have the primary stress on the syllable of the suffix. (a) Immediately preceding (b) immediately following (c) root (d) additional Ans. (a) : In linguistics, word ending with Bound Morpheme ’logy' have the primary stress on the Immediately Preceding syllable of the suffix. Bound Morphemes Bound Morphemes cannot stand alone, and are usually tied to another morpheme. Bound Morpheme

Derivational

_________________ Prefixes

Suffixes

Inflectional

Suffixes _____________

57.

In 1798 edition of The Lyrical Ballads did not contain which of the following: (a) 19 poems of William Wordsworth (b) Archaic spellings and terms (c) Name of either poet (d) "The Idiot Boy" ________________________ Ans. (c) : 'Lyrical Ballads' Published in 1798 is often considered the beginning of the Romantic period because Wordsworth and Coleridge’s poetry make a distinct change in form and subject matter from and Subject matter from Neoclassical Poetry. It is collections of 23 poems, in which 19 poems of Wordsworth and 4 poems of Coleridge. Thus, In 1798 edition of 'The Lyrical Ballads' did not contain 'Name of either poet'

UPHESC (Assistant Prof.) Exam 2021

58.

Who of the following distinguishes between the ’’text of pleasure” and "text of bliss”? (a) Roland Barthes (b) Horace _____(c) Longinus__________(d) T.S. Eliot ________ Ans. (a) : French literary theorist Roland Barthes distinguishes between the “Text of pleasure” and “Text of bliss”. The text of pleasure corresponds to the Readerly Text which does not challenge the reader's position as a subject. While the Writerly Text provides bliss which explodes literary codes and allows the reader to break out of his or her subject position. 59. What two images does Emily Dickinson use to symbolize ’’success” in ’’Success is Counted Sweetest”? (a) The Olive branch and the laurel (b) The laurel and the victorious army (c) The nectar and the victorious army _____(d) The nectar and the olive branch ___________ Ans. (c) : "Success is counted sweetest" is a lyric poem by Emily Dickinson written in 1859 and published anonymously in 1864. The poem uses the two image of to comprehend nectar and the victorious army. Here nectar is referring to the Sweetness of victory. ________ 60. ”O powerful Western Star!” In the above line of Watt Whitman’s poem Bloom ’d”, the Fallen Star’ refers to: (a) John F. Kennedy (b) Abraham Lincoln (c) Manhatten _____(d) The Poet himself _______________________ Ans. (b) : In Walt Whitman's poem "When Lilacs Last in the Dooryard Bloom ‘d” the 'Fallen Star' refers to Abraham Lincoln. It is an elegy to president of America Lincoln in a way that is all the more profound for seeing the president's death as only a smaller, albeit highly symbolic. 61. In which of the following novels does one come across the line:” A pure hand needs no glove to cover it?” (a) The Scarlet Letter (b) The Old Man and the Sea (c) Beloved (d) The Adventures of Huckleberry Finn Ans. (a) : 'The Scarlet Letter, A Romance’ is a work of historical fiction by American author Nathaniel Hawthorne Published in 1850. "A pure hand needs no glove to cover it" has taken from ‘The Scarlet Letter’ which means that if a person is honest and truthful they have nothing to hide. Therefore, no glove needed. Dimmesdale does not confers his sin like the first time nor does he confess the second time around, on the scaffold. 62. Who is Santiago’s model in The Old Man the Sea? (a) African Lions (b) JoeDiMaggi'O (c) Yellow Lions (d) Manolin

13

YCT

Ans. (b) :Joe Di Maggi’O is Santiago's model in 'The Old Man and the Sea', Santiago expresses his admiration for Joe Di Maggi’O. Di Maggi’O represents skill and similar family background in that his father was a fisherman and he probably grew up poor. 63. "He who was living is now dead We who were living are now dying With a little patience.” (T.S. Ehot, ’’The Waste Land”) (a) Tiresias (b) Fisher King (c) Christ (d) SirParcifal Ans. (c) : Christ is the one being spoken about in "He who was living is now dead" we modem folks are in a similar position as Christ but instead of being dead, we live in a sort of half death as "We who were Waving are now dying with a little patience. This line appears is Section-5 ' What the Thunder Said' poem of T S Eliot 'The Waste Land' (1922). _________________________ 64. How many stories each pilgrim was supposed to tell in The Canterbury Tales? (a) One story (b) Two stories (c) Three stories (d) Four stories Ans. (d) : Tour Stories’ each pilgrim was supposed to tell in 'The Canterbury Tales' by Geoffrey Chaucer. Each of the pilgrims will tell two tales on the way to Canterbury and two more on the way back, whomever the Host decides has told the most meaningful and comforting stories will receive a meal paid for by the rest of the pilgrims upon their return 65. Specify the source of the title of the novel Far From the Madding Crowd from amongst: (a) Paradise Lost (b) "The Rime of Ancient Mariner" (c) "Elegy Written in a Country Churchyard" (d) "The Lady of Shallot" ___________________ Ans. (c) : The title of the novel 'For from the Madding Crowd' (1874) comes from Thomas Gray's famous 18th century poem "Elegy Written in a Country Churchyard" (1751). 66. In writing The Glass Menagerie, Tennessee Williams drew on the earlier (a) "The Gentleman Caller" (b) Happy August the Tenth" (c) "Miss Coynte of Greene" (d) "Women Awakening Elizabeth Ans. (a) : 'The Glass Menagerie' is a memory play by Tennessee Williams that Premiered in 1944 and catapulted William from obscurity to fame. In the play, William drew on an earlier short story as well as a screen play he had written under the title of 'The Gentleman Caller' 'The Glass Menagerie' was William's first Successful play. UPHESC (Assistant Prof.) Exam 2021

67.

Which of the following is not correct with reference to Gandhi’s Hind Swaraj? (a) Gandhi is arguing with the extremists who want to make a violent revolution. (b) Indians allowed the Britishers to come and rule, says Gandhi (c) By "Western Civilization" Gandhi means the modem, capitalist culture (d) Gandhi instead of citing western scholars cites Indian scriptures to bring his point ________home. ________________________________ Ans. (d) : In M. K. Gandhi's Hind Swaraj' (1909) is not appropriate option (d) Gandhi instead of citing western Scholars cites Indian scripture to bring his point home. ‘Hind Swaraj’ is the most seminal work of Mahatma Gandhi and most original contribution to political theory after Kautilya's Arthashastra. 68. Raja Rao maintains that telling the Indian tale in English involves: (a) A complete mastery of two languages, the English and one Indian (b) The ability to mentally translate from one language to the other (c) A knowledge of religions and spiritual texts. (d) Describing in an Indian language not one's own, a spirit that is one's own. Ans. (d) : In the context of Indian tale Raja Rao maintains that telling the Indian tale in English involves describing in an Indian language not one's own, a spirit that is one's own. 69. True poetry, according to Sir Aurobindo, (a) is written under the influence of the Uanishadas. (b) is like a divine mantra. (c) cannot be translated. _____(d) is dreamlike. __________________________ Ans. (b) : According to sir Aurobindo, “True poetry is like a divine Mantra.” Sir Aurobindo emphazies that, “Poetry is a psychological phenomenon, the poetic impulse a highly charged force of expression of the mind and soul of man, and therefore in trying to follow out its line of evolution it is the development of the psychological motive and power. Hence option (b) is correct. 70. In Shashi Deshpande’s That Long Silence, Jaya is renamed after her marriage, (a) Kusum (b) Suhasini (c) Indu (d) Sarita Ans. (b) : And after Marriage, Jaya lives in the family life thinking about her family and her husband Mohan renamed his wife Jaya as "Suhasini" as he wanted her to be 'soft, Smiling, placid motherly women. This novel won the Sahitya Academi award in the year is 1990. In this novel, the soul protagonist is 'Jaya' . That Long Silence' (1988) novel has written by Shashi Deshpande.

14

YCT

UP Higher Education Service Commission Asst. Prof. Exam. 2016

ENGLISH (Solved Paper) 1.

Thalia was muse of comedy. Melpomene was muse of tragedy. Polymnia was muse of religious hyms. Erato was muse of erotic poetry. Terpsichore was muse of choral song and dance. ______ 3. George Bernard Shaw derives his theory of Life Force from: (a) Charles Darwin (b) Henry Bergson (c) Heidegger ________(d) Emile Zola ________ Ans: (*) George Bernard Shaw’s theory of life force or creative evolution and that of the well known French philosopher Henri Bergson were congenial in terms of a dynamic view of life. Shaw wrote - ’’Bergson is the established philosopher of my sect I have called it the Life Force and the Evolutionary Appetite. Bergson called it the Elen Vitale ” UPHESC opted answer (a). ______________________ 4. From where have the following lines been taken? ”1 imagine this midnight moment’s frest : Something else is alive Beside the clock’s loneliness And this blank page where my fingers move.” (a) Ted Hughes : ’The Thought Fox’ (b) Philip Larkin : ’Next Please’ (c) T.S. Eliot : ’A Cooking Egg’ (d) W.B. Yeats : ’Byzantium’ _________________ Ans: (a) The lines ”1 imagine this midnight moment’s ” have been taken from Ted Hughes’ poem ’The Thought Fox’. Ted Hughes was a modernist English poet and became Poet Laureate in 1984. 5. Which of the following is not a ’closet drama’? (a) Milton’s Samson Agonistes (b) Shelley’s Prometheus Unbound (c) Shakespeare’s King Lear (d) Thomas Hardy's The Dynasts ______________ Ans: (c) Closet drama is a drama suited primarily for reading rather than production. John Milton's 'Samson Agonistes (1671), Shelley’s 'Prometheus Unbound’ (1820) and Thomas Hardy’s 'The Dynasts' (1903) are closet drama while William Shakespeare’s 'King Lear’ (1605) is one of his great tragedies, written with purpose of stage performance. 6. The statement that the Metaphysical poets, in their poems, presented ’’heterogeneous ideas yoked by violence together” was made by(a) Dryden (b) Dr. Johnson (c) T.S. Eliot (d) Coleridge 15 YCT

Match List-I with List-II and select the correct answer by using the codes given below the lists. The lines are from various Shakespearean plays: List-I (A) To be or not to be, that is the question ! (B) The lunatic, the lover and the poet, Are of imagination all compact (C) As flies to wanton boys, are we to the gods ! They kill us for their sport. (D) Yet, I do fear thy nature; It is too full of ’the’ milk of human kindness.. . List-II (1) Macbeth (2) King Lear (3) A Midsummer Night’s Dream (4) Hamlet Codes : A B C D (a) 2 3 4 1 (b) 4 3 2 1 (c) 3 2 1 4 (d) 1 2 3 4 _____________________ Ans: (b) To be or not to be - ’Hamlet’ Prince Hamlet’s soliloquy speech in Act 3, Scene 1 of the play depicts his nature of ’indecision’ which becomes his tragic flaw later. • The lunatic, the lover - ’A Midsummer Night’s Dream’ In these lines Theseus comments that love and imagination are similar. These lines take place in Act 5, Scene 1. • As flies to wanton boys ’King Lear’ These words are said by Gloucester in Act 4, scene 1 of King Lear. • Yet I do fear -’Macbeth’ Lady Macbeth says this line right after reading a letter from Macbeth. This speech happens in Act 1, scene 5 of ’Macbeth’. __________________________________ 2. Who is the muse of epic poetry? (a) Calliope (b) Clio (c) Urania ___________(d) Euterpe___________ Ans: (a) The muses were the Greek goddesses of poetic inspiration, the adored deities of song, dance and memory on whose mercy the creativity, wisdom and insight of all artist and thinkers depended. In the ’Theogony’ Heroid tells about nine musesCalliope, the eldest muse was muse of epic. Clio muse was muse of history. Urania was muse of astronomy. Euterpe was muse of lyric poetry.

UPHESC Asst. Prof. Exam. 2016

[Exam. Date : 15 Dec 2018]

Ans: (b) Dr. Samuel Johnson in his ’Life of Cowley' while referring to 'Wit' or Metaphysical Conceit stated it as "the most heterogeneous ideas yoked by violence together. _____________________________________ 7. Pick out the incorrect of the following: (a) William Shakespeare : 1564-1616 (b) John Milton: 1608-1674 (c) William Wordsworth: 1789-1850 (d) T.S. Eliot: 1888-1965____________________ Ans: (c) William Wordsworth was bom on 7 April 1770, in Cockermouth, Cumberland. He died at Rydal Mount from pleurisy disease on 23 April 1850. He was appointed Poet Laureate in 1843 till his death. 8. In which of the following does D.H. Lawrence’s novel, Sons and Lovers end? (a) Mrs Morel dies of cancer (b) Paul is married to Miriam (c) Paul is married to Clara (d) William elopes with Mrs Baxter ____________ Ans: (a) The final chapter of D. H. Lawrence's novel, 'Sons and Lovers' is named 'Derelict'. Mrs. Morel dies of cancer after her children Paul and Anne decide to give her an overdose of morphia to speed up her death. Paul decides to stay single after her mother's death.________________________________________ 9. In Philip Larkins’ poem, The Whitsun Weddings, Whitsun is a festival that comes after Christmas on the (a) fifth Sunday (b) sixth Sunday (c) fourth Sunday _____(d) seventh Sunday Ans: (*) Whitsun or Whit Sunday, in the seventh Sunday after Easter (Pentecost), deep into spring, when people often marry. The poem 'The Whitsun Weddings' was written by famous Movement Poet Philip Larkin and first published in his collection 'The Whitsun Weddings" in 1963. 10. Who brought out the Female Spectator as a response to Addison’s and Steele’s periodical? (a) Eliza Heywood (b) Sarah Fielding (c) Fanny Burney _____(d) AphraBehn _______ Ans: (a) Eliza Heywood published a monthly journal entitled 'The Female Spectator' from April 1744 to May 1746. It appeared as a response to Addison's and Steele's periodical - 'The Spectator' (1711 to 1712). The four characters appeared in this periodical areThe Female Spectator, Euphrosine, Mira and a Widow of Quality. ____________________________________ 11. Which character in literature is associated with hilarious linguistic blunders and word play? (a) Mrs Grundy (b) MrsMalaprop (c) Mrs Backbite ______(d) Lady Bracknell Ans: (b) The character Mrs. Malaprop is a humorous aunt who gets mixed up in the schemes and dreams of young lovers in Richard Sheridan's 1775 comedy 'The Rivals'. One of the funniest aspects of Mrs. Malaprop's character is that she often uses an incorrect word to express herself. Mrs. Backbite appears in Richard Sheridan's play 'The School for Scandal'. Lady Bracknell is in Oscar Wilde's 'The Importance of Being Earnest'. UPHESC Asst. Prof. Exam. 2016

Lady Grundy has been referred to in various works, including 'Speed the Plough', 'Erewhon' and 'Hard Times'. 12. Who of the following was not a Renaissance theologian? (a) Huldreych Zwingli (b) Martin Luther (c) John Calvin _______(d) William Caxton Ans: (d) Huldreych Zwingli (1484-1531) was bom in Wildhaus, Switzerland. He was a leading figure in the Protestant Reformation in Switzerland. He was a strong proponent of Renaissance Humanism. Martin Luther (1483-1546) was a German Professor of theology and a seminal figure of Reformation. John Calvin (1509-1564) was a French theologian and reformer in Geneva during the Protestant Reformation. William Caxton (1422-1491) was an English merchant who established printing press in England. Hence, correct option is (d). ______________________ 13. Which of one of the following is not a picaresque narrative? (a) Thomas Nashe's The Unfortunate Traveller (b) Tobias Smollette's Roderick Random (c) Henry Fielding's Tom Jones (d) Oliver Goldsmith's The Vicar of Wakefield Ans: (d) Picaresque novel is an early form of novel, usually relating the adventures of a rogue or lowborn adventurer as he drifts from place to place and from one social milieu to another in his efforts to survive. Thomas Nashe's The Unfortunate Traveller (1594), Tobias Smallete's Roderick Random (1748) and Henry Fielding's Tom Jones (1749) are examples of the picaresque novel. Oliver Goldsmith's The Vicar of Wakefield (1768) is a comic and satirical novel. ______ 14. The concept of ’White man’s burden’ was involved by (a) Rudyard Kipling (b) E.M. Forster (c) Paul Scott ________(d) Conrad ___________ Ans: (a) The concept of 'White man's burden' was expounded by Rudyard Kipling in his poem published in 1899 with the same title. The concept attempts to justify the imperialist and expansionist policies. 15. The term ’langue’ was first used by _____ (a) Freud (b) Jung (c) Saussure __________(d) Chomsky _________ Ans: (c) In linguistics and language, langue is an abstract system of signs (the underlying structure of language). Swiss linguist Ferdinand de Saussure first used the term 'langue' in his 'Course in General Linguistics' (1916). _____________________________ 16. According to ’’art is twice removed from reality.” (a) Plato (b) Longinus (c) Aristotle __________(d) Horace ___________ Ans: (a) Plato, in his book 'Republic' gave the 'theory of mimesis' in which he refused the importance of art saying - "art is twice removed from reality". It means art is an imitation of an imitation. Aristotle replied to the charges made by Plato against art. Despite being agree with the theory of imitation, he distinguished between the various arts.

16

YCT

17. Which of the following statements about Longinus* The Sublime is not true? (a) It is a book on rhetoric (b) It has been written in the 4 th Century A.D. (c) The treatise has been one of the most influential (d) Longinus is inspired by the Latin writer ________Quintilian to a certain extent_______________ Ans: (b) ’On the Sublime’ is a rhetorical treatise during the first century A.D. written by Longinus. The treatise was written in Greek. It was the most influential rhetorical text through much of the period of the Second Sophistic. To a certain extent, Longinus is inspired by the Latin rhetoric writer, Quintilian. 18. Which of the following statements best describes the term ’Womanisin’? (a) ’’Woman is a Womb.” (b) ’’Womanist is to feminist as purple is to lavender.” (c) ”1 am a woman Phenomenally.” (d) ’’Every woman has known the torment of ________getting up to speak.” _____________________ Ans: (b) Womanism is derived from the term ’Womenist’ coined by Alice Walker. She states that womanism is just a darker shade of Feminism i.e. black feminism. In her book ’In Search of Our Mothers’ Alice says - ’’Womanist is to feminist as purple is to lavender.” While feminism focuses strictly on gender discrimination, womanism opposes discrimination against women in the areas of race, class and gender. _______________________________ 19. Who wrote the following lines? ’’The great contention of criticism is to find the faults of the moderns, and the beauties of the ancients.” (a) Aristotle (b) Samuel Johnson (c) S.T. Coleridge _____(d) F.R. Leavis _______ Ans: (b) Dr. Samuel Johnson in his ’Preface to the Plays of William Shakespeare’ states that the great contention of criticism is to find the faults of the moderns and the beauties of the ancient. ____________ 20. ’’Willing suspension of disbelief’ is a term coined by: (a) John Keats (b) Matthew Arnold (c) Samuel Taylor Coleridge (d) William Wordsworth ____________________ Ans: (c) Samuel Taylor Coleridge coined the term ’Willing Suspension of disbelief in his ’’Biographia Literaria”. He proposes that the images in poetry have a force and logic of their own that urge the reader to enter the world of poetic illusion and to suspend judgment as to whether the images of that poetic world have a real existence. ______________________ 21. Which of the following according to Aristotle is the soul of tragedy? (a) Reasoning (b) Plot (c) Song_____________(d) Character ________ Ans: (b) According to Aristotle, tragedy has six main elements - plot, character, diction, thought, spectatle and song, of which the first two are primary. Here, he UPHESC Asst. Prof. Exam. 2016

considers plot the most important element. By plot Aristotle means the arrangement of incidents.________ 22. Post-colonial criticism considers issues of: (a) appropriate and indispensable resources of epics (b) inappropriate silences about colonized people (c) probability theory (d) influence of science on literature ___________ Ans: (b) Post colonial critics are very vocal to refuse the concepts of ’White Mythology’ or an attempt of dominance over entire world by European and American thinkers. Post colonial criticism has embraced a number of aims - most fundamentally, to re-examine the history of colonialism from the perspective of the colonized. These critics attack on imperialism and inappropriate silences about colonized people. Edward Said, Gayatri Chakravorty Spivak and Homi Bhabha are prominent ’Postcolonial’ critics. _______________________________________ 23. Which of the following terms refers to a surprise development in the action? (a) Peripeteia (b) Denouement (c) Anagnorisis _______(d) Parodos __________ Ans: (a) In Greek tragedy, Aristotle derived some terminologies that are following• Peripeteia- Peripeteia is a sudden reversal, often in fortunes of the protagonist. Peripetia is, therefore the turning point in Greek tragedy. It is also referred to as a surprise development in the action. • Denouement-It is the final outcome of the main dramatic complication in a tragedy. • Anagnorisis-Anagnorisis is the moment of recognition. The protagonist of a tragedy recognizes that his trouble is his own fault. • Parodos- A Parodos was one of two gangways on which chorus and actors made their entrances from either side into the orchestra. _____________________ 24. Who of the following makes a distinction between the text of pleasure and the text of bliss? (a) Longinus (b) Horace (c) Roland Barthes (d) T.S. Eliot _________ Ans: (c) According to Roland Barthes text of pleasure is the text that contents, fills, grants euphoria, the text that comes from culture and does not break with it, is linked to a comfortable practice of reading. On the other hand, text of bliss is the text that imposes a state of less, the text that discomforts, wasettles the reader’s historical, cultural, psychological assumptions, the consistency of his tastes, values, memories, brings to a crisis his relation with language. Barthes is famous for his essay ’The Death of Author’ 1967. His other important works areThe Fashion System, Writing Degree Zero, Elements of Semiology, The Pleasure of the Text, Criticism and Truth. _______________________________________ 25. Who is Dryden’s mouthpiece for his views about drama in ’Of Dramatic Poesy’? (a) Neander (b) Phaedrus (c) Lysis (d) Cratylus

17

YCT

political point, whereas Derrida is precisely concerned Ans: (a) John Dryden’s ’An Essay on Dramatic Poesy’ with the later. presents a brief discussion on Neo-classical theory of Jacques Lacan - Psychoanalysis and Structuralism. Literature. ’An Essay on Dramatic Poesy’ is written in Luce Irigaray - French Feminism. the form of a dialogue among four gentlemen : Jean Francois Lyotard - Post-Structuralism and PostEugenius, Crites, Lisideius and Neander who Modernism. represent Charles Sackville, Sir Robert Howard, Sir 29. Identify the American critic who deconstructs Charles Sidney and Dryden himself respectively. identity categories: Neander, Mouthpiece of Dryden, steps in to support (a) Raymond Williams English drama precisely because of its subplots, (b) Eve Kosofksy Sedgwick mixture of mirth and tragedy (in tragicomedy) and (c) Judith Butler spirited, multiple characters. ______________________ (d) Atan Sinfield ___________________________ 26. The first anthropologist to see the potential of Ans: (b & c) Judith Butler explores the production of Saussure’s analysis of language was? identities such as homosexual and heterosexual and (a) A.J. Griemas (b) Gerard Genette the limiting nature of identity categories. An identity (c) Levi-Strauss _______(d) Tzvetan Todrov category for them is a result of certain exclusions and Ans: (c) Ferdinand de Saussure’s work in structural concealments, and thus a site of regulation. Butler linguistics and semiology greatly influenced Levibelieves that identity forms through repetition or Strauss. Claude Levi-Strauss is widely regarded as the imitation and is not original. father of structural anthropology. In 1972, his book Eve Kosofsky Sedwick in her ‘Epistemology of the Structuralism and Ecology detailed the tenets of what Closet’ opined that the binary opposition would become structural anthropdogy. He proposed (man/woman, gay/straight) on which discourse and that culture, like language, is composed of hidden subjectivity are founded are revealed to be not fixed, rules that govern the behaviour of its practitioners. but fluid, fictional - and can, therefore be 27. Who said, ’’Delight is the chief, if not only end of destabilized. In this way Sedgwick also deconstructs poesy”? the identity of sexes. ____________________________ (a) Dryden (b) Keats 30. Which of the following is not one of the (c) Shelley ___________(d) Wordsworth _______ influences on Emerson’s thinking? Ans: (a) Deliberating the function of poetry, thinkers (a) Romanticism (b) Pragmatism put different opinions. For Plato, the function of (c) Idealism __________(d) Stoicism __________ poetry is to instruct the reader. Aristotle wanted Ans: (b) In America, followers of Dewey claim that poetry to delight, Horace to do both and Longinus to Emerson is the fountainhead of American transport. Dryden was of the opinion that the final end pragmatism, Stenley Cavell, takes a powerful position of poetry is delight and transport rather than with evidents that Emerson is not a pragmatic. instruction. He valued poetry as a work of art rather Emerson was an idealist. He believed that ultimately than mere imitation. Dryden believes ’’Delight is the some form of mind was the foundation for the rest of chief, if not the only end of poesy”. reality. His spiritual teaching about Self-Reliance and Unlike the opinion of Dryden, the Romantics valued the Over-Soul are all based on his idealism. poetry for life’s sake. Shelley remarked The American Romantic and Transcendental ’’Shakespeare, Dante and Milton are philosophers of movements of the nineteenth century were a reaction the very loftiest power.” against the 18th century Age of Enlightenment’s emphasis on science and rationalism as ways of Keats believes that the function of poetry is ’’that is discovering truth. Emerson was one of the pioneer of should be a friend to soothe the cares, and lift the these movements. thoughts of men”. Like stoics, Emerson also stressed on value and Wordsworth says ’’poetry is the breath and finer spirit therefore influence of stoicism is evident in his works. of all knowledge, the impassioned expression that is in the countenance of all science”. _________________ 31. Who of the following was not a transcendentalist? 28. Who of the following is associated with the Yale School of Deconstruction? (a) Ralph Waldo Emerson (a) Paul de Man (b) Margaret Fuller (b) Jacques Lacan (c) Mark Twain (c) Luce Irigaray (d) Nathaniel Hawthorne ____________________ (d) Jean-Francois Lyotard ___________________ Ans: (c) Transcendentalism is essentially a kind of Ans: (a) Yale School of Deconstruction refers to a practice by which the world of facts and the categories moment in the 1970s when the work of Jacques of common sense are temporarily exchange for the Derrida was taken up and experimented with by four world of ideas and the categories of imagination. prominent literary critics - Paul de Men, J. Hillis It developed in the early nineteenth century in Miller, Geoffrey Hartman and Harold Bloom. The America. Ralph Waldo Emerson, Frederic Henry Hodge, George Ripley, George Putnam, Margaret Yale School of Deconstruction tends to be a highly Fuller, Henry David Thoreau and Nathaniel Hawthorne playful and erudite form of close reading, with very were prominent transcendentalists. little interest in making either a philosophical or UPHESC Asst. Prof. Exam. 2016

18

YCT

Mark Twain, original name Samuel Langhorne Clemens was an American novelist famous for his novels ‘An Adventury of Tom Sawyer’ and ‘Huckleberry Finn’. 32. ’Mending Wall’ by Robert Frost was first published in: (a) North of Boston (b) The Mountain Interval (c) A Witness Tree (d) Steeple Bush ___________________________ Ans: (a) ‘Mending Wall’ is a poem by American poet Robert Frost. It published in Frost’s second poem collection ‘North of Boston’. Frost’s poetry collectionsA Boy’s Will 1913 North of Boston 1914 Mountain Interval 1916 New Hampshire 1923 West Running Brook 1928 A Further Range 1936 A Witness Tree 1942 Steeple Bush 1947 In the Clearing 1962 33. Examine the following statements and identify the one which is not true. (a) Ernest Hemingway was awarded the Pulitzer Prize for Fiction in 1953 for his novel The Old Man and the Sea (b) The Old Man and the Sea was written in 1951 and published in 1952 (c) The Old Man and the Sea is the first novel by Hemingway (d) The Old Man and The Sea is narrated by an ________anonymous narrator _____________________ Ans: (c) ‘The Old Man and the Sea’ is a short novel written by the American author Ernest Hemingway, in 1951 in Cayo Blanco and published in 1952. Narrated by an anonymous narrator, the novel tells the story of a battle between an old fisherman Santiago and a large Merlin fish. It was his last major work of fiction. In 1953, ‘The Old Man and the Sea’ was awarded the Pulitzer Prize for Fiction and afterwards Hemingway was conferred on Nobel Prize in 1954. Ernest Hemingway worksin Our Time 1925 The Sun Also Rises 1926 A Farewell to Arms 1929 For Whom the Bell Tolls 1940 The Old Man and the Sea 1952 _______________ 34. Which of the following is not true of Hawthorne’s The Scarlet Letter? (a) It tells the story of Hester Pryne (b) It was published in 1850 (c) Pearl is a symbol of love and passion (d) By punishing Hester with the scarlet letter, the Puritan community is promising redemption at _______her death ______________________________ Ans: (d) ‘The Scarlet Letter’ is a novel written by American novelist Nathaniel Hawthorne, published in 1850. Its story revolves around the Puritan set of norms, sin (adultery) and wish for revenge. UPHESC Asst. Prof. Exam. 2016

Hester Prynne, Arthur Dimmesdale, Rojer Chillingwirth and Hester’s daughter Pearl are major characters in this novel. Pearl, Hester’s daughter, is presented as a symbol of love and passion which Puritan community regards as an act of sin (adultery) and punishes Hester for her deed. _________________ 35. How many sections are there in Walt Whitman’s poem, ”A Passage to India”? (a) Five (b) Seven (c) Nine _____________ (d) Ten______________ Ans: (c) ‘A Passage to India’ is a poem by American poet Walt Whitman, appeared in 1871-72 edition of ‘Leaves of Grass’ poem collection, divided in nine sections, this poem inspired E. M. Forster to write his 1924 novel, ‘A Passage to India’. The poem accounts new achievements of the modem world like Suez Canal, the great American Railway, his great faith in God and exploration of new places. 36. The sub-title of O’Neill’s play, The Hairy Ape is: (a) ’’Time - The Modem” (b) ”A Comedy of Modem Life” (c) ”A Comedy of Ancient and Modem Life” (d) ”A Comedy of Ancient Life” ______________ Ans: (c) ‘The Hairy Ape’ is a play by Eugene O’ Neill, published in 1922. The sub-title of the play is ‘A Comedy of Ancient and Modem Life’. The play comprises eight scenes. It is considered one of the prime achievements of Expressionism on stage. Yank, Paddy, Long and Mildred Douglas are major characters. Eugene O’ Neill was an American playwright. He was awarded Nobel Prize in Literature in 1936 and received Pulitzer Prize for Drama four times (1920, 1922, 1928, 1957). ‘Beyond the Horizon’, ‘The Hairy Ape’, ‘Desire Under the Elms’, ‘Mourning Becomes Electra’, ‘Ah, Wilderness’ are his famous plays. ______ 37. In Emily Dickinson’s poem, ”1 Never Lost As Much But Twice”, how many losses has the persona suffered? (a) Two (b) Three (c) Four _____________ (d) Five _____________ Ans: (a) ‘I Never Lost As Much But Twice’ is an eight lines poem by Emily Dickenson. She reveals in this poem that she has lost her two friends and begs before God to return them. Emily Dickinson was an American poet during nineteenth century. Her poems were heavily influenced by the Metaphysical poets of seventeenth century England, as well as her reading of the Book of Revelation and her upbringing in a Puritan New England town, which encouraged a Calvinist, Orthodox and conservative approach to Christianity. ‘I taste a liquor never brewed’, ‘I measure every Grief I meet’, ‘Like Brooms of Steel’, ‘Because I could not stop for Death’ are some of her poems. _____________ 38. What type of poem is Whitman’s ’’When Lilacs Last”? (a) An elegy (b) A sonnet (c) A panegyric (d) A dirge

19

YCT

Ans: (b)“Just listen to the frozen air, Or in wild despair, pick an armful of Darkness to bring it here to lie. . ..” Therefore the persona in Kamala Das poem, “My Grandmother’s House” wants to bring from her grandmother’s house an armful of darkness. “My Grandmother’s House” is a short poem from Kamala Das which focuses on love lost, nostalgia and emotional pain. ‘The Sirens’, ‘Summer in Calcutta’, ‘The Descendants’, ‘The Old Playhouse’ are her other poems. _______________________________________ 43. ’’Beneath it we have played,” (Toru Dutt, ”Our Casuarina Tree”) In the above line, the pronoun ’we’ stands for (a) the poet and the baboon (b) the poet and the tree (c) the poet and her brother (d) the poet, her sister and her brother __________ Ans: (d) Tom Dutt was an Indian poetess. She was youngest among three siblings. She is known for her poems - ‘Our Casuarina Tree’, A Shenf Gleaned in French Fields’ and ‘Ancient Ballads and Legends of Hindustan’. In the following lines of ‘Our Casuarina Tree’“But not because of its magnificence Dear is the Casurina to my soul : Beneath it we have played; though years may roll, O sweet companions, loved with love Intense .....” Here she is referring to her sister and brother with whom she had spent frolic time under the tree. _______ 44. How many similes have been used in Sarojini Naidu’s ’’The Palanquin Bearers”? (a) 12 (b) 8 (c) 6 ________________(d) 7 ________________ Ans: (b) Simile is a figure of speech involving the comparison of one thing with another thing of a different kind using ‘like’ or ‘as’. In ‘Palanquin Bearers’ of Sarojini Naidu there are seven similes have been used‘like a flower’, Tike a bird’, Tike a laugh’, Tike a pearl’, Tike a star’, Tike a bean’, Tike a tear’. ‘Like a pearl’ simile is used twice in the poem, therefore the commission has considered ‘b’ as right option. 45. The first gift given by Raju to Rosie in the novel, The Guide, was a (a) silver box (b) silver coin (c) silver brooch ______(d) silver ring _________ Ans: (c) Raju gifted Rosie a silver brooch in the novel ‘The Guide’ written by R. K. Narayan. Narayan was awarded the first Sahitya Akademi Award in English for this novel in 1958. ‘The Guide’ is based on a fictional story of Raju, Marco, Rosie, Gaffur, are other important characters. 46. What symbolizes the supreme symbol of the essence and efflorescence of God in Savitri, Canto I? (a) Rose (b) Light (c) Love (d) Nature 20 YCT

Ans: (a) ‘When Lilacs Last in the Dooryard Bloom’d’ is a long poem written by American poet Walt Whitman as an elegy to President Abraham Lincoln, though it never mentions the president by name. Like most elegies, it develops from the personal to the impersonal, from an intense feeling of grief to the thought of reconciliation. The poem appeared in his poem collection ‘Sequel to Drum-Taps’ in 1865. _____ 39. Who is the nicknamed ’blue roses’ in the play Glass Menagerie? (a) Amanda (b) Laura (c) Mrs Wingfield (d) Tom Ans: (b) ‘The Glass Menagerie’ is a play by American playwright Tennessee Williams. It was first performed in 1944. Tom, Amanda, Laura, Jim, are main characters in this play. Jim Nicknames Laura ‘blue roses’ is this play. The name ‘blue roses’ turns Laura’s defect into an asset : her unusual, otherworldly qualities are seen as special rather than debilitating. __________________________ 40. In Keki N. Daruwalla’s poem, ’’The Ghaghra in Spate”, who floats over the rooftop? (a) a woman (b) a cow (c) a buffalo__________(d) a goat ____________ Ans: (c) ‘The Ghaghara in Spate’ is a poem by Keki N. Daruwalla which depicts the rural landscape during monsoon season when rivers in India often wreck havoc. “And through the village The Ghaghra steers her course; Thatch and dung-cakes turn to river scum, A buffalo floats over to the roof top Where the men are stranded.” These lines depict how a buffalo floats over the rooftop. Keki N. Daruwalla is an Indian poet, bom in Lahore in 1937. ‘Under Orion’, ‘Apparition in April’, Tn Morning Dew’, ‘Crossing of Rivers’ are some of his renowned poem collections. 41. In A K Ramanujan’s poem, ’A River’, the river which flows through Madurai is: (a) Vankaya (b) Vankai (c) Vaika (d) Vainkai Ans: (c) ‘A River’ poem by A. K. Ramanujan is a poem figures Vaikai river which flows through Madurai. River is unnamed in this poem. The poet depicts how the river washes away three village houses, one pregnant woman and a couple of cows named Gopi and Brinda with it. A. K. Ramanujan was an Indian poet, bom in Mysore in 1929. ‘The Striders’, ‘Relations’, ‘Second Sight’ are his famous poem collections. __________________ 42. The persona in Kamala Das’ poem, ”My Grandmother’s House” wants to bring from her grandmother’s house (a) a collection of books (b) an armful of darkness (c) a handful of dust (d) a pair of windows UPHESC Asst. Prof. Exam. 2016

Ans: (d) The story of Kanthapura is narrated by a woman, Achakka in Raja Rao’s novel ’Kanthapura’. The novel is narrated in the form of a Sthala Purana by her. Kanthapura’ explores Gandhian philosophy of truth, non-violence and sacrifice. Murthy is the protagonist of the novel. __________________________ 51. Who is quoted in the epigraph of Shashi Deshpande’s That Long Silence? (a) Elizabeth Robins (b) Kate Millet (c) BetyFriedan _______(d) Robyn Warhol _____ Ans: (a) Elizabeth Robins was an American actress and novelist. Her speech extract is quoted in the epigraph of Shashi Deshpade’s ’That Long Silence’ "If I were a man and cared to know the world I lived in, I almost think it would make me a shade uneasy - the weight of that long silence of one-half of the world.” ’That Long Silence’ is significant for its portrayal of frustration, suppression and gender discrimination with Indian women. The novel was awarded Sahitya Akademi Award in 1990. _________________________ 52. Kamala Das’ ”An Introduction” is a strong indictment against: (a) Capitalist society (b) Globalised world (c) Patriarchal society (d) Traditional society Ans: (c) Kamala Das’ ”An Introduction” is a 60 lines poem contained within a single stanza. This poem is a very clear feminist statement that advocates for free choices for all women. It is an autobiographical and confessional poem that voices out her concern about patriarchy, starting from politics to sexual politics. 53. Identify the correct order of appearance of the poems in Nissim Ezekiel’s The Collected Poems: (a) ’A Time to Change', ’Enterprise', 'Poet, Lover and Birdwatcher', 'Background Casually' (b) 'Enterprise', 'Poet, Lover and Birdwatcher', 'Background Casually', 'A Time to Change' (c) 'Poet, Lover and Birdwatcher', 'Background Casually', 'Enterprise', ’A Time to Change' (d) 'Background Casually', ’A Time to Change’, ’Poet, Lover and Birdwatcher', 'Enterprise' Ans: (a) Poem Poem Publication Collection Year A Time to A Time to 1952 Change Change The Unfinished 1960 Enterprise Man Poet, Lover and Exact Name 1965 Birdwatcher Background Hymns in 1976 Casually Darkness 54. Which novel of Anita Desai in the following list does not end on a tragic note? (a) Cry, the Peacock (b) Fire on the Mountain (c) Where Shall We Go This Summer (d) Baumgartner's Bombay Ans: (c) 'Where Shall We Go This Summer?' is a novel by Anita Desai. Its story is based on a middleaged pregnant woman Sita, who is fed up with modem life style and seeks relief in nature. The novel ends up 21 YCT

Ans: (b) ‘Savitri : A Legend and a Symbol’ by Aurobindo is an epic poem. Aurobindo takes the mythical story of Savitri and Satyavan and describes his vision of existence and explores the reason for ignorance, darkness, suffering and pain, the purpose of life on earth and the prospect of a glorious future for humanity. In Savitri, Book 1, Canto 1, the supreme symbol of the essence and efflorescence of God is dawn i.e. light. 47. Which novel of Anita Desai is considered to be "an epic on Calcutta”? (a) Bye-Bye, Blackbird (b) Voices in the City (c) Cry the Peacock (d) Fire on the Mountain ______________________ Ans: (b) ’Voices in the City’ published in 1965, in the second novel by Indian writer Anita Desai. It is a story of three siblings - Monisha, Nirode and Amla. It is an epic on Calcutta giving its comprehensive description - its important landmarks such as Howrah, Victoria Memorial, Grand Hotel etc, crowds, noisiness, frenzied pace and lack of places to think and reflect. Anita Desai’s other important novels areBye-Bye Blackbird 1971 Cry the Peacock 1963 F ire on the Mountain 1977 The Peacock Garden 1974 The Village by the Sea 1982 Journey to Ithaca 1995 Fasting, Feasting _______ 1999 ________________ 48. Aurobindo’s Savitri : A Legend and Symbol is a an epic poem in (a) Couplets (b) Ballad meter (c) Blank verse ________(d) Free verse _________ Ans: (c) ’Savitri : A Legend and a Symbol’ is an epic poem in blank verse by Aurobindo. ’Savitri’ poem is composed of almost 24,000 lines. It published in 1940. The poem is referred to as the supreme revelation of Sri Aurobindo’s vision. 49. What are the stylistic similarities of Mulk Raj Anand’s The Untouchable to high-modernist works? (a) Contemporary idiom (b) Stream of consciousness technique (c) Unreliable narrator (d) Comedic parody _________________________ Ans: (b) Stream of consciousness is a phrase used by William James in his ’Principles of Psychology’ to describe the unbroken flow of perceptions, thoughts and feelings in the waking mind. It is a narrative technique through which the author attempts the fluid and eruptive nature of human thought. Mulk Raj Anand’s ’The Untouchable’ begins with the thinking trance of Bakha. He thoughts uncongenially of his home as he lay half awake in the morning of an autumn day. 50. The story of Kanthapura is narrated in flashback by .” (a) Kenchamma (b) Moorthy (c) Ratna (d) Achakka UPHESC Asst. Prof. Exam. 2016

with a happy note that she is reunited with her husband and agrees to go with him. • ’Cry the Peacock’ begins with the death of Toto and ends with Gautam’s murder and Maya’s insanity. • ’Fire on the Mountain’ ends with rape and murder of an old woman Ila. • ’Baumgartner’s Bombay’ is a story of a German Jew refugee Baumgartner, who arrives India for safety. In the end, he is stabbed and filled by another German. 55. Language is a symbol system based on (a) pure conventions (b) arbitrary conventions (c) pure or arbitrary conventions (d) no conventions need be followed ___________ Ans: (c) According to Robert Henry Robins ’’language is a symbol system based on pure or arbitrary conventions .... Infinitely extendable and modifiable according to the changing needs and conditions of the speakers”. According to this definition, language is a symbol system. Every language selects some symbols for its selected sounds. 56. The phonetic transcription of the word ’realm’ is (a) /rielm/ (b) /relm/ (c) /re:1m/ ___________(d) /realm/ ___________ Ans: (b) A phonetic transcription looks quite unlike a regular transcription, The transcriber notes the way the spoken words are pronounced, using a special alphabet of phonetic symbols. The most common is the international phonetic alphabet. Transcription of realm is - /relm/. _________________ 57. The sentence pattern of ’He is now here in my room’ is: (a) SVCAA (b) SVOCA (c) SVACA __________(d) SVAAA __________ Ans: (c) A sentence pattern is an arrangement of words. This arrangement needs to be in a grammatically correct structure. He / is / now / here I in my room (S) (V) (Adv.) (Com.) (Adjunct) Subject + Verb + Adverb + Complement + Adjunct Hence, sentence pattern is - SVACA _______________ 58. Literary stylistics is the study of (a) Variation in the form of a word (b) Variation in language (c) Variation in the grammatical structure (d) Variation in morphemes __________________ Ans: (b) Literary stylistics is a practice of analyzing the language of literature using linguistic concepts and categories, with the goal of explaining how literary meanings are created by specific language choices. Therefore, literary stylistics is the study of variation in language and literary forms such as poetry, drama and prose.________________________________________ 59. Language behaviour is known as (a) parole (b) langue (c) competence _______(d) sign _____________ Ans: (a) According to Swiss linguist Ferdinand de Sassure ’language’ is the faculty of speech or ability to speak and it has two aspects - ’langue’ and ’parole’. UPHESC Asst. Prof. Exam. 2016

Langue implies the language system and parole implies the act of speaking that is, language behaviour. Noam Chomsky coined the term ’Competence’ in linguistics which means the tacit knowledge of the language. Sign in linguistics is any unit of language used to designate objects or phenomena of reality. 60. In linguistics, the diachronic approach is (a) non-historical (b) synchronic (c) historical _________(d) ahistorical ________ Ans: (c) In linguistics, the diachronic approach is the study of a language through different periods in history. Diachronic linguistics refers to the study of how a language evolves over a period of time. Tracing the development of English from the Old English period to the twentieth century is a diachronic study. 61. Morphology deals with (a) speech sound (b) word-structure (c) sound system of a language (d) phonemes of all human languages __________ Ans: (b) Morphology is the study of word structure, the way words are formed and their relationship to other words in the same language. Sound system of a language is called phonology and a phoneme is a basic of linguistics that studies how humans produce and perceive sounds. 62. What is the technical term for the ’tum-te-tum’ rhythm of English? (a) syllable time (b) prosody (c) tempo ____________(d) stress-time ________ Ans: (d) Technical term for the ’tum-te-tum’ rhythm is ’stress-time’ because the stresses fall at roughly regular intervals in the stream of speech. A syllable-timed language is a language whose syllables take approximately equal amounts of time to pronounce. Tempo is a measure of the number of speech units of a given type produced within a given amount of time. Prosody is the study of all the elements of language that contribute toward acoustic and rhythmic effects, chiefly in poetry. 63. The noun phrase, ’a brightly and beautifully shining moon’, replacing the premodifier by the semantically equivalent post-modifier, can be restructured as (a) a shining moon brightly (b) a moon brightly shining (c) a moon shining brightly and beautifully (d) a brightly shining moon beautifully _________ Ans: (c) Noun phrase - A noun phrase is a group of two or more words headed by a noun that includes modifiers (e.g. ’the’, ’a’, ’of them’, ’with her’). Modifiers- Modifiers are used to give extra information about a noun. These are of two kinds pre modifiers and post modifiers. Here in ’a brightly and beautifully shining moon’ premodifier, ’brightly and beautifully shining’ can be restructured as - ’a moon shining brightly and beautifully’.

22

YCT

64.

What is the term used to describe the creative capacity of language to invent new words and sentences? (a) structure (b) syntax (c) duality ___________(d) productivity _______ Ans: (d) Productivity is a general term in linguistics referring to the limitless ability to use language - any natural language - to say new things. It is also known as open-endedness or creativity. The term productivity is also applied in a narrower sense to particular forms or constructions that can be used to produce new instances of the same type. In this sense, productivity is most commonly discussed in connection with word - formation. 65. What kinds of sounds are produced when the vocal chords vibrate while the egressive airstream passes through larynx? (a) Voiceless (b) Vowels (c) Voiced ___________ (d) Consonants _______ Ans: (c) All sounds are either voiced or voiceless. Voiced sounds - those sounds that make our vocal chords vibrate while the egressive airstream passes through larynx. For example - |b| sound is voiced in ’bet’. Voiceless sound- Voiceless sounds are produced from air passing through the mouth at different points. |p| sound is voiceless in ’pet’. Consonant sound- A consonant sound is one in which the air flow is cut off, either partially or completely, when the sound is produced. Vowel sound- A vowel sound is one in which the air flow is unobstructed when the sound is made. 66. In which of the following plays does Shakespeare adhere to the three unities? (a) Hamlet (b) King Lear (c) The Tempest (d) As You Like It Ans: (c) • Three unities are based on conventions established by writers of classical Greek drama. The unity of time implies that the length of time taken by the action in a play should correspond approximately to the length of time taken to perform the play. ’The Tempest’ can be performed in about two hours. • The unity of place requires that the scene should remain unchanged throughout the play. In ’The Tempest’ the entire action, with the exception of the first scene, is confined to the island. • The third unity, the unity of action, prescribes that a play which adhered to the classical unities should have only one plot and that all other incidents in the play should be subordinate to it. In ’The Tempest’ the main plot revolves around Prospero’s effort to regain his dukedom and to undo the evil. • The Tempest is a play by William Shakespeare written in 1611. • Prospero, Miranda, Caliban, Ariel, Alonso, Gonzalo are main characters. UPHESC Asst. Prof. Exam. 2016

67. Who says : ’’Let not the royal bed of Denmark be A couch for luxury and damned incest.” (a) Horatio (b) Ghost (c) Hamlet ___________(d) Polonius __________ Ans: (b) In Act 1, Scene 5 of Shakespeare’s play ’Hamlet’ the ghost says to prince Hamlet”If thou hast nature in thee, bear it not. Let not the royal bed of Denmark be a Couch for luxary and dammed incest.” The ghost explains Hamlet how he was poisoned by his brother, Claudius and after that Claudius married his widow, Gertrude, Hamlet's mother. _____________ 68. The first edition of the Paradise Lost consisted of books. (a) 08 (b) 09 (c) 10 _______________(d) 11 _______________ Ans: (c) The first edition of John Milton’s ’Paradise Lost’ published in 1667 consisted of 10 books. A second edition followed in 1674, arranged into twelve books. Commenting on ’Paradise Lost' William Blake wrote that Milton was a true poet and of the Devil's party without knowing it. _____________________________ 69. The Freytag’s Pyramid analyses the structure of a (a) Two act play (b) Three act play (c) Four act play ______(d) Five act play ______ Ans: (d) Freytag’s Pyramid is a dramatic structural framework developed by Gustav Freytag, a German playwright of the mid-nineteenth century. Freytag’s Pyramid analyses the structure of a five act play and there are five elements of his pyramid - 1. Introduction 2. Rising Movement 3. Climax 4. Falling Action 5. Catastrophe or Denouement.______________ 70. Which one of the following statement is incorrect? (a) Deconstruction was a response to structuralism (b) Deconstruction attacked the assumption that structures of meaning were stable universal or a historical (c) Deconstruction is not a method, but an activity of reading (d) Deconstruction shows that all texts are _______meaningless____________________________ Ans: (d) According to Jacques Derrida, deconstruction seems to center around the idea that language and meaning are often inadequate in trying to convey the message or idea a communication is trying to express. Derrida's deconstruction was a response to structuralism. Structuralism viewed language as a number of signs, composed of a signified and a signifier. Derrida proposed that signs always referred to other signs, existing only in relation to each other, and there was therefore no ultimate foundation or centre. Although, he regarded language and meaning inadequate but never called these meaningless.

23

YCT

UP Higher Education Service Commission Asst. Prof. Exam. 2014

ENGLISH (Solved Paper)

[Exam. Date : 11 Jan 2015]

1.

(a) Spondee (b) Stichomythia The eminent Victorian poet, Charles Swinburne (c) Melodrama ________(d) Irony _____________ called Shakespeare’s ................... ’’the most elemental and primeval” of all his plays. Ans: (b) ’Stichomythia’ is an arrangement of dialogue in which single lines of verse or parts of line are (a) King Lear (b) Macbeth (c) Hamlet ___________(d) Othello ___________ spoken by alternate speakers. In Act III Scene IV Hamlet and Gertrude exchange Ans: (a) Charles Swinburne called Shakespeare’s dialogue that is an example of Stichomythia. In this ’King Lear’ play ’’the most elemental and primeval of scene, Hamlet and Gertrude argue about her marriage all his plays. to Claudius. 2. John Keats, ’Endymion’ is dedicated to: 6. Which of the following plays, according to (a) Leigh Hunt (b) John Milton Martin Esslin, ’’does not tell a story. It explores (c) William Shakespeare (d) Thomas Chatterton a static situation.” Ans: (d) ’Endymion’ the mythological poem of the (a) Harold Pinter’s The Birthday Party English Romantic poet John Keats - four thousand (b) Samuel Beckett’s Waiting for Godot lines about young love of Endymion and the moon (c) Henrik Ibsen’s A Doll's House goddess Dianna. Keats worked on the poem in 1817 (d) G. B. Shaw’s Man and Superman ___________ and published in - 1818 with dedicated to the memory Ans: (b) The most repeated critique of ’Waiting for of Thomas Chatterton. Godot’ is voiced in Irish critic Vivian Mercier’s 3. The term ’Objective Correlative’ was: succinct summary : ’’Nothing happen, twice”. The (a) coined by Dryden and introduced by T. S. Eliot play sub-titled A Tragic Comedy in two acts, does (b) coined by Washington Allston and introduced not, in the words of Martin Esslin, ’’tell a story, it by Keats explores a static situation” that is encapsulated by the words of Estragon : ’’Nothing happen, nobody comes, (c) coined by Washington Allston and introduced nobody goes, it’s awful.” b y T . S. Eliot (d) coined by Dr. Johnson and introduced by T. S. 7. Match List-I with List-II and select the correct answer by using the codes given below the lists: ________Eliot ___________________________________ List-I List-II Ans: (c) The term ’Objective Correlative’ which had (A) Arnold (1) Gerontion been coined by the American painter and poet (B) W. B. Yeats (2) Sohrab and Rustum Washington Allston, was introduced by T. S. Eliot, (C) W. H. Auden (3) Lapis Lazuli rather casually into his essay ’Hamlet and His (D) T. S. Eliot (4) In Praise of Problems’ (1919). Limestone 4. In which month does the novel Mrs. Dalloway Codes: take place? D ABC (a) April (b) June (a) 2 3 4 1 (c) October ___________(d) December _________ 4 (b) 1 2 3 Ans: (b) All the action of Mrs. Dalloway takes place (c) 2 4 1 3 in London during one day and one night in mid- June 1 (d) 4 3 2 1923. Heroine of the novel is Clarissa Dalloway who is Ans: (a) ’Sohrab and Rustum’ a narrative poem in an upper-class housewife married to Richard, a politician blank verse by Matthew Arnold published in - 1853 in the conservative party. Mrs. Dalloway is a novel by in his collection ’Poems’. ’Lapis Lazuli’ written on 25 th Virginia Woolf that was first published in - 1925. July 1936 by W. B. Yeats was published in his ’’Last 5. In drama, a dialogue consisting of single lines Poems”. The poem is dedicated to Harry Clifton. ’In spoken alternately by two characters, especially Praise of Limestone’ (1948) is a poem of 93 unrhymed in dispute (e.g. in Shakespeare’s Hamlet, lines by W. H. Auden, one of the most highly between Hamlet and the Queen) is an example regarded of his past 1939 works. ’Gerontion’ is a poem by T. S. Eliot that was first published in 1920. of:

UPHESC Asst. Prof. Exam. 2014

24

YCT

8.

Who of the following became a clerk at the East Ans: (c) Marlowe is best known for his brilliant India Company? Doctor. Faustus, the story of a university man who (a) William Hazlitt sold his soul to the Devil In both Doctor Faustus and (b) Charles Lamb the earlier two part play. ’Tamburlaine’ Marlowe (c) Samuel Taylor Coleridge created what Ben Jonson called his ’’mighty line”. (d) Thomas De Quincey ______________________ Marlowe created and mastered a new theatrical language - a superb, unrhymed iambic pentameter, or Ans: (b) Charles Lamb has been acclaimed by blank verse - for more expressive than anything. This Common Consent as the prince among the English was a language capable of remarkable intensity, essayist. Lamb was a career clerk with East India intellectual rigor and emotional complexity. Ben Company and known for his persistent unpunctuality. Jonson admiringly said this new kind of writing He was one of several English authors whose literary ’Marlowe’s mighty line’. excursions came as spin offs of the enormous drudgery of running the Indian empire from the 11. ”To fly/steal is woman’s gesture, to steal into mildewed chambers of East India House in London. language to make it fly.” This is the statement in: 9. Match the opening lines of the poems with their (a) Simone de Beauvoir’s The Second Sex correct titles: (b) Virginia Woolfs A Room of One's Own List-I List-II (c) Julia Kristeva’s Powers of Horror (Opening Lines) (Titles of Poems) (d) Helene Cixous’ The Newly Born Woman ______ (1) ”A Prayer for My (I) What dire offence from am’rous causes Daughter” Ans: (d) ’The Laugh of the Medusa’ is an essay by French feminist critic Helene Cixous’. This essay was springs, originally written in French as ”Le Rire de la Medusa” (2) ’’Last Ride (II) Season of mists in 1975 was translated into English by Paula Cohen Together” And mellow and Keith Cohen in 1976. The famous statement of Fruitfulness,.... this essay is - ”To fly/steal is woman’s gesture, to (Ill)Once more the storm (3) ”To Autumn' steal into language to make it fly”. is howling and half hid 12. Which of the following is incorrectly matched? (IV) I said (4) ’’The Rape of (a) Matthew Arnold ’Culture and Then, dearest, since the Lock” Anarchy’ ’tis so, (b) C. M. Bowra ’The Romantic Imagination’ Codes: (c) T . S . Eliot ’Essay on Criticism’ I II III IV (a) 3 4 1 2 (d) G. Wilson Knight ’The Wheel of, Fire’ (b) 4 3 1 2 Ans: (c) ’Essay on Criticism’ is one of the first major (c) 2 1 4 3 poems written by English writer Alexander Pope, (d) 1 2 3 4 published in 1711. Ans: (b) ’The Rape of the Lock’ is a mock heroic 13. ’Cultural Poetics’ is the preferred name for: narrative poem written by Alexender Pope, that was (a) Logocentricism (b) New Historicism first published in 1712. Opening line of the poem is (c) Structuralism ______(d) Holistic approach ’’What dire offence from am’rous causes springs. ’To Ans: (b) The term ’Cultural Politics’ refers to the way Autumn’ is a poem by John Keats composed in 1818 that culture - including people’s attitudes, opinions, and published in - 1820. Opening line of the poem is beliefs and perspectives, as well as the media and arts ’’Season of mists and mellow fruitfulness”. ’A Prayer - shops society and political opinion and give rise to for My Daughter’ is a poem by William Butler Yeats social economic and legal realties. ’Cultural Poetics’ written in - 1919 and published in - 1921. Opening is the preferred name for New Historicism. line of this poem is ’’Once more the storm is howling 14. Which critic has made a very clear distiriction and half hid”. ’The Last Ride Together’ is a poem by between referential and emotive language? English poet Robert Browning, first published in his (a) Northrop Frye (b) T. S. Eliot 1855 collection Men and Women. Opening line of this (c) I. A. Richards ______(d) Dr. Johnson ________ poem is ”1 said Then dearest since ’tis so.” Ans: (c) Ivor Armstrong Richards has made a very 10. Who coined the phrase ’’Marlowe’s Mighty clear distinction between referential and emotive language. According to Richards in the referential or Line’’? scientific, the word faithfully recalls the object. In the (a) Dr. Samuel Johnson (b) Robert Greene emotive use, the word evokes emotions. The emotive (c) Ben Jonson (d) Thom 25 YCT UPHESC Asst. Prof. Exam. 2014

use of language is used in poetry and other literary works. In the scientific use of language, the reference should be correct and the relation of references should be logical. 15. Read the following statement and identify to whom they belong: ’’There is always an appeal open from criticism to nature.” (a) Milton (b) Dr. Johnson (c) Dryden ___________(d) Jeremy Taylor _____ Ans: (b) In ’Preface to Shakespeare’ Dr. Samuel Johnson writes ’’That this is a practice contrary to the rules of criticism will be readily allowed; but there is always an appeal open from criticism to nature.” 16. An overall term for a class of novels written by Gabriel Garcia Marquez, Gunter Grass and John Fowles is” (a) Non-fiction novel (b) Epistolary novel (c) Non-veau roman (d) Magic realism Ans: (d) Magic realism is an approach to literature that weaves fantasy and myth into everyday life. It is also associated with realistic and figurative artworks paintings, drawing and sculpture - that suggest hidden meanings. In 1925, critic Franz Roh coined the term ’Magic Realism’ to describe the work of German artists who depicted routine subjects with eerie detachment. In 1955, literary critic Angel Flores adopted the term magical realism to describe the writings of Latin American authors who transformed ’the common and the everyday into the awesome and the unreal’. Jorge Luis Borges, Isabel Allende, Gunter Grass, Gabriel Garcia Marquez, John Fowles, Italo Calvino etc. are some famous authors of this tradition. 17. ’Kunstler roman’ is: (a) a form of German play (b) a form of play having the influence of Roman traditions (c) an artist’s novel (d) an important concept in deconstructive criticism Ans: (c) ’Kunstler roman’ is an important subtype of the Bildingsroman which represents the growth of a novelist or other artists from childhood into the stage of maturity that signalizes the recognition of the protagonist’s artistic desting and mastery of an artistic craft. Prominent examples of Kunstler roman are ’Sons and Lovers’ by D. H. Lawrence and ’A Portrait of the Artist as a Young Man’ by James Joyce. 18. T. S. Eliot’s ’The Sacred Wood’ embodies the principles of: (a) New Criticism (b) Structuralism (c) Surrealism (d) Postmodernism UPHESC Asst. Prof. Exam. 2014

Ans: (a) ’New Criticism’ is an approach to literature made popular in the 20th Century that evolved out of Formalist criticism. This term, coined by the publication of John Crowe Ramson’s ’The New Criticism’ in 1941, came to be applied to a theory and practice that was prominent in American literary criticism until late in the 1960. The movement derived in considerable part from elements in I. A. Richard’s Principal of Literary Criticism (1924) and ’Practical Criticism’ (1929) and from the critical essays of T. S. Eliot’s. ’The Sacred Wood’ is a collection of 20 essays by T. S. Eliot, first published in 1920. This essay embodies the principles of New Criticism. 19. What is the term for ’recognition’ in Aristotle’s poetics? (a) Hamartia (b) Mimesis (c) Anagnorisis _______(d) Eudaimonia _______ Ans: (c) The reversal frequently depends on a discovery in Aristotle’s term Anagnorisis. This is the recognition by the protagonist of something of great importance hither to unknown to him or to her. 20. Who made the following statement? ’’Poetry is not a turning loose of emotion, but an escape from emotion.” (a) John Dryden (b) Matthew Arnold (c) T. S. Eliot ________(d) I. A. Richards Ans: (c) T. S. Eliot was an American-English poet, playwright, literary critic and editor. He defines poetry as ’’Poetry is not a turning loose of emotion, but an escape from emotion.” 21. Tennessee Williams’ 'The Glass Menagerie 1 is: (a) a ’memory play’ (b) a romance (c) an expressionistic play (d) a surrealistic play _______________________ Ans: (a) Tennessee Williams was an American playwright and author of many stage classics. ’The Glass Menagerie’ (1945) is a four character memory play by Tennessee Williams. The play is set in 194143 in New York St. Louis and Las Angeles. 22. Who has written the play ’’Who’s Afraid of Virginia Woolf’? (a) Saul Bellow (b) Virginia Woolf (c) Edward Albee _____(d) Samuel Becken Ans: (c) Edward Albee is an American playwright known for famous works such as ’’Who is Afraid of Virginia Woolf’ (1962). It is a play by Albee which examines the breakdown of the marriage of a middle aged couple Martha and George. The play is in three acts. 23. When it comes, the landscape listens Shadows hold their breath When it goes, ’tis like the distance On the look of death

26

YCT

27. "Hamatreya" by Emerson is based on a passage These lines are from Emily Dickinson's poem: (a) ’There’s a Certain Slant of Light’ from: (b) ’Success is Counted Sweetest’ (a) The Bible (b) Bhagwad Geeta (c) ’Because I Could Not Stop for Death’ (c) Vishnu Purana (d) Homer’s Odyssey (d) 'I Felt a Funeral in My Brain’ ______________ Ans: (c) The poem ’’Hamatreya” was based on a Ans: (a) These lines ’’When It Comes, the landscape passage from the Vishnu Purana. Emerson copied the listens ................ look of death” are from Emily passage into his journal in - 1845. ’Hamatreya’ first Dickinson’s poem ’There’s a Certain Slant of Light’. appeared in print in ’Poems’ published by Chapman in Emily Dickinson was an American poet. London and by Munroe in Boston late in - 1846. 24. Which poem of Walt Whitman is a sequel to his 28. Mildred Douglas is a character in: war poems? (a) The Hairy Ape (a) ’’When Lilacs Last in the Courtyard Bloom’d” (b) ’’Out of the Cradle Endlessly Rocking” (b) The Iceman Cometh (c) ”A Passage to India” (c) Desire Under the Elms (d) ’’One, Self I Sing” _______________________ (d) Mourning Becomes Electra Ans: (a) ’’When Lilacs Last in the Courtyard Ans: (a) ’The Hairy Ape’ is a play by Eugene O’ Neill Bloomed” is a sequel to his war poems. The poem was that was first performed in - 1922 and published the published in 1865 under the section - titled Drumfollowing year. It is considered one of the prime Taps in ’Leaves of Grass’. It is an elegy mourning the achievements of Expressionism on stage. Mildred death of Abraham Lincoln. Douglas, the entitled daughter of the president of 25. Match the following playwright with their Nazareth Steel, a man who also serves as the chairman works: to the board of directors that controls the ocean liner List-I List-II upon which Yank, Paddy and Long work. (Playwrights) (Works) (I) Tennessee Williams (1) Buried Child 29. "Ah, but let her cover the mark as she will, the (2) Cat on a Hot Tin (II) Eugene O’Neil pang of it will be always in her heart." This is a Roof line from: (III) Sam Shepard (3) The Zoo Story (a) The Scarlet Letter (4) The Emperor Jones (IV) Edward Albee (b) The House of the Seven Gables Which is the correct combination according to (c) The Blithedale Romance the code? (d) A Life of Pierce Codes: Ans: (a) This line ’Ah, but let her ................. in her I II III IV heart’ quotes from ’The Scarlet Letter’ by Nathaniel 1 4 (a) 2 3 Hawthorne. This novel was published in - 1850. It is 4 2 1 (b) 3 4 1 (c) 2 3 considered a masterpiece of American literature and a 4 2 (d) 1 3 classic moral study. The novel is set in a village in Ans: (c) ’Cat on a Hot Tin Roof is a three act play Puritan New England. The main character is Hester written by Tennessee Williams; an adaptation of his Prynne. 1952 short story ’Three Players of Summer Game’. 30. Who among the following wrote a tribute poem The Emperor Jones is a 1920 play by American to Hawthorne, called "The Bells of Lynn"? dramatist Eugene O’ Neill. Buried Child is a play (a) Emerson (b) H. W. Longfellow written by Sam Shepard that was first presented in (c) Edgar Allan Poe (d) Harold Bloom 1978. ’The Zoo Story’ is a one act play by American Ans: (b) Henry Wadsworth Longfellow was one of playwright Edward Albee. th 26. The famous line "Success is counted sweetest/By the most popular American poets of the 19 Century those who ne'er succeed" is taken from a poem and a founder of the Atlantic. He wrote a tribute poem to Hawthorne called - ’The Bells of Lynn’ Heard at by: Nahant. (a) Emerson (b) Robert Frost Opening line of the poem is(c) Walt Whitman _____(d) Emily Dickinson O Curfew of the setting sun! O Bells of Lynn! Ans: (d) ’Success is Counted Sweetest’ is a lyric poem by Emily Dickinson written in 1859 and published 31. Dominique Lapierre wrote an appealing, anonymously in 1864 in the Brooklyn Daily Union. It account of the sordid squalor of an Indian city. was republished in the anthology A Masque of Poets Identify the city: (1878) as part of a series of books published without (a) Calcutta (b) Bombay writer’s name. (c) Udaipur (d) Delhi 27 YCT UPHESC Asst. Prof. Exam. 2014

Ans: (a) Dominique Lapierre is a French author Lapierre and Larry Callins wrote several books together. Two of Lapierre’s books - ’Is Paris Burning’ and ’City of Joy’. The City of Joy is about the unsung heroes of the Pilkhana Slum in Kolkata. 32. H. A. L. Fisher wrote about this writer: ” ........... this child of the green valley of the Ganges has by sheer piece of native genius earned for herself the right to be enrolled in the great fellowship of English poets.” Who is the poet? (a) SarojiniNaidu (b) KamalaDas (c) Torn Dutt (d) BinaAggarwal Ans: (c) Torn Dutt or Tarulatha Dutt was an Indian poet bom in the Bengal province in - 1856 to the well known Rambagan Dutt family. She may be reedited with being the first Indian English poet extensively use Indian myth. Torn mainly writes in English and French. H.A.L. Fisher writes, ’’this child of the green valley of the Ganges has by sheer ............... ” 33. The dominant theme of Ramanujan's poem "Conventions of Despair" is: (a) alienation (b) atheism (c) karma (d) celebration of life Ans: (a) Ramanujan’s poetry is a depiction of human sentiments, feelings and emotion in a direct way without any glass or sophistication. His poetry has technical excellence and bears the stamps of his individuality. ’Conventions of Despair’ is a protest poem. The dominant theme of this poem is alienation. Opening line of the poem instruct us to be itinerant and modem to fit in the current or contemporary world and the stay away from the bitter sense of anguish and dissatisfaction. 34. Which one of the following poems is not composed by Nissim Ezekiel? (a) ’Philosophy’ (b) ’Night of the Scorpion’ (c) ’The Mistress’ (d) ’The Visitor’____________________________ Ans: (c) Poem - ’The Mistress’ is not composed by Nissim Ezekiel. This poem was composed by John Wilmot in - 1691. Opening line of this poem isAn age in her embraces passed Would seem a winter’s day 35.

Ans: (c) ’The Guide’ is a novel written in English by the Indian author - R. K. Narayan. The novel was written in 1956 and published in 1958. The novel is set in Malgudi a fictional town in India. Moving between present and past and narrated alternately in third person and first person. ’The Guide’ tells the life story of Raju. 36. Who was the first recipient of the Sahitya Akademi Award for English Literature? (a) Mulk Raj Anand (b) R. K. Narayan (c) Raja Rao _________(d) Nayantara Sehgal Ans: (b) R. K. Narayan was the first recipient of the Sahitya Akademi Award for English Literature in 1960 for his novel ’The Guide’. 37. The book 'Listening to Grasshoppers' has been written by: (a) ArundhatiRoy (b) Jhumpa Lahiri (c) Anita Desai _______(d) Salman Rushdie Ans: (a) ’Listening to Grasshopper : Field Notes on Democracy’ (2009) is a collection of essays written by Booker Prize winner - Arundhati Roy, an Indian author best known for his novel - ’’The God of Small Things”. 38. Who wrote the following lines? "Philosophy, Poverty and Poetry, three Companions shared my basement room." (a) Kebi N. Daruwalla (b) A. K. Ramanujan (c) Nissim Ezekiel (d) Dom Moraes Ans: (c) The following lines - ’’Philosophy, Poverty and Poetry, three Companions shared my basement room” appear in one of the biographical poem of Nissim Ezekiel’s - ’’Background Casually”. 39. Who wrote "The Great Indian Way : A Life of Mahatma Gandhi"! (a) Mulk Raj Anand (b) Khushwant Singh (c) Raja Rao (d) Manohar Malgonkar _____________________ Ans: (c) ’The Great Indian Way : A Life of Mahatma Gandhi’ is written by Raja Rao, and Indian writer of English language, novels and short story. He got Sahitya Akademi Award in 1964. 40. Who advised Sarojini Naidu 'to be a genuine Indian Poet of the Deccan, not a clever, machine-made imitator of English Classics'? (a) TaraAliBaig (b) Mahatma Gandhi The following novels, except one, portray the (c) Edmund Gosse_____(d) Tagore ___________ condition of Indians settled abroad. Mark the Ans: (c) Sarojini Naidu was a poet, a politician and an one which does not activist, called the Nightingale of India. It is said that (a) The Wife Edmund Gosse, and English poet guided Naidu into (b) The Serpent and the Rope ”a genuine Indian poet of Deccan, not a clever, (c) The Guide machine made imitator of English Classics”. (d) Bye-bye Blackbird

UPHESC Asst. Prof. Exam. 2014

28

YCT

41. Match the following List-I (I) Roland Barthes (II) Jonathan Culler

List-II (1) The Savage Mind (2) The Fight for English (3) Structuralist (4) Mythologies

Ans: (b) ’An Outline of English Phonetics’ is written by Daniel Jones. This work has been selected by scholars as being culturally important, and is the part of the knowledge base of civilization as we know it. 46. Who has applied the behaviourist approach and explained language as a pattern of stimulus and response? (a) Bloomfield (b) Halliday (c) Sassure ___________(d) Derrida ___________ Ans: (a) Leonard Bloomfield, a well-known linguist and a behaviorist, was the one who connects linguistics and behaviourism. Bloomfield suggested that language behaviour is based on stimulusresponse. Bloomfield is also credited for expounding American structuralism. 47. Linguistics is: (a) an empirical science (b) a social science (c) both an empirical and a social science (d) inductive science ________________________ Ans: (c) Linguistic is the scientific study of language. It is both an empirical and a social science. 48. Who introduced the semiotic notion of Intertextuality? (a) Roland Harthes (b) Northrop Frye (c) Ferdinand de Saussure (d) Julia Kristeva __________________________ Ans: (d) The development of the semiotic idea of intertextuality is largely attributed to the French semiotician Julia Kristeva who in her pioneering of intertextuality, brought on the rise of paststructuralism. 49. Which of the following is not by Michael Halliday? (a) English as a Global Language (b) An Introduction to Functional Grammar (c) Learning How to Mean (d) Explorations in the Function of Language Ans: (a) ’English as a Global Language’ is a book by David Crystal, published in 1997. Rest other books belong to Michael Halliday. 50. Who is the author of Syntactic Structures? (a) Noam Chomsky (b) David Crystal (c) Roman Jakobson (d) Hob Hodge ________ Ans: (a) Syntactic Structures is an influential work in linguistics by American linguist Noam Chomsky originally published in 1957. It is an elaboration of his teacher’s Zelling Harris’s model of transformational generative grammar. 51. ’The Rambler’ and ’The Idler’ are periodicals started by: (a) Addison (b) Goldsmith (c) Dr. Johnson (d) Steele

(III)David Crystal (IV) Claude LeviStrauss Codes: I II III IV (a) 4 1 2 3 2 (b) 1 4 3 (c) 4 3 2 1 (d) 2 4 1 3 Ans: (c) ’Mythologies’ is a 1957 book by Roland Barthes. Stricturalist Poetics : Structuralism, linguistics and the study of literature is a 1975 book of critical literary theory by the critic - Jonathan Culler. The Fight for English is a book by David Crystal and The Savage Mind is by Claude Levi-Strauss. 42. ’’John found the boy studying in the library.” This sentence is an example of (a) ambiguity (b) passive voice (c) ungrammatical sentence (d) circumlocution Ans: (d) ’John found the boy studying in the library’ is an example of Circumlocution. Circumlocution is a phrase that circles around a specific idea with multiple words rather than directly evoking it with fewer and apter words. 43. A sound produced with the Velic opening and oral closure is a/an: (a) oral sound (b) nasal consonant (c) nasalized vowel (d) voiced sound Ans: (b) A sound produced with the Velic opening and oral closure is a nasal consonant. 44. The Founding father of semiotics is: (a) Ferdinand de Saussure (b) Chomsky (c) Derrida ____________(d) Bakhtin_________ Ans: (a) One of the founding fathers of semiotics was the Swiss linguist - Ferdinand de Saussure. He developed a science he called semiology but that term has been replaced in recent years by Peirce’s term ’’Semiotics”. Semiotics is the theory and study of signs and symbols, especially as elements of language or other systems of communication. Examples - traffic signs, emojis etc. 45. Who is the author of the book ”Outline of English Phonetics" 2 (a) Gimson (b) Daniel Jones (c) Sir William Jone (d) Chomsky 29 UPHESC Asst. Prof. Exam. 2014

YCT

Ans: (c) ’The Idler’ was a series of 103 essays, all but 56. To whom is T. S. Eliot’s ’’The Waste Land” dedicated? twelve of them by Samuel Johnson published in the (a) W.H. Auden London weekly The Universal Chronicle between (b) His Wife 1759 and 1760. The Rambler was a periodical by (c) Ezra Pound Samuel Johnson. (d) Victims of World War I 52. To whom are the following lines attributed ”1 began to write poetry with the ignoble aim of Ans: (c) ’The Waste Land’ is Eliot’s epic poem of 434 lines in free verse. It is published in 1922. It is wooing a man.” dedicated to Ezra Pound. The dedication is : ’’Far Ezra (a) Kamala Markandya (b) Sarojini Naidu Pound : il migliar fabbro”. It means - ’’the better (c) ToruDutt (d) Kamala Das Craftsman”. Ans: (d) The following lines are attributed to Kamala 57. The term ’curtal sonnet’ Das as ”1 began to write poetry with the ignoble aim (a) John Milton (b) William Blake of wooing a man”. Kamala Surayya, popularly known (c) G. M. Hopkins _____(d) Matthew Arnold by her one time pen name Madhavikutty and married Ans: (c) The Curtal Sonnet is a form invented by name - Kamala Das was an Indian poet in English. Gerard Manley Hopkins, and used in three of his 53. Shakespeare depended primarily on two sources poems. It is an eleven lines sonnet but rather than the for his legendary and historical plays. Which is first eleven lines of a standard sonnet, it consist of the most important one? precisely 3/4 of the structure of a Petrachan sonnet (a) King Arthur’s Legend shrunk proportionally. (b) Holinshed’s Chronicles 58. In what publication was the novel Great (c) Pluutarch’s Lives Expectations originally serialized? (d) Legends of Charlemagne _________________ (a) Simple Wisdom Ans: (b) Plutarch’s ’Lives’ and Holinshed’s (b) All the Year Round ’Chronicles’ are two major sources of William (c) Home and Away (d) The English Almanac ____________________ Shakespeare’s legendary and historical play. Holinshed’s ’Chronicles’ is the most important. It is Ans: (b) ’Great Expectations’ is Dicken’s novel. This known as - Holinshed’s Chronicles of England, novel originally serialized weekly serial in ’All the Year Round’ December 1, 1860 to August 3, 1861. Scotland and Ireland. The Chronicles have This novel can be called - bildungsroman. Hero of the traditionally been a source of interest to many because novel is - Pip whose real name is Philip Pirrip. of their extensive links to Shakespeare’s history plays as well as King Lear, Macbeth and Cymbeline. 59. Who has written Iliad*! (a) Virgil (b) Homer 54. Which of the following novels is called a ’novel (c) Plato _____________(d) Aristotle __________ without a hero’? Ans: (b) The ’Iliad’ is an Ancient Greek epic poem by (a) Pickwick Papers (b) Northanger Abbey Homer that was first published in 762 BC. (c) Mill on the Floss (d) Vanity Fair Ans: (d) ’Vanity Fair’ novel of early 19th Century 60. What is the name of Tess’ illegitimate child in Hardy’s Tess of the D'Urbervilles? English society by William Makepeace Thackeray, (a) Abraham (b) Sorrow published in 1848. The book is a densely populated (c) Pity _____________ (d) Grace ____________ multilayered panorama of manners and human Ans: (b) Tess of the D’Urbervilles : A pure women frailties, sub titled - A Novel Without a Hero, Vanity Faithfully presented’ is a novel by Thomas Hardy. Fair metaphorically represents the human condition. This novel was originally published in 1891. The 55. ’True wit is nature to advantage dress’d name of Tess’ illegitimate child is Sorrow. It is Tess’s What oft was thought but ne’er so well baby from Alec. He only lives a few week, and Tess expressed. has to baptized and bury him herself. These lines have been written by: 61. Which of the following is not pseudonym of the (a) Shakespeare (b) Dryden author? (c) Pope (d) Dr. Johnson (a) Mark Twain (b) George Eliot Ans: (c) Alexander Pope in his poem ’An Essay on (c) Saki (d) Ruskin Criticism’ defined ’true wit’ as a nature of advantage Ans: (d) ’Ruskin’ is not pseudonym of the author dress’d /What off was thought, but ne’er so well whether other option as - Mark Twain - S. L. expressed. In other words the best poet makes Clemens George Eliot - Mary Ann Evans, Saki memorable lines out of what everybody already know. Hector Hugh Munro are the pseudonym. UPHESC Asst. Prof. Exam. 2014

30

YCT

62.

The Symbolist Movement in Literature written Ans: (c) The phrase - Busy as a Bee was first used by by: Geoffrey Chaucer in 1392, in his classic ’’The (a) W. B. Yeats (b) Ezra Pound Canterbury Tales”. The phrase - 'Cool as a Cucumber’ (c) Arthur Symons (d) W. H. Auden was first recorded in a poem by the British poet John Ans: (c) The Symbolist Movement in Literature Gay ’New Song on New Similies in 1732. ’Cool as a written by Arthur Symons, first published in 1899 and Cucumber could see the rest of woman kind'. John with additional material in 1919. Its first two editions Lyly used the phrase - 'Two Peas in a Pod’ in Euphues were vital influences on W. B. Yeats and T. S. Eliot. and his England, 1580. Shakespeare used the phrase 63. To which category do the two works of William A Sea Change in his dramatic romance - ’The Shakespeare Venus and Adonis and The Rape of Tempest' as - a sea change is a change brought about Lucrece belong? by sea : Full fathom five thy father lies ........./ (a) Narrative Poems (b) Comedies Nothing of him that doth fade/But doth suffer a sea (c) Historical Plays (d) Tragedies change. Ans: (a) William Shakespeare’s ’Venus and Adonis’ 67. Hamlet, lying wounded, says to his friend, (1593) is a narrative poem. It is dedicated to Henry ’Horatio, I am dead’. Wriothesley and ’The Rape of Lucrece’ (1594) is a This is an example of: narrative poem about Lucretia. It is dedicated to the (a) protasis (b) anacrusis Right honorable Henry Wriothesley, Earl of (c) prolepsis (d) pun Southampton and Baron of Tichfield. 64. Simon embodies human goodness and is a Ans: (c) ’Prolepsis’ is a figures of speech in which the speaker raises an objection to their own argument and character in: then immediately answers it. It is the representation of (a) Thackeray’s Vanity Fair a thing as existing before it actually does or did so, as (b) Golding’s Lord of the Flies in "Horatio, I am dead." (c) Hardy’s Tess (d) Jane Austen’s Emma 68. Which one of the following pairs is not correctly Ans: (b) ’Lord of the Flies’ is a 1954 novel by Nobel matched? Prize winning British author - William Golding. The (a) Derrida - Deconstruction novel was published in 17 September 1954. Simon (b) Elaine Showalter - Gynocriticism Jack, Piggy, Sam, Roger, Eric are the major characters (c) Edward Said - Orientalism in the novel. Simon embodies a kind of innate, (d) Structuralism - Matthew Arnold spiritual human goodness that is deeply connected Ans: (d) Structuralism is an intellectual movement with nature. which began in France in - 1950s and is first seen in 65. Which one of the following pairs is not correctly the work of the anthropologist Claude Levi Strauss matched? and the literary critic - Ronald Barthes. (a) Matthew Arnold Dover Beach 69. The critical essay ’’Under Western Eyes : (b) Ted Hughes Hawk Roosting Feminist Scholarship and Colonial Discourses” (c) W. H. Auden Among School is written by: Children (a) Alice Walker (d) Robert Browning Rabbi Ben Ezra (b) Vandana Shiva Ans: (c) ’Among School Children' is a poem by (c) Chandra Talpade Mohanty William Butler Yeats. Yeats published this poem in (d) Carol Gilligan his famous 1928 Collections of poems, The Tower. Ans: (c) 'Under Western Eyes : Feminist Scholarship 66. Match of the following lists: and Colonial Discourses' (1984) is an academic essay List-I List-II by Chandra Talpade Mohanty, an Indian-American (The Phrase) (Author) (I) Busy as a Bee (1) John Gay feminist scholar. In this essays Mohanty argues that (II) Cool as a Cucumber (2) William Western feminist scholarship has reduced all women Shakespeare of the third world into a single, collective other. (III) Two Peas in a Pod (3) Geoffrey Chaucer 70. Tennyson’s Ulysses has been primarily (IV) A Sea Change (4) John Lyly influenced by: The correct combination is: (a) Milton (b) Alexander Pope I II III IV (c) John Dryden (d) Dante (a) 2 4 3 1 Ans: (d) 'Ulysses’ is Tennyson’s dramatic monologue (b) 3 4 2 1 in blank verse. It is written in 1833 and published in (c) 3 14 2 1842. It is based on Dante's Inferno [Canto XXVI]. (d) 2 3 14 31 YCT UPHESC Asst. Prof. Exam. 2014

UPPSC Govt. Degree College Asst. Prof. Exam. 2021

ENGLISH Solved Paper 1.

Which of the following is NOT correct? (a) The Terza Rima is a stanza of four lines (b) The Chaucerian stanza is of seven lines (c) The Spenserian stanza is of nine lines (d) The Ottava Rima is a stanza of eight lines Ans. (a) : All the given options are correct except option (a) the ’Terza Rima’ is a stanza of four lines. The term ’Terza Rima’ is coined by Italian poet Dante Alighieri for his narrative poem ‘The Divine Comedy’. It is a rhyming verse form consists of three line stanza with an interlocking three-line rhyme scheme. The last word of the second line in one tercet provides the rhyme for the first and third lines in the tercet that follows (aba, bcb, cdc). Terza Rima has been used by Geoffrey Chaucer in his short poem ’’Complaint to His Lady”. P.B. Shelley has used ’’Terza Rima in his famous ’Ode to the West Wind.” _______________________________ 2. Which of the novels given below does NOT fall under the category of ’Gothic Novels’? (a) The Pickwick Papers (b) The Castle of Otranto (c) The Moor _____(d) The Old English Baron ___________________ Ans. (a) : The given novel ’The Pickwick Papers’ does not fall under the category of ’Gothic Novels’ This was Charles Dickens’ first novel, known as ’’The Posthumous Papers of the Pickwick Club,” first published serially from 1836 to 1837 under the pseudonym ’Boz’ and in book form in 1837. The other given novels belong to ’Gothic novel’ written by _ _ The Castle of Otranto - Horace Walpole The Moor - Laurie R. King The Old English Baron - Clara Reeve Hence, the correct answer is option (a). ______________ 3. What is the name of the father of Estella in the novel Great Expectations? (a) Magwitch (b) Compeyson (c) Jaggers _ _ _ _ _(d) Uncle Pumblechook _____________________ Ans. (a) : Abel Magwitch is the name of the father of Estella in the novel, ’Great Expectations’ written by Charles Dickens. Estella Havisham, in simple known as Estella, a significant character like the protagonist Philip Pirrip, nicknamed ’Pip’. Abel Magwitch is a fictional character, an escaped convict who plays a major role in the growth and development of Philip Pirrip in the novel ’’Great Expectations” (1861). Hence, the correct answer is option (a). UPPSC GDC (Asst. Prof.) Exam. 2021

[Exam. Date 15 March 2022]

4.

Consider the following events and arrange these in chronological order: I. The publication of Lyrical Ballads II. Reform Bill III. Restoration of Charles II IV. French Revolution. Select correct answer from the codes given below. (a) III, II, I, IV (b) II, I, III, IV (c) III, IV, I, II ________(d) III, I, II, IV Ans. (c) : The correct chronological order with their events is option (c) III, IV, I, II. III. Restoration of Charles II 1660 IV. French Revolution 1789 I. The Publication of Lyrical Ballads1798 II. Reform Bill 1832 Restoration of Charles II took place in 1660 when he returned from exile in continental Europe. Lyrical Ballads (a collection of 23 poems) 19 poems composed by William Wordsworth and 4 poems by S.T. Coleridge, published in 1798. Hence, the correct answer is option (c). ______________ 5. In scene I of Bernard Shaw’s Saint Joan, Sir Robert de Baudricourt is blustering about the lack of what? (a) Eggs (b) Milk ______(c) Money ____________(d) Piety ____________ Ans. (a) : In scene I of Bernard Shaw’s ‘Saint Joan’ Sir Robert de Baudricourt is blustering about the lack of eggs. Saint Joan is a chronicle play in six scenes and an epilogue by G. B. Shaw performed in 1923 and published in 1924. It was inspired by the canonization of Joan of Arc in 1920 nearly five centuries after her death in 1431. Joan, a simple peasant girl, claims to experience visions of Saint Margaret and Saint Catherine. Michael Holroyd has characterised the play as ”a tragedy without villains” and also as G. B. Shaw’s ’’only tragedy”. Hence, the correct answer is option (a). ______________ 6. Which of the following critics has initiated and emphasized the need of ’practical criticism’ as the fundamental requirement of a critic? (a) I. A. Richards (b) Rene Wellek (c) William Welsh (d) F. R. Leavis Ans. (d) : F. R. Leavis has initiated and emphasized the need of ’Practical criticism’ as the fundamental requirement of a critic. ’Practical criticism’ is a form of literary analysis which focuses exclusively on the text, ignoring such extraneous factors as authorial intention and historical context. The term originates with an experiment performed on Cambridge literature students by F. R. Leavis. Hence, the correct answer is option (d).

32

YCT

7.

"Do as you will, but I’ll be gone" who said this and to whom in the play Volpone? (a) Volpone to Mosca (b) Mr. Would be to Mrs. Would be (c) Corvino to Mosca _ _ _ _ _(d) Corvino Celia ___________________________ Ans. (c) : The given statement- ”Do as you will, but I will be gone”- is spoken by Corvino to Mosca in the play ’ Volpone (The Fox). It is a comedy in blank verse by Ben Jonson. It was first acted in 1605 and printed in 1607. It is often considered Jonson’s masterpiece. It was drawn on the elements of city comedy and beast fable. It is ranked among the finest Jacobean Era comedies. Hence, the correct answer is option (c). ______________ 8. The liberty which poets take is (a) Poetic justice (b) Poetic license ______(c) Affective fallacy (d) Rhyming scheme Ans. (b) : The liberty which poets take is ’Poetic License’. The word ’’Poetic’ derives from the Latin word ’Poeta’ which means ’Poet’ or ’maker’. The word ’License’ comes from the Latin word-’Licentia’ which means ”to be permitted.” John Dryden defines ’Poetic license’ as the liberty which poet have assumed to themselves in all ages of speaking thing in verse which are beyond the severity of prose. In a broader sense ’’Poetic License” is applied not only to diction but to all the ways in which poets and other literary authors are held to be free to violate the conventional rules of language for special effects. Hence, correct answer is option (b). 9. In which of the following poems occurs the phrase "the still sad music of humanity"? (a) Ode: Intimations of Immortality (b) Michael: A Pastoral Poem (c) The Solitary Reaper _____(d) Tintem Abbey __________________________ Ans. (d) : The above phrase ’The still sad music of humanity’ has been derived from the famous dramatic lyric ” Tintem Abbey” composed by nature poet William Wordsworth. The full title of the poem is ’’Lines Composed a Few Miles above Tintem Abbey, on Revisiting the Banks of the Wye during a Tour, July 13, 1798.” The poem is written in ’Decasyllabic blank verse’ and comprises ’verse paragraphs’. It opens with the speakers declaration that ’’five years have past; five summers, with the length of five long winters ! and again I here....”. Hence, the correct answer is option (d). ______________ 10. With reference to Francis Bacon, which of the following statements is/are correct? 1. Knight in 1603 2. Solicitor General in 1607 3. Attorney General in 1 6 1 3 4. Viscount - St. Albans in 1625 Select the correct answer using the codes given below: (a) 1 only (b) 2 only (c) 1 and 3 only (d) 1 and 4 only UPPSC GDC (Asst. Prof.) Exam. 2021

Ans. (c) : With reference to Francis Bacon, the option (c) (1 and 3 only) is correct. Francis Bacon was bom on 22 January, 1561 and died on 9 April, 1626 (aged 65). He is known as the father of English essay and the father of Empiricism. He belongs to Elizabethan and Jacobean age. Alexander Pope in his ’Essay on Man’ called Bacon. ’’The wisest, brightest and meanest of mankind”. Bacon was knighted in 1603 and become Attorney General in 1613. Hence, the correct answer is option (c). ______________ 11. The title ’The Second Coming’ originates from which Book of ’The Bible’? (a) Genesis (b) Exodus ______(c) Revelation _________(d) None of these _____ Ans. (c) : The title ’ The Second Coming’ originates from Revelation Book of 'The Bible'. W.B. Yeats was one of the Irish poets who composed a famous poem ‘The Second Coming’ in 1919, soon after the end of the First World War, first printed in ‘The Dial’ in November 1920 and afterwards included in his 1921 collection of verses Michael Robartes and the Dancer. The poem ’Second Coming’ refers to the idea that Jesus will return to earth towards the end of time to bring justice and order. Hence, the correct answer is option (c). 12. Who of the following was the daughter of Sir Leslie Stephen? (a) Virginia Woolf (b) Emily Bronte ______(c) Jane Austen ________(d) George Eliot ______ Ans. (a) : Virginia Woolf (Adeline Virginia Stephen) was the daughter of Sir Leslie Stephen. She is recognised as one of the most innovative writers of the 20th century. After her father's death, she moved to Bloomsbury. ’Bloomsbury Group' is the name applied to an informal association of writers, artists, intellectuals and philosophers in the first half of the 20 th century, many of them lived in Bloomsbury, including Virginia Woolf, John Maynard Keynes, E.M. Forster and Lytton Strachey. Hence, the correct answer is options (a). 13. Who said, "A poetry of revolt against moral ideas is poetry of revolt against life’’.? (a) T.S Eliot (b) Arnold ______(c) F.R. Leavis ________(d) Coleridge ________ Ans. (b) : The given statement, "A poetry of revolt against moral ideas is poetry of revolt against life,” is said by Matthew Arnold. As a critic, Matthew Arnold is essentially a moralist and has very definite ideas about what should and should not be. He is known as the first modem critic and could be called "the critic's critic”. The purpose of literary criticism, in his views, was to know the best that is known and thought in the world, and by its turn making this known, to create a current of true and fresh ideas. Hence, the correct answer is option (b). ______________ 14. Which of the following is NOT true about Spenserian Stanza? (a) A form invented by Spenser (b) First eight lines are iambic pentameter

33

YCT

(c) First eight lines are iambic octameter (d) last line is iambic hexameter Ans. (c) : All the given options are correct except option (c). First eight lines are lambic octameter, is not correct in relation to Spenserian stanza. Spenserian stanza is a poetic device coined by Edmund Spenser for his epic poem ’The Faerie Queene’. It is a stanza of nine lines poem - eight lines are in lambic pentameter followed by a single Alexandrine line in lambic Hexameter. An Alexandrine line is a 12 syllable lambic lines. The rhyming is ababbcbcc. Hence, the correct answer is option (c). 15. In The Tempest, what are Miranda and Ferdinand doing in the final scene? (a) Playing cards (b) Carrying wood (c) Playing chess (d) Playing tag Ans. (c) : In ’The Tempest’ Miranda and Ferdinand were playing chess in the final scene. The Tempest is the last play by English dramatist William Shakespeare. There are five Acts in the play, first written and performed about 1611 and published in the First Folio of 1623. It is classified as a comedy in the First Folio. 16. Who said. "If Pope be not a poet, where is Poetry to be found"? (a) Dr. Johnson (b) Thomas Gray _____(c) Coleridge _________(d) Wordsworth ______ Ans. (a) : Dr. Samuel Johnson is a famous literary critic, wrote in ’’The Life of Pope”, ‘If Pope be not a poet, where is poetry to be found?’ Many felt the same way, believing that none could equal Pope in his mastery of the heroic couplet. The Oxford Dictionary of National Biography calls Samuel Johnson ’’arguably the most distinguished man of letters in English history.” Hence, the correct answer is option (a). 17. "Disappointment in love" deals with the love affair of (a) Honeycomb (b) Sir Roger _____(c) Sir Andrews _______(d) Addison _________ Ans. (b) : ’’Disappointment in love” deals with the love affairs of Sir Roger. ’Sir Roger at Home’ is a famous essay written by Joseph Addison. It was published on July 2, 1711 and its number is 106. In this essay Joseph Addison says that Sir Roger was a very hospitable man. Sir Roger was the chief character in the Coverlev Paper. Sir Roger de Coverley is the first member of the Spectator Club. He is a gentle man of 56 years of Worcestershire, a non-metropolitan country of England with very strong family history. Hence, the correct answer is option (b). 18. In John Osborne’s Look Back in Anger (1956), England is depicted as (a) A flourishing nation (b) A prosperous nation (c) An emasculated nation (d) An affluent nation UPPSC GDC (Asst. Prof.) Exam. 2021

Ans. (c) : In John Osborne’s Look Back in Anger (1956) England is depicted as an emasculated nation. ’’Look Back in Anger” is a realist play in three acts performed in 1956, published in 1957. Alienation is an important theme in John Osborne’s Look Back in Anger. Jimmy Porter, the main character of the play, typifies the overeducated, underemployed worker. Hence, the correct answer is option (c).______________ 19. Who among the following critics considers criticism to be "dissemination of ideas, an unprejudiced and impartial effort to study and spread the best that is known and thought in the world"? (a) Matthew Arnold (b) I. A. Richards (c) F. R. Leavis _______(d) T. S. Eliot _______ Ans. (a) : Matthew Arnold was a British poet and critic during the Victorian era. He considers criticism to be, ’’dissemination of ideas, an unprejudiced and impartial effort to study and spread the best that is known and thought in the world”. In 1847, Arnold became private secretary to Lord Lansdowne, Lord president of the council. In 1849, he published his first book of poetry, ’The Strayed Reveller,’ and in 1850 “Memorial Verses.” In 1852, he published his second volume of poems, Empedocles on Etna and Other Poems. Hence, the correct answer is option (a). 20. Who is Shock in Alexander Pope’s "The Rape of the Lock"? (a) Belinda’s Horse (b) The baron’s Horse _____(c) The poet’s Mouse (d) Belinda's Lapdog Ans. (d) : Shock is the name of Belinda’s Lapdog in Alexander Pope’s ’’The Rape of the Lock”. It is a Mockheroic narrative poem, commonly cited examples of High Burlesque, first published anonymously in Lintot’s Miscellaneous Poems and Translations (May 1712) in two cantos (334 lines); a revised edition written by ’Pope’ followed in March 1714 as five cantos (794 lines). At the suggestion of John Caryl, Pope has composed this poem. Belinda is the heroine whose lock has been cut or raped by Lord Peter. Hence, the correct answer is option (d). 21. Which of the following writers remarked after reading The Rape of the Lock that he did not know, "Whether to laugh or weep"? (a) Wilson Knight (b) Hazlitt _____(c) Ian Jack __________(d) Cleanth Brooks Ans. (b) : William Hazlitt was an English essayist and literary critic, remarked after reading, The Rape of The Lock that he did not know, ” Whether to laugh or weep”. The Rape of the Lock is a famous mock heroic poem in V cantos. William Hazlitt is a social and literary critic, best known for his Informal Essays, he published “An Essay of the Principles of Human Action” in 1805 and “Free Thoughts in Public Affairs” in 1806. Hence, the correct answer is option (b).

34

YCT

22.

Which of the following expressions is by Tennyson? (a) The best lack all conviction, while the worst are full of passionate intensity. (b) And may there be no sadness of farewell, when I embark (c) I was ever a fighter, so-one fight more, The best and the last! (d) Oh Sir, she smiled, no doubt, Whenever I passed her, but who passed without much the same smile Ans. (b) : Alfred Lord Tennyson was a Victorian poet, he became the Poet Laureate in the reign of Queen Victoria. He is famous for Victorian compromise. All the given expressions do not belong to Tennyson except option (b) "And may there be no sadness of farewell, When I embark...” This famous line has been written by him in his famous poem “Crossing the Bar”. The poem “Crossing the Bar” is an elegy, in it, the narrator uses an extended -metaphor to compare death with crossing the ’sandbar’ between the river of life, with its outgoing ’flood’, and the ocean that lies beyond death, the “boundless deep,” to which we return. Hence, the correct answer is option (b). 23. Who out of the following writers thought Tom Jones ’’too vicious to be read by women"? (a) S. T. Coleridge (b) Samuel Johnson (c) T. S. Eliot (d) James Boswell Ans. (b) : Dr. Samuel Johnson thought Tom Jones ’’too vicious to be read by women”. Tom Jones is a picaresque novel, by Henry Fielding. Its second title is ’Foundling’ first published on 28th February 1749 in London. It is generally regarded as Fielding’s greatest book and as an influential English novel. Notable works of Henry Fielding are l.Shamela 1741 2. Joseph Andrews 1742 3. Amelia 1751 4. Tom Jones 1749 5. The Life and Death of Jonathan 1743 Wild the Great Hence, the correct answer is option (b). 24. Whose speech is this? ’’Present fears/Are less than horrible imaginings” (a) Macbeth (b) Banquo (c) Macduff (d) Duncan Ans. (a) : The given famous speech ” Present fears/Are less than horrible imaginings” is spoken by Macbeth in the play ’Macbeth’ in Act 1 scene III. The play Macbeth is a tragedy written by William Shakespeare in about 1606, first published in the First Folio of 1623 by Heminges and Condell. The source of the play is Holinshed’s Chronicles (1587). Hero of the play is Macbeth and heroine of the play is Lady Macbeth. Hence, the correct answer is option (a). 25. Who amongst the following writers does NOT belong to the group known as ’’University Wits”? (a) Henry Medwall (b) Thomas Nashe (c) Robert Greene (d) JohnLyly UPPSC GDC (Asst. Prof.) Exam. 2021

Ans. (a) : Henry Medwall is remembered for his ’Fulgens and Lucrece’, the first known ’secular’ play in English. He does not belong to the group known as ’’University Wits”. The University Wits is a phrase used to name a group of English playwrights and pamphleteers who were educated at the University of Oxford and Cambridge. Prominent members of this group were Christopher Marlowe, Robert Greene. Thomas Nashe, John Lviv, Thomas Lodge, George Peele and Thomas Kyd. Hence, the correct answer of option (a). 26. Who in Richard II kills Richard when he hears of Henry’s coronation? (a) Bolingbroke (b) Piers Exton (c) Anmerie (d) Thomas Mowbray Ans. (b) : ’Piers Exton’ kills Richard when he hears of Henry’s coronation in the play ’’Richard II”. It is a history play written around 1595, based on the life of King Richard II of England and chronicles his downfall and the machination of his nobles. William Shakespeare has written 37 plays and 154 sonnets in his entire career. The play Richard II was performed at the Globe Theatre on 12 June, 1631. The Historical plays of Shakespeare cover the period of English history of England over a period of about 350 years. Hence, the correct answer is option (b). 27. Who says, ’’the progress of an artist is a continual self-sacrifice, a continual extinction of personality.”? (a) T.S. Eliot (b) I. A. Richards (c) F.R. Leavis (d) Matthew Arnold Ans. (a) : ’’Tradition and the Individual” is the most famous works of T. S. Eliot in which he says, ’’the progress of an artist is a continual self-sacrifice, a continual extinction of personality”. Tradition and the Individual Talent (1919) is an essay first published in the Egoist (1919) and later in Eliot’s first book of criticism, The Sacred Wood (1920). The essay is also available in Eliot’s “Selected Prose” and ’’Selected Essays”. Hence, the correct answer is option (a). 28. A foot in prosody is a basic unit of (a) rhyme (b) length (c) rhythmic measurement (d) None of these Ans. (c) : A foot in prosody is a basic unit of rhythmic measurement. A ‘foot’ usually contains one stressed syllable and at least one unstressed syllable. The four standard feet distinguished in English are 1. lambic An unstressed syllable followed by a stressed syllable. 2. Anapestic Two unstressed syllables followed by a stressed syllable. 3. Trochaic A stressed syllable followed by an unstressed syllable. 4. Dactylic A stressed syllable followed by two unstressed syllables. By. M. H. Abrams Hence, the correct answer is option (c).

35

YCT

29.

Which of the following is NOT correct? (a) The still sad music of humanity’ -Wordsworth (b) ’Charms strike the sight, but merit wins the soul’- Coleridge (c) ’Our two souls therefore which are one’-Donne (d) ’Our little systems have their day’-Tennyson Ans. (b) : All the given option are correct except option (b). ’’Charms strike the sight, but merit wins the soul”, Coleridge. This line has been taken from the mock-epic poem, ’’The Rape of the Lock” by Alexander Pope, first published anonymously in 1712, consisted of two contos; a revised edition published in 1714, was expanded in five contos; and finally in 1717 with the addition of Clarissa’s speech on good humour. The poem was much translated and contributed to the growing popularity of mock-heroic in Europe. Hence, the correct answer is option (b) 30. Some imagists are influenced by Japanese haiku which has exactly (a) 17 syllables (b) 19 syllables (c) 21 syllables (d) 23 syllables Ans. (a) : Haiku is a Japanese poem of seventeen syllables in three lines of five, seven and five, traditionally evoking images of the natural world. The Haiku greatly influenced Ezra Pound and other Imagists, who set out to reproduce both the brevity and the precision of the image in the Japanese original. Ezra Pound’s ”In a Station of the Metro” is well known instance of the haiku in the loosened English form. Hence, the correct answer is option (a) _______________ 31. Robert Browning’s ‘Rabbi Ben Ezra’ is a defence of (a) Youth against old age (b) Old age against youth (c) Power against knowledge (d) Knowledge against Ans. (b) : Robert Browning’s poem Rabbi Ben Erza is a Dramatic Monologue, a defence of ’’Old age against youth.” It is about Abraham ibn Erza one of the greatest poets and scholars of the 12the century. He wrote on grammar astronomy, the astrolabe etc. The poem was published in Browning’s Dramatis Personae in 1864. The poem’s final metaphor describe life as pot that is fashioned by the master’s hand and argues that the value of a person should be measured not by the work done but by the character that has been molded. Hence, the correct answer is option (b). ______________ 32. What is the sub-title of the novel Tom Jones? (a) The Infant (b) The Orphan ______(c) The Foundling ______(d) None of above Ans. (c) : The Foundling is the sub-title of the novel ‘Tom Jones’ by an English playwright and novelist ‘Henry Fielding’. It is a Picaresque novel, published on 28 Feb, 1749 in London. This novel consists of 18 books, open with the narrator stating that the purpose of the novel will be to explore ‘human nature’. Hence, the correct answer is option (c). UPPSC GDC (Asst. Prof.) Exam. 2021

33.

What is the name of Miss Havisham’s Manor in Great Expectations? (a) Satis House (b) Lockmont ______(c) Larchmout _________(d) Satyr House ______ Ans. (a) : Satis House is the name of Miss Havisham’s Manor in ‘Great Expectations’. ‘Great Expectations’ is Dicken’s novel (Weekly Serial in All the Year Round, December 1, 1860 to August 3, 1861). This novel can be called Bildungsroman, deals with the formative year of Philip pirrip (Pip) who is the narrator. The setting place of the novel is in Kent and London (mid-nineteenth century). It is a story of Pip, an orphan who lives with his sister Mrs. Joe Gargery and works with his brother-in-law Mr. Joe Gargery, Miss Havisham’s attempt to stop time is shown as the symbol of the stopped clocks at Satis House. Hence, the correct answer is option (a). 34. What is the rhyming scheme of Robert Browning’s ”My Last Duchess”? (a) Enjambed blank verse (b) Enjambed rhyming couplets (c) End stopped rhyming couplets (d) End stopped blank verse Ans. (b) : Enjambed rhyming couplets is the rhyming scheme of Robert Browning’s “My Last Duchess.” Robert Browning belongs to the Victorian age. My Last Duchess is a famous dramatic monologue, published in 1842 in his dramatic lyrics. The poem is pictorial in nature. There are 56 lines in the poem written in ‘rhyming couplets’ (a pair of end-rhymed lines of verse that are self-contained in grammatical structure and meaning). A couplet may be formal (closed), in which case each of the two line is end-stopped, or it may be run-on-couplet (open couplet) with the meaning of the first line continuing to the second enjambment. Hence, the correct answer is option (b). 35. Who has made the following statement? ’’The mind of the poet is the shred of platinum.” (a) Aristotle (b) Wordsworth (c) T. S. Eliot _________(d) Dryden __________ Ans. (c) : The given statement “The mind of the poet is the shred of Platinum” is spoken by Thomas Steam Eliot (T.S. Eliot). This line occurs in his essay on poetic theory ’’Tradition and the Individual Talent”, first published in 1919 and soon after included in ’’the Sacred Wood: Essay on Poetry and criticism” (1920) Notable works of T.S. Eliot are :=> The Love Song of J. Alfred Prufrock (1915). => The Waste Land (1922) => The Hollow Men (1925) => Ash Wednesday (1930) => Four Quartets (1943) => Murder in the Cathedral (1935) => The Cocktail Party 1949 etc. Hence, the correct answer is option (c).

36

YCT

36.

The ’Stream of Consciousness’ method is NOT Ans. (b) : “The emotion of art is impersonal, and the used by poet can not reach this impersonality without (a) Nathan Asch in ’’Pay Day” surrendering himself wholly to the work to be done”, is (b) Conrad Aiken in ’’Blue Voyage” said by famous modem English critic and poet T.S. Eliot (Thomas Steams Eliot). This statement is used by (c) Virginia Woolf in ’’Mrs. Dalloway” _____(d) Rebecca West in ’’Harriet Hume” __________ Eliot in his essay ‘Tradition and the Individual Ans. (d) : All the given authors have used “Stream of Talent’. T.S. Eliot was a poet essayist, playwright , Consciousness” method in their works except option (d) literary critic and editor. He got the Novel Prize in Literature in 1948. He is one of the giants of modem Rebecca West in “Harriet Hume”. ‘Stream of Consciousness’ is a phrase used by William literature highly distinguished as a poet. James in his book ’’The Principles of Psychology” 1890 His famous works are to describe the unbroken flow of perceptions, memories, 1. The Waste Land (1922) thoughts and feeling in the waking mind. It has been 2. Murder in the Cathedral (1935) adopted to describe a narrative method in modem 3. Four Quartets etc (1909-1935) fiction. The best practitioner of this technique are Hence, the correct answer is option (b), Virginia Woolf in ’Mrs. Dalloway’, Dorothy Richardson 40. Wherein was Lamb’s Essay first published and Henry James. serially? Hence, the correct answer is option (d). (a) in the Lavander Magazine 37. Which of the following is NOT correctly (b) in blackwood Magazine matched? (c) in the London Magazine (a) ’We are such stuff as dreams are made on’ - _____(d) in quarterly Magazine ___________________ The Tempest Ans. (c) : In ’The London Magazine' Lamb’s essays (b) ’Fair is foul and foul is fair’ - Macbeth were first published. ‘The London Magazine’ is the title (c) ’If music be the food of love, play on’ - of six different publications that have appeared in Twelfth Night succession since 1732. All six have focused on the arts (d) ’Some are bom great, others achieve literature and miscellaneous topics. Charles Lamb was greatness’ - Richared II an English essayist, poet and antiquarian, best known Ans. (d) : All the given option are correctly matched for his Essays of Elia and for the children’s book “Tales except option (d) “Some are bom great, others achieve from Shakespeare” co-authored with his sister, Mary greatness” - Richard II. This line has been spoken by Lamb. Malvolio in the play Twelfth Night written by William Major works :Shakespeare. It is a comedy, centres on the twins Viola 1. The Old Familiar Faces and Sebastian who are separated in a shipwreck. Its 2. A Tale of Rosamund Gray and Old Blind Margaret main source is a prose tale Apollonius and Silla (1581) (1798) by Bamabe Riche an English version of an Italian tale 3. On the Tragedies of Shakespeare (1811) by Cinthio (novelist). The sub- title of the play is ‘What 4. Essays of Elia (1823) You Will’. The setting is the court of Orsino, Duke of 5. The Last Essays of Elia (1833) Illyria and the house of the wealthy countess Olivia, 6. Christ’s Hospital Five And Thirty Years Ago whom he is courting. Hence, the correct answer is option (d). _____________ 7. Poor Relations. Hence, the correct answer is option (c).______________ 38. The typical imagist poem is written in 41. Which of the following novels was declared (a) heroic couplet (b) blank verse unfit and illegal for human study by the court ? _____(c) free verse _________(d) sonnet ___________ (a) The White Peacock (b) The Rainbow Ans. (c) : The Typical Imagist poem is written in Free _____ (c) Women in Love (d) The Tresspasser Verse. Free verse is also known as ‘Verse Libre’ or ’Open form’. It is unrhymed and metrically irregular. Ans. (b) : The Rainbow, the novel was declared unfit Hence the lines are unequal in length. T.S. Eliot used it and illegal for human study by the court. in much part of his 'The Waste Land’ (1922) and Walt 'The Rainbow’ was written by D.H. Lawrence, Whitman used it in ‘Leaves of Grass’. In easy way, we published in 1915. The novel was officially banned can say, Free Verse is an open form of poetry, it does after it was labeled obscene and unsold copies were not use consistent meter patterns, rhyme, or any musical confiscated. The story line traces three generations of pattern. the Brangwen family living in the Midlands of England Hence, the correct answer is option (c). ______________ from 1840 to 1905. 39. ’’The emotion of art is impersonal, and the poet 42. Find the right combination can not reach this impersonality without (a) A Prayer for My Daughter - W.B. Yeats surrendering himself wholly to the work to be (b) Sonnet an Sonnet - Shakespeare done.” Who said? (c) The Canonization -S.T Coleridge (a) Wordsworth (b) T. S. Eliot (d) The Indian Summers - John Keats (c) S. T. Coleridge (d) I. A. Richards 37 YCT UPPSC GDC (Asst. Prof.) Exam. 2021

Ans. (a) : The right combination is option (a). “A Prayer for My Daughter” is a poem by ’’William Butler Yeats written in 1919 and published in 1921 as part of ‘Yeats’ collection - Michael Robartes and the Dancer. W.B. Yeats was an Irish poet, dramatist, writer and one of the foremost figures of 20th century literature. It is written to Anne, his daughter with Georgie Hyde Lees, whom Yeats married after his last marriage proposal to Maud Gonne was rejected in 1916. 43. Which of the following critics considers a poet as "a man speaking to men” ? (a) William Wordsworth (b) Sir Philip Sidney _____(c) T. S. Eliot _________(d) Matthew Arnold Ans. (a) : William Wordsworth consider a poet as ‘a man speaking to men’. He wrote these words in the ’’Preface” to the second edition (1800) of the ’’Lyrical Ballads”. The ’’Preface” is often considered a manifesto of the romantic movement in English literature. 44. Who says in the ’Duchess of Malfi’, ’’Diamonds are of most value, they have passed through most jewellers’ hands” ? (a) The Duchess (b) Julia _____(c) Cariola ___________(d) Delio ___________ Ans. (a) : ‘The Duchess’ says in the ‘Duchess of Malfi’ “Diamonds are of most value, they have passed through most jewellers’ hands.” ‘The Duchess of Malfi’ is five act tragedy by English dramatist John Webster, performed in 1613-14 and published in 1623. ‘The Duchess of Malfi’ tells the story of the spirited duchess and her love for her trustworthy steward Antonio. 45. Which of the following movements in literature and arts had its origin in France ? 1. Surrealism 2. Literature of the Absurd 3. Imagism Select the correct answer using the codes given below. Codes : (a) 1 and 2 (b) 1 only (c) 2 and 3 (d) 3 only Ans. (a) : ‘Literature of the Absurd’ and ‘Surrealism’ movements in literature and arts had its origin in France. In literature, it refers to a style of writing in which authors focus on the meaninglessness of the universe and human life. Hence, option (a) is correct. 46. On seeing whom does Miranda exclaim, ”O, father, surely that is spirit, Lord ! How it looks about” ? (a) Caliban (b) Ferdinand (c) Alonso (d) Stephano UPPSC GDC (Asst. Prof.) Exam. 2021

Ans. (b) : On seeing Ferdinand, Miranda exclaim, “O, father, surely that is spirit, Lord! How it looks about” Miranda, charming and graceful girl, she is one of the most charming heroines of Shakespeare’s play ’The Tempest’. 47. ”An Apology for Poetry is a work of literary criticism by Philip Sidney, Philip Sidney is also Known as (a) Classical poet (b) Elizabethan poet (c) Augustan poet (d) Restoration poet Ans. (b) : ‘An Apology for Poetry’ is a work of literary criticism by Philip Sidney, Philip Sidney is also known as an Elizabethan Poet. ‘An Apology for poetry’ (or The Defence of Poesy) is a work of literary criticism by Sidney in approximately 1580 and first published in 1595, after his death. 48. The soul of tragedy, according to Aristotle is (a) Characters (b) Thought (c) Spectator (d) Plot Ans. (d) : The soul of tragedy, according to Aristotle is Plot. Aristotle determined six elements of tragedy-Plot, character, diction, thought, spectacle and song. Plot is ‘ the soul of tragedy’, because action is paramount to the significance of drama and all other elements are subsidiary. 49. The following famous quote praises the style of an important essayist. Fill in the blank with an appropriate name: Whoever wishes to attain an English style familiar but not coarse and elegant but not ostentatious, must give his days and nights to the volumes of . (a) Walter Pater (b) Charles Lamb (c) Joseph Addison (d) Richard Steele Ans. (c) : The famous quote praises the style, Whoever wishes to attain an English style, familiar but not coarse and elegant but not ostentations, must give his days and nights to the volumes of Joseph Addison. He was an English essayist, poet, playwright politician and best known as ’’The Noblest purifier of English literature”. 50. Which character in William Golding’s Lord of the Flies maintains, ’’life is scientific”? (a) Simon (b) Ralph (c) Piggy ____________(d) Jack ____________ Ans. (c) : ‘Piggy’, a character in William Golding’s ‘Lord of the Flies’ maintains, “Life is scientific.” Piggy is the first boy Ralph encounters on the island after the crash and remains the most free and loyal friend throughout ’Lord of the Flies’. He is an intellectual with eyesight, a weight problem and asthma, he is the most physical vulnerable of all the boy despite his greater intelligence.

38

YCT

51.

Which of the following poems is written in Terza Rima ? (a) P. B. Shelley’s Ode to the West Wind ? (b) John Keats ’Ode to a Nightingale’ (c) William Wordsworth’s ’The Solitary Reaper’ _____(d) Tennyson's 'Ulysses' ____________________ Ans. (a) : ’P.B. Shelley's ’Ode to the West Wind’ is written in Terza Rima. It was published in 1920, considered a prime example of the poet’s passionate language and symbolic imagery, the ode invokes the spirit of the west wind. This ode introduced a new stanzaic form composed of five sonnets, each of which has four tarcets (units of three lines each). The scheme is based on Italian terza rima, rhyming aba, bcb, cdc, and ded followed by a rhyming ee couplet. _____________________________ 52. The line ’’And the dead tree given no shelter, the crickets no relief’ appears in which of the following works of T.S. Eliot? (a) The Waste Land (b) The Love song of J. Alfred Prufrock (c) Ash Wednesday (d) The Hollow Men Ans. (a) : The line "And the dead tree given no shelter the crickets no relief' appears in ‘The Waste Land’ is a work of T.S. Eliot. ‘The Waste Land’ is a long poem by T.S. Eliot published in 1922 first in London in the Criterion (October) next in New York City in The Dial (November). The 433 lines, five part poem was dedicated to fellow poet Ezra Pound. The Waste Land expresses with great power the disillusionment and disgust of the period after World War I. __________________________________ 53. Which of the following is NOT correct matched? (a) Francis Bacon - Father of English essayists (b) Charles Lamb - Prince of English essayists (c) Bertrand Russell -Nobel prize in literature _____(d) Walter Pater -Poor relations ______________ Ans. (d) : In the given option, option (d) is not correctly matched. ’Poor Relations" is written by Charles Lamb. The story depicts the troubles of family members who are poor and deprived of their rightful wealth. It is one of the most interesting essays of Lamb, Lamb begins the essay with an element of humour and ends it with an element of pathos. He has given many metaphorical phrases to describe the poor relation. _____ 54. Who called Chaucer ” The father of English poetry’? (a) Spenser (b) Yeats (c) Arnold (d) Milton Ans. (c) : Matthew Arnold called Geoffrey Chaucer “The father of English poetry”. Geoffrey Chaucer was an English poet author and civil servant best known for ‘The Canterbury Tales’. He was the first to write what became generally well known and recognized poems and stories in the language of the common people of his time in medieval English.

UPPSC GDC (Asst. Prof.) Exam. 2021

55.

In Pinter’s ‘The Birthday Party’, what does Meg serve for breakfast in Act I ? (a) Toast and Jam (b) Eggs and Bacon (c) Pancakes and Waffles (d) Cornflakes and Fined Bread Ans. (d) : In Pinter’s The Birthday Party, Meg serves for breakfast - ’cornflakes and Fined Bread in Act 1. The Birthday Party is a drama in three acts by Harold Pinter, produced in 1958 and in 1959. Pinter’s first fulllength play established his trademark ‘comedy of menace’ in which a character is suddenly threatened by the vague horrors at large in the outside world. 56. Caroline age is concerned with the reign of (a) Charles I (b) Victoria (c) George (d) None of the above Ans. (a) : Caroline age is concerned with the reign of Charles I, the age of Caroline is an age in British history that is named for Charles I. It lasted from 1625 to 1649. It is made up of three poetic schools - Metaphysical, Cavalier and Puritan. The word "Caroline" comes from the name "Charles", king of England from 1625 to 1649. Thus, the appropriate answer is option (a). ______ 57. The Lotos-Eaters is based on (a) Homer’s Odyssey (b) Dante’s Inferno (c) Homer’s The Iliad _____(d) Dante’s Divine Comedy _________________ Ans. (a) : Tennyson’s ‘The Lotos-Eaters’ is based on an episode in book 9 of Homer’s Odyssey. The Odyssey, in which Odysseus and his men travel through the land of the lotos eaters. They attempt to return home from the Trojan war as well as Odysseus’ sailors returning home after the fall of Troy are forced to land in a strange country after a strong wind propels them post the Island of Cythera. Thus, appropriate answer is option (a). 58. Which of the following is NOT correctly matched ? (a) Then felt like some watcher of the skies When a new planet swims into his ken Tennyson (b) Water, water, everywhere, Nor any drop to drink - Coleridge (c) Oh, lift me as a wave, a leaf, a cloud !Shelley (d) The Sounding Cataract Haunted me like a ________passion -Wordsworth ___________________ Ans. (a) : In the given option (a) is not correct while others are correct. "Then felt I like _____ swims in to his ken", has been taken from John Keats, ‘On First Looking into Chapman’s Homer" (Lines 9-10). This line inspires Keats to more and more ambitious goals. "The planet ’swims’ into the astronomers" Ken is an example of extreme metaphorical language called Catachresis, which is a radical misuse of words. Thus, appropriate answer is option (a).

39

YCT

59.

Detective fiction in English was first written by (a) Edgar Allan Poe (b) Wilkie Collins _____(c) Charles Dickens (d) Georges Eliot _____ Ans. (b) : Detective fiction in English was first written by Wilkie Collins. He was an English novelist and playwright known especially for ’The Woman in White’ (1859), a mystery novel and early 'Sensation novel’ and for ’The Moonstone’ (1868), which has been proposed as the first modern English detective novel. Thus, appropriate answer is option (b). 65. In Pinter’s The Birthday Party, where does Petey meet Goldberg and McCann ? (a) On the beach (b) Intown _____(c) On the road________(d) The pier _________ Ans. (a) : In Pinter’s ’The Birthday Party', Petey meets Goldberg and McCann on the beach. 'The Birthday Party’ (1957) is a play by Harold Pinter, first published in London by Encore Publishing in 1959. The play focuses on human existence that is devoid of purpose and meaning through the image of the protagonist. Thus, the correct option is (a). _____________________ 66. ’’Anagnorisis” is a term used by Aristotle for describing. (a) The reversal of fortune (b) The happy resolution of the plot (c) The moment of recognition by the protagonist (d) The convergence of the main plot and sub________plot _________________________________ Ans. (c) : ’’Anagnorisis” is a term used by Aristotle for describing the moment of recognition by the protagonist. ’Anagnorisis’ is a Greek word for ’recognition’ or ’discovery’ by the protagonist. It is used by Aristotle in his Poetics to denote the turning point in a drama. Thus, the correct option is (c). _____________ 67. Identity Alice in the following sentence from Lamb’s ’’Dream children”. The children of Alice call Bartrum father (a) Alice in Wonderland (b) Charles Lamb’s sister (c) Charles Lamb’s beloved wife (d) Charles Lamb’s beloved whom he couldn’t ________marry ________________________________ Ans. (d) : Alice is beloved of Charles Lamb whom he couldn't marry. Charles Lamb (1775-1834) was an English essayist, poet and antiquarian, best known for his Essays of Elia and for the children’s book, ’’Tales from Shakespeare.” The essay ’’Dream-children: A Reverie” definitely implies a dream element. The term ‘reverie’ as already noted, indicates day-dreaming where as ‘dream children’ are the children of daydreaming. Hence, the correct answer is option (d). 68. ”His Wessex is the special environment of his characters whose development could not be understood without reference to it.” Who comments about Hardy ? (a) A.C Rickett (b) George Sampson (c) J. B. Priestly (d) David Cecil YCT 40

What is the full name of Darcy in the novel Tride and Prejudice’ ? (a) Henry Darcy (b) John Darcy _____(c) Jonathan Darcy (d) Fitzwilliam Darcy Ans. (d) : Fitzwilliam Darcy is the full name of Darcy in the novel ’Pride and Prejudice’ by Jane Austen. Fitzwilliam Darcy is a very arrogant man of England’s upper class. He is lord of Pemberley, an expensive estate located in the county of Derbyshire. He is fairly tall and handsome, but his demeanor is unfriendly and inappropriate. Thus, the correct answer is option (d). 60. Graham Greene’s novels are marked by (a) Protestantism (b) Paganism _____(c) Buddhism _________(d) Catholicism ______ Ans. (d) : Graham Greene’s novels are marked by Catholicism. Greene objected strongly being described as a Roman Catholic novelist, rather than as a novelist who happened to be Catholic. Catholic religious themes are at the root of much of his writing. For example ’Brighton Rock’, ’The Power and the Glory’, ‘The Heart of the Matter’ and ’The End of the Affair.’ Thus, the appropriate answer is option (d). ___________ 61. Who defines criticism in the following words : ’’Criticism is the art of interpreting art”? (a) Aristotle (b) Plato _____(c) Walter Pater _______(d) Water Releigh Ans. (c) : Walter Pater defines ’’Criticism is the art of interpreting art”. Walter Horatio Pater was an English essayist, art and literary critic, and fiction writer. His first and most often reprinted book is ’’Studies in the History of the ’’Renaissance.” ”He belongs to Aesthetic Movement. He emphasized on ’’art for art’s sake”. Thus, the correct answer is option (c). 62. ’’The Aristotelian poetics is the poetics of the drama and specially of tragedy.” Who commented ? (a) Dr. Johnson (b) A.C. Bradley _____(c) Northrope Fry (d) Wimsatt and Brooks. Ans. (d) : Wimsatt and Brooks commented that ’’The Aristotelian poetics is the poetics of the drama and specially of tragedy” When he defined tragic hero concept in Shakespeare’s tragedies. Thus, the correct option is (d). 63. Who was the translator of the German national poet Goethe’s works into English ? (a) Thomas Browne (b) John Stuart Mill _____(c) Samuel Beckett (d) Thomas Carlyle Ans. (d) : Thomas Carlyle was the translator of the German national poet Goethe’s works into English. ’’The Life of Friedrich Schiller” translated into English by Goethe. Thomas Carlyle was a Scottish cultural critic, essayist, historian, lecturer, Mathematician, Philosopher and translator. He has translated the works of Goethe, Johann Karl Musaus and Jean Paul. Thus, the appropriate answer is option (d). UPPSC GDC (Asst. Prof.) Exam. 2021

64.

Ans. (c) : J. B. Priestly comments on Thomas Hardy, ”His Wessex is the special environment of his characters whose development could not be understood without reference to it.” Thus, the correct answer is option (c). ______________ 69. Who among the following writers is NOT Irish? (a) Samuel Beckett (b) James Joyce (c) W. B. Yeats _______(d) T.S. Eliot ________ Ans. (d) : T.S. Eliot was an American English poet, playwright, literary critic and editor. He is best known as a leader of the modernist movement in poetry. He moved to England in 1914 at the age of 25 and went on to settle, work and marry there. He became a British citizen in 1927 at the age of 39. While the other writers are Irish. Thus, the correct option is (d). 70. ’’Let nature be your teacher”, occurs in which of the following poems ? (a) Tintem Abbey (b) The Excursion (c) The Prelude (d) Solitary Reaper Ans. (*) : The line, ’’Let Nature be your teacher”, is taken from the famous poem of William Wordsworth, ’’The Tables Turned”, published in 1798 in the collection ‘Lyrical Ballads’. In this line, the speaker is telling his friend that nature has more ability to teach than books and that is why he should go outside rather than seek refuge in dry pages. Note All the above given answer is not correct with the given line but UPPSC has given the answer option (a). Hence, the correct answer is option (a).______________ 71. Which of the following is NOT correctly matched ? (a) Mexico - Power and the Glory (b) Chandrapore A Passage to India (c) Coral Island - Lord of the Flies _____(d) Canada ______ Hard Times ___________ Ans. (d) : From the given options, options (d) is not matched correctly. Charles Dickens’ novel ’Hard Times’ is set in the Coketown - an Industrial mill town in England instead of Canada. The other novels and their settings are matched correctly. Thus, the correct option is (d). _____________________ 72. About whom is the following statement made ? ”He is a gentleman that is very singular in his behaviour.” (a) Sir Roger de Coverley (b) Captain Sentry (c) Andrew Freeport (d) Will Honeycomb _______________________ Ans. (a) : The given statement -”He is a gentleman that is very singular in his behavior”- is made about Sir Roger de Coverley. Sir Roger de Coverley, a member of the Spectator Club, is a character made up by Richard Steele. This line is taken from Richard Steele’s The Spectator Club (1711). Thus, the correct option is (a). UPPSC GDC (Asst. Prof.) Exam. 2021

73.

The Lives of Saints were depicted in (a) Morality plays (b) Miracle plays (c) interludes (d) Ballads Ans. (b) : The Lives of Saints were depicted in Miracle plays. Miracle play, also called Saint’s play, is one of three principal kinds of vernacular drama of the European Middle Ages. A Miracle play presents a real or fictitious account of the life, miracles or martyrdom of a saint. 74. Who says ”God will come and we will be saved. If we drop him, he will punish us,” in the play Waiting for Godot ? (a) Estragon (b) Vladimir (c) Pozzo (d) Lucky Ans. (b) : Vladimir says, ”God will come and we will be saved. If we drop him, he will punish us,” in the play Waiting for Godot. It is a tragicomedy in two acts by Irish writer Samuel Beckett, published in 1952 in French as ’En attendant Godot’ and first produced in 1953. The play consists of conversations between Vladimir and Estragon, who are waiting for the arrival of the mysterious Godot. 75. Who called Keats one of the ’’inheriters of unfulfilled renown’? (a) Wordsworth (b) Coleridge (c) Shelley (d) Byron Ans. (c) : Percy Bysshe Shelley called John Keats one of the ’’inheriters of unfulfilled renown.” This phrase is taken from Percy Bysshe Shelley’s ‘Adonais : An Elegy on the Death of John Keats’. It was written and published in 1821 to commemorate the death of his friend and fellow poet John Keats earlier that year. 76. In which Chapter of the Poetics, Aristotle compares tragedy with epic and show its superiority ? (a) XXVII (b) XXV (c) XXVI (d) XXIV th Ans, (c) : In XXVI (26 ) Chapter of the Poetics? Aristotle compares tragedy with epic and show its superiority. Aristotle compares tragedy with epic and finds that ’despite having so much similarities in both of them, there are some differences on the basic of their presentation, meter, length, spectacle and melody’ which make tragedy superior to epic. Aristotle considers tragedy over the epic as ’’all the elements of an epic are found in tragedy, but all the elements of tragedy are not found in the epic.”

41

\ | Tragedy Voy-Epic YCT

77.

Wordsworth’s "Ode on Intimations of Immortality" hold the view that (a) Soul is non-existent (b) Childhood is immortal (c) Soul lives in heaven before coming into the world (d) When we grow older we do not forget our experiences of Heaven Ans. (c) : William Wordsworth’s "Ode on Intimations of Immortality" holds the view that Soul lives in heaven before coming into the world. The poem, published in the collection Poems in Two Volumes in 1807, expresses the thought of narrator as "When he was a child, he saw the whole world shining with heavenly beauty and wonders where that beauty has gone now, he is an adult." 78. How many issues of "The Spectator" were published during its tenure ? (a) 555 (b) 455 (c) 355 ______________(d) 655 _____________ Ans. (a) : 555 issues of ’The Spectator’ were published during its tenure (1711 to 1712) beginning on 1st March 1711. The Spectator, a periodical published in London by the essayists Sir Richard Steele and Joseph Addition from March 01, 1711 to Dec. 6, 1712. These issues were collected into seven volumes. 79. Which of the following poems includes a Choric Song ? (a) InMemoriam (b) Mariana (c) Locksley Hall (d) The Lotos Eaters Ans. (d) : From the given options, the poem Lotos Eaters includes a choric song. “Lotos Eaters” by Alfred Lord Tennyson has eight choric songs in total. It was published in the collection Poems (1832 ; dated 1833). The poem is based on an episode in Book 9 of Homer’s Odyssey. 80. Match List-I with List-II and select the correct answers using codes gives below the lists :List-II List-I A. 1798 1. The Great Tradition 2. Principle of Literary B. 1865 Criticism 3. Function of Criticism at C. 1924 Present Time D. 1948 4. Preface to the Lyrical Ballads Codes A B D C 2 4 (a) 1 3 2 1 3 (b) 4 2 1 (c) 4 3 2 1 4 (d) 3 UPPSC GDC (Asst. Prof.) Exam. 2021

Ans. (b) : The correct match of List-I with List-II is in option (A4, B3, C2, DI). List-II List-I A 1798 4 Preface to the Lyrical Ballads B 1865 of Criticism at 3 Function Present Time c 1924 2 Principle of Literary Criticism D 1948 1 The Great Tradition Note:-‘Lyrical Ballads,’ collection of 23 poems, first published in 1798 by S.T. Coleridge and William Wordsworth while ‘Preface to Lyrical Ballads’ was published in the second edition in 1800 by William Wordsworth. 81. In which language did Samuel Beckett first write most of his novels before he rendered them in English? (a) Latin (b) French (c) Greek (d) German Ans. (b) : Samuel Beckett first wrote most of his novels in French before he rendered them in English. Samuel Barclay Beckett, author, critic and playwright, winner of the Nobel Prize for Literature in 1969, wrote in both French and English and is perhaps best known for his plays, especially ‘En attendant Godot (Waiting for Godot) 1952/ 82. Which of the following is NOT correctly matched? Character Characteristic A. Sir Roger Valet-de - Has looks of a Privycounsellor Chambre B. Sir Roger’s Butler - Grey headed C. Sir Roger’s Groom - One of the gravest men D. Sir Roger’s Like his brother Coachman (a) Both A and D (b) Both A and B (c) Both B and C (d) Both C and D Ans. (a) : From the following characters and characteristics, ’A’ and ’D’ are not matched correctly while correctly matched options are Character Characteristic A. Sir Roger Valet-de His Brother -Chambre B. Sir Roger’s Butler Grey-headed One of the gravest men C. Sir Roger’s Groom D. Sir Roger’s Coachman Has looks of a Privycounsellor Sir Roger de Coverley (56 year), a fictional character in ‘The Spectator’ no. 106, (1711), by Joseph Addison and Richard Steele. Thus, correct answer is option (a). 83. Imagism evolved under the influence of poetic theory of (a) LA. Richards (b) F.R. Leavis (c) John Crowe Ransom (d) T.E. Hulme

42

YCT

Ans. (d) : Imagism evolves under the influence of poetic theory of T.E. Huime. Imagism was a movement in early twentieth century Anglo-American poetry that favored precision of imagery and clear, sharp language, as opposed to decorous language and unnecessary meter and rhyme. Though Ezra Pound is noted as the founder of imagism, the movement was rooted in ideas first developed by English philosopher and poet T.E. Hulme. 84. Which out of the following novels was originally a youthful work entitled for ’First Impressions’? (a) Pride and Prejudice (b) Power and The Glory (c) The Rainbow (d) Mrs. Dalloway Ans. (a) : ‘Pride and Prejudice’, a Novel by Jane Austen, was originally a youthful work entitled ’First Impressions’ published in three volumes in 1813. The narrative describes the clash between Elizabeth Bennet, the daughter of a country gentleman and Fitzwilliam Darcy, a rich and aristocratic Landowner. 85. The line, ’’The best lack all conviction while the worst/Are full of passionate intensity, ’’are taken from, (a) The Tower (b) Sailing to Byzantium (c) The Second Coming (d) Easter 1916 Ans. (c) : The lines ’’The best lack all conviction, while the worst/Are full of passionate intensity,” are taken from W.B. Yeats most famous poem ’’The Second Coming.” The Second Coming, first printed in ‘The Dial’ (Nov 1920), and published in his collection of verses entitled Michael Robartes and the Dancer (1921). Through this two-stanza poem in blank verse, Yeats believed that history is cyclical and with its imagery of swirling chaos and terror, prophesies the cataclysmic end of an era. 86. In the opening dialogue of the novel A Passage to India, who is the most open to the possibility of Indian - English friendship ? (a) Aziz (b) Hamidullah (c) Mahmoud Ali (d) Hamidullah’s wife Ans. (b) : In the opening dialogue of the novel ‘A Passage to India,’ Hamidullah is the most open to the possibility of Indian-English friendship. Hamidullah, Aziz’s uncle and friend, educated in law at Cambridge University, he declares at the beginning of the novel that it is easier to be a friend of an Englishman in England than India. Aziz comes to agree with him. A Passage to India, a novel by E.M. Forster, published in 1924 and examines racism and colonialism. 87. Tom Jones consists of (a) Fourteen books (b) Seventeen books (c) Eighteen books (d) Twenty books UPPSC GDC (Asst. Prof.) Exam. 2021

Ans. (c) : Tom Jones, in full title ‘The History of Tom Jones, a Foundling,’ a comic novel by Henry Fielding, published in 1749, consists of Eighteen books. It is a Bildungsroman and a Picaresque novel. S.T. Coleridge argued that it has one of the ’’three most perfect plots ever planned”, alongside Oedipus Tyrannus and The Alchemist. Henry Fielding (1707-1754), novelist and playwright, who with Samuel Richardson, is considered a founder of the English novel. 88. In 1698 who attacked the Restoration drama in a pamphlet entitled -’A short view of the immorality and profaneness of the English stage ? I. John Milton II. John Bunyan III. Jeremy Collier IV. John Dryden Codes : (a) Only I (b) Only III (c) Only IV (d) Only II Ans. (b) : In 1698 Jeremy Collier attacked the Restoration Drama in a pamphlet entitled ’A short view of the immorality and profaneness of the English stage’. In his notorious pamphlet, Collier attacked William Wycherley, John Dryden, William Congreve, John Vanbrugh, and Thomas D' Urfey, censuring them for indecency, for profane language for abusing the clergy, and for undermining public morality by sympathetic presentation of vice. 89. Sei anus and Catiline the two post Shakespearean tragedies were written by (a) Ben Jonson (b) Middleton (c) Webster (d) Dekker Ans. (a) : Seianus and Catiline, the two post Shakespearean tragedies were written by Ben Jonson. Seianus His Fall, a 1603 play by Ben Jonson, a tragedy about Lucius Aelius Sejanus, the favourite of the Roman emperor Tiberius. Catiline His Conspiracy is a Jacobean tragedy written by Ben Jonson and published in quarto in 1611. Seianus His Fall and Catiline His Conspiracy both are Roman tragedies. 90. Which of the following is kind of truth that Francis Bacon NOT talk of in his essay ”Of Truth” ? (a) Theological truth (b) Philosophical truth (c) Truth of civil business (d) Truth of spiritual business Ans. (d) : From the following options, ’’Truth of Spiritual business” is a kind of truth that Francis Bacon does not talk of in his essay ”Of Truth”. Of Truth (1625) is Bacon’s great work of prose which shows his keen observation of human beings with their attributes of truth and lie. In the beginning, he states that people generally do not care for the truth.

43

YCT

UPPSC Govt Degree College Asst Prof. Exam. 2017

ENGLISH (Solved Paper) 1.

’’And I have felt A presence that disturbs me with the joy”. The above lines have been quoted from: (a) Arnold (b) Tennyson (c) Wordsworth _______(d) Larkin ___________ Ans. (c) The given lines have been quoted from William Wordsworth poem ’Tintem Abbey’. It was published in 13 July, 1798. ’Tintem Abbey’ is the young ’Wordsworth’s first great statement of his principle (great) theme: that the memory of pure communion with nature in childhood works upon the mind even in adulthood, when access to that pure communion has been lost and that the maturity of mind present in adulthood. 2. Who says that ”1 was declared the belle of the ball” in The Birthday Party? (a) Meg (b) Goldberg (c) Petey ____________(d) Lulu _____________ Ans. (a) ’The Birthday Party’ is about Stanley Webber, an erstwhile piano player who lives in a rundown boarding house run by Meg. The given statement was said by Meg. It was written in 1957 by Harold Pinter. It is one of his best-known and most frequently performed plays. ______________________ 3. E.M. Forster’s A Passage to India deals with (a) ancient Indian culture (b) arrival of Britishers in India (c) relationship between the Britishers and the Indians (d) discovery of sea route to India _____________ Ans. (c) ’A Passage to India’ is a 1924 novel by English author E. M. Forster set against the backdrop of the British Raj and the Indian independence movement in the 1920. It’s main theme is ’Unity’. Ideas of division and unity are important in ’A Passage to India’ in both a social and spiritual sense. The social and cultural divisions between English and Indians are clear but India itself is also internally divided. 4. ’’And see the children sport upon the shore. And hear the mighty waters rolling evermore.” The above lines are quoted from: (a) Wordsworth’s ’’Immortality Ode” (b) Shelley’s ”Ode to the West wind” (c) Coleridge’s ’’Dejection” (d) Browning’s ’’Prospice” ___________________ Ans. (a) The given lines have been taken from ”Ode on Intimations of Immortality from Recollections of Early Childhood” in a poem by William Wordsworth, completed in 1804 and published in poems in two volumes. It was originally published in 1807. UP GDC Asst. Prof. Exam. 2017

[Exam. Date 3 Nov 2019]

5.

T.S. Eliot’s The Family Reunion deals with (a) Forgive and forget (b) Crime, remorse and purification (c) Crime and revenge (d) Love and hatred _________________________ Ans. (b) ’The Family Reunion’ is a play by T.S. Eliot, written mostly in blank verse, it incorporates elements from Greek drama and mid-twentieth century detective plays to portray the hero’s journey from guilt to redemption. The primary theme of ’The Family Reunion’ is dualistic love-hate relationship between generations and among siblings of the same. 6. Hardy believed in the Philosophy of: (a) Immanent Will (b) Character is Destiny (c) FreeWill (d) Man as the master of his own fate___________ Ans. (a) Thomas Hardy strongly believed in the incoherence of the empirical world. In his major fiction Hardy illustrated his personal philosophy of chance, a belief that chance, a blind force of Nature can change man’s destiny. Chance is for Hardy everything for which man has no control. Since chance is the immanent will of the universe, man’s will cannot overcome chance. Thomas Hardy (18401928) was an English novelist and poet. 7. Who among the 18th century writers is known as ’’the wasp of Twickenham”? (a) Dryden (b) Pope (c) Johnson __________(d) Swift ____________ Ans. (b) Alexander Pope (1688-1744) was a poet and satirist of the Augustan period and one of its greatest artistic exponents. He is best known for his poems ’An Essay on Criticism’ (1711), ’The Rape of the Lock’ (1712-14), ’The Dunciad’ (1728) and ’An Essay on Man’ (1733-34). _______________________________ 8. The main theme of St. Joan is: (a) Religion and faith (b) Hypocrisy of the Church (c) Reason v/s Religion (d) Atheism _______________________________ Ans. (a) ’Saint Joan’ is a play by G.B. Shaw about 15th century French military figure Joan of Arc. G. B. Shaw characterized ’Saint Joan’ as ”A chronicle play in 6 Scenes and an Epilogue.” The source of Joan’s motivation in life was her religious faith in what she held to be Christian truths.

44

YCT

D.H. Lawrence’s novel The Rainbow follows 14 .............. the rarer action is three generations of the In virtue than in vengeance!” (a) Bennet family This is the summing of Shakespeare’s (b) Allworthy family philosophy of life. Which play does express this (c) Brangwen family Philosophy? (d) Monchensey family ______________________ (a) Hamlet (b) Twelfth Night (c) Winter’s Tale ______(d) The Tempest ______ Ans. (c) ’The Rainbow’ is a novel by D. H. Lawrence first published in 1915. It follows three generations of Ans. (d) ’The Tempest’ is a play written by William the Brangwen family living in Nottinghamshire. This Shakespeare. It is a play about magic betrayal, love novel was officially banned after it was labelled and forgiveness. It is set on island somewhere near obscene and unsold copies were confiscated. Italy where Prospero the one time Duke of Milan and 10. The lines ’’Therefore I am still I A lover of the his beautiful daughter Miranda - live with sprite meadows and the woods” are composed by called Ariel and a strange Wildman called Caliban. (a) Byron (b) Coleridge 15. One of the sources of The Waste Land is: (c) Thomson _________(d) Wordsworth _______ (a) Solomon’s Mines (b) The Golden Bough Ans. (d) The given lines have been taken from the (c) Lawrence of Arabia (d) The Arabian Nights poem ’Tintem Abbey’. It is a poem by William Ans. (b) ’The Waste Land' was quickly recognized as Wordsworth. The title ’’Lines composed a few miles a major statement of modernist poetics, both for its above Tintem Abbey, On Revisiting the Banks of the broad symbolic significance and for Eliot’s masterful Wye during a tour July 13, 1798” is often abbreviated use of formal techniques that earlier modernists had simply to ’Tintem Abbey’, although that building does only begun to attempt. One of the sources of 'The not appear within the poem. Waste Land' is ’’The Golden Bough”. 11. In which play does Shakespeare attack the 16. What is the alternative title of Twelfth Night? Puritans? (a) Much Ade About Nothing (a) The Merchant of Venice (b) All is well that ends well (b) Twelfth Night (c) What you will (c) As You Like It (d) Measure for Measure (d) Much Ado About Nothing ________________ Ans. (c) The alternative title of 'Twelfth Night' is Ans. (b) 'Twelfth Night, What You Will' is a romantic "What you will”. It is a romantic comedy by William comedy by William Shakespeare, believed to have Shakespeare, believed to have been written around been written around 1601-02. The play centres on the 1601-02. _____________________________________ twins Viola and Sebastian, who are separated in a 17. Which of the following poems of Wordsworth Shipwreck. In this play Shakespeare attacks the deals with the evolution of the poet’s interest in Puritans. nature? 12. ’Wessex’ is: (a) "Ode on Intimations of Immortality” (a) the home town of George Eliot (b) "Tintem Abbey” (b) the region where Bronte sisters lived (c) 'The World is too much with us” (c) the region in which Hardy's novels are set (d) "The Solitary Reaper" ____________________ (d) name of a country in Scotland _____________ Ans. (b) 'Lines Written a Few Miles above Tintem Ans. (c) Wessex Tales is an 1888 collection of tales, Abbey' is a poem by William Wordsworth. This poem written by English novelist and poet Thomas Hardy, of Wordsworth deals with the evolution of the poet’s many of which are set before Hardy's birth in 1840. It interest in nature. It is famous for its abbey and for the is a collection of five stories. Thomas Hardy was an poets and painters such as Wordsworth and Turner English novelist and poet. He was influenced both in who visited it two hundred years ago in the Romantic his novels and in his poetry by Romanticism. Period. 13. Who is the unifying link in The Waste Land? 18. What kind of plays did Shakespeare write (a) King Fischer (b) Tiresias towards the end of his career? (c) Oedipus King______(d) King Arthur _______ (a) Tragedies (b) Comedies Ans. (b) 'The Waste Land’ is a poem by T.S. Eliot. It (c) History Plays ______ (d) Romance _________ can be viewed as a poem about brokenness and loss and Eliot’s numerous allusions to the First World War Ans. (d) William Shakespeare wrote Romance in the suggest that he played a significant part in bringing end of his career. The Tempest, the last play about this social, psychological and emotional Shakespeare wrote alone, may be read as the collapse. Tiresias is the unifying link in 'The Waste playwright's farewell to the stage. Shakespeare retired Land'. from writing around 1613.

9.

UP GDC Asst. Prof. Exam. 2017

45

YCT

19.

How does the beauty of the antique urn move the pot in "Ode on a Grecian Urn"? (a) It surprises (b) It depresses (c) It enhances and elevates (d) None of these __________________________ Ans. (c) ’Ode on a Grecian Um’ is a poem written by the English Romantic poet John Keats in 1819, first published anonymously in Annals of the Fine Arts for 1819. The poet concludes that the um will say to future generations of mankind ’’Beauty is Truth, Truth Beauty.” _____________________________________ 20. What is the alternative title of Jonson’s Volpone? (a) The Fox (b) The Dog (c) The Snake ________(d) The Bull __________ Ans. (a) Volpone is a comedy play by English playwright Ben Jonson first produced in 1605-06, drawing on elements of city comedy and beast fable. The alternative title of Volpone is ’The Fox’. It was first published in 1606. 21. Which amongst the following works of Keats, contains the line. "Forever will thou love and she be fair"? (a) ”Ode to a Nightingale” (b) ”To Psyche” (c) "Ode to autumn” (d) "Ode on a Grecian Um” __________________ Ans. (d) The given line has been taken from John Keats poem "Ode on a Grecian Um". ’Ode on a Grecian Um’ is a poem written by the English Romantic poet John Keats in 1819, first published anonymously in Annals of the Fine Arts for 1819. The poet concludes that the um will say to future generations of mankind "Beauty is Truth, Truth Beauty.” _____________________________________ 22. Shaw has claimed St. Joan to be a (a) high tragedy (b) farce (c) comedy __________(d) romance __________ Ans. (a) ’Saint Joan' is a play by G.B. Shaw about 15th century French military figure Joan of Arc. G. B. Shaw characterized 'Saint Joan’ as "A chronicle play in 6 Scenes and an Epilogue." The Source of Joan's motivation in life was her religious faith in what she held to be Christian truths. G.B. Shaw has claimed St. Joan to be a high tragedy. 23. Who is famed for the use of ’conceit’ in his poems? (a) Chaucer (b) Spencer (c) Wyatt ____________(d) John Donne _______ Ans. (d) John Donne is famed for the use of 'conceit' in his poems. In the 17th century, a school of poets known as the metaphysical poets also often used conceits in their poetry. Conceit is an often unconventional, logically complex or surprising metaphor whose delights are more intellectual than sensual. UP GDC Asst. Prof. Exam. 2017

24.

T.S. Eliot has borrowed the mythical source in The Family Reunion from: (a) Aeschylus (b) Sophocles (c) Plato _____________(d) Herodotus ________ Ans. (a) T.S. Eliot has borrowed the mythical source in 'The Family Reunion' from Aeschylus. Aeschylus was an ancient Greek author of Greek tragedy and is often described as the father of tragedy. 25. Which of the following books heralded the Romantic Revival? (a) The Prelude (b) The Lyrical Ballads (c) The Seasons _______(d) None of these ______ Ans. (b) 'Lyrical Ballads' is a collection of poems by William Wordsworth and S.T. Coleridge, first published in 1798 and generally considered to have marked the beginning of the English Romantic movement in literature. 26. "Ode on Intimations of Immortality" describes: (a) Soul is non-existent (b) Soul lives in heaven before coming into the world (c) The childhood is immortal (d) All the above ___________________________ Ans. (b) "Ode : Intimations of Immortality From Recollections of Early Childhood” is a poem by William Wordsworth, completed in 1804 and published in ’Poems, in Two Volumes' (1807). The poem is an irregular Pindaric Ode. The poem describes that soul lives in heaven before coming into the world. The idea of pre-existence within the poem contains only a limited theological component. 27. Who authored the novel The Third Man? (a) Graham Greene (b) Charles Dickens (c) Virginia Woolf (d) D.H. Lawrence Ans. (a) The novel ’The Third Man' is written by Henry Graham Greene, an English writer and journalist of the 20th century. 'The Third Man' represents a growing trend in Fiction. At the time of its publication towards realism following the modernism that held sway between the World Wars. __________________________________ 28. The Douchess of Malfi is a: (a) Tragedy of revenge (b) Tragedy of lust (c) Tragedy of jealousy (d) Tragedy of greed Ans. (a) 'The Douchess of Malfi' is a Jacobean revenge tragedy written by English dramatist John Webster in (1612-1613). It was first performed privately at the Blackfriars Theatre, then later to a larger audience at the Globe. 29. Who, amongst the following, proposed to popularize philosophy by bringing it to the coffee houses? (a) Goldsmith (b) Johnson (c) Steele (d) Addison

46

YCT

Ans. (d) Joseph Addison (1672-1719) was an English essayist, poet, playwright and politician. He proposed to popularize philosophy by bringing it to the coffee houses. They were public places where men would meet for conversation and commerce. Various topics were discussed there included politics, daily gossip, fashion, current events and debates surrounding philosophy and the natural sciences. 30. Who is the propounder of "context theory"? (a) T.S. Eliot (b) I.A. Richards (c) F.R. Leavis _______(d) William Empson Ans. (b) Given the nature of context within his model, Richards envisions misunderstanding as the inevitable result of ’’inadequate contextual control”, that is meaning Shifts due to the change in sense or relevance derived from linguistic cues. _____________ 31. Fill in the gap Phillip Larkin is a/an poet. (a) Victorian (b) New Lines (c) Georgian _________(d) Elizabethan _______ Ans. (b) Phillip Larkin is a modem poet writing in the language of common people about the life and matters around him. He based his poetry on the actual experiences of man. Poets in New Lines included Enright, Conquest, Kingsley Amis, Donald Davie, Phillip Larkin, and John Wain. The polemical introduction to New Lines particularly targeted the 1940s poets, the generation of Dylan Thomas. 32. Pope’s The Rape of the Lock is based on: (a) A trivial matter of the court that led to a quarrel between two families. (b) Greek mythology (c) French Folklore (d) None of these __________________________ Ans. (a) A trivial matter of the court that led to a quarrel between two families. It is based on an actual incident and written to reconcile the families that had been estranged by it, ’The Rape of the Lock’ recounts the story of a young woman who has a lock of hair stolen by an ardent young man. 33. Which one is the shortest of Shakespeare’s supreme tragedies? (a) Hamlet (b) Macbeth (c) Othello ___________(d) King Lear ________ Ans. (b) There are four supreme tragedies of Shakespeare - Hamlet, Othello, King Lear and Macbeth. Macbeth is the shortest of Shakespeare’s supreme tragedies. It is thought to have been first performed in 1606. 34. God is referred to as the "President of the immortals" in the novel: (a) The Mayor of Casterbridge (b) Far From the Madding Crowd (c) Tess (d) The White Devil UP GDC Asst. Prof. Exam. 2017

Ans. (c) God is referred to as the 'President of the immortals' in the novel 'Tess of the d'Urbervilles' (Tess). Tess of the d’Urbervilles : A pure woman faithfully presented is a novel by Thomas Hardy. 35. Who claimed to have taken all knowledge to be his province? (a) Shakespeare (b) Bacon (c) Johnson __________(d) Jonson ___________ Ans. (b) Francis Bacon (1561-1626), and English lawyer, politician, essayist and philosopher was one of the leading figures in natural philosopher and the field of scientific methodology that took place from the Renaissance to the early modem era. He declared; ”1 have taken all knowledge to be my province” and after words he dedicated himself to a wholesale revaluation and re-structuring of traditional learning. 36. Who amongst the following novelists, is included in F.R. Leavis’ The Great Tradition? (a) E.M. Forster (b) Graham Greene (c) H.G. Wells ________(d) Joseph Conrad Ans. (d) In 'The Great Tradition' (1948), F. R Leavis reassessed English fiction, proclaiming Jane Austen, George Eliot, Henry James and Joseph Conard as the great novelists of the past and D. H. Lawrence as their only successor. ________________________________ 37. "Stream of Consciousness" in literature denotes (a) A mode of narration that captures full flow of a character’s mental process (b) A state of mind (c) A manner of linguistic expression in prose (d) Dramatic Monologue ____________________ Ans. (a) 'Stream of Consciousness' presents thoughts and feelings that flow through the mind of a character. There is an unending and uneven flow of mind. William James was the first one to use this technique. The other practitioners of this techniques are Virginia Woolf (Mrs. Dalloway), Dorothy Richardson (Piligrimage) and Henry James. 38. Things fall apart; the centre cannot hold; This line occurs in: (a) "Bizantium” (b) ’’Sailing to Bizantium” (c) "Prayer for my Daughter" (d) "The Second Coming” ___________________ Ans. (d) 'Things fall apart; the centre cannot hold’ this line appears in William Butler Yeats' poem "The Second Coming". Violence, prophecy and meaninglessness are the major themes foregrounded in this poem. African novelist Chinua Achebe draws the title of his novel ’Things Fall Apart’ from this poem, _______________________________________ 39. In Waiting for Godot, who says that if "Godot comes ..... we’ll be saved"? (a) Estragon (b) Vladimir (c) Pozzo (d) Lucky

47

YCT

Ans. (b) In ’Waiting for Godot’ Vladimir says that when Godot comes they will be saved, although from what is not defined. ’Waiting for Godot’ is a play by Samuel Beckett. 40. Which of the following is Henry Fielding’s earliest novel? (a) Jonathan Wild (b) Tom Jones (c) Joseph Andrews (d) Amelia ___________ Ans. (c) Joseph Andrews, or The History of the Adventures of Joseph Andrews and of his friend Mr. Abraham Adams, was the first full-length novel by the English author Henry Fielding, to be published and among the early novels in the English language. 41. Who has described Bacon as ’’the wisest and meanest of mankind”? (a) Pope (b) Dr. Johnson (c) Dryden ___________(d) None of these ______ Ans. (a) Alexander Pope, in Epistle IV of his ’Essay on Man’ refers to Sir Francis Bacon as ’the wisest, brightest, meanest of mankind. 42. The criterion of F.R. Leavis’ The Greai Tradition is (a) Sublime subject matter (b) Moral interest (c) Reader - response (d) Truth to life Ans. (a) F. R. Leavis in ’The Great Tradition’ (1948), seeks to establish an order of importance and excellence in the novel in the manner of Arnold and Eliot. The criterion of 'The Great Tradition' is sublime subject matter. 43. The English Reformation aimed at (a) Abolition of morality (b) Social Reform (c) Economic Reform (d) Religious Reform _______________________ Ans. (d) The English Reformation was part of the Protestant Reformation. Many Christian Churches in Europe broke away from Rome. Earlier the Roman Catholic Church had supreme powers. Henry VIII broke ties with the church and became head of the English Church. This was done in 1534 in the Acts of Supremacy. 44. ’’Nature never did betray the heart that loved her.” This line occurs in: (a) "Ode on Intimations of Immortality" (b) "Tintem Abbey" (c) "The Solitary Reaper" (d) "The Daffodils" _________________________ Ans. (b) 'Nature never did betray the heart that loved her’, this line occurs in 'Tintem Abbey'. ’Lines written a few Miles Above Tintem Abbey' is a poem by William Wordsworth, on revisiting the banks of the Wye (river) during a tour, July 13, 1798. UP GDC Asst. Prof. Exam. 2017

45.

John Webster belongs to the (a) Elizabethan period (b) Restoration period (c) Caroline period (d) Jacobean period Ans. (d) John Webster was an English Jacobean dramatist best known for his tragedies ’The White Devil' and 'The Duchess of Malfi' which are often regarded as masterpieces of the early 17th century English stage. 46. Francis Bacon is best known for his (a) lyrical style (b) aphoristic style (c) complex style______(d) ornamental style Ans. (b) Francis Bacon, a lawyer, statesman, philosopher and master of the English tongue. He is remembered in literary terms for the sharp worldly wisdom of a few dozen essays. Bacon is best known for his aphoristic style. An aphoristic style means a compact, condensed and epigrammatic style. 47. Which of the following is not a novel by Graham Greene? (a) The Power and the Glory (b) England Made Me (c) The Heart of Africa (d) The Honorary Consul ____________________ Ans. (c) ’The Heart of Arica' is not a novel by Graham Greene. All the three novels; 'The Power and the Glory', 'England Made Me’ and 'The Honorary Consul’ are written by Graham Greene._______________________ 48. Sidney wrote ”An Apology For Poetry” in response to the views of: (a) Stephen Gosson (b) Socrates (c) John Lily _________(d) None of these ______ Ans. (a) Philip Sidney defends poetry in his essay 'An Apology For Poetry' from the accusations made by Stephen Gosson in his 'School of Abuse'. There, Gosson makes some objections against poetry. Sidney replies to the objections made by Gosson very emphatically, defending poetry in his essay. _________ 49. The Renaissance literally means. (a) Revival of Old Belief (b) Reform of religion (c) Rebirth of Classical learning (d) Resurgence of the Dark Ages ______________ Ans. (c) Renaissance is a French word meaning "rebirth". It refers to a period in European Civilization that was marked by a revival of classical learning and wisdom. 50. Which of the following poems of S.T. Coleridge is the tale of sin and expiation? (a) "Ode to Dejection" (b) "Rime of the Ancient Mariner" (c) "Frost at Midnight" (d) None of these __________________________ Ans. (b) ’The Rime of the Ancient Mariner’ is one of the most significant works of Romantic poetry by S. T. Coleridge. In the poem, a tale waits for the readers a story of committing sin, reception of punishment and its redemption.

48

YCT

(a) ’’The Love Song of Alfred J. Prufrock” 51. Who expressed this view? (b) ”Ash Wednesday” ’’For art’s sake I would not face the toil of (c) ’’The Waste Land” writing a single sentence.” (d) ’’The Hollow Men” ______________________ (a) T.S. Eliot (b) Osborne (c) G.B. Shaw ________(d) None of these ______ Ans. (c) These lines occur in ’The Waste Land’ is a poem by T.S. Eliot, widely regarded as one of the Ans. (c) George Bernard Shaw expressed this view most important poem of the 20th Century and a central ’’For art’s sake I would not face the toil of writing a work of modernist poetry. Given lines are occurred in single sentence”. first-section of the poem. The poem is divided into Source- George Bernard Shaw’s ’Man and Superman’ five sections. The first - ’’The Burial of the Dead”, (1903)._______________________________________ The second - ”A Game of Chess”, The third - ’’The 52. Elizabeth rejected Mr. Collins’ proposal because fire Sermon”, The forth - ’’Death by Water” and The his letter revealed his fifth - ’’What the thunder Said”. ___________________ (a) self-centredness (b) arrogance 57. In the play The Birthday Party, the gift that (c) wickedness _______(d) self-confidence Stanley receives is a Ans. (b) Mr. Collins’ proposes to Elizabeth (In Pride (a) toy drum (b) flute and Prejudice). (c) toy guitar _________(d) toy violin _________ He outlines his motivation for proposing and promises Ans. (a) Toy Drum that Meg gives to Stanley for his never to bring up the fact she brings so little money to birthday is a clear representation of her fondness for the marriage. order. ’The Birthday Party’ is the first full-length play Tom between discomfort and the desire to laugh at his by Harold Pinter, first published in London by Encore officious manner, Elizabeth politely refuses him. Publishing in 1959. Pinter began writing ’The 53. What is the strength of members, as mentioned Birthday Party’ in the summer of 1957 while touring in Doctor in the House. by Steele, in the Spectator Club? (a) Four (b) Five 58. In A Passage to India, E.M. Forster calls India a (c) Six ______________(d) Seven ____________ (a) glory (b) muddle (c) continent _________(d) riddle ____________ Ans. (c) Sir Richard Steele’s essay ’The Spectator Club’ describes six members whose activities and Ans. (b) ’A Passage to India’ is a 1924 novel by English author E. M. Forster set in the 1920s during characters are elaborated upon in various other the period of the British Raj and the Indian essays by Steele and his collaborator, Joseph independence movement. The main theme of ’A Addison. Steele describes six members of the club Passage to India’ is Ideas of division and unity in both they are Sir Roger de Coverley, Captain Sentry, Sir a social and spiritual sense. Writer pointed out his Andrew Freeport, Will Honeycomb, the Clergyman thoughts about India and called India a muddle. and the Student of Law. 59. Who is called ’the prince’ of English essayists? 54. F.R. Leavis edited the literary journal. (a) Bacon (b) Brown (a) Criterion (b) Sewanee Review (c) Lamb (d) Hazlitt (c) New Criticism _____(d) Scrutiny __________ Ans. (c) Charles Lamb is called ’the prince’ of English Ans. (d) Scrutiny : A Quarterly Review was a essayists. He was an English essayist, poet and literature periodical founded in 1932 by L. C. Knights antiquarian, best known for his ’Essay of Elia’ and for and F. R. Leavis, who remained its principal editor the Children’s book Tales from Shakespeare, countil the final issue in 1953. authored with his sister Mary Lamb. Hugh Walker 55. The Oxford Movement is also known as the calls him the essayist par excellence who should taken (a) Tractarian Movement as model. Lamb is the shining star in the sky of (b) University Movement English essay. (c) Intellectual Movement 60. Who expressed the opinion that Chaucer has (d) None of these __________________________ ’’truth” but lacks ’’high seriousness”? Ans. (a) The Oxford Movement was a movement of (a) Matthew Arnold High Church members of the church of England (b) T.S. Eliot which eventually developed into Anglo-Catholicism. (c) F.R. Leavis The movement’s philosophy was known as (d) William Wordsworth ____________________ Tractarianism after its series of publications, the tracts Ans. (a) Matthew Arnold expressed the opinion that for the Times, published from 1833 to 1841. Chaucer has ’’truth” but lacks ’’high seriousness. He 56. And the dead tree gives no shelter, the cricket no has a very high opinion of Chaucer. It is Chaucer who establishes romantic poetry in England. Chaucer is the relief. father of the splendid English poetry. Arnold argues And the dry stone no sound of water. that this immortal poet lacks high seriousness. These lines occur in: UP GDC Asst. Prof. Exam. 2017

49

YCT

61.

Who defined Romanticism as ’’addition of strangeness to beauty”? (a) William Wordsworth (b) Walter Pater (c) Graham Hough ______(d) None of these Ans. (b) Walter Pater defined Romanticism as ’’addition of strangeness to beauty”. Romanticism have given us strange concepts of in ordinary things of beauty, for example Wordsworth has given mystical concept of nature, Coleridge has given supernatural aspect of nature. Walter Pater works on Renaissance subjects were popular but controversial in his times. 62. Which of the following poems of Shelley was inspired by a real bird? (a) ”Ode to the Westwind” (b) ’’The Indian Summer” (c) ”To A Skylark” (d) None of these __________________________ Ans. (c) ’To A Skylark’ is a poem of Percy Bysshe Shelley that was inspired by a real bird. It is a lyric poem published in 1820 with Prometheus Unbound. It consists of 21- five lines of stanza. Skylark is symbol of the joyous spirit of the divine. It cannot be understood by ordinary. 63. Look Back in Anger was published in (a) 1957 (b) 1956 (c) 1950 _____________(d) 1952 _____________ Ans. (b) ’Look Back in Anger’ is a realist play written by John Osborne. It was published in 1956. It focuses on the life and marital struggles of an intelligent and educated but disaffected young man of working-class origin. 64. George Orwell in his writings usually hits at: (a) British Democracy (b) American Pragmatism (c) Stalinism (d) French Liberalism _______________________ Ans. (c) The son of a British civil servant, George Orwell spent his first days in India. His work is characterized by lucid prose, biting social criticism, opposition to totalitarianism and outspoken support of democratic socialism. George Orwell in his writings usually hits at Stalinism. Stalinism is the ideology and policies adopted by Stalin based on centralization, totalitarianism and the pursuit of communism. 65. Who claimed to write fiction that could be called ”comic epic in prose”? (a) Samuel Richardson (b) Henry Fielding (c) Lawrence Sterne (d) Smollett __________ Ans. (b) Henry Fielding claimed to write fiction that could be called ’’comic epic in prose”. He was an English novelist and dramatist known for his rich earthly humour and satirical power. He never stopped writing political satire and satires of current art and letters. UP GDC Asst. Prof. Exam. 2017

66.

’Hamartia’ in Aristotelian criticism denotes: (a) the great virtue (b) the beneficial argument (c) the fatal flaw (d) the hurting reason _______________________ Ans. (c) Hamartia is the term that denotes the downfall of a tragic hero or heroine. Hamartia, also called tragic flaw, (hamartia from Greek hamartarian ”to err”). So in Aristotelian criticism Hamartia denotes the fatal flaw. 67. One of the basic themes of the twentieth century literature is: (a) Togetherness (b) Isolation (c) Exploration _______(d) Revelry __________ Ans. (b) Focusing on literary-culture production emerging from or responding to the 20th century, broadly construed, 20th century literature modernism is a major literary movement of the first part of the 20th century literature. Isolation is one of the basic theme of the 20th century literature. 68. Which of the following poems of Wordsworth mourns the loss of childhood vision? (a) ”Ode on Intimations of Immortality” (b) ’’Tintem Abbey” (c) ’’Three Years She Grew” (d) None of these __________________________ Ans. (a) ”Ode on Intimations of Immortality” poem by William Wordsworth, published in the collection poems in two volumes in 1807. The Ode sings of the mature narrator’s how to breaking realization that childhoods special relationship to nature and experience has been lost forever. This poem mourns the loss of child vision. 69. Who speaks the Epilogue in The Tempest? (a) Alonso (b) Sebastian (c) Prospero __________(d) Ariel _____________ Ans. (c) ’Prospero’ speaks the Epilogue in ’The Tempest’. Epilogue begins with Prospero’s declaration that he is going to overthrow all his personal charms as a magician. It is true he has got back his dukedom and pardoned all the conspirators. The Epilogue is often used to tie up loose ends and clarify any issues that remain unresolved. 70. Who stands for ’pride’ in Pride and Prejudice (1813)? (a) Mr. Collins (b) Mr. Darcy (c) Mr. Philips ________(d) Mr. Charles Bingley Ans. (b) The Traditional View of the book is that Elizabeth Bennet stands for the Prejudice in the title and that Mr. Darcy stand for the Pride. This seems to be correct; Elizabeth judges Mr. Darcy too quickly and with too little information and Mr. Darcy acts as if he were superior to the people in Hertfordshire. 71. Whose essays are known as ’’dispersed meditations”? (a) Addison (b) Steele (c) Bacon (d) Lamb

50

YCT

Ans. (c) The essays of Francis Bacon show the example of Dispersed Meditation. It is the style, where though the discussing matter is some the ideas that are coming one by one are not well organized or ideas don’t come consecutively. He described his essay as ’’dispersed meditation” as ’’brief notes set down rather significantly than curiously”. 72. T.S. Eliot propounds his ’’impersonal theory of poetry” in the essay (a) ’’Hamlet and His Problems” (b) ’’Tradition and the Individual Talent” (c) ’’The Function of Criticism” (d) ’’The Perfect Critic” Ans. (b) Eliot’s essay ’’Tradition and the Individual Talent” was published in the year 1919, in which his views on impersonality of the artist first appeared. The impersonal theory propounded by T.S. Eliot is an attempt to break free from the subjective approach of historical, biographical and sociological schools of criticism. 73. What was the leading principle in the age of Pope? (a) Reason and good sense (b) Romantic thought (c) Loose morals (d) All of these Ans. (a) The leading principle in the age of Pope was reason and good sense. This was an age of new prose forms such as periodicals travelogues, political allegories and romantic tales. The works of this age were concerned with facts and reason and less concern was shown to emotion and imagination. 74. Which of the following poems is a fearless challenge to death? (a) "My Last Duchess" (b) "Prospice" (c) "Rabbi Ben Ezra" (d) None of these Ans. (b) ’Prospice’ the poem which is a fearless challenge to death. Robert Browning begins by describing the oppressive imagery of it - "Fog in my throat”, "the press of the storm, the post of the foe.” This poem is a dramatic monologue and highly optimistic. ____________________________________ 75. Who speaks the following lines in Twelfth Night? ”If music be the food of love, play on, Give me excess of it...” (a) Viola (b) Olivia (c) Valentine _________(d) Orsino ___________ Ans. (d) These lines are spoken by Duke Orsino of Illyria, presenting over the Merry, mixed-up world of ’Twelfth Night’, opens the play with these festive sentiments. Duke Orsino is head over heels in love with the Countess Olivia. However, she’s placed herself into a period of mourning, so Orsino can’t woo her. UP GDC Asst. Prof. Exam. 2017

76.

Who amongst the following is not a writer of ’Stream - of - Consciousness’ novels? (a) James Joyce (b) Dorothy Richardson (c) D.H. Lawrence (d) Virginia Woolf Ans. (c) Except D. H. Lawrence, James Joyce, Dorothy Richardson and Virginia Woolf are the writer of ’Stream of Consciousness’ novels. In literary criticism stream of consciousness is a narrative mode or method that attempts to depict the multitudinous thoughts and feelings which pass through the mind of the narrator. Lawrence’s writing explores issues such as sexuality, emotional health, vitality and instinct. 77. Humour and pathos are ever present in the essays of: (a) Bacon (b) Addison (c) Hazlitt (d) Lamb Ans. (d) Lamb was a master of humor and pathos, both of which are blended together in his essays, as they did in his life. This is his unique style, which we rarely find in any other literary work of English literature. 78. ’’Anagnorisis” in Aristotelian criticism signifies: (a) downfall of the tragic hero (b) tragic flaw (c) a change from ignorance to knowledge (d) error of judgement ______________________ Ans. (c) Anagnorisis in a literary work, the startling discovery that produces a change from ignorance to knowledge. It is discussed by Aristotle in poetics as an essential part of the plot of a tragedy, although anagnorisis occurs in comedy, epic and at a later date in novel as well. 79. Who said that ”it was his ambition to bring philosophy out of closets and libraries, schools and colleges to dwell in clubs and assemblies, at tea-tables and coffee houses”? (a) Addison (b) Dr. Johnson (c) Dryden ___________(d) Pope _____________ Ans. (a) Joseph Addition’s essays should not be read as profound pieces, they are meant as vehicles of instruction with two particular intention. In September 10 he wrote, "It was his ............ and coffee houses. 80. Who holds the view that ’’Beauty engenders a peaceful and pleasurable state of mind called synnesthesis”? (a) T.S. Eliot (b) LA. Richards (c) F.R. Leavis (d) None of these Ans. (b) Ivor Armstrong Richards, poet, dramatist speculative, philosopher, psychologist and semanticist is among the first of 20th century critics to bring to English criticism a scientific, precision and objectivity. In ’The Principles of Literary Criticism’ (1924), Richards expresses the view that "Beauty engenders a peaceful and pleasurable state of mind called synnesthesis”.

51

YCT

81. The Civil War in England led to the closing of theatres in the year: (a) 1640 (b) 1641 (c) 1642 _____________(d) 1645 _____________ Ans. (c) The English Civil War was a series of civil wars and political machinations between Parliamentarians and Royalists, mainly over the manner of England’s governance and issues of religious freedom. It was part of the wider wars of the three Kingdoms. In September 1642, just after the first English Civil War had begun, the long parliament ordered the closure of all London theatres. 82. Yeats envisages the end of Christian era in: (a) "Prayer for My Daughter” (b) "Sailing to Bizantium" (c) "Bizantium" (d) "The Second Coming" Ans. (d) William Butler Yeats envisages the end of Christian era in his most masterful, highly regarded poem "The Second Coming". He was an Irish poet, dramatist, prose writer and one of the foremost figures of 20th century literature. In 1923 he was awarded the Nobel Prize in literature. 83. Who, amongst Shakespeare’s heroes, is described as ’’Bellona’s bridegroom”? (a) Ferdinand (b) Macbeth (c) Duke Orsino (d) None of these Ans. (b) Macbeth is described as "Bellona’s bridegroom". Bellona was the Roman goddess of war. Here the thane of Ross Praises Macbeth’s unsurpassed skill on battlefield, referring to him as "Bellona’s Bridegroom". 84. Who is charged of attempted rape of Miss Adela Quested in A Passage for India? (a) Fielding (b) Ronny (c) Dr. Aziz (d) Godbole Ans. (c) Dr. Aziz, fictional character, a humble Muslim surgeon in ’A Passage to India’ by E. M. Forster. Aziz represents the native Indian Community in conflict with the British ruling class. The central event of the novel is his trial for the alleged rape of a visiting Englishwoman, Adela Quested. 85. The Restoration Comedy was written for (a) the limited court society (b) an educated middle class (c) the groundlings (d) all sections of society Ans. (a) ’Restoration Comedy’ is English comedy written and performed in the restoration period from 1660 to 1710. It depicts a stylish society, mainly the upper and middle classes, its focus is on elegance, with character of fashion and rank. Its topics are social intrigue mainly marital and sexual. UP GDC Asst. Prof. Exam. 2017

86.

In which of his poems W.B. Yeats refers to himself as an old man having a heart ’’sick with desire I and fastened to a dying animal”? (a) "The Second Coming" (b) "Sailing to Byzantium” (c) "Prayer for My Daughter" (d) None of these Ans. (b) ’Sailing to Byzantium’ was written in 1926 and included in Yeats greatest single collection, 1928’s ’The Tower’. ’Sailing to Byzantium’ is Yeats definitive statement about the agony of old age and the imaginative and spiritual work required to remain a vital individual even when the heart is "fastened to a dying animals”. 87. What is the sin of the Duchess of Malfi in the eyes of her brothers? (a) Disobeying the King’s orders (b) Marrying her steward (c) Wedding a man of higher rank (d) None of these Ans. (b) The Duchess of Malfi - The Protagonist, sister to Ferdinand and the Cardinal. At beginning she is a widow whose brothers take every precaution to keep from marriage, though later she secretly marries Antonio. Antonio is the steward of the Duchess of Malfi’s palace. 88. Who is the real benefactor of Pip in Dickens’ Great Expectations? (a) Miss Havisham (b) Estella (c) Mr. Jaggers (d) Magwitch Ans. (d) Magwitch is eventually revealed to be Pip’s banefactor, even though Pip has long assumed that Miss Havisham is the one funding his education, after Magwithch started making money as a sheep farmer, he became obsessed with using the money to transform Pip into a gentleman. 89. Lamb narrates the story of Mr. Billet in the essay: (a) "Poor Relations" (b) "Dream Children" (c) "In Praise of Chimney Sweepers" (d) "The Old and the New School Masters" Ans. (a) Lamb narrates the story of Mr. Billet in the essay ’Poor Relation’. The opening of this essay is full of wit but we are more inclined to cry than to laugh when we read the story. In this essay, Lamb narrates about Mr. John Billet, a relative of Lamb’s father. 90. I.A. Richards reiterates the distinction between the referential and emotive use of language in: (a) The Meaning of Meaning (b) Principles of Literary Criticism (c) The Philosophy of Rhetoric (d) Science and Poetry

52

YCT

Ans. (a) Sir Roger de Coverley, fictional character, Ans. (b) In his critical work ’Principles of Literary Criticism’ (1924), I. A. Richards reiterates the devised by Joseph Addition, who portrayed him as the distinction between the referential and emotive use of Ostensible author of papers and letters that were published in Addition and Richard Steele’s influential language. periodical ’The Spectator’. As imagined by Addition, Richards writes that both referential and emotive Sir Roger was a baronet of Worcestershire, also a language can hold truth, but the types of truth are member of the fictitious Spectator Club. different. 91. The expression ’naughty nineties’ refers to the 96. Who described Mona Lisa as ’older than the decade: rocks among which she sits’? (a) 1690s (b) 1790s (a) Coleridge (b) John Ruskin (c) Ben Jonson ________(d) Walter Pater _______ (c) 1890s _____________(d) 1990s _____________ Ans. (c) The expression ’naughty nineties’ refers to the Ans. (d) Walter Pater described ’Mona Lisa’ as ’older than the rocks among which she sits. He was an decade of the 1890s. It is an American nostalgic term. English essayist and literary critic, novelist and Despite the term, the decade was marked by an economic crisis, which greatly worsened when the associated with the Pre-Raphaelite Brotherhood and panic of 1893 set off a widespread economic was one of the most vociferous defender of the ’’art depression in the United States that lasted until 1896. for art’s sake”. 92. The land of the Lotus Eaters symbolizes 97. The conceit ’’Fruits of much grief they are, (a) the life of mirth and celebration emblems of more” occurs in: (b) the life of activity (a) ”A Valediction : Forbidding Mourning” (c) rejection of activity in favour of a trance like (b) ’’The Canonization” existence (c) ”A Valediction : Of My Name” (d) None of these ____________________________ (d) ”A Valediction : Of Weeping” ______________ Ans. (c) The land of the Lotus Eaters symbolizes Ans. (d) ’A Valediction : Of Weeping’ is one of the rejection of activity in favour of a trance like Donne’s best poem. In it he uses memorable images to speak about a relationship and its two partners. This existence. In Greek mythology, the lotus-eaters were a poem is also a great example of Donne’s use of conceits. race of people living on an island dominated by the lotus tree. The lotus fruits and flowers were the The Conceit occurs in the first stanza of the poem. primary food of the island and were a narcotic apathy. 98. G.B. Shaw’s St. Joan was produced in the year: (a) 1923 (b) 1915 93. Ben Jonson’s Volpone is a: (a) Tragi-Comedy (c) 1920 _____________(d) 1922 _____________ (b) Romantic Comedy Ans. (a) Saint Joan is a chronicle play in six scenes (c) Comedy of Humours and an epilogue. G.B. Shaw produced in 1923 and (d) Sentimental Comedy ______________________ published in 1924. It was inspired by the canonization of Joan of Arc in 1920, nearly five centuries after her Ans. (c) ’Volpone’ is a Comedy of Humours, Ben death in 1431. ___________________________________ Jonson first produced in 1605-1606, drawing on elements of city comedy and beast fable. A merciless 99. The locale of Graham Greene’s The Power and satire of greed and lust, it remains Jonson’s most the Glory (1940) is: performed play and it is ranked among the finest (a) Britain (b) Mexico Jacobean era comedies. ___________________________ (c) America __________(d) France ____________ 94. Which of the following novels is concerned with Ans. (b) ’The Power and the Glory’ is a 1940 novel by the conflict between two competing impulses : British author Graham Greene. This novel tells the instinct of civilization and the instinct of savagery? story of a renegade Roman Catholic ’Whisky Priest’ (a) The Lord of the Flies living in the Mexican state of Tabasco in 1930, at time (b) The Power and the Glory when the Mexican government was attempting to (c) The Rainbow suppress the Catholic Church. (d) The Great Expectations ___________________ 100. Who defined the motto of the periodical essay Ans. (a) ’The Lord of the Flies' is a 1954 novel by ”to temper wit with morality and to enliven Nobel Prize winning British author William Golding. morality with wit”? At an allegorical level, the central theme is the (a) Defoe (b) Steele conflicting human impulses towards civilization and (c) Cowley (d) Addison social organization, living by rules, peacefully and in Ans. (d) Addition’s boast in the spectator no. 10 that harmony and towards the will power. there were already 3000 papers distributed everyday 95. Who introduced the character of Sir Roger first with some 60,000 readers. He promised this sizable in The Coverley Papers? thoughtful and above all new relationship that he endeavor on their behalf ”to enliven morality with wit, (a) Addison (b) Steele (c) Lamb (d) Dryden and temper wit with morality. UP GDC Asst. Prof. Exam. 2017

YCT

(c) denounces woman 101. Who, according to L.C. Knights, characterizes (d) None of these __________________________ the earliest instance of ’dissociation of sensibility’? Ans. (b) Russel in his essay ’Ideas that have Harmed (a) Bacon (b) Milton Mankind’ expresses his views, he talks about Pride on (c) Donne ___________(d) Johnson __________ Sex and class too. According to him, men believe that Ans. (c) According to L.C. Knights, John Donne woman are irrational, foolish and unwise. Russel does characterizes the earliest instance of ’dissociation of not agree with common people, he denounces sensibility’. Eliot used the term to describe the manner superiority of male sex. by which the nature and substance of English poetry 106. In which of the following poems of W.B. Yeats changed ’’between the time of Donne or Lord Herbert do these lines occur? of Chebury and the time of Tennyson and Browning. ”It’s certain that fine women eat A crazy salad 102. Lovers are compared to ’’stiff twin compasses” with their meal. Whereby the Horn of Plenty is in the poem: undone.” (a) ”A Valediction : Of Weeping” (a) ’’The Second Coming” (b) ”A Valediction : Forbidding Mourning” (b) ”A Prayer for My Daughter” (c) ”A Valediction : Of My Name” (c) ’’Bizantium” (d) ”A Valediction : Of The Book” ____________ (d) ’’Sailing to Bizantium” ___________________ Ans. (b) In his poem ’A Valediction : Forbidding Ans. (b) These lines occur in W. B. Yeats poem ’A Mourning’, John Donne uses the simile of ’’stiff twin Prayer for My Daughter’. He wrote this poem in 1919, compasses” to describe two lovers who are physically shortly after his daughter’s birth and World War II. So parted, but united in their souls. The woman like fixed the ongoing unsettling feel is visible in the foot which remains firm in the centre. While the man background and the poet’s mind. The poem appeared is like ’other foot’ roaming for and wide. for the first time in his poetry collection, Michael 103. Which Greek dramatic device is used in T.S. Robartes and the Dancer in 1921. Eliot’s The Family Reunion? 107. Which of the following novels of Jane Austen (a) The Furies (b) Chorus begins with the sentence: (c) Anagnorisis _______(d) None of these ______ ”It is a truth universally acknowledged that a Ans. (*) ’The Family Reunion’ is a play by T.S. Eliot, single man in possession of a good fortune must first published in 1939. In this play, Eliot uses be in want of a wife.” following Greek dramatic devices(a) Pride and Prejudice (b) Emma • The Furies - The protagonist, Harry is pursued by (c) Mansfield Park (d) None of these ______ the Eumenides - the avenging furies. Ans. (a) Pride and Prejudice begins with the given • Chorus - Harry’s uncles and aunts occasionally sentence. It is a romantic novel published in 1813 by detach themselves from the action and chant a Jane Austen. This novel is set in rural England in commentary on the plot, in the manner of a Greek early 19th century and it follows the Bennet family, chorus. which includes five very different sisters. • Anagnorisis - anagnorisis is the point in a play in 108. The theme of ’’Dream Children” is: which a principal character recognizes or discovers (a) Reconciliation (b) Regret and loss true identity or the true nature of their own (c) Sense of fulfillment (d) Forgiveness _______ circumstances. It happens with Harry also with the Ans. (b) The theme of Lamb’s essay is regret and loss help of his aunt Agatha. : regret for unfulfilled joy, unfulfilled love, lost hope, 104. Who said the following? lost opportunity and lost joys of life. There are three ’’All I want is to answer to my blood, direct, topics describing the theme of regret and loss at work without fribbling intervention of mind, or in this essay. moral, or what-not.” 109. ’’Bliss was it in that dawn to be alive. But to be (a) E.M. Forster (b) Rudyard Kipling young was very heaven!” (c) D.H. Lawrence (d) Graham Greene These lines occur in: Ans. (c) Following lines are expressed by D. H. (a) ’’Tintem Abbey” Lawrence. Sometimes intellectual aversion in the (b) ’’Laodamin” artist passes over into a conscious creed. It was so (c) ’’The French Revolution” with D. H. Lawrence. My great religion is a belief in (d) ”Ode on Intimations of Immortality” ________ the blood, the flesh as being wiser than the intellect. 105. Russel in his essay ’’Ideas that have Harmed Ans. (c) These lines occur in ’The French Revolution’. It is a poem written by William Wordsworth. Mankind” Wordsworth was one of those poets who greeted the (a) glorifies man French Revolution with enthusiasm. (b) denounces superiority of male sex 54 YCT UP GDC Asst. Prof. Exam. 2017

110. Which of the following plays of Shakespeare has Ans. (c) W. H. Auden ’In Memory of W. B. Yeats’ a masque? says that ’The words of the dead man are modified in (a) Macbeth (b) The Tempest the guts of the living’. ’In Memory of W. B. Yeats’, (c) Twelfth Night (d) Richard II Auden discusses the death of Y. B. Yeats. Ans. (b) Traditionally Shakespeare’s plays are 116. ’’The Sleep Walking Scene” occurs in categorized as Comedy, History, Romance and (a) Othello (b) Twelfth Night Tragedy. One of these additional categories of (c) The Tempest ______(d) Macbeth __________ Shakespeare plays is masque. Shakespeare’s plays Ans. (d) ’The Sleep Walking Scene’ is a critically generally accepted as including masques are : ’Henry celebrated scene from William Shakespeare’s tragedy VIII’, ’A Midsummer Night’s Dream’, ’Romeo and Macbeth. The first scene in the tragedy’s 5 th act, the Juliet’ and ’The Tempest’. sleepwalking scene is written principally in prose, and 111. Which of the following novels highlights the follows the guilt-wracked, sleepwalking lady Macbeth corruption of the church and the government? as she recollects horrific images and impressions from (a) A Passage to India her past. (b) The Lord of the Flies 117. The title A Passage to India has been taken (c) The Power and the Glory from: (d) The Great Expectations __________________ (a) the poem of Pope Ans. (c) Graham Greene’s ’The Power and the Glory’ (b) the poem of Frost (1940), the story of a fugitive ’whisky priest’ in 1930s (c) the poem of Walt Whitman Mexico, is a short Pathos-laden novel about religious (d) the poem of Emily Dickinson ______________ persecution after the Mexican Revolution. Ans. (c) The title of ’A Passage to India’ is a reference 112. Who influenced most the satirical works of to Walt Whitman’s poem ’A Passage to India’. In the Pope? poem Whitman takes his reader on an imaginary (a) Horace (b) Juvenal journey through time and space. (c) Boileau __________(d) None of these ______ 118. ’’Get stewed Ans. (a) Pope learned Latin and Greek in childhood Books are a load of crap.” and all his life wrote imitations and translations of Who said the above lines? classical authors such as Homer, Virgil, Horace, (a) Philip Larkin (b) W.B. Yeats Quintilian and Ovid. Horace influenced most the (c) W.H, Auden (d) T.S. Eliot _________ satirical works of Pope. Ans. (a) ’’Get stewed Books are a load of crap" are the 113. ’’Nothing happens, nobody comes, nobody goes, lines from the Philip Larkin poem ’A Study of it’s awful.” This line occurs in: Reading Habits’, written in August 1960 and (a) Look Back in Anger published in Larkin’s 1964 volume ’The Whitsun (b) The Birthday Party Wedding’. ____________________________________ (c) Waiting for Godot (d) None of these __________________________ 119. Which of the following plays of Shakespeare observes three dramatic unities? Ans. (c) This line occurs in ’’Waiting for Godot”. It is (a) Macbeth (b) The Tempest a play by Samuel Beckett that premiered in France in (c) Richard II ________ (d) Twelfth Night January 1953. The play, Beckett’s first explores the Ans. (b) In the given plays Shakespeare observes meaning and meaninglessness of life through its three dramatic unities in ’The Tempest’. The classical repetitive plot and dialogue. ’Waiting for Godot’ is an unites, Aristotelian unities or three unities represent a enigmatic but very significant play. prescriptive theory of dramatic tragedy that was 114. The policemen in The Power and the Glory are th introduced in Italy in the 16 century. The three called: unities are(a) redshirts (b) yellowshirts (c) blackshirts ________(d) greenshirts ________ 1. Unity of Time 2. Unity of Place Ans. (a) ’The Power and the Glory’ by Graham 3. Unity of Action. Greene is a novel. This novel is unified partially by 120. What is the sub-title of Pride and Prejudice? the failing efforts of several characters to (a) The first Rejection (b) The first Expectation communicate significantly with one another. (c) The first Expression (d) The first Impression Greene uses the symbol ’redshirts’ to the policemen in Ans. (d) "Pride and Prejudice" was first written in the novel. 1797 under the title "First Impressions". It was later 115. ’’The words of the dead man are modified in the revised and published under the title "Pride and guts of the living”, said: Prejudice" in 1813. First impressions do play on (a) T.S. Eliot (b) W.B. Yeats important role in the novel. (c) W.H. Auden (d) Philip Larkin UP GDC Asst. Prof. Exam. 2017

55

YCT

RPSC Assistant Professor Exam. 2020

ENGLISH-I Solved Paper Direction: Fill in the blanks (1-6) Choosing correct idiomatic expression from those given: 1. were invited for the interview and soon there was a mayhem. (a) Too many irons in the fire (b) Every John, Jack and Jerry (c) All and sundry _____(d) Each and every _________________________ Ans. (c) : In the given blank space, the correct idiomatic expression is ’All and sundry’ which means everyone. Ex: "He has borne a lot of unfair criticism from all and sundry. Thus, appropriate answer is (c). 2. I don’t like what the new management has done, but to , sales have certainly improved. (a) play to the gallery (b) warm oneself into favour (c) give the devil his due _____(d) know which side the bread is buttered ______ Ans. (c) : In the given blank space, the correct idiomatic expression is ’give the devil his due’ means-If someone or something generally considered bad or undeserving has any redeeming features these should be acknowledged. Thus, the correct option is (c). ________ 3. There is certainly more in the restaurant since they extended the dining area. (a) bone of contention (b) holding the head high _____(c) elbowroom ______(d) snapping one’s fingers Ans. (c) : In the given blank space ’the correct idiomatic expression is ’elbow room’. There is certainly more elbow room in the restaurant since they extended the dining area. Elbow room : 1) enough space in which to move easily or space to move around in. 2) freedom to do what you want to do or need do in a particular situation. Thus, the correct option is (c). 4. ”To stir up a hornet’s nest’ means . (a) to do a disservice to others (b) to exit the hostility of others (c) to be totally at a loss _____(d) to be bitten by bugs ______________________ Ans. (b) : In the given blank space, the correct idiomatic expression is "To stir up a hornet’s nest’ which means to exit the hostility of others. Explain- Hostility is unfriendly or aggressive behaviour towards people or ideas. Thus, the correct option is (b). 5. ’A raw deal’ is . (a) an unripe statement (b) early stages of an assignment (c) unjust treatment _____(d) undecided contract ______________________ Ans. (c) :In the given blank space, the correct idiomatic expression is ’A raw deal’ which means unjust treatment; To receive unfair or poor treatment in a particular situation. Ex: Mandly really got a raw deal at that job. They passed her over for so many promotions. Thus, the correct option is (c). RPSC Assistant Professor Exam. 2021

[Exam Date : 06.10.2021

6.

An ’open question’ is . (a) A permanent problem (b) A question without any generally agreed answer (c) A question put to all _____(d) A permanent problem ____________________ Ans. (b) : In the given blank space, the correct idiomatic expression is 'open question' which means 6a question without any generally agreed answer’. Thus, the correct option is (b). ______________________ Direction: Identify the basic sentence pattern of the following sentences by choosing from the given options (7-9): 7. I envy you, your beautiful garden. (a) SVC (b) SVO (c) SVOO ____________(d) SVOC ___________ Ans. (c) : Envy is mostly negative feeling of desire for something that someone else has and you do not have. It is always followed by the person or thing that envied. 2

envy

sub(s) Verb (v)

you, Indirect object (o)

your

beautiful garden

Direct object (o)

So this construction is based on SVOO pattern. Subject + Verb + Indirect object + Direct object 8.

A little kindness goes a long way. (a) SVC (b) SVA (c) SVO ______________(d) SVAO ___________ Ans. (b) : The basic sentence pattern of the sentence is, ’a little kindness goes a long way’. SVA- [Subject + verb + Adverb] Example are - John/sat/up.(SVA). Thus, the correct option is (b)._____________________________________ 9. I have always found him friendly. (a) SVO (b) SVC (c) SVOC ____________(d) SVOO ___________ Ans. (c) : The basic sentence pattern of the sentence is SVOC. Thus, the correct option is (c). I have always found him friendly. [Subject + verb + object + complement] (SVOC). ______ Direction: change the sentences as stated against each (10-21). The options are given, choose the grammatically correct one. 10. Walk carefully lest you should fall, (change into compound) (a) Walk carefully or you will fall. (b) Carefully walking will avoid the fall. (c) fall you will without carefulness. _____(d) If you do not walk carefully you might fall. Ans. (a) : "Walk carefully or you will fall" is the correct compound sentence for the given sentence. Explain- A compound sentence contains at least two independent clauses. These two independent clauses can be combined with a comma and a coordinating conjunction or with a semicolon. ____________________ 11. On being challenged they ran away, (change into complex) (a) They were challenged so they ran away.

56

YCT

(b) When they were challenged they ran away. (c) Challengingly they ran. _____(d) The ran because they were challenged. ______ Ans. (b) : When they were challenged they ran away (complex sentence) Explain- A complex sentence contains at least one independent clause and at least one dependent clause, when they were challenged they ran away dependent clause independent clause 12. I believe that God exists, (change into a simple sentence) (a) I believe and God exists. (b) God exists because we believe it. (c) I believe in God’s existence. _____(d) God exists when we believe it._____________ Ans. (c) : The sentence ’I believe that God exists’ can be converted into a simple sentence by writing it as ‘I believe in God’s existence’ because, Coordinating Conjunctions are not used in simple sentences. Instead it has a single main verb in whole sentence. Hence, the correct option is (c). _____________________ 13. Someone has made a hole in the fence, (change into passive voice) (a) A hole had been made in the fence (by someone) (b) A hole has been made in the fence. (c) The fence had been holed. _____(d) Someone holed the fence. _________________ Ans. (b) : [Obj + has/have + been + V 3 + sub] is passive voice construction for the above sentence, structure of Present Perfect Tense in Active Voice. Structure - (Sub + has/have + V 3 + Obj + ....). Thus, the correct Passive Voice- A hole has been made in the fence. Doer of the action will not be used in passive form because it’s not relevant. ___________________________ 14. We must leave the decision to your own judgment, (change into passive voice) (a) The decision has to be left to you to judge. (b) The decision is in your hands. (c) It is for you to decide what to do. (d) The decision must be left to your own _________ judgment. ______________________________ Ans. (d) : This sentence has given in the Modal verb word ’must’ which show ’compulsion’ in Active voice. Structure in Active [Subject + Modal + Vi + object] Passive St. [Obj. + Modal + be + V 3 + (by + Agent)] Thus, the correct passive voice is- ’ The decision must be left to your own judgment.’ ______________________ 15. Things were sent flying in all directions by the explosion, (change into active voice) (a) The explosion sent things flying in all directions. (b) Things in all directions flew with the explosion. (c) The explosion had sent flying things in all directions. _____(d) Flying things were sent in all directions. Ans. (a) : The sentence, in passive voice (has verb of Past Continuous Tense) will be converted into active voice in verb of Simple Past Tense. [Sub + Verb 2 + Object.] The correct active voice of the sentence is ’ The explosion sent things flying in all directions.’ Thus, option (a) is correct. RPSC Assistant Professor Exam. 2021

16.

He explained, "I know the place well because I used to live here.” (change into indirect speech) (a) he said that he had lived here and because of that he knew the place well. (b) he explained he knew the place well as he used to live here. (c) he explained that he knew the place well because he used to live there. (d) he explained that he knew the place well as he _________had lived here. __________________________ Ans. (c) : He explained that he knew the place well because he used to live there. (Indirect speech) When the reporting a principle verb is in Past Tense, all Present Tense of the direct speech are changed into corresponding Past Tense after converting into indirect speech, and ’here’ will be changed into ’there’. Thus, the correct option is (c). ______________________ 17. ’If you get the job will you move to Delhi?’ He asked, (change to indirect speech) (a) He asked if whether he got the job he would moved to Delhi. (b) He asked whether, if I got the job, I’d move to Delhi. (c) He asked if he got the job he moved to Delhi. (d) He asked if I got the job would I moved to _________Delhi. _________________________________ Ans. (b) : The most appropriate indirect speech is - He asked whether, if I got the job, I’d move to Delhi. • ’’Whether” is used as a conjunction for reporting verb and reported speech where "if’ is condition for the given sentence . Hence, the correct answer is option (b). ______________ 18. My friends warned me not to leave my car unlocked, (change into direct speech) (a) "You cannot leave without locking your car." said my friends. (b) "You had better not leave your car unlocked," said my friends. (c) "Unlocked, no never," said my friends about my car. (d) My friends said, "I had not left my car _________unlocked. ______________________________ Ans. (b) : "You had better not leave your car unlocked," said my friends. (Direct speech) We use “had better” plus infinitive (bare) without “to” to give advice. It is used to give advice about the present or future. So, this type of structure is treated as an Imperative sentence while transforming into reported speech and vice-versa. Structure: Had better + Not + verb Change Direct speech into Indirect speech same steps1. IDS (R.V.) change > Sense (R.S.) change 2. Connective > To 3. Change in Tense -> NO 4. Change on pronoun -> Yes Thus, option (b) is correct. _________________________ 19. There is nothing particularly impressive about this picture, (change into interrogative) (a) Is there anything particularly impressive about this picture? (b) Is thee nothing that impresses us about this picture? (c) Are you not impressed by this picture? (d) Does this picture particularly impress you?

57

YCT

Ans. (a) : Is there anything particularly impressive about this picture? (Interrogative sentence) Explain - ”An interrogative sentence is formed by according the words of its declarative counterpart: While transforming into interrogative an assertive sentence is changed into bare interrogative. Note : nothing is changed into anything. 20. Virtue is its own reward, (change into interrogative) (a) Has not virtue its own reward? (b) What is virtue’s reward? (c) is not virtue its own reward? _____(d) Virtue is own reward, Is not? _____________ Ans. (c) : Is not virtue its own reward? (Interrogative sentence) Rule In Transformation of Sentence, if in Affirmative Assertive Sentence converted into Negative Interrogative sentence. ___________________________ 21. Everyone is dependent, (change to negative) (a) Everyone is not independent. (b) Nobody is independent. (c) Someone is dependent. _____(d) Nothing depends. ______________________ Ans. (b) : If we find the word "Every” in an Affirmative sentence, we will write the word ”No" instead of that word [Every] and make the opposite word of verb and adjective at the time of the negative sentence. Example Affirmative - Everybody is dependent. Negative - Nobody is independent. Thus, the correct option is (b). Direction: Answer the following questions by choosing the correct answer from the options given: 22. Aristotle in his Poetics states that the poet should prefer ____in his narration. (a) improbable possibilities (b) probable impossibilities (c) possible probabilities _____(d) impossible probabilities _________________ Ans. (b) : The correct answers is in the blank space option is (b) [Probable Impossibility]. Probable Impossibility refers to a situation that is impossible to happen in the real world, but is probable in the universe of imaginary events that is assumed to exist. This lets the reader or audience suspend her judgment concerning the impossibility of the narrative [Poetics (XXIV)]. *Note: RPSC has chosen option (a) as a correct answer but option (b) probable impossibilities is correct answer. 23. Command of which poetic device is the mark of a genius according to Aristotle? (a) Irony (b) Metalepsis _____(c) Metaphor _________(d) Justice __________ Ans. (c) : Command of ‘Metaphor’ poetic device is the mark of a genius according to Aristotle. Aristotle wrote in his poetics, “To be a master of metaphor, is the greatest thing by far. It is the one thing that cannot be learnt from others, and it is also a sign of genius.” ______________________________________ 24. Which of the following is not a popular term introduced by Horace in his Ars Poetica? (a) deus ex machina (b) in medias res (c) aboro (d) at picture RPSC Assistant Professor Exam. 2021

Ans. (a) : "Ars Poetica" or "The Art of Poetry" is a poem written by Horace 19B.C, in which he advises Poets on the art of writing poetry and drama. The Ars Poetica has "exercised a great influence in later ages on Eurpean literature, notably on French drama" and has inspired poets and authors since it was written. It has been used in literary criticism since the Renaissance. The popular terms introduce by Horace in his ’Ars Poetica are ‘in medias res’, ‘Ab oro’, and ‘at pictura’. Thus, option (a) is correct. Note: deus ex machina, (Latin : “God from the machine”) a person or thing that appears or it introduced into a situation suddenly and unexpectedly and provides an artificial or contrived solution to an apparently insoluble difficulty. ____________________ 25. What does Horace mean by the phrase ’Purple Prose’? (a) Royal language (b) Decayed language _____(c) Dead language _____(d) Flowery language Ans. (d) : A generally prejorative term for writing or speech characterized by Ornate, Flowery, or Hyperbolic language is known as Purple Prose. Bryan Garner notes that Purple Prose ” derives from the Latin phrase purpureus pannus, which appear in the (Ars poetica) of Horace (65-68B.C.)" (Gamer’s modem Anerican usage, 2009). Thus, the correct option is (d). 26. According to Longinus which aspect does not lead to faulty language? (a) puerility (b) frigidity _____(c) simplicity _________(d) turgidity _________ Ans. (c) : According to Longinus ’simplicity' is not faulty language. Longinus Critically applauds and condemns certain literary work as promotes an "Elevation of Style and an essence of ’Simplicity'. _____ 27. The Rasa theory is stated in which chapter of the Natya Shastra? (a) 14 (b) 18 (c) 6 ________________(d) 1 _______________ Ans. (c) : The Rasa Theory has a dedicated section (chapter-6) in the Sanskrit text ’Natya Shastra’, an ancient scripture from the 1st millennium BCE attributed to Bharata Muni . Thus, the correct option is (c). _____________________ 28. Sahrdaya according to Bharat is ____. (a) the human soul (b) a mental state (c) a model spectator type of Rasa _____(d) a type of Rasa ____________________ Ans. (c) : Sahrdaya according to Bharat is a Type of Rasa, According to Bharata’s Natyashastra there is eight fundamental feeling or mental states referred to as Sthayibhvas which can be experienced by human beings. They are Delight (Riti) Laughter (Hasya), Sorrow (Shoka), Anger (Krodha), Heroism (Utsaha), Fear (Bhaya), Disgust (Jugupsa) and wonder (Vismaya). 29. The word ’ekstasis’ finds a place in the critical canon of (a) Longinus (b) Aristotle _____(c) Horace ___________(d) Neoptolemous Ans. (a) : The word ‘ekstasis’ finds a place in the critical canon of ‘Longinus’. Ecstasy comes from the Greek word 'ekstasis' formed of the prefix ek (outside or beyond) and stasis ('standing', 'Position'),

58

YCT

30.

In whose opinion the ’theatre is not an hospital’ while reacting to the Aristotelian concept of Catharsis? (a) J. Hillis Miller (b) J.K. Atkins _____(c) Michael Foucault (d) F.L, Lucas _______ Ans. (d) : F. L. Lucas remarks ” The theatre is not a hospital.” However many scholars do not accept this view of art as a moralizing agent. F.L. Lucas in his study of Aristotle’s theory of tragedy notes that by the term ’’Catharsis” in the sense of ’’purgation” is to be conceived ” not in the modem, but in the older wider English sense which included the partial removal of excess humours. _________________ 31. Who, fervently supporting poetry against spurious charges of corrupting men’s mind, says we ’’need not say that poetry abuseth man’s wit, but that man’s wit abuseth poetry”? (a) Henry Howard (b) Sir Walter Raleigh _____(c) Sir Philip Sidney (d) Gabriel Harvey Ans. (c) : Sir Philip Sidney, fervently supporting poetry against spurious charges of corrupting men’s mind, says “poetry abuseth man’s wit, but that, man’s wit abuseth poetry”. _______________________________ 32. Which critic was of the opinion that ’’the greatness of literature cannot be judged solely by literary standards, though we must remember that whether it is literature or not can be determined only by literary standards” (a) Matthew Arnold (b) T.S. Eliot _____(c) LA. Richards ______(d) Allen Tate _______ Ans. (b) : T.S. Eliot Quotes in his essay "Religion and Literature" "The greatness of literature cannot be judged solely by literary standards though we must remember that whether it is literature or not can be determined only by literary standards.” Thus, the correct option is (b). _____________________ 33. The literary principle of ’paradox’ was first fore grounded by ? (a) Yvor Winters (b) Cleanth Brooks _____(c) L.C. knight ________(d) Kenneth Burke Ans. (b) : The literary principle of ’paradox’ was first fore grounded by ’Cleanth Brooks’. Cleanth Brooks is an active member of the new criticism movement, outlines the use of reading poems through paradox as a method of critical interpretation. In his essay ’’The Language of paradox", Cleanth Brooks emphasizes how the language of poetry is different from that of sciences, claiming that he is interested is own seeing that the paradoxes spring from the very nature of the poet’s Language. It is a language in which the connotations play as great a part as the denotations. _________________________ 34. Which commentator calls Shakespeare’s Hamlet ”an indifferent play, the lines but mean”? (a) Abraham Wright (b) T.S. Eliot (c) Maynard Mack (d) John Calvin Ans. (a) : Hamlet has not always been so highly regarded as it is today. Abraham Wright put it in the 1630s, Hamlet was “an different play, the lines but mean.” He mean to say that the play was not very good, boring and tedious. Thus, the correct option is (a) RPSC Assistant Professor Exam. 2021

35.

’Markedness”, a state of standing out as nontypical was a formalist concept categorised by ? (a) Mikhail Bakhtin (b) Vladimir Propp _____(c) Boris Eichenbaum (d) Roman Jakobson Ans. (d) : Markedness theory, as Roman Jakobson conceived it, is a qualitative theory of oppositional binary relations. Marked and Unmarked Terms are very useful when analysis the representation of gender in text as words which are marked are these which are ascribed less prestige than the standard form of a word. ________ 36. Northrope Frye in his first book, Fearful symmetry reinterprets the poetry of ? (a) Thomas Chatterton (b) George Cumberland (c) William Blake _____(d) Thomas Gray _________________________ Ans. (c) : Fearful symmetry : A study of ’William Blake’ is a 1947 book by Canadian literary critic ’Northrope Frye’ whose subject is the work of English poet and visual artist William Blake. The Book has been hailed as one of the most important contributions to the study of William Blake and one of the first that embarked on the interpretation of many of Blake’s most obscure works. 37. G. Wilson Knight’s The Wheel of Fire is a collection of essays on ? (a) Shakespeare’s plays (b) T.S. Eliot’s poetry (c) Dante’s The Divine Comedy _____(d) Goethe’s Faust _________________________ Ans. (a) : G. Wilson Knight’s The Wheel of Fire is a collection of essays on Shakespeare’s plays, first published in 1930 by oxford university press with an introduction by T.S. Eliot. George Richard Wilson Knight (1897-1985) was an English literary critic and academic, known particularly for his interpretation of mythic content in literature, and ’The Wheel of Fire (1930)’, a collection of essay on Shakespeare’s play. Hence, the correct answer is option (a).______________ 38. Who is the author of the critical treatise A Philosophical Inquiry into the Origin of our Ideas of the Sublime and Beautiful? (a) Immanuel Kant (b) Edmund Spenser _____(c) Edmund Burke _____(d) S.T. Coleridge Ans. (c) : ’A Philosophical Enquiry into the Origin of our Ideas of the Sublime and Beautiful’ is a 1757 treatise on aesthetics written by ’Edmund Burke. It was the first complete philosophical exposition for separating the beautiful and the sublime into their own respective rational categories. It attracted the attention of prominent thinkers such as Denis Diderot and Immanuel Kant. 39. Leavis’ New Bearings in English Poetry and Revaluation show the strong influence of which poet on the writer? (a) Milton (b) T.S. Eliot _____(c) D.H. Lawrence (d) William Wordsworth Ans. (b) : Leavis’ criticism falls into two phases. In the first, influenced by T.S. Eliot, he denoted his attention to English verse. In New Bearing in English poetry (1932) he attacked English late Victorian poetry and proclaimed the importance of the work of T. S. Eliot, Ezra pound, and Gerard Manley Hopkins, emphasizing

59

YCT

wit and the play of intellect rather than late-Romantic sensuousness. In Revaluation : Tradition and Development in English poetry (1936), he extended his survey of English poetry back to 17th century. Hence, the current answer is option (b). ______________ 40. Pope’s "An Essay on Criticism" was fiercely attacked. However, one of the following did not do so. Who was is? (a) John Dryden (b) Thomas Rymer _____(c) John Dennis________(d) Jonathan Swift Ans. (a) : Pope’s ”An Essay on Criticism” was fiercely attacked by many critics like Rymer Dennis, and Swift but never criticized by John Dryden. ’An Essay on Criticism’ is one of the first major poems written by the English writer Alexander Pope (16881744), published in 1711. It’s famous quotations ”To err is human to forgive divine,” A Little knowledge is a dangerous thing," and "Fools rush in where angels fear to tread.” John Dennis was English critic and dramatist whose insistence upon the importance of passion in poetry led to a long quarrel with Alexander Pope. _______________ 41. The Chicago School of critics are also called ___. (a) Structuralists (b) Deconstructionists _____(c) Neo Aristotelians (d) New Historioists Ans. (c) : The Chicago School of critics also called Neo Aristotelians was developed in the 1920s, 30s, and 40s, at the university of Chicago. Chicago critics, also called the Chicago school group of pluralist essentially formalist American literary critics. This group contained Richard Mckeon, Elder Olson, Ronald Salmon Crane, Bernard Weinberg, and Norman Maclean. _____________ 42. Which of the notable dichotomy was not propounded by Ferdinand Saussure? (a) Langue and Parole (b) Text and context (c) Signifier and signified _____(d) Synchrony and diachrony _________________ Ans. (b) : ’Text and Context', a notable dichotomy was not propounded by Ferdinand de Saussure while ’Text and Context’ was propounded by ’George Steiner’. Ferdinand de Saussure views language as having an inner duality which is manifested by the interaction of the ’ Synchronic' and Diachronic’ , the 'Langue and Parole and the Signifier and Signified’. Thus, the correct option is (b). ______________________ 43. Match the following concepts with the writer who introduced them? A. Touchstone E. T.S Eliot method B. Intentional F. Cleanth Fallacy Brooks C. The Heresy of G. Matthew Paraphrase Arnold D. Dissociation H. Wimsatt & of Sensibility Beardsley A B C D (a) E F G H (b) F G H E (c) G H F E (d) H E F G RPSC Assistant Professor Exam. 2021

Ans. (c) : The correct sequence of concepts with the writer who introduced firstConcepts Writes A. Touchstone Method G. Matthew Arnold B. Intentional Fallacy H. Wimsatt and Beardsley c. The Heresy of F. Cleanth Brooks Paraphrase D. Dissociation Of E. T. S. Eliot Sensibility Thus, the appropriate answer is (c). 44. Where did Wordsworth first present his views on the nature and function of poetry? (a) In a separately brought out supplement (b) In Table Talk (c) In an Advertisement appended to the first edition of Lyrical Ballads. _____(d) In a rejoinder to Coleridge’s contention. _____ Ans. (c) : Wordsworth presents his view on the nature and fimction of poetry first in-The ’Advertisement’ appended to the first edition of the Lyrical Ballads (1798). It is a landmark of literary criticism. In this critical piece, Wordsworth throws much light on the nature and function of a poet. He is highly conscious of the distinction between a common man and man of genius. ____________ 45. Which of the following writers was noi associated with the Aesthetic Movement during the late 19th century? (a) Matthew Arnold (b) Walter Pater _ _ _ _ _(c) Oscar Wilde _______(d) William Morris Ans. (a) : Aesthetic (also the Aesthetic movement) was an art movement in the late 19th century which privileged the aesthetic value of literature, music and the arts over their socio - political function. According to Aestheticism, art should be produced to be beautiful, rather than to serve a moral, allegorical, or other didactic purpose, a sentiment exemplified by the slogan "art for art's sake.” Aestheticism originated in 1860s England with a radical group of artists and designers, including William Morris and Dante Gabriel Rossetti. It flourished in the 1870s and 1880s, gaining prominence and the support of notable writers such as Walter Pater and Oscar Wild. Mathew Arnold (1822-1888) was never associated with Aesthetic movement during 19th century. He was an English Victorian poet and literary and social critic. 46. The term that John Ruskin coined to describe the ascription of human emotions to inanimate objects and impersonal natural forces in called: (a) Sympathetic Fallacy (b) Naturalist Fallacy _ _ _ _ _(c) Affective Fallacy (d) Pathetic Fallacy Ans. (d) : ’Pathetic Fallacy’- The assignment of human feelings to inanimate objects, as coined by the Victorian literary critic John Ruskin. For him, a poet's tendency to project his or her emotions outward onto the working of the natural world was a kind of False vision. Today the term is used more naturally, and the phenomenon is usually accepted as a integral part of the poet's craft. It is related to Personification and anthropomorphism, but emphasizes the relationship between the poet’s emotional state and what he or she sees in the object or objects. _____ 47. Who said, "Poetry lifts the veil from the hidden beauty of the world, and makes familiar objects be as if they were not familiar"?

60

YCT

(a) S.T. Coleridge (b) P.B. Shelley _____(c) John Keats (d) William Wordsworth Ans. (b) : P.B. Shelley says these lines ’’Poetry lifts the veil from the hidden beauty of the world, and makes familiar objects be as if they were not familiar”. These lines are taken from the essay ’A Defence of Poetry written in 1821. 48. Keat’s idea that the poet submerges his/her senses and perceptions in the experience of another entity or even object corresponds to his theory of _ _ _. (a) dissociation of sensibility (b) unification of sensibility (c) pathetic fallacy _____(d) negative capability _____________________ Ans. (d) : Negative Capability is a phrase first used by Romantic poet John Keats in (1817) to explain the capacity of the greatest writers Keats countered this theory of Romanticism with the theory ’’empathy" where by the poet submerges of another entity or even object. "Ode on a Grecian Um" and empathetically conveys its experience and feelings. ________________ 49. Gerald Manley Hopkins described Alfred Tennyson pejoratively as a ____because of what Hopkin’s considered his ’uninspired poetry’. (a) Pseudo-Hellenists (b) Parnassian _____(c) Malapropist _______(d) Dionysianist______ Ans. (b) : Gerald Manley Hopkins described Alfred Lord Tennyson pejoratively as a Parnassian (or pamassism) was a French literary style that began during the positivist period of the 19fll century, occurring after Romanticism and prior to symbolism. The style was influenced by the author Theophile Gautier as well as by the philosophical ideas of Arthur Schopenhauer. 50. Gerald Manley Hopkins derived his concept of Inscape and Instress from the ideas of the medieval philosopher____. (a) Imago Dei (b) Duns Scotus _____(c) Thomas Merton (d) Walter Pater _ _ _ _ _ Ans. (b) : ‘Inscape’ and ‘Distress’ are complementary and enigmatic concepts about individuality and uniqueness derived by the poet ’Gerard Manley Hopkins from the ideas of the medieval philosopher ’’Duns Scotus.” Thus, the correct option is (B). ____________________ 51. John Dryden wrote his defense of ”An Essay on Dramatic Poesy” in response to an attack by (a) Lord Byron (b) Sir Robert Howard _____(c) Sir Walter Scott (d) Thomas Shadwell Ans. (b) : John Dryden wrote his defense of ‘An Essay of Dramatic Poesy’ in response to an attack by sir Robert Howard. The three ‘Persons of Wit and Quantity’ are sir Robert Horward (Crities), Charles Sackville (Eugenious) and sir Charles Sedley (Lisideius), while the fourth character Neander has been identified as Dryden himself. Thus, the appropriate answer is (B). _________ 52. Dryden in his ’’Essay on Dramatic Poesy” does not debate on. (a) the unities of time, place and action. (b) catharsis and poetic justice. (c) rigid distinctions between genres. (d) the use of rhyme in drama. RPSC Assistant Professor Exam. 2021

Ans. (b) : ’Essay on Dramatic Poesy’ by Dryden is an attempt to justify drama is a legitimate form of ’Poetry’ comparable to the Epic, as well as defend English Drama against that of the ancients and the French. In his essay, Dryden does not debate on Catharsis and Poetic Justice. Catharsis (a term coined by Aristotle) is the process of releasing strong or pent-up emotions through art. In literature, Poetic Justice is an ideal form of justice, in which the good characters are rewarded and the bad characters are punished, by an ironic twist of fate. Writers employ poetic justice to confirm to moral principles._____________________________________ 53. Dr. Samuel Johnson in his Lives of Poets is not critical of this poet and his work. (a) Milton’s "Lycidas" (b) Gray’s "Odes" (c) Abraham Cowley’s "Metaphysics" _____(d) Jonathan Swift’s "Satire" _________________ Ans. (c) : Samuel Johnson’s ‘The Lives of the Poets' is his last great work, Prefaces, Biographical and critical, to the works of the English poets. The lives are ordered chronologically by date of death, not birth and range in length from a few pages to an entire volume. • Among the major lives are those of Abraham Cowley, John Milton, John Dryden, Joseph Addision, and Alexander Pope. • His dislike of some of the poets who lives he wrote, such as John Milton, Thomas Gray and Jonathan Swift etc. • He was justly proud of The Life of Cowley, especially of its lengthy discussion of the 17th century Metaphysical poets, of whom Cowley may be considered the last representative. ________________ 54. ’’The poet’s mind is in fact a receptacle for seeking and storing up numberless feelings, phrases, images, which remain there until all the particles which can unite to form a new compound are present together.” These lines are from (a) Watler Pater, Appreciations (b) Matthew Arnold’s, Study of Poetry (c) T. S. Eliot’s, Tradition and Individual Talent _____(d) John Ruskin, Modem Painters ____________ Ans. (c) : The given lines are taken from "Tradition and the Individual Talent" by ’T.S. Eliot. Perhaps his best-known essay, "Tradition and the Individual Talent" was first published in (1919) and soon after included in ‘The Sacred Wood’, ‘Essay on Poetry and Criticism’ (1920). Eliot attempts to do two things in this essay: he first redefines "Tradition" by emphasizing the importance of history to writing and understanding poetry, and he then argues that poetry should essentially "Impersonal," that is separate and distinct from the personality of its writer. 55. ”On Some Technical Elements of Style in Literature” is an essay written by(a) J.B. Priestley (b) H.G. Wells _____(c) Charles Dickens (d) R.L. Stevenson Ans. (d) : "On Some Technical Elements of Style in Literature" is an essay by Robert Louis Stevenson, who was a Scottish novelist, essayist, poet and travel writer

61

YCT

whose writing style was known as knowledgeable and comprehensible. Thus, correct option is (D). 56. The Sense of an Ending is a book by: (a) L.C. Knights (b) F.R. Leavis (c) Cleanth Brooks _____(d) Frank Kermode (Julian Barnes ) ___________ Ans. (d) : ’’The sense of an Ending : Studies in the I Theory of Fiction” is a book by Frank Kermode. It was first published in 1967 by Oxford University press. 57. The book A Grammar of Motives was written by (a) Noam Chomsky (b) Frank Palmer _____(c) Kenneth Burke (d) R.S. Crane _______ Ans. (c) : 'A Grammar of motives' is written by 'Kenneth Burke.' American philosopher, literary theorist, poet, and essayist 'Kenneth Burke' provided a method to examine the discourse around everyday action and motives. In his book A Grammar of motives, he uses examples and reference from literature, Psychology, and Politics, and Economics to examine. 58. Dryden’s "As Essay of Dramatic Poesy" was written to counter the remarks made by Samuel Sorbiere about the English stage. Who else other than Dryden, replied to the charge? (a) Dr. Johnson (b) Jonathan Swift _____(c) Thomas Sprat ______(d) Alexander Pope Ans. (c) : Dryden’s “An Essay of Dramatic Poesy” was written to counter the remarks made by Samuel Sorbiere about the English stage. Thomas Sprat is other than Dryden, replied to the charge. Thomas Sprat was an English Churchman and writer, Bishop of Rochester from 1684. Thus, option (C) is correct. _______________________ 59. ’On the Artificial Comedy of the Last Century’ is an essay written by: (a) Charles lamb (b) Leigh Hunt _____(c) William Hazlitt _____(d) Robert Southey Ans. (a) : ‘On the Artificial Comedy of the Last Century’ is an essay written by Charles Lamb. Lamb in his essay says that the Restoration Comedies are a world of themselves almost as much as fairy land. Lamb was anxious to reconcile his enjoyment of the plays of Wycherley and William Congreve with the Moral disapproved of his contemparies. Thus, the correct option is (A). 60. Characters of Shakespeare’s Plays is a book of criticism written by: (a) Charles Lamb (b) William Hazlitt _____(c) S.T. Coleridge (d) Thomas De Quincey Ans. (b) : ‘Character of Shakespeare’s Play’ is an 1817 book of criticism of Shakespeare’s play written by early nineteenth century English essayist and literary critic William Hazlitt. 'Characters of Shakespeare’s Plays' argues against a century and a half of criticism that saw Shakespeare as a 'Child of Nature', deficient in art and full of faults. 61. Which notorious critic of Shakespeare said that his plays were "trivial and positively bad" and what was the title of work/essay? (a) Leo Tolstoy-On Shakespeare and Drama (b) George Bernard Shaw in Saturday Review (c) J.R.R. Tolkien in his Letters RPSC Assistant Professor Exam. 2021

(d) Thomas De Quincey in "On the knocking at the Gate” in 'Macbeth' Ans. (a) : In the essay, published as "On Shakespeare and Drama” in 1906. Tolstoy called Shakespeare's plays "trivial and positively bad" labeled his enduring popularity "pernicious" and dismissed Shakespeare himself as "an insignificant, inartistic writer" who was "not only not moral but immoral". Hence, the correct answer is option (a). 62. According to S.T. Coleridge the function of a critic is____. (a) to interpret and to judge (b) to appreciate and to interpret (c) to evaluate and to judge _____(d) to value and devalue ____________________ Ans. (b) : According to S.T. Coleridge the function of a critic is to Appreciate and to Interpret. Coleridge was the first to introduce psychology into literary criticism. He was interested in the study of the process of poetic creation, the very principles of creative activity and for his purpose freely drew upon philosophy and psychology._______________________ 63. Arrange the following essays by T.S. Eliot chronologically: A. Metaphysical Poets B. Tradition and the Individual Talent C. Hamlet and his Problems D. Homage to John Dryden: Three essays on the poetry of the Seventeenth Century (a) A B C D (b) B C A D (c) C A B D_________(d) D A B C _______ Ans. (b) : The following is a list of books of essay by T.S. Eliot arranged chronologically by first edition. 1. Tradition and the Individual by T.S. Eliot (1919). 2. Hamlet and his Problems by T.S. Eliot (1920). 3. Metaphysical Poets by T.S Eliot (1921). 4. Homage to John Dryden; Three essay on the Poetry of the Seventeenth Century by T.S. Eliot (1924). Thus, the correct option is (B). ____________________ 64. Which of the following concept was not included in those that I.A. Richards used to substantiate interpretive criticism. (a) Metaphor (b) Allusiveness _____(c) Ambiguity ________(d) Pastiche _________ Ans. (d) : I. A. Richards (26 Feb 1893-7 Sep 1979) was an English educator, literary critic and Rhetorician. LA. Richards included the concepts of Metaphor, Allusiveness and Ambiguity to substantiate interpretive criticism. He doesn't use the concept of Pastiche, which means a work of art, piece of writing, etc. That is created by deliberately copying the style of somebody/something else or a work of art etc., that consists of a variety of different styles. Thus, the correct option is (D). 65. Who among the following has not contributed to the cannon of structuralist literary criticism? (a) Claude Levi Strauss (b) Jorge Luis Borges _____(c) Vladimir Propp (d) Algirdas Greimas Ans. (b) : Structuralist criticism is a research method, a type of textual research, that literary critics use to interpret texts. A genre of discourse employed by literary critics used to share the results of this interpretive efforts.

62

YCT

Jorge Luis Borges, who was an Argentine short-story write, essayist, poet and translator and a key figure in Spanishlanguage and International Literature has not contributed to the cannon of Structuralism literary criticism. ____________ 66. ’Poetry is the first and last of all knowledge, it is as immortal as the heart of man.” Who opined this? (a) P.B. Shelley (b) S.T. Coleridge _____(c) William Wordsworth (d) John Keats _______ Ans. (c) : ’Poetry is the first and last of all knowledgeIt is as immortal as the heart of man’ was opined by William Wordsworth. ’’Preface to Lyrical Ballads.” (1798). In "preface to Lyrical Ballades" Wordsworth argues that poetry is more closely united with the universal truth of human experience than with any other discipline (subject). He contrasts this truth of human existence (via poetry) with the truth/knowledge gained by Scientists, Botanists and Historian. _______________ 67. ’’Poetry thus makes immortal all that is best and most beautiful in the world”? Who said this? (a) P.B. Shelley (b) S.T. Coleridge (c) William Wordsworth _____(d) John Keats _____________________________ Ans. (a) : Quote by P.B. Shelley in "A Defence of poetry’’-’Poetry thus makes immortal all that is best and most beautiful in the world’. P.B Shelley is expressing his views about poetry that all the beautiful and the best things are immortalized by poetry. Hence, the correct answer is option (a.) ______________ 68. Who said that there should be Five acts in a play”, ’’only three actors at a time”, ’’Scenes of bloodshed offstage” and ’’hexameter verse for war poems”. (a) Aristotle (b) Dryden _____(c) Longinus __________(d) Horace __________ Ans. (d) : Horace gives a set of principles that a writer should follow as for as composition of creative drama is concerned • Horace feels that the proper length of a play is five acts; indeed, that a play should have a maximum of five acts. • According to Horace, a play should not have more than three characters on stage at the same time. • Bloodshed scene should be offstage so it may not be frightening for the audience. • Hexameter verse for war poems. Hence, the correct answer is option (d). ______________ 69. The Road to Xanadu is written by (a) S.T. Coleridge (b) Herbert Read _____(c) J.L. Lowes _________(d) William Empson Ans. (c) : ’The Road to Xanadu’ is written by ’John Livingstone Lowes’. His most famous work is The Road to Xanadu: A study in the ways of the Imagination (Houghton Mifflin 1927), which examines the sources of Coleridge’s ‘The Rime of the Ancient Mariner’ and ‘Kubla Khan’. Thus, the correct option is (C). _____________________ 70. ’’Timber or Discoveries made upon Men and Matter” is a critical work of: (a) Thomas More (b) Ben Jonson (c) Thomas Kyd (d) Henry Howard RPSC Assistant Professor Exam. 2021

Ans. (b) : ’Timber on Discoveries made upon Men and matter’ is a critical work of Ben Jonson in which he merely attacked contemporary stage practice. Timber is a collection of meditations and commentaries upon a range of issues, from the nature of fortune, fame, opinion and wisdom to observations on dramatic theory and poetics. 71. ” A linguistic system is a series of differences of sound combined with a series of differences of ideas.” who said this? (a) Leonard Bloomfield (b) F.R. Leavis (c) Ferdinand de Saussure _____(d) Allan Tate _____________________________ Ans. (c) : In his book "Course in General Linguistics" Ferdinand de Saussure quotes "A linguistic system is a series of differences of sound combined with a series of differences of ideas". According to Saussure, the value of a linguistic sign does not come from its intrinsic signification and it cannot be determined by the sound image alone. Instead the linguistic value of sign is determined by other factors within its environment by the other linguistic signs. ______ 72. According to which school of thought the critic ought to be considered to ’create’ the finished work by his reading of it, and not to remain simply an inert consumer of a ’ready-made’ product: (a) post-modernism (b) New Historicism (c) Reader-response Theory _____(d) Dialogism __________ Ans. (c) : New criticism was a formalist movement in literary theory that dominated American literary criticism in the middle decades of the 20 thcentry. Indicative of the Reader-response school of theory. Terence Hawks writes that the fundamental close reading technique is based on the assumption that "the subject and the object of study-the reader and the textare stable and independent forms, rather than product of the unconscious process of signification, "an assumption which he identifies as the" ideology of liberal humanism," Which is attributed to the new critics who are "accused of attempting to disguise the interests at work in their critical processes. For Hawks, ideally, a critic ought to be considered to "[create] the finished work by his reading of it, and [not to] remain simply an inert consumer of a ready-made product. 73. Which; among the following books cannot be said to incorporate ideas seminal to New Criticism? (a) LA. Richards’ Principles of Literary Criticism (b) Cleanth Brooks’ The Well Wrought urn" Studies in the Structure of Poetry (c) William Empson’s Seven Types of Ambiguity (d) Stephen Greenblatt and Catherine Gallagher’s _________Practicing New historicism _______________ Ans. (d) : Catherine Gallagher and Stephen Greenblatt focus on five central aspects of new historicism: For almost twenty years, new historicism has been a highly controversial and influential force in literary and cultural studies. New criticism is named after John Crowe Ransom’s "The New criticism" (1941). Early seminal works in this tradition were those of English Critics LA. Richards

63

YCT

(Practicle Criticism, 1929) and William Empson (seven types of Ambiguity, 1930) Hence, the correct answer is option (d). ______________ 74. T.S. Eliot’s concept of tradition is countered by Harold Bloom’s . (a) hereby of paraphrase (b) anxiety of influence (c) affective fallacy _____(d) dissociation of sensibility _________________ Ans. (b) : T.S. Eliot’s concept of tradition is countered by Harold Bloom’s term ‘Anxiety of Influence’ (1973). For T.S. Eliot term ’’Tradition” is imbued with a special and complex character. It represent a ’’simultaneous order” by which Eliot means a historical timelessness- a fusion of past and present- and at the same time, a sense of present temporality. Thus, the correct option is (B). _____________________ 75. ’’The business of the poet is not to find new emotions, but to use the ordinary ones and in working them into poetry’, to express feelings which are not in actual emotions at all.” T.S. Eliot says this in which of his texts? (a) "Hamlet and His Problems” (b) "The Metaphysical Poets” (c) "Tradition and Individual Talent” _____(d) "The Frontiers of Criticism" _______________ Ans. (c) : According to T.S. Eliot in his essay Tradition and the Individual Talent "The business of the poet is not to find new emotions, but to use the ordinary ones and in working them into poetry’, to express feelings which are not in actual emotions at all”. Hence, the correct answer is option (c). 76. Who said that, ’’There is nothing outside of the text”? (a) Jacques Derrida (b) Roland Barthes _____(c) Cleanth Brooks _____(d) William Empson Ans. (a) : Jacques Derrida said that “There is nothing outside of the text or nothing outside context”. Jacques Derrida argues that the reading of the text should free itself, in its axis, from the classical categories of history, from the history of ideas and literature of the West that imprisons the text. Thus, the correct option is (A). _____________________ 77. ’’The most striking aspect of linguistic competence is what we may call the ’creativity of language’, that is the speaker’s ability to produce new sentences... ’’This is a quote by who is here talking about the of human language. (a) T.S. Eliot; Creative expressiveness (b) Noam Chomsky; ’Generative capacity (c) William Empson; Ambiguities _____(d) Jean Baudrillard; Simulative ______________ Ans. (b) : "The most striking aspect of ‘Linguistic Competence’ is what we may call the ’creativity of language’ that is the speakers ability to produce new sentences, sentence that are immediately understood by other speakers although they bear no physical resemblance to sentences which are familiar" is a quote by Noam Chomsky, who is here talking about the generative capacity of human language. _____________ 78. Post-modernist thinkers developed concepts like ’difference’ and ’hyper-reality’ to subvert . RPSC Assistant Professor Exam. 2021

(a) myths and parables (b) existing social phenomenon (c) critically acceptable knowledge _____(d) grand narratives and univocity of being _____ Ans. (d) : Post-modernist thinkers developed concepts like ’difference’ repetition, trace and ’Hyper-reality’ to subvert- "Grand narratives,” univocity of being, and epistemic certainty. Thus, the correct option is (D). _____________________ 79. ’’What is an Author?”, an essay, is seen as a challenge to Barthe’s depiction of a historical progression that will liberate the reader from the domination by the author. Who wrote this essay? (a) Jacques Derrida (b) Michel Foucault _____(c) Gilles Deleuze ______(d) Virginia Woolf Ans. (b) : "What is an Author ?’’ is a lecture on literary theory given at the society franchise de philosophies on 22 February 1969 by French Philosopher Michel Foucault. Many considers this essay as a response to Roland Barthes’ essay ’The Death of the Author’. The work considers the relationship between author, text and reader. Thus, the correct option is (B). _____________________ 80. Who coined the term ’gender performativity’? (a) Simone de Beauvoir (b) Elaine Showalter _____(c) Julia Kristeva ______(d) Susan Gubar ______ Ans. (*) : Gender Performativity is a term first used by the feminist philosopher "Judith Butler in her 1990 book ‘Gender Trouble’. She argues that being bom male or female does not determine behavior instead, people learn to behave in particular ways to fit into society. The idea of gender is an act, or performance. This act is the way a person, walks, talks, dresses, and behaves, she calls this acting "Gender Performativity. *Note : RPSC has chosen option (c) Julia Kristeva the correct option while Judith Butler is the feminist writer who coined the term "Gender Performativity". Hence, there is no correct answer among the given options. _________________________________________ 81. Gayatri Chakravorty Spivak introduced a major concept in postcolonial theory through which minority groups, nationalities mobilize themselves on the basis of shared gendered, cultural or political identity. What is it? (a) Orientalism (b) Alterity _____(c) Subaltemism (d) Strategic essentialism Ans. (d) : Strategic Essentialism, a major term in postcolonial theory, was introduced in the 1980s by the Indian literary critic and theorist ’Gayatri Chakravorty Spivak’. It refers to a political tactic in which minority groups, nationalities, or ethnic groups mobilize on the basis of shared gendered, cultural, or political identity to represent themselves. Thus, the correct option is (D). _____________________ 82. A concept propounded by Homi K. Bhabha in postcolonial theory which is a sign of double articulation, a strategy which appropriates the other as it visualizes power is . (a) Mimicry (b) Liminality _____(c) Hybridity __________(d) Ambivalence _____ Ans. (a) : Jacgues Lacan asserts, "The effect of mimicry is camouflage It is not question of harmonizing with the background, but against a mottled background." Colonial mimicry comes from the colonists desire for a

64

YCT

reformed, recognizable other as subject of a difference that is, as Bhabha writes almost the same. Homi K. Bhabha’s Minicry is a sign of a double articulation, a strategy which appropriates the other as it visualize power. Thus, the correct option is (A). ____________________ 83. Match the following ’’Texts’ in I with their ’Author’s’ in II: List-I List-II A. The Female Eunuch E. Toril Moi B. Sexual/ Textual F. Luce Irigary Politics c . This Sex Which is G. Germaine Greer Not One D. The Beauty Myth H. Naomi Wolf D ABC (a) F H G E (b) E F G H (c) G E F H (d) H G E F ______________ Ans. (c) : In 1979, Australian-born author ’Germaine Greer’ wrote ’The Female Eunuch’, a book that Challenged a woman’s traditional role in society and provided an important framework for the feminist movement of the 1970s. ’Sexual/Textual Politics’ is a book by ’Toril Moi’. This book examines the strength and limitations of the two main strands in feminist criticism, the Anglo-American and the French, paying particular attention to the works of Cixous, Irigaray and Kristeva. ‘This Sex Which is not One’ book by Luce Irigaray elaborates on some of the major themes of speculum of the other Woman, her landmark work on the status of woman in western philosophical discourse and in psychoanalytic theory. ’The Beauty Myth’ is a book by ’Naomi Wolf. According to Wolf, the myth of beauty is an order that dictates behavior and not appearance: above all the myth of beauty determines the urgency and duty to base the identity and value of women on their beauty. The Beauty Myth is just one of several social fiction that emerged the 1930. Thus, the correct option is (C). ____________________ 84. ’’The Personal is Political” is an essay written by: (a) Betty Friedan (b) Carol Hanisch _____(c) Sylvia Plath _______(d) Alice Walker _____ Ans. (b) : The Second-Wave-Feminist Carol Hanisch Essay- titled "The Personal is Political” appeared in the anthology Notes From the Second year : Woman’s Liberation in 1970, and is often credited with creating the phrase "The Personal is Political” means “All relationships between Men and Women are based on power and dominance”. Thus, the correct option is (B). ____________________ 85. What is the term that Foucault introduces in his book The Order of Things Which refers to the orderly ’unconscious’ structures underlying the production of knowledge in a particular time and place? (a) Heterotopia (b) Discursive formation (c) Archive (d) Episteme RPSC Assistant Professor Exam. 2021

Ans. (d) : Episteme- The term which Foucault introduces in his book. ‘The Order of Things’, refers to the orderly ’unconscious’ Structures underlying the production of scientific knowledge in a particular time and place. There are three Episteme : the Traditional, the Classical and the Modem, the forms of knowledge associated with each of them. ______________________________ 86. what word does Bakhtin use to describe the process through which meaning is evolved out of interactions among the author, the work and the reader? (a) Polyphony (b) Dialogism _____(c) Heteroglossia ______(d) Chronotopia ______ Ans. (b) : Dialogics or dialogism, according to Bakhtim, means the process which meaning is evolved out of interactions among the author, the work and the reader or listener. For Bakhtin, dialogism characterises the entire social world. Authentic human life is an open-ended dialogue. The world thus, merges into an open-ended, multivoiced, dialogical whole. _________________________ 87. ’Individualism’, ’Experimentation’, ’Absurdity’, ’Symbolism’ are the main characteristics of which kind of literature? (a) Post Modernist (b) Post Structural _____(c) Modernist _________(d) Structuralist ______ Ans. (c) : ‘Individualism’, Experimentation, Absurdity and Symbolism are the main characteristics of Modernist. Modernism is an artistic and cultural movement that flourished in the first decades of the 20th century, about the time of WWI. Modernism is a cultural trend. It is the movement in visual arts, music, literature and drama which rejected the old Victorian standard of how art should be made. Modernism was developed in the first three decades of the 20th century. It was developed in Europe especially in the Great Britain. _____________________________ 88. Who has coined the term ’backlash’ for a strategy that ’blames the victim’ by suggesting that women’s liberation movement is itself the cause of many problems that plague women? (a) Shulamith Firestone (b) Helene Cixous _____(c) Kathie Sarachild (d) Susan Faludi _____ Ans. (d) : "Backlash”- The Undeclared War Against American Women’ is a (1991) book by "Susan Faludi’ in which the author presents evidence demonstration the existence of a media-driven "backlash” against the feminist advances of the 1970s. Faludi argues that the backlash uses a strategy of "blaming the victim” by suggesting that the women’s liberation movement itself is the cause of many of the problems alleged to be plaguing women in the late 1980s. She also argues that many of these problems are illusory constructed by the media without reliable evidence. Faludi also identifies backlash as a historical trend recurring when women have made substantial gains in their efforts to obtain equal rights. The book won the National Book Critics Circle Award for non-fiction in 1991. A 15th anniversary edition was released in 2006. 89. The Wretched of the Earth, a book that analyses the demunanizing effects of colonization, by Frantz Fanons derives its title from YCT 65

(a) The Opening lyrics of ’’The International”, a left-wing anthem (b) Ovid's Metamorphoses (c) Dante's Divine Comedy (d) Lines from the anthem "This Land is Your _________ Land' __________________________________ Ans. (a) : ‘The Wretched of the Earth’ is a book that analysis the dehumanizing effects of colonization by Frantz Fanons, derives its title from ‘The opening lyrics of The Internationale’ a left wing anthem. ‘The Internationale’ is the most famous socialist song and one of the most widely recognized songs in the world. The original French words were written in 1871 by Eugene Pottier. Thus, the correct option is (A).

90.

Oppositional Aesthetics Readings from a Hyphenated Space is a book about ’diasporics’ by: (a) Chandra Talpade Mohanty (b) Uma Parmeshwaran (c) Arun Prabha Mukherjee _____(d) Gayatri Spivak ____________________ Ans. (c) : 'Oppositional Aesthetics Reading from a Hyphenated space'-is a book by 'Arun Prabha Mukherjee'. In these closely argued essays, talking examples from writing and film, 'Arun Prabha Mukherjee' Considers the place of the third world person-both as artistic creator and as a subject of artistic endeavour- in the West, works of non- mainstream, immigrant artists, she urges, should be understood on their own term.___________________________________ 91. Avtar Brah has written a seminal book on Diaspora called (a) Diaspora and Identity (b) Cartographies of Diaspora (c) Diaspora and Transnationalism _____(d) Diaspora and Hybridity __________________ Ans. (b) : 'Cartographies of Diaspora: Contesting Identities’ is a book by 'Avtar Brah'. Cartographies of Diaspora throws light on these debates by exploring the intersections of race, gender, class. 92. Which among the following is not a Diaspora critic? (a) William Safran (b) Paul Gilroy _____(c) Robin Cohen _______(d) Raymond Williams Ans. (d) : Diaspora criticism takes the concept 'Diaspora' as its object of inquiry and provides a framework for discussing displaced communities in a way that takes contemporary social, cultural and economic pressures into account. Some of the main diasporic crities include william safran, Jonathan Boyarin, Paul Gilroy, James clifford, Stuart Hall, Rey Chow, Avtar Brah, Vijay Mishra and Robin Cohen. Raymond williams was a welsh socialist writer, novelist and critic influential within the New Left and in wider culture. 93. Cultural Materialism makes analysis based in critical theory, in the tradition of the _ _ _? (a) Chicago School (b) Frankfurt School _____(c) Prague School ______(d) Rinzai School Ans. (b) : Cultural Materialism in literary theory and cultural studies traces its origin to the work of the left wing literary critic Raymond Williams. Cultural materialism makes analysis based in critical theory in the tradition of the Frankfurt school. RPSC Assistant Professor Exam. 2021

94.

Who says of ‘Cultural Materialism’ that it is a ’’Politicized form of historiography” (a) Herbert Marcuse (b) Antonio Gramsci _ _ _ _ _(c) Graham Holdemess (d) Alan Sinfield _____ Ans. (c) : British Critic Graham Holdemess defines ‘Cultural Materialism’ as a "politicized form of historiography." Through its insistences on the importance of an engagement with issues of gender, sexuality, race and class, ‘Cultural Materialism’ has had a significant impact on the field of literary studies especially in Britain. ______________________________ 95. ’’The Poststructuralist orientation to history is now emerging in literary studies I characterize chiastically, as a reciprocal concern with the historicity of texts and the texuality of history” is a statement by the new historicist _ _ _. (a) Stephen Greenblatt (b) Louis Adrian Montrose (c) Clifford Geertz _____(d) Catherine Gallagher _____________________ Ans. (b) : Louis Montrose, in professing the Renaissance: The poetics and politics of culture claimed that new historicism deals with the "textuality of history and the historicity of texts." While "historicity of texts" refers to the "cultural specificity and social embedment of all modes of writing," the rootedness of a text in the social- historical, political and cultural ambiance of its production, "textuality of history" refers to the fictionality and constructedness of history.____________ 96. Post modernists typically argue that(a) Universalist narratives which attempt to explain the world are invariably false. (b) The world is socially constructed in a variety of ways. (c) Differences of viewpoint should be celebrated. (d) Grand narratives are useful sources of _________information. ____________________________ Ans. (d) : Postmodernist philosophers, in general, argue that truth is always contingent on historical and social context than being absolute and universal and that truth is always partial and at issue rather than being complete and certain. ______________________________________ 97. All liberal feminists advocate (a) equal treatment in the public sphere (b) the strict segregation of the sexes (c) discrimination in economic terms only _____(d) no role in political matters ________________ Ans. (a) : All liberal feminists advocate equal treatment in the public sphere. Liberal feminists argue that society holds the false belief that women are, by nature less intellectual. Liberal feminism believes in “Pragmatic reform against gender discrimination through the promotion, from equal right engaging and formulating laws and policies that will ensure equality”. 98. Who wrote that panopticism is a one-way power/ knowledge relationship between those under surveillance and the watchers who could not be watched? (a) Jeremy Bentham (b) Michel Foucault _ _ _ _ _(c) Erving Goffman (d) None of these _____ Ans. (b) : Paul - Michel Foucault was a French philosopher, historian of ideas, writer, political activist and literary critic, wrote that panopticism in one way

66

YCT

power/knowledge relationship between those under surveillance and watchers. It has become the most widely used metaphor for surveillance, becoming almost its synonym. The panopticon has become particularly famous through Foucaults concept of panopticism resulting in Benthanm often being understood through the reading of Foucault. __________ 99. Who first welcomed us to ’’The desert of the real”? (a) Frederich Nietzsche (b) Jacques Derrida (c) Jean Baudrillard _ _ _ _ _(d) Jean-Francois Lyotard ____________________ Ans. (c) : Jean Baudrillard was French sociologist, philosopher and cultural theorist. Jeam Baudrillard first welcome us to “The desert of the real”. The first invitation Slavoj Zizek christened the 9/11 attacks on the World Trade center towers ’’The Desert of the real” (referring to a line in the film Matrix) thus informing us of a new phase in our relationship with the real (that is, our lame attempts to name, symbolize and hense domesticate it). __________________________________ 100. Who seeks an assumed binary opposition in an author’s narrative then rereads the text against the author’s wishes from the perspective of the marginalized position, finally showing that the correct interpretation is undecidable? (a) Roland Barthes (b) Emmanuel Levinas (c) Jean-Francois Lyotard _____(d) Jacques Derrida _________________________ Ans. (d) : Jacques Derrida was an Algerian-born French philosopher best known for ‘Deconstruction Theory’ who seeks an assumed binary apposition in an authors narrative then re-reads the text against the author’s wishes from the perspective of the marginalized position, finally showing that the correct interpretation is undecidable. _______________________ 101. Feminist Criticism does/is NOT (a) concerned with the marginalization of women in a patriarchal culture. (b) try to explain how the subordination of women is reflected in or challenged by literary texts. (c) try to examine the experiences of women from all races, classes and cultures. (d) explain how and when women become _________ responsible mothers. _____________________ Ans. (d) : Among the given option, Feminist criticism does not explain how and when women become responsible mothers. Rather Feminist Criticism is concerned with the marginalization of women in a patriarchal culture. It try to explain how the subordination of women is reflected in or challenged by literary texts and also tries to examine the experiences of women from all races, classes and cultures. Virginia Woolf is rightly considered the founder of modem ‘Feminist Literary Criticism’. 102. Which one of the following is not one of Elaine Showalter’s four models of sexual difference? (a) Biological (b) Occasional (c) Linguistic (d) Psychoanalytic RPSC Assistant Professor Exam. 2021

Ans. (b) : ‘Occasional’ is not one of Elaine Showalter's four models of sexual difference. The four models of sexual difference by Elaine Showalter are - Biological, Linguistic, Cultural and Psychoanalytic. 103. Which of the following statements is true of the Ego, according to Freud? (a) It exists prior to the Id. (b) It follows the pleasure principle. (c) It lends its libidinal energy to super ego. (d) It is not affected by reality, logic or the _________everyday world. _________________________ Ans. (d) : According to Sigmund Freud Ego, lends its libidinal energy to super ego. Sigmund Freud was an Austrian neurologist and the founder of psychoanalysis a clinical method for treating psychopathology a through dialogue between a patient and a psychoanalyst. Freud was bom to Galician Jewish parents in the Moravian town of Freiberg, in the Austrian Empire. 104. In ’’the imaginary order” the subject’s own image permanently catches and captivates the subject. In the Lacanian model this statement refers to which stage’? (a) Symbology (b) Mirror _ _ _ _ _(c) Castration _________(d) Alterity __________ Ans. (b) : Mirror stage : The Mirror stage is concept in the psychoanalytic theory of Jacques Lacan, this theory believes that infants recognize themselves in a mirror (literal). In the "imaginary order,” the subject own image permanently catches and captivates the subject, Lacan explain that ’’the mirror stage is a phenomenon to which I assign a two fold value._____________________ 105. Which one of the following is not one of the concepts explored by Freud? (a) Oedipus complex (b) Slips _ _ _ _ _(c) Myths _____ ______(d) Unconscious ______ Ans. (c) : All the above concepts explored by Sigmund Freud except the term ‘Myths’. Freud has explored the concepts of Oedipus complex. Slips and unconscious. Oedipus complex is a concept of psychoanalytic theory. Freud introduced Oedipus complex in his book "Interpretation of Dreams”. 106. Which one is not a book written by Sigmund Freud? (a) The Interpretation of Dreams (b) Civilization and its Discontents (c) Studies in Hysteria _____(d) The Future of Disillusion _________________ Ans. (d) : ’The Future of Disillusion’ is not written by Sigmund Freud. It is a book written by G.A. Cohen which was published in 1991. Instead of 'The Future of Disillusion', Sigmund Freud has written 'The Future of an Illusion'. _____ 107. ’’Representation of the world, like the world itself, is the work of men; they describe it from their own point of view, which they confuse with absolute truth:. This statement is made by (a) Simone de Beauvoir (b) Mary Wollstonecraft (c) Elaine Showalter _____(d) Sigmund Freud _________________________ Ans. (a) : The line "Representation of the world, like the world itself, is the work of men; they describe it from their own point of view, which they confuse with absolute truth,” is said by Simone de Beauvoir in her 1949 book "The Second Sex", a cornerstone of 20 th century feminism.

67

YCT

108. Who said that, ’’Who speaks is not who writes, and who writes is not who is”? (a) Michel Foucault (b) Jaques Derrida _____(c) Roland Barthes _____(d) Homi K, Bhabha Ans. (c) : Roland Barthes Quotes ’’who speaks is not who writes, and who writes is not who is,”. The above line is taken from his essay ’The Death of the Author' published in 1967. This essay holds that an author’s intentions and biographical facts (author’s politics religion etc). _____________________________________ 109. Of Grammatology, by Derrida, opens with a preface and an exergue. The preface outlines the structure of the text and tests the theoretical matrix by a reading of age of ____. (a) Shakespeare (b) The meta physicals _____(c) Rousseau __________(d) Kafka ___________ Ans. (c) : Of Grammatology, by Derrida, opens with a preface and an exergue. The preface outlines the structure of the text and tests the theoretical matrix by a reading of age of Rousseau. ’’Thus ’’Grammatology” (a term that Derrida uses to refer to the science of writing) can liberate our ideas of writing from being subordinated to our ideas of speech. Grammatology is a method of investigating the origin of language that enables our concepts of speech. _________ 110. ’’People know what they do; frequently they know why they do what they do; but what they don’t know is what they do does” say Michel Foucault in his book: (a) Discipline and Punish (b) Madness and Civilization (c) The Order of Things _____(d) Jhe Archaeology of Knowledge ___________ Ans. (b) : Michel Foucault quotes in his book’ Madness and Civilization’, "people know what they do, frequently they know why they do what they do; but what they don't know is what they do does." __________ 111. In ’’Towards a Feminist Poetics” Elaine Showalter introduces the word ____to construct ’a female framework for the analysis of women’s literature”. (a) Comparative criticism(b) Expressive criticism _____(c) Mimetic criticism (d) Gynocriticism Ans. (d) : In "Towards a Feminist Poetics" Elaine Showalter introduces the word Gynocriticism to construct 'a female framework for the analysis of women's literature." Showalter coined the term ’Gynocriticism’ to describe literary criticism based in a feminine perspective, probably the best description showalter gives of Gynocriticism is in ’Towards a Feminist Poetics’. 112. Who was the first to propose a science focused on the study of signs? (a) Vladimir Propp (b) Jacques Derrida _____(c) Roland Barthes (d) Ferdinand de Saussure Ans. (d) : The first to propose a science focused on the study of signs was Swiss linguist Ferdinand de Saussure. The study was called Semiotics or Semiology, in which Saussure describes the signs and its arbitrary relation to reality. ________________________________ 113. Who argued that the literary critic’s relationship to literary discourse is the same as the linguist’s relationship to language? RPSC Assistant Professor Exam. 2021

(a) Tzvetan Todorov (b) Roland Barthes _ _ _ _ _(c) Vladimir Propp _____(d) Roman Jakobson Ans. (a) : Tzvetan Todorov argued that the literary critic’s relationship to literary discourse is the same as the linguist's relationship to language. Linguistics describes the system underlying language use while literature makes use of the relevant aspects of these linguistic features to express what the writer has to say. 114. Claude levi-Strauss gave a structural reading of which famous story? (a) King Lear (b) Oedipus Rex _ _ _ _ _(c) The Iliad __________(d) Paradise Lost _____ Ans. (b) : Claude Levi - Strauss gave a structural reading of ‘Oedipus Rex’, a play by Sophocles, first performed around 429 BC. Levi-Strauss reads the myth as being not at all about killing or incest, but about a contrast between the belief that human beings were originally bom from the earth. ______________________ 115. Homi Bhabha's essay ’’The Commitment to Theory” is (a) a defense of poststructuralist theory's political value. (b) A critique of poststructuralist theory of elitism. (c) a critique of postcolonial theory of the subaltern. (d) an attestation of liminality and ambivalence. Ans. (a) : Homi Bhabha's essay "The Commitment to theory" is a defense of poststructuralist theory’s political value. "Commitment to Theory"- Here an attempt has been made to explore and define the notions about cultural identity following the turn of events in the 19thcentury Europe. • The main points factored here are: • To rethink on the "perspective of identity of our culture" in the post colonial world. • Begin by differentiating ‘cultural diversity’ and ‘cultural identity’. • Culture- as a strategy for survival in transnational and transrational. _______________________________ 116. Which among the following is a common convention of postmodern writing? (a) Straight forward plots and realistic situations. (b) Clear and obvious meanings (c) A respect for previous styles and conventions _____(d) Metafiction and intertextuality _____________ Ans. (d) : Postmodern literature is a form of literature that is characterized by the use of Metafiction, unreliable narration, self- reflexivity, intertexuality, and which often thematizes both historical and political issues. This style of experimental literature emerged in the united states in the (1960). ______________________ 117. Which collection of essays by New Historicists focuses on the Romantic period? (a) Rethinking Historicism: Critical Readings in Romantic history (b) The Romantic Ideology: A Critical Investigation (c) The New Historicism _____(d) Practicing New Historicism _______________ Ans. (a) : Rethinking Historicism : Critical Readings in Romantic history a collection of essays by New Historicists focuses on the Romantic period.

68

YCT

New historicism seeks to find meaning in a text by Thought cleaves the interstellar gloom considering the work within the framework of the And sits in Sirius' disc all night, prevailing ideas and assumption of its historical era. Till day makes him retrace his flight, These critics focus on revealing the historically specific With smell of burning on every plume, model of truth and authority (not a ’’truth” but a Back past the sun to an earthly room. ’’cultural construct”) reflected in a given work. ________ His gains in heaven are what they are, 118. Who came up with the term ’Self- fashioning’? Yet some say love by being thrall (a) Clifford Geertz (b) Jerome McGann And simply staying possesses all _____(c) Stephen Greenblatt (d) Catherine Gallagher In several beauty that Thought fares far Ans. (c) : Self-fashioning, a term introduced by To find fused in another star. ”Stephen Greenblatt’ (Renaissance, self-fashioning, 122. What physical attributes does the poet give to 1980), is used to describe the process of constructing personify Love and Thought in stanza 1? one's identity and public persona according to set of (a) hills and walls (b) arms and wings socially acceptable standards. _____________________ _____(c) earth and sky ______(d) fear and need _____ 119. In ’’Postmodern Blackness”, bell hooks argues Ans. (b) : Poet gives physical attributes to Love and that post-modern theory is good for: thought to personify arms and wings. This poem of (a) Maintaining ’’status quo” Marvell’s has eight stanza, whereas the two others have (b) Critiquing essentialism nine and four respectively. In poetry, personification is (c) Legal practioners used to allow non-human things to take on human traits _____(d) Critiquing language_____________________ and emotion, poets can use personification to make Ans. (b) : Postmodern Blackness is a philosophical inanimate objects, such as mirror, express feelings and essay written by author bell hooks regarding postmodernism what constitutes ones identity, and the perform action. perception of people, particularly Africans, who 123. Which emotional stance of love is contrasted with the ’dauntlessness’ of Thought? espouse the postmodern philosophy. The Critique of (a) gloominess (b) possessiveness essentialism encouraged by postmodernist thought is useful for African-Americans concerned with _____(c) fearfulness ________(d) abrasiveness______ reformulating outmoded notions.___________________ Ans. (c) : Fearfulness emotional stance of love is 120. Which theoretical book is often cited as the contrasted with the 'dauntlessness' of thought. book that created the field of post colonialism? Fearfulness define- The quality of being frightened or (a) Homi Bhabha’s Location of Culture worried about something._________________________ (b) Joseph Conrad's Heart of Darkness 124. How does the poet describe the task of Thought (c) Edward Said’s Orientalism as Herculean? (d) Franz Fanon's The Wretched of the Earth (a) He can shake his ankles. Ans. (c) : The field of postcolonial studies has been (b) He can bum every plume. gaining prominence since the 1970. Some would date (c) He can retrace his flight. its rise in the Western academy from the publication of _____(d) He can cleave interstellar gloom. __________ Edward Said's, influential critique of western construction of the orient in his 1978 book, Ans. (d) : The poet describe the task of thought as Herculean as he can cleave interstellar gloom. Thus, the Orientalism. correct option is (d). _____________________________ 121. If one asks of a text, ”How is the indigenous culture of the land being portrayed?” Which 125. The phrase/s that the poet uses to show Love’s endearing gestures are: critical theory would you be using? (a) fusing with another star (a) New Historicism (b) Psycho-analysis (b) encircling arms, being in the world’s embrace _____(c) Deconstruction _____(d) Post-colonialism (c) being shut within walls, hills, turf Ans. (d) : Post colonialism is the critical academic study of the cultural, political and economic legacy of _____(d) eager to possess victory _________________ colonialism and imperialism, focusing on the human Ans. (b) : Love's endearing gestures are-encircling consequences of the control and exploitation of arms, being in the world's embrace. Thus, the correct colonised people and their lands ___________________ option is (b). ___________________________________ Direction: Read the following poem and answer the 126. ’Sirius disc’is_____. question (122-131) that follow by choosing from (a) a luminous star (b) the edge of the moon the given options: _____(c) the setting sun (d) a celestial abode _____ Love has earth to which she clings Ans. (a) : 'Sirius' disc’ is a luminous star. Sirius is the With hills and circling arms about brightest star in the night sky. Its name is derived from Wall within wall to shut fear out, the Greek origin. But Thought has need of no such things, 127. How many time has the poet used a gender For Thought has a pair of dauntless wings. defining pronoun for Love in the poem? On snow and sand and turf, I see (a) thrice (b) once Where love has left a printed trace _____(c) twice _____________(d) never ___________ With straining in the world’s embrace Ans. (b) : The poet used a gender defining pronoun for And such is love and glad to be. Love in the poem for one time only. But Thought has shaken his ankles free. YCT 69 RPSC Assistant Professor Exam. 2021

128. The contrast being cited in the poem between Ans. (d) : Here no article is required. Every singular Love and Thought is not about? countable noun in English must have an article, whereas (a) sojourning the skies and remaining non- countable noun and plural noun do not require an earthbound article. (b) their fearfulness and dauntlessness 135. It has surely rained last night, there are (c) their influences during the day and at night drops imprinted on the window panes. _____(d) being free and being bound ______________ (a) some (b) few Ans. (c) : The contrast being cited in the poem between _____(c) a little ____________(d) most ____________ Love and thought is not about-their influences during Ans. (c) : It has surely rained a little last night, there the day and at night. _____________________________ are drops imprinted on the window panes. We use 'a 129. Which figure of speech is repeatedly used in the little' to show that we are talking about a small quantity last stanza? or amount of something. (a) Onomatopoeia (b) Alliteration Additional Information: There is an error in the _____(c) Metaphor _________(d) Synecdoche ______ construction of given sentence : It has surely rained a Ans. (b) : The figure of speech repeatedly used in the little last night. • ‘has’ should not be used in a sentence with such a last stanza of the poem is Alliteration. Alliteration is figure of speech in which a series of words - last night, last day or last (time period). words, usually two or more neighboring words, have the So, the sentence should be in past Tense. ____________ same first Consonant sound, however, sometimes 136. Our college is quite a landmark, can tell repetition of sounds occur inside a word. The sound you where it is. created by alliteration establish a certain mood, emotion (a) eachone (b) anyone and rhythm in text. _____(c) someone __________(d) each and every Ans. (b) : Our college is quite a landmark, anyone can 130. Which two senses are evoked in the poem? tell you where it is. We use anyone, anybody and (a) Touch and Smell (b) Taste and Sight _____(c) Hearing and Seeing (d) Smelling as Tasting anything to refer to both an open, unlimited set of things Ans. (a) : There are two senses that are evoked in the or people and specific things or people. Direction: Fill in the blanks in the following sentences poem. Touch and smell. Thus, the correct option is (a). (137-140) by choosing the correct modal 131. The narrator in the poem ends in favour of auxiliary from the given options: Love because 137. He have sent the message immediately (a) Love understands the emotion of fear. rather, than delaying its dispatch. (b) Love is able to achieve a more lasting bond. (a) can't (b) shall (c) Love is enthralled by heaven's beauty. _____(d) Love needs to perform less movements._____ _____(c) could _____________(d) ought ___________ Ans. (b) : The narrator in the poem ends in favour of Ans. (c) : Here correct modal Auxiliary should be Love because Love is able to achieve a more lasting ‘could’. Now the sentence is, ‘He could have sent the bond. Thus, the correct option is (b). _______________ message immediately rather, than delaying its dispatch.’ Direction: Fill in the blanks in the following questions Could have is used to talk about something that was or (132-136) by choosing the correct wasn’t possible in the past or capable of doing something but we didn’t. _________________________ article/determiner. 138. A banyan tree to be here when we were 132. I’ve read the prescribed text times. children. (a) some (b) much (a) must (b) may _____(c) several ___________(d) good deal ________ _____ (c) should ____________(d) used ____________ Ans. (c) : I've read the prescribed text several times. Ans. (d) : A banyan tree used to be here when we were Explain- A determiner is a word that is placed just before a noun to give additional information about that children. The verb used to is a modal verb. It is only noun and that's why they are also classified as found in the past tense. Adjectives. ____________________________________ Explain- Need, Dare, ought to and used to are known as Marginal Modal Auxiliaries or Marginal Modal Verb. 133. I have gone through both the books, has 139. When fire breaks out one not use the lift the information I need. but take the emergency stairs down. (a) both (b) every (a) have (b) should _____(c) that ______________(d) neither __________ Ans. (d) : Correct Determiner in accordance with the _____(c) could _____________(d) shan't ___________ given sentence is ‘neither’. I have gone through both Ans. (b) : Here correct modal Auxiliary should be ‘Should’. Now the sentence is: ‘When fire breaks out the books, neither has the information I need. Neither as a determiner: Neither allows us to make a one should not use the lift but take the emergency stairs negative statement about two people or things at the down.’ same time. Neither goes before singular countable Explain- "Should" is a modal verb most commonly used nouns. We use it to say ‘not either’ in relation to two to make recommendations or give advice. It can also be things. _______________________________________ used to express obligation as well as expectation. ______ 140. Did he criticize my arrangements? 134. He is on his way to work. (a) need (b) ought (a) the (b) a (c) dare (d) must (c) those (d) no article YCT RPSC Assistant Professor Exam. 2021 70

Ans. (c) : Did he dare criticize my arrangements? Ans. (b) : This new machine will certainly Step up Explain- ’’Dare” is another verb that can be used with all production. We use the phrasal verb 'step up' for the characteristics of an auxiliary, when it means "to something, if we increase it or increase its intensity. have boldness or courage (to do something),’’ to be so Direction: Fill in the blanks in the following sentences bold as,” It is often followed by an infinitive without (146-150) by choosing the correct tense of the "to". ______________________ verb/sequence of tenses from the given options: 146. After she her tea she felt refreshed. Subject + dare + zero inf initive Affirmative (a) have had (b) have taken Dare + Subject + zero inf initive + ? Interrogative _____(c) had ______________(d) had took _________ No priest dares hint at a providence which does not Ans. (c) : After she had her tea she felt refreshed. respect English utility. ___________________________ Explain- The past perfect tense in English is composed Direction: Fill in the blanks in the following sentences of two parts; the past tense of the verb to have (had) + (141-145) by choosing the correct the past participle of the main verb. _________________ preposition/phrasal verb from the given 147. Let’s hope it won’t still when we leave. options: (a) have rained (b) was raining 141. The sky is overcast by clouds, I’m afraid _____(c) have be raining _____(d) be raining ________ we’re a heavy downpour. Ans. (d) : Let's hope it won't still be raining when we (a) allowing for (b) in for leave. _____(c) asking for _________(d) calling for _______ Explain- We use the Future continuous Tense to talk Ans. (b) : Correct phrasal verb from the given options about action which will be in progress at a time in the is in for. The sentence is -‘The sky is overcast by future. clouds, I’m afraid we are in for a heavy downpour.’ 148. Unless it rains, we will have to the plants be in for something (phrasal verb): daily. to be going to experience something unpleasant very (a) waters (b) watered soon. _____(c) water _____________(d) watering _________ Ex: The weather Forecast says we’re in for heavy rain Ans. (c) : Unless it rains, we will have to water the this evening. ___________________________________ plants daily. 142. The newly elected leader his associate to Explain- Unless is a conditional word (like if), so we work for the good of their enterprise. don't use will or would in the subordinate clause. ______ (a) called out (b) calledin 149. Now the streets as parking places since _____(c) called to __________(d) called upon ______ the pavements have been encroached upon by Ans. (d) : The Appropriate phrasal verb is called upon. the shopkeepers. The newly elected leader called upon his associates to (a) have being used (b) being used work for the good of their enterprise. call _____(c) been using ________(d) are being used on/upon: to ask formally for someone to do something. Ex-They called on the company to reverse its decision. Ans. (d) : Here the correct tense of the verb from the Thus, the correct option is (d). _____________________ given option is are being used. Now the streets are being used as parking places since the pavements have 143. She returned her parent company after been encroached upon by the shopkeepers. the completion of her deputation abroad. Note: Here In the given sentence, are being used has (a) in (b) to been used as a passive construction of present (c) up _______________(d) by ______________ continuous tense. _______________________________ Ans. (b) : The Appropriate Preposition is ‘to’ for this 150. you obeyed orders this disaster would sentence. The sentence is - ‘She returned to her parent not have happened. company after the completion of her deputation abroad.’ (a) Have (b) Had Explain- we use preposition ’To’ for movement. When (c) Should ___________(d) Would __________ indicating that there is movement from one place to Ans. (b) :Here the correct tense of the verb from the another. In other words, the preposition ’to’ with verb given options is ‘Had’. The sentence is - Had you such as drive, walk, go, like, fly, sail etc. ____________ obeyed orders this disaster would not have happened In 144. The referee was booked his decision the given sentence, type III conditional has been used. regarding a penalty he had awarded the team. Rule: In a type 3 conditional sentence, the tense in (a) in (b) by _____(c) for _______________(d) at ______________ ‘if clause is in past perfect and the tense in the main clause is in the perfect conditional or the perfect Ans. (c) : The Appropriate Preposition is ‘for’ for this continuous conditional used. sentence. The referee was booked for his decision It is used when is impossible to full fill a condition regarding a penalty he had awarded the team. which is given in the if-clause If someone is booked for, he/she is registered in official Type -III conditional sentences structure record for doing something wrong. Main Clause If Clause Ex. My Cousin is booked for speeding last week. would + have+ Past participle Past Perfect 145. This new machine will certainly___________ 3 Could + have+ Past participle (if+sub +had + V ) production. Might + have+ Past participle or (a) standup (b) step up (c) turn in (d) make for (Had + sub+ V3 ) RPSC Assistant Professor Exam. 2021

71

YCT

RPSC Assistant Professor Exam. 2020

ENGLISH-II Solved Paper 1.

was an early English comedy written by a schoolmaster to be performed by his students, which illustrates the early Renaissance belief in the educational advantages of acting. (a) Ralph Roister Doister (b) Gammer Gurton’s Needle (c) The Old Wives’ Tale _____(d) The Shoemaker's Holiday ________________ Ans. (a) : Ralph Roister Doister is a first English Comedy by Nicholas Udall, which was once regarded as the first comedy to be written in English language. The date of its composition is disputed, but the balance of opinion suggests that it was written in about 1552, when Udall was a schoolmaster in London, and some theories the play was intended for public performance by his pupils who were all male, as were most actors in that period. The work was not published until 1567, eleven years after its author’s death. ________________ 2. Who said, ’’The hero of Shakespeare’s great classical trilogy is Rome”? (a) Wilson knight (b) L.C. Knights _____(c) Dover Wilson ______(d) Edward Dowden Ans. (a) : Wilson Knight was an English literary critic and academic, known particularly for his interpretation of mythic content in Literature. ‘The Wheel of Fire’, a collection of essay on Shakespeare’s play. He was also an actor and theatrical director, and considered an outstanding lecture. _____________________________ 3. Match the following prose writers with their works: i. Bishop John Fisher ii. Roger Ascham iii. Thomas More iv. Richard Hooker A. Of the laws of Ecclesiastical Polity B. Froissart’s Chronicles C. Toxophilus D. History of Richard HI i ii iii iv (a) B C D A (b) B A D C (c) C B D A (d) D C B A ______________________ Ans. (a) : The correct match of the prose writers with their works are Jean Froissart - Froissart’s Chronicles Roger Ascham - Toxophilus Thomas More - History of Richard III Richard Hooker - Of the Laws of Ecclesiastical Polity 4. The poets of the early 16th century, such as Sir Thomas Wyatt, Henry Howard, Sir Walter Raleigh, Sir Philip Sidney, Mary Sidney, Michael Drayton, and Sir John Davies, have in the recent time been called poets. (a) Reformation (b) Pre-Shakespearean (c) Golden (d) Silver RPSC Assistant Professor Exam. 2020

[Exam Date : 06.10.2021

Ans. (d) : The term Silver poets is used for a group of 16th century English poet known for their excellence in writing short, elegant, lyrical poems. The Silver Poets were a group of English writers including Sir Thomas Wyatt, Henry Howard (Earl of Surrey), Sir Philip Sidney, Sir Walter Raleigh, and John Davis. __________ 5. Match the sonnet sequence with their authors: i. Edmund Spenser ii. Samuel Daniel iii. Michael Drayton iv. Henry Constable A. Idea B. Amoretti D. Sonnets to Delia C. Diana iii iv i ii D C (a) B A D A (b) B C A C (c) B D D B (d) C A Ans. (c) : Amorettithis a sonnet cycle written by Edmund Spenser in the 16 Century. The cycle describes his courtship and eventual marriage to Elizabeth Boyle. Amoretti was first published in (1595) in London by William Ponsonby. It was printed as part of a volume entitled Amoretti and Epithalamion. Samuel Daniel’s sonnet sequence Delia (1591). Michael Draytons’ Idea is one of the major Elizabethan Sonnet sequences, reprinted here in an attractive new edition. Idea is a sonnet cycle of love poetry and some of the finest verse in the English Language. Henry Constable (1562 October 1613) was an English poet, known particularly for Diana, one of the first English Sonnet sequences. 6. Which of the following is a feature of the Renaissance? i. Individualism ii. Interest in History iii. Ptolemaic Astronomy iv. Sprezzatura (a) All of these (b) i and ii _____(c) i, iii and iv ________(d) i, ii and iv________ Ans. (d) : Characteristics of the ‘Renaissance’ include a renewed interest in classical antiquity; a rise in humanist philosophy (a belief in self, human worth and individual dignity); and radical changes in ideas about religion, politics, and science. _____________________ 7. The phrase ’’God's anointed” in Absalom and Achitophel endorses . (a) the politics of the Whigs (b) the politics of the Tory (c) divine Right Theory of Kings _____(d) the ideology of Protestantism _____________ Ans. (d) : Absalom and Achitophel was written by John Dryden. The poem is accidentally inconclusive because it projects its ending into the future: "Shoud conclude with the Reconcilement of Absalom to David. ’’These proportion together constitute the legend of the unsuccessful ending of ‘Absalom and Achitophel’. The phrase ‘God’s anointed’ means to bring physical harm or death.

72

YCT

8.

Who is called ’’the greatest man of a little age”? (a) John Milton (b) John Dryden _____(c) Charles II _________(d) None of these Ans. (b) : Dryden is called as, “the greatest man of a little age”, because Dryden was the dominant literary figure and influential person of his age. He established the ‘Heroic Couplet’ as a standard form of English poetry by writing successful satires, religious pieces, Fables, Epigrams, Prologues and play with it. He also introduced the Alexandrine and Triplet into the forms in English Literature. ____________________________ 9. was the year of the Popish Plot. (a) 1678 (b) 1668 (c) 1665 _____________(d) 1664 ____________ Ans. (a) : The Popish Plot was a fictitious conspiracy invented by Titus Oates between 1678 to 1681 gripped the kingdom of England and Scotland in Anti-Catholic hysteria. ______________________________________ 10. was the patron of John Locke. (a) Earl of Shaftesbury (b) Duke of Monmouth (c) Duke of Buckingham _____(d) None of these _______________________ Ans. (a) : Earl of Shaftesbury was the patron of John Locke. He was a prominent English politician during the interregnum and the region the king Charles IL 11. In The Conquest of Granada, who conquers Granada? (a) the English (b) the French _____(c) the Spaniards ______(d) the Portuguese Ans. (c) : The Conquest of Granada of the Spaniards is a full title of Dryden’s ‘The Conquest of Granada’, in two parts (1670, 1671), had all the requisite elements of poetry, battle, courage, death, and murder. It is notable both as a defining example of the "heroic drama" pioneered by Dryden, and as the subject of later satire. The plot deals with the Spanish conquest of Granada in 1492 and the fall of Muhammad XII of Granada, the last Islamic ruler on the Iberian peninsula. ___________ 12. The 18 century Gothic novel The Old English Baron was written by . (a) Fanny Burney (b) Clara Reeve _____(c) Anne Radcliffe _____(d) Maria Edgeworth Ans. (b) : The Old English Baron is an early Gothic novel by the English author Clara Reeve. Clara Reeve (23 January 1729-3 December 1807) was an English novelist best known for the Gothic novel, The Old English Baron (1777). She also wrote an innovative history of prose fiction The Progress of Romance (1785). Her first work was a translation of Latin. 13. Which of Laurence Sterne’s novels was incomplete at the time of his death? (a) A Sentimental Joumey(b) Tristram Shandy _____(c) Both of these _ _ _ _ _(d) Neither of these Ans. (a) : A Sentimental Journey was the novel which was incomplete at time of Sterne’s death. He is best known for his novels, The Life and Opinions of Tristram Shandy, Gentleman and A Sentimental Journey, Through France and Italy. ________________ 14. Who wrote the poem ’’Jerusalem” which was termed ”a perfectly mad poem” by Robert Southey? (a) John Keats (b) Walter Scott (c) Samuel Johnson (d) William Blake RPSC Assistant Professor Exam. 2020

Ans. (d) : William Blake wrote poem ‘Jerusalem’ which was termed "a perfectly mad poem" by Robert Southey. He drew illustrations for Madoc, is an 1805 epic poem composed by Robert Southey. It is based on the legend of Madoc, a supposed Welsh prince who fled and sailed to America in the 12th century. 15. Who is considered to be the ’Dickens of the Elizabethan stage’ for his intimate knowledge of the common man? (a) Thomas Dekker (b) Christopher Marlowe (c) William Shakespeare _____(d) Ben Jonson _______________________ Ans. (a) : Thomas Dekker is considered the ‘Dickens of the Elizabethan Age’. Thomas Dekker was a play writer and a pamphlet writer of the Elizabethan age. The Elizabethan Age also referred to ‘as the golden age of England’ was a time of peace and prosperity in England. Renaissance took place between 1558 to 1603 and is named for ‘Queen Elizabeth I’ This age is also well known because of the outpouring of work from William Shakespeare. However, many other writers and artist also had significant work and success during this time. _________________________________________ 16. Who of the following wrote a Mock-Elegy on the Death of Horace Walpole’s Cat? (a) Charles Lamb (b) Lewis Carroll _____(c) Thomas Gray ______(d) Walter Scott______ Ans. (c) : Thomas Gray was the poet of Transitional Period and also known as Graveyard Poet, wrote a Mock-Elegy ‘On the Death of Walpole’ Cat’. He also wrote ‘Ode on the Death of a favorite Cat, Dr. Owned in a Tub of Gold Fishes’, a mock elegy concerning Horace Walpole’s cat. ___________________________ 17. Wordsworth states that the central aim of the poem in the Lyrical ballads is to ’’choose incidents and situations from life”? (a) everyday (b) rustic _____(c) common __________(d) ordinary _________ Ans. (c) : Wordsworth wrote his introduction as a way of explaining why the poems in "Lyrical Ballads" were so different from popular poetry that had come before. In ‘Lyrical Ballads’, aim of the preface, is the primary object which Wordsworth proposed to propagate through the poems were to select incidents and situations from common life. ______________________ 18. What according to Coleridge is ’’the living power and prime Agenda of all Human perception”? (a) Fancy (b) Primary imagination (c) Secondary imagination _____(d) None of these ________________________ Ans. (b) : According to Coleridge Primary Imagination is the living power and prime agenda of all human perceptions. It was mentioned in Biographia Literaria, where he analyzed Primary and Secondary imagination and the concept of Fancy and Imagination. __________________________________ 19. There is a strange union of change and stasis in ”To Autumn” which Keats himself termed as

73

(a) Stationing (c) Flux

(b) Freezing (d) Repose YCT

Ans. (a) : To Autumn is an ode by the English Romantic poet John Keats, written in 1819. In the words of Walter Jackson Bate, ” a union of process and stasis”, ’’energy caught in repose”, an effect that Keats himself termed "stationing". The poet wants to describe Autumn as a period of sadness which comes after happy Spring. So, the poet wants to say that happiness comes after sadness and sadness comes after happiness. __________ 20. Who said, ’’Poetry redeems from decay the visitations of the divine in the man”? (a) William Godwin (b) P.B. Shelley (c) Matthew Arnold (d) Leigh Hunt Ans. (b) : P.B. Shelley thought that "Poetry redeems from decay the visitations of the divine in man". It also shows that poetry can help people to express themselves. This quote is taken from Shelley’s A Defence of Poetry. ______________________________ 21. Who authored the long essay commenting on the ’Porter Scene’ in Macbeth? (a) Richard Steele (b) Thomas de Quincey _____(c) Oliver Goldsmith (d) Dover Wilson Ans. (b) : On the Knocking at the Gate in Macbeth is an essay in Shakespearean criticism by the English author Thomas De Quincey, first published in the October (1823) edition of the London Magazine. It has been called "Thomas De Quincey’s finest single critical piece" and "one of the most penetrating critical footnotes in our literature. The reference has been taken from Macbeth, concerns Act II, scene III. ____________ 22. Harold Skimpole in Bleak House is a caricature of _ _ _. (a) Leigh Hunt (b) William Hazlitt _____(c) Charles Lamb ______(d) Thomas de Quincey Ans. (a) : Harold Skimpole in Bleak House is a caricature of Leigh Hunt. He was a musician who was always sponging of his friend. Dickens was believed to have based this character on the author Leigh Hunt, who was much offended. 23. The New Pygmalion is a work by _____ (a) G.B. Shaw (b) William Hazlitt _____(c) G.K. Chesterton (d) J.B. Priestley _____ Ans. (b) : The New Pygmalion is a work by William Hazlitt. In Greek mythology Pygmalion is a legendary figure of Cyprus, who was a King and a Sculptor. He is the most familiar from Ovid's narrative poem Metamorphoses, in which Pygmalion was a sculptor, who fell in love with a statue he had carved. 24. Which poet illustrated empathy in these lines, "If a sparrow comes before my window. I take part in its existence and pick about the gravel"? (a) Tennyson (b) Keats (c) Wordsworth (d) Shelley Ans. (b) : Keats once wrote, “If a sparrow................. pick about the gravel.” This suggests that Keats was always on the lookout for beautiful things wherever they occurred. Keats seemed to believe its easy to see beauty in a majestic mountain or a stunning sunset, but the activities of a common bird pecking at a Window contain just as much beauty. ______________________ 25. Mary Shelley’s Frankenstein was published in 7a)~”1790 (b) 1806 (c) 1818 (d) 1827 RPSC Assistant Professor Exam. 2020

Ans. (c) : Mary Shelley’s Frankenstein was published in 1818. The sub-title of Frankenstein is The Modern Prometheus. Frankenstein tells the story of Victor Frankenstein, a young scientist who creates a Sapient Creature in an unorthodox scientific experiment. 26. Arrange the following chronologically 1. "Ode to Nightingale" 2. "Ode to Indolence" 3. "To Autumn" 4. "Ode to Grecian Urn" 5. "Ode to Psyche" 6. "Ode to Melancholy" (a) 4, 2, 6, 1,5, 3 (b) 1,2, 4, 5, 6, 3 (c) 1,5, 2, 4, 6, 3 (d) 1,4, 2, 5, 6, 3 Ans. (a) : In 1819, John Keats composed six odes, which are among his most famous and well-regarded poems. Keats wrote the first five poems: Ode on a Grecian Urn (1819 pub.), Ode on Indolence(1848pub.), Ode on Melancholy (1819pub.), Ode to a Nightingale (1819pub.) and Ode to Psyche(1819pub.) in quick succession during the spring, and he composed To Autumn (1820 pub.)in September. __________________ 27. In which society does Arnold place himself in Culture and Anarchy? (a) Barbarians (b) Philistines (c) Populace _____(d) A fourth class that is yet to emerge ________ Ans. (b) : In Culture and Anarchy, Matthew Arnold places himself as a Philistines. The term Philistines, Arnold, seeing himself as a member of the Middle Class. Hence, option (b) is correct. _________________ 28. News from Nowhere, which combines Utopian socialism with science fiction, was written by (a) Wilkie Collins (b) Charles Kingsley _____(c) William Morris _____(d) Charles Reade Ans. (c) : News from Nowhere (1890) is a classic work combining utopian socialism and science fiction written by the artist, designer and socialist pioneer William Morris. He was a founding member of the socialist League, an organization founded in 1885 on the tenets of socialism, a political and economic ideology that advocates for property and natural resources to be owned and regulated by the community or state, rather than private individuals. In ‘New from Nowhere’ the narrator of the book is transported from 19th to 21st century England. 29. Who wrote the "Parliamentary (Pelliser) Novels"? (a) Arnold Bennett (b) Bulwer Lytton (c) Benjamin Disraeli (d) Anthony Trollope Ans. (d) : ‘The Palliser novels’ are novels written in series by Anthony Trollope. These were more commonly known as the Parliamentary novels prior to their (1976) television dramatisation by the BBC broadcast as The Pallisers. ________________________ 30. Life of Charlotte Bronte, one of the best biographies in English, was written by ____. (a) George Eliot (b) Charles Reade _____(c) Mrs. Gaskell _______(d) Anthony Trollope Ans. (c) : The Life of Charlotte Bronte is the posthumous biography of Charlotte Bronte by fellow novelist Elizabeth Gaskell. The first edition was published in 1857 by Smith. A major source was the hundreds of letters sent by Bronte to her lifelong friend Ellen Nussey.

74

YCT

31.

Arrange the following novels chronologically: i. David Copperfield ii. Pickwick Papers iii. Nicholas Nickleby iv. Great Expectations (a) ii, iii, i, iv (b) ii, iv, iii, i _ _ _ _ _(c) iii, ii, iv, i __________(d) iv, i, ii, iii ________ Ans. (a) : The correct chronological order of Charles Dickens’ Novels are : i. Pickwick Papers (1836 - 3 7 ) ii. Nicholas Nickleby (1838 - 39) iii. David Copperfield (1849 - 50) iv. Great Expectations (1860- 61) Thus, the appropriate answer is option (a). ____________ 32. Which of the following poems by Alfred Tennyson begins with the words "I hate 11? (a) "To Virgil" (b) "Maud" (c) "The Idylls of the King" _____(d) "Enoch Arden" __________________________ Ans. (b) : In Maud a poem by Alfred Tennyson begins with the words “I hate”. This poem was composed in 1854 and published in Maud and other Poems in 1855. It is a strange and turbulent Monodrama provoked a storm of protest, many of the poets admirers were shocked by the morbidity, hysteria and bellicosity of the hero. Yet Maud was Tennyson’s favourite among his all poems. _________ 33. Who of the following had an elliptical style? (a) Robert Browning (b) Alfred Tennyson _ _ _ _ _(c) Alexander Pope (d) John Dryden ______ Ans. (a) : Robert Browning is well-known for using elliptical style. It is a literary critical term. Thus, option (a ) will be correct answer. 34. Which of the following novels has a "ChineseBox" structure? (a) Middlemarch (b) Jane Eyre _____(c) Mansfield Park _____(d) Wuthering Heights Ans. (d) : Chinese-Box structure has been used in the novel Wuthering Heights by Emily Bronte. The Novel is organised in such a way that it is a narrative within a narrative what some critics would called Chinese- Box Structure. ______________________________________ 35. In Jude the Obscure. Jude dreams of going to the university at . (a) Westminster (b) Christminster _____(c) Oxford ____________ (d) Cambridge _______ Ans. (b) : Jude the Obscure is a novel written by Thomas Hardy, Jude dreams of Christminster are dreams of discovering his place in the world. He (Jude) is the protagonist of the novel. ______________________ 36. Thackeray’s Vanity Fair is a . (a) bildungsroman (b) romance _ _ _ _ _(c) social satire ________(d) historical fiction Ans. (c) : Thackeary’s Vanity Fair is a social satire,. Its subtitled A Novel Without a Hero. Vanity Fair metaphorically represents the human condition. Vanity Fair, is a novel of early 19 th century English society by William Makepeace Thackeray, published serially in monthly installments from 1847 and in book form in 1848. 37. "Mr. Bennett and Mrs. Brown" is a satirical essay which is one of the several accounts of Virgina Woolf about the state of . (a) contemporary England (b) women's Education RPSC Assistant Professor Exam. 2020

(c) women's Writing _____(d) modem fiction ________________________ Ans. (a) : Mr. Bennett and Mrs. Brown is a satirical essay which is one of the several accounts of Virginia Woolf about the state of contemporary England. Mr. Bennett and Mrs. Brown (1924) is an essay by Virginia Woolf. "Mr. Bennett and Mrs. Brown' is just one of several closely related version of Woolfs account of the state of the modem novel, and it seems appropriate to read the essay with other versions of the arguments of the argument in mind. _________________ 38. is an imaginative biography of Elizabeth Barrett Browning’s cocker spaniel, written by Virginia Woolf. (a) Flush (b) Orlando _____(c) Melymbrosia _______(d) Night and day Ans. (a) : Flush: A Biography is an imaginative biography of Elizabeth Barrett Browning's cocker, spaniel, written by Virginia Woolf. It was written after the completion of her emotionally draining The Waves the work returned Woolf to the imaginative consideration of English history that she had begun in Orlando: A Biography, and to which she would return in between the acts. _______________________________ 39. Which poem by T.S. Eliot presents the thoughts of "an old man in a dry month"? (a) "Ash Wednesday" (b) "Conversation Galante" (c) "Gerontion" _____(d) "Sweeney Among the Nightingales" ________ Ans. (c) : Gerontion is a poem by T.S. Eliot presents the thoughts of an old man in a dry month. The title Gerontion, means in Greek little old man and the poem is a dramatic monologue. __________________________ 40. The Truth about an Author is a playful "literary autobiography" of (a) Arnold Bennett (b) P.G. Wodehouse _____(c) G.K. Chesterton (d) Somerset Maugham Ans. (a) : The Truth about an Author is a playful ‘literary autobiography’ of Arnold Bennett. Arnold Bennett (27 May 1867-27 March 1931) was an English author, best known as a novelist. He was a prolific writer between the start of his career in 1898 and his death he completed 34 novels, seven volumes of short stories, 13 plays (some in collaboration with other writers) and a daily journal totalling more than a million words. He wrote articles and stories for more than 100 different newspapers and periodicals and wrote for the Cinema in the 1920's. 41. Which of the following is Conrad’s novel not set against the backdrop of seafaring? (a) Lord Jim (b) Typhoon (c) The Rover (d) Under Western Eyes Ans. (d) : Joseph Conrad’s novel Under Western Eyes is not set against the backdrop of Seafaring. The novel takes place in St. Petersburg, Russia and Geneva, Switzerland and is viewed as Conrad’s response to the themes explored in Fyodor Dostoevsky’s Crime and Punishment. Joseph Conrad was a Polish-British writer regarded as one of the greatest novelists to write in English language. Though he did not speak English fluently until his twenties, he came to be regarded a master prose stylist who brought a non-English sensibility into English literature.

75

YCT

42.

Who of the following authors greatly (d) Murder in the Cathedral, The Importance of Being Earnest, The Riders to the Sea, Back to appreciated French culture and submitted too willingly to the influences of French realism? ________Methuselah. ___________________________ (a) Arnold Bennett (b) Joseph Conrad Ans. (c) : The correct chronological orders are _____(c) E.M. Forster _______(d) John Galsworthy The Importance of Being Earnest (1895) Ans. (a) : Arnold Bennett was greatly appreciated for The Riders to the Sea (1904) French Culture and submitted to willingly to the Back to Methuselah (1922) influences of French realism of a well defined current. Murder in the Cathedral (1935) He is making a distinction between realism like the Thus, correct option is (C). _______________________ French realists, Bennett too owes much of the 47. Who is the narrator in Melville’s Moby Dick? authenticity of his works. _________________________ (a) Captain Ahab (b) Elijah 43. Which of the following works is not by _____(c) Ishmael ___________(d) Gabrial __________ Somerset Maugham? Ans. (c) : The narrator of Moby Dick is Ishmael, a man 1. Corridors of Power who witnesses Ahab’s guest to hunt Moby Dick. As the 2. Liza of Lambeth narrator of Moby Dick, Ishmael has long provoked 3. The Razor’s Edge critical problems. He disappears from the dramatic 4. An Appointment as Sor action midway through the book, he reports scenes and (a) All of these are by Maugham (b) 4 and 5 conversations which he could not have witnessed and _____(c) 3 alone________ __________(d) 1 and 4 his style varies erratically. ________________________ Ans. (*) : Corridors of Power is a work by C.P. Snow. 48. Who is the representative figure of the ’’Jazz It is not work of William Somerset Maugham. This Age”? book is the 9th book in C.P. Snow’s, Strangers and (a) Sherwood Anderson (b) F. Scott Fitzgerald Brothers series. It’s little had become a household _____(c) Saul Bellow _______(d) Wallace Stevens phrase referring to the centres of government and power Ans. (b) : ‘The Jazz Age’ was the term coined by F. after snow coined it in his earlier novel Homecomings. Note:- This particular the question was dropped by Scott Fitzgerald to describe the flamboyant anything RPSC because of some discrepancies. _______________ goes culture that characterized the 1920s. 44. Edward Thomas, W.H. Davies and John This term was popularized with the publication of his Drinkwater were major writers of the _____ short story collection ‘Tales of the Jazz Age’._________ 49. American Civil War was fought in period. (a) 1815-1820 (b) 1830-1840 (a) Edwardian (b) Georgian (c) 1861-1865 ________(d) 1825-1833 _______ (c) I World War _______(d) II World War Ans. (b) : Edward Thomas, W.H Davies and John Ans. (c) : The American Civil War (April 12, 1861Drink Water were major writers of the Georgian period. Mays, 1865) was a Civil War in the United States Georgian Poetry, a variety of lyrical poetry produced in fought between the Union (states that remained loyal to the early 20 th century by an assortment of British poets, the federal union or ’’the North”) and the confederacy. including Lascelles Abercrompie, Hilaire Belloc, 50. Who coined the phrase ’’Lost Generation”? (a) Hemingway (b) Gertrude Stein Edmund Charles Blunden, Rupert Brooke, William Henry Davies, Ralph Hodgson, John Drinkwater, James _____(c) F. Scott Fitzgerald (d) Sherwood Anderson Elroy Flecker, Wilfred Wilson Gibson. Ans. (b) : Gertrude Stein is credited with coining the 45. The Dog Beneath the Skin, or Where is term and it was subsequently popularized by Ernest Frances? is a play by _____. Hemingway who used it in the epigraph for his 1926 (a) A.A. Milne (b) Elmer Rice novel. The Sun Also Rises. _____(c) Auden and Isherwood (d) John Masefield 51. Who says the ’’Earth is the right place for Ans. (c) : The Dog Beneath the Skin or Where is love”? Francis?\ is a play in three acts, by W.H. Auden and (a) Sylvia Plath (b) Langston Hughes Christopher Isherwood, was the first collaboration of _____(c) Wallace Stevens (d) Robert Frost______ Auden Isherwood and an important contribution to Ans. (d) : Robert Frost says that "Earth's the right place English poetic drama in the 1930s. It was published in for love”. The speaker says that he was also a swinger 1935 and first performed by the group theatre in 1936. of birches when he was a boy and wishes to be so now. 46. Which of the following is in the correct when he becomes confused, he would like to go toward’s chronological order? heaven by climbing a birch tree and then coming back 1. The Importance of Being Earnest again, because earth is the right place for love. 2. The Riders to the Sea 52. In Saul Bellow’s novel Herzog (1964), Moses 3. Back to Methuselah Herzog is a 4. Murder in the Cathedral (a) Christian (b) Hindu (a) The Riders to the Sea, The Importance of _____ (c) Jew ______________ (d) Afro-American Being Earnest, Back to Methuselah, Murder Ans. (c) : Herzog(1964) is a novel by Saul Bellow, in the Cathedral. (b) Back to Methuselah, The Importance of composed in part of letters from the protagonist Moses Being Earnest, The Riders to the Sea, Murder E. Herzog belongs to Jew family. It won the U.S. National Book Award for Fiction and the Prix in the Cathedral. (c) The Importance of Being Earnest, The Riders International in 2005, Time Magazine named it one of to the Sea, Back to Methuselah, Murder in the the 100 best novels in the English language since time’s founding in 1923. Cathedral. YCT RPSC Assistant Professor Exam. 2020 76

53.

When did Hemingway receive the Nobel Prize for Literature? (a) 1952 (b) 1954 (c) 1956 _________ (d) 1958 ____________ Ans. (b) : The Nobel Prize in Literature 1954 was awarded to Ernest Miller Hemingway "for his mastery of the art of narrative, most recently demonstrated in The Old Man and the Sea, and for the influence that he has exerted on contemporary style." ________________ 54. What was the original title of Hemingway’s novel The Old Man and the Sea? (a) Fiesta (b) The Assistant _____(c) The Sea in Being (d) A Farewell to Arms Ans. (c) : The original title of Hemingway’s novel The Old Man and The Sea is ‘The Sea in Being’ (1952). It was the last major work of fiction written by Hemingway that was published during his lifetime. 55. Who is the central character in Hemingway’s novel The Old Man and the Sea? (a) Santiago (b) Marlin _____(c) Manolin __________(d) None of these _____ Ans. (a) : The central character is an old Cuban fisherman named Santiago, who has not caught a fish for 84 days. The family of his apprentice, Manolin, has forced the boy to leave the old fisherman, though Manolin continues to support him with food and bait. 56. Who is the narrator in F. Scott Fitzgerald’s novel The Great Gatsby (1925)? (a) Gatsby (b) Nick _____(c) Buchannan ________(d) None of these Ans. (b) : The narrator in F. Scott Fitzgerald’s novel The Great Gatsby is Nick Carrayway, a Yale University graduate from the Midwest who moves to New York after World War I to pursue a career’s in Bonds. _______ 57. Which one is a great patriotic poem by Frost? (a) "Mending Wall" (b) "Birches" (c) "The Gift Outright" (d) "Directive" Ans. (c) : Frost has summarized in ’The Gift Outright’, a short poem of 16 lines, the history of colonization in America and the growth of love and devotion that the settlers came to shower on this land. It is one of the best patriotic poems ever written about America and the American people. 58. Who is the central figure in O’ Neill’s The Hairy Ape? (a) Mildred (b) Yank (c) The Secretary of I.W.W. _____(d) None of these _________________________ Ans. (b) : The Hairy Ape is a 1922 expressionist play by American playwright Eugene O'Neill. It is about a beastly, unthinking laborer known as Yank, the protagonist of the play, as he searches for a sense of belonging in a world controlled by the rich. 59. Absalom, Absalom! is a novel written by (a) Steinback (b) Faulkner _____(c) Hemingway _______(d) Fitzgerald ________ Ans. (b) : Absalom, Absalom\ is a novel by the American author William Faulkner, first published in 1936._________________________________________ 60. The poem ’Chicago’ is written by (a) Ezra Pound (b) E.E. Cummings (c) Carl Sandburg (d) Carlos William RPSC Assistant Professor Exam. 2020

Ans. (c) : The poem Chicago is written by Carl Sandburg is a poem of admiration and self-defense. The poem itself is a typical example of middle class life that existed in Chicago. ______________________________ 61. Native Son (1940) is written by (a) Jean Toomer (b) Richard Wright _____(c) Ralph Ellison ______(d) Stephen Crane Ans. (b) : Native Son (1940) is a novel written by the American author Richard Wright. It tells the story of 20 year old Thomas, a black youth living in utter poverty in a poor area on Chicago's South side in the 1930s. 62. Which American poet is hailed as the representative poet of America? (a) Robert Frost (b) R.W. Emerson _____(c) Walt Whitman _____(d) Edgar Allan Poe Ans. (c) : Walt Whitman is hailed as the representative poet of America, because as a humanist, he was a part of the transition between transcendentalism and realism, incorporating both views in his works, Walt Whitman among the most influential poets in the American canon, often called the father of free verse. ___________ 63. Arthur Miller’s Death of A Salesman premiered on Broadway in ____ (a) 1945 (b) 1947 (c) 1949 _____________(d) 1948 ____________ Ans. (c) : Death of a Salesman is a 1949 stage play written by American playwright Arthur Miller. The play premiered on Broadway in February 1949, running for 742 performances. It won the 1949 Pulitzer Prize for drama and Tony Award for the best play. It is considered by some critics to be one of the greatest plays of the 20th century. _________________________ 64. Who was the first black woman who won the Nobel Prize for Literature? (a) Toni Morrison (b) Jane Austen _____(c) Ann Petry _________(d) Frances Harper Ans. (a) : Toni Morrison was the first black woman who received for Nobel Prize for Literature in 1993. Her critical essay is Playing in the Dark : Whiteness and the Literature Imagination (1922). She emphasizes that "My work requires me to think about how free I can be as an African-American woman writer in my genderized, sexuality wholly racialized world." 65. Frost’s poem The Road Not Taken is included in his poetical collection (a) A Boy’s Will (b) A Witness Tree _____(c) North of Boston (d) Mountain Interval Ans. (d) : The Road Not Taken is a narrative poem by Robert Frost, first published in the August 1915, issue of The Atlantic Monthly, and later published as the first poem in the collection Mountain Interval (1916). 66. Rabbit Angstrom novels are written by (a) Harper Lee (b) John Updike _____(c) Henry Miller (d) R. Ellison ________ Ans. (b) : Rabbit, Run is a (1960) novel by John Updike. Four novels by John Updike are Rabbit, Run (1960) and its sequels. Rabbit Angstrom is an ordinary middle-class man lost in the sterility of the modem world. He is protagonist of the novel. 67. One of the functions of protest poetry was to

77

(a) urge African Americans to fight their oppressors. (b) encourage society's strife for equality for all. YCT

of unnecessary violence by white people onto innocent (c) extol the virtues of living in the free North. (d) argue that slavery was not so bad for black people standing up for their rights. ________everyone. _____________________________ Hence, the correct option is (D). ___________________ Ans. (b) : ‘Protest poetry’ is any form of poetry which 72. Who wrote Incidents in the Life of a Slave Girl? (a) Lucy Terry (b) William Wells Brown has, as one of its main function, the objective of finding fault with some existing current event or circumstance. _____(c) Harriet Wilson (d) Harriet Jacobs _______ The kind of poetry often focuses on the misdeeds Ans. (d) : In 1861 Harriet Jacobs, the first African performed by a government upon its people. American female slave to author her own narrative, 68. The fact that Claude McKay visited Russia in published Incidents in the Life of a Slave Girl, which 1922 exemplifies the following theme of depicted her resistance to her masters’ sexual exploitation and her ultimate achievement of freedom Modernism? (a) Collectivism versus the authority of the for herself and her two children. ___________________ individual. 73. Race relations in the North are attacked in ? (b) The wearing away of traditional class (a) Harriet Jacob’s Incidents in the Life of a Slave structures. Girl. (c) The impact WWI and the 1918 Bolshevik (b) Harriet Wilson’s Our Nig. Revolution in Russia. (c) William Well’s Brown’s Clotel. _____(d) The disassociated, anomic self.____________ _____(d) Toni Morrison’s Beloved ________________ Ans. (c) : The fact that Claude McKay visited Russia in Ans. (b) : The year 1859 was a year of important first 1922 exemplifies the impact of WWI and the 1918 for African-American women writing and Harriet E. Bolshevik Revolution in Russia. Festus Claudius “Claude Wilson’s Our Nig became the first novel by an AfricanMckay” was a Jamaican American writer and poet. He American to be published in the United States. Harriet became a central figure in the Harlem Renaissance. E. Wilsons addressed race relations in the North. She 69. The theme of Phillis Wheatley’s "On Being extended the slave narratives attack on chattel relation below the Mason-Dixon line by offering a scathing Brought from Africa to America" is . (a) Slaves are capable of becoming good revelation of northern racism. Christians. 74. Who introduced the character of the "Tragic (b) Slaves should rebel against the Christian mulatto"? religion. (a) William Wells Brown (c) Slaves are the children of Cain. (b) Lydia Maria Child _____(d) Christians should free their slaves. _________ (c) Harriet Jacobs Ans. (a) : The theme of Phillis Wheatley’s On Being _____(d) Harriet Beecher Stowe __________________ Brought mixes themes of slavery, Christianity, and Ans. (b) : Lydia Maria Child’s 1842 short story The salvation, and although its unusual for Wheatley to Quadroons is generally credited as the first work of write about being a slave taken from Africa to America literature to feature a Tragic mulatto, to gamer support this poem strategically addresses ideas of liberty, for emancipation and equal rights. religion, and racial equality._______________________ The "Tragic mulatto" is an archetypical mixed 70. Why is the couple in Arna Bontemps’s "A race person (a ’’mulatto’’), who is assumed to be sad or even suicidal because he/she fails to completely fit in Summer of Tragedy" getting dressed up? the ’’white world” or the "black world". (a) To go to a party. (b) To go pay old man Stevenson. 75. Until recent years it was thought that Harriet (c) To end their lives Jacobs* Incidents in the Life of a Slave Girl was _____(d) To go to church ________________________ (a) based on a New England captivity narrative. Ans. (c) : The story, A Summer of Tragedy was written (b) an anonymous narrative. in 1933 by Arna Bontemps. It was written at a time of (c) fiction written by Lydia Maria Child. great economics depression and color segregation in _____(d) written by Jacob’s son. __________________ America. The story begins as the couple is dressing up Ans. (c) : Incidents in the Life of a Slave Girl, written to go for a journey. It is not a normal journey however, by herself, is an autobiography by Harriet Jacobs, a as Jeff Shakes and trembles and trembles at the thought mother and fugitive slave, published in 1861 by L. of it. It’s journey continues to end their lives. Maria Child, who edited the book for its author. 71. Sonia Sanchez’s "Right On : White America" 76. W.E.B. Du Bois argued that a liberal arts is protesting . college education was needed for . (a) the extermination of Native Americans (a) the "Talented Tenth" (b) that there is a Black America and a White (b) all African Americans America. (c) African American women _____(d) only white Americans ___________________ (c) black on black violence (d) the fact that America still has a frontier Ans. (a) : William Edward Burghardt Du Bois (W.E.B. ________mentality. ____________________________ Du Bois) argued that a liberal arts college education Ans. (d) : Sonia Sanchez’s Right on: White America is was needed for the ’Talented Tenth’. protesting the fact that America still has a frontier Talented Tenth, (1903) concept, espoused by black mentality. Her many famous protest poem, one of the educator and author W.E.B. Du Bois, emphasizing the most famous is called Right on : White America and necessity for higher education to develop the leadership was written around 1970. This poem discusses America capacity among the most able ten percent (10%) of still being a “pioneer land”. In America, there was a lot black Americans. RPSC Assistant Professor Exam. 2020

78

YCT

77.

Who won the Nobel Prize for Literature in 1973. (a) Patrick White (b) A.B. Facey _____(c) Morris West _______(d) David Malouf Ans. (a) : The Nobel Prize in Literature (1973) was awarded to Patrick White "for an Epic and psychological narrative art which has introduced a new continent into literature.” __________________________ 78. The Playmaker, The Chant of Jimmie Blacksmith and Bring Larks and Heroes are novels written by (a) Peter Carey (b) Patrick White _____(c) David Malouf ______(d) Thomas Keneally Ans. (d) : Thomas Keneally (bom October 7, 1935, Sydney, Australia), an Australian writer, best known for his historical novels. His reputation as a historical novelist was established with Bring Larks and Heroes (1967), about Australia's early years as an English penal colony. 'The Chant of Jimmie Blacksmith (1972; film 1980)' Keneally international acclaim; it is based on the actual story of a half-caste Aboriginal who rebels against white racism by going on a murder spree. 79. Who is now one of Australia’s best-known women writer in whose honour a prestigious writing award is named? (a) Dymphna Cusack (b) C.J. Dennis _ _ _ _ _(c) Nancy Keesing _____(d) Miles Franklin Ans. (d) : Stella Maria Sarah Miles Franklin (14 October 1879 - 19 September 1954), known as Miles Franklin, was an Australian writer and feminist who is best known for her novel My Brilliant Carrier, published by Blackwood's of Edinburgh in 1901. ______ 80. Johnno (1975) was the first novel of _____ (a) Morris Lurie (b) Robert Drewe _ _ _ _ _(c) Peter Carey ________(d) David Malouf Ans. (d) : Johnno is a semi-autobiographical novel written by Australian author David Malouf and was first published in 1975. It was Malouf s first novel. 81. Who wrote ’The Idea of perfection’ which won the 2001 Orange Prize for Fiction and ’The Secret River’ which was a story based on her own convict ancestors? (a) Elizabeth Jolley (b) Janette Turner Hospital (c) Elizabeth Stead _____(d) Kate Grenville __________________________ Ans. (d) : Catherine Elizabeth Grenville (bom 1950) is an Australian author. She has published fifteen books including fiction, non-fiction, biography and books about the writing process. In 2001, she won the Orange Prize for The Idea of Perfection, and in 2006 she won the Commonwealth Writers' Prize for The Secret River. 82. Which among the following is not written by Atwood? (a) Surfacing (b) The Blind Assassin _____(c) The Edible Woman (d) In the Skin of a Lion Ans. (d) : In the Skin of a Lion is a novel by CanadianSri-Lankan writer Michael Ondaatje. It was first published in 1987 by McClelland and Stewart. The novel fictionalizes the lives of the immigrants who played a large role in the building of the city of Toronto in the early 1900s. In the Skin of a Lion is thus an expose of the migrant condition. "It is a novel about the wearing and the removal of masks; the shedding of skin, the transformations and translations of identity. RPSC Assistant Professor Exam. 2020

83.

Which 2005 Australian novel is narrated by Death? (a) Shantaram (b) The Book Thief _ _ _ _ _(c) A Town Like Alice (d) The Potato Factory Ans. (b) : The Book Thief is a historical novel by the Australian author Markus Zusak, and it is one of his most popular works published in 2005. The novel is about the adventures of Liesel Meminger in Nazi Germany during the Second World War. The Book Thief is narrated by Death, the day after the death of the main character, Liesel. Death is on omniscient narrator who switches between first person and a closed third person point of view. ______________ 84. Sinclair Ross* great novel As for Me and My House is set during which era? (a) Early 1920s (b) Turn of the century (c) The Great War _____(d) The Great Depression ____________________ Ans. (d) : Sinclair Ross' great novel As for Me and My House is set during The Great Depression era in a prairie town that mimics the Canadian prairies. As a result the story takes place on the backdrop of a period that was facing harsh social economic changes. 85. Which of the following novels by Michael Ondaatje won the Booker Prize? (a) Collected works of Billy the Kid (b) In the Skin of a Lion (c) The English Patient _____(d) Running in the Family ___________________ Ans. (c) : "The English Patient", the wartime love story by Michael Ondaatje, won the Golden Man Booker Prize. Note : The Golden Man Booker was one of the prizes awarded in commemoration of the 50th anniversary of the booker Prize. 86. Who is the writer of the novel Family Matters? (a) Salman Rushdie (b) Amitav Ghosh _ _ _ _ _(c) Rohinton Mistry (d) Michael Ondaatje Ans. (c) : Family Matters is the third novel, published by Indian bom author Rohinton Mistry. It was first published by McClelland and Stewart in 2002. Subsequent editions were published by Faber in UK. In a tiny two-room flat in modern-day Mumbai and presented a compelling portrayal of a family of Parsis living in exigent circumstances. _____________________ 87. Name the first novel of Mulk Raj Anand which carried a preface by E.M. Forster that won him world-wide recognition (a) Coolie (b) The Village _____(c) Untouchable (d) Two Leaves and a Bud Ans. (c) : Mulk Raj Anand wrote his first novel, ’Untouchable', published in 1935, is a chilling expose of the lives of India's Untouchable Caste. The introduction of the novel was written by his friend E.M. Forster, whom he met while working on T.S. Eliot's Magazine Criterion. Mulk Raj Anand wrote about publication of his novel Untouchable to his friend K. Natwar Singh, "as you know this book was written in Gandhiji's Ashram in 1927 was turned down by 19 publishers in U.K. Note:- RPSC has considered option (b) as the right answer but in reality option (c) will be correct answer because the Preface of Untouchable was written by E.M. Forster.

79

YCT

88.

The Old Woman and the Cow by Mulk Raj Anand shows the transformation of a cow into a (a) Lioness (b) Tigress _____(c) Buffalo ___________(d) Goddess _________ Ans. (b) : ’The Old Woman and the Cow’ by Mulk Raj Anand shows the transformation of a cow into a Tigress. The Old woman and The Cow is the only one of Anand’s novel to have a woman as its central and to present women's problems and sufferings, The novel shows the transformation in the character of Gauri. Note:- RPSC has accepted option (c) as a correct answer but the correct answer will be option (b) because in this particular work, Anand shows the transformation of a cow into a tigress. ___________________________ 89. Who was the first recipient of the Sahitya Akademi Award for the best writing in English? (a) R.K. Narayan (b) Mulk Raj Anand _____(c) Raja Rao ________(d) Khuswant Singh Ans. (a) : The Sahitya Akademi Awards in English began in 1960. The first recipient was R.K. Narayan for his novel The Guide. ____________________________ 90. Who wrote an Introduction to R.K. Naryan’s novel The Financial Expert? (a) Graham Greene (b) E. M. Forster _____(c) Mulk Raj Anand (d) Rajaj Rao ________ Ans. (a) : The Financial Expert is a novel by R.K. Narayan, with a two page introduction by ’Graham Greene’, published in 1952. The novel tells the story of the rise and fall of ’Margayya’ (central character). Narayan's sixth novel, The Financial Expert, is his masterpiece, and Walsh calls Margayya, the hero of the novel, "Probably Narayan's greatest single comic creation". 91. Who said, ’’Ezekiel is a poet of the city, Bombay”? (a) K. S. Iyengar (b) Linda Hess _____(c) R. Parthasarthy _____(d) KamalaDass _____ Ans. (b) : ’Linda Hess’ said, "Ezekiel is a poet of the city, Bombay" perhaps, because his most of the works' plot is based on Bombay. Nissim Ezekiel (16 December 1924 -9 January 2004) was an Indian Jewish poet, actor, playwright, editor and art critic. He was a foundational figure in postcolonial India's literary history, specifically for Indian poetry in English. ______ 92. Which one of R. K. Narayan's novels became the most popular one? (a) The Guide (b) The Bachelor of Arts (c) The Vendor of Sweets _____(d) The Financial Expert____________________ Ans. (a) : The Guide (1958) one of the R.K. Narayan's novels became the most popular one. Among the bestreceived of R.K. Narayan's 34 novels are The English Teacher (1945) Waiting for the Mahatma (1955), The Guide (1958), The Man Eater ofMalgudi (1961), The Vendor of Sweets (1967) and A Tiger for Malgudi (1983). _______________________________________ 93. Rabindranath Tagore got Nobel Prize in (a) 1912 (b) 1913 (c) 1917 _____________(d) 1923 ____________ Ans. (b) : Rabindranath Tagore won the Nobel prize for Literature in (1913) for his collection Gitanjali published in London in 1912. The prize gained even RPSC Assistant Professor Exam. 2020

more significant by being given to an Indian for the first time. This honour established Tagore's Literary reputation worldwide. ___________________________ 94. Mulk Raj Anand's autographical novel is (a) Seven Summers (b) The Untouchable (c) The Coolie _____(d) Twop Leaves and a Bud _________________ Ans. (a) : In 1950 Mulk Raj Anand embarked on a project to write a seven part autobiography titled Seven Ages of Man, of which he was only able to complete four parts beginning in 1951 with Seven Summers, followed by Morning Face, "Confession of a Lover" and Bubble. 95. Aurobindo’s Savitri is (a) A romantic poem (b) An epic poem _____(c) A short story _______(d) A religious novel Ans. (b) : Savitri : A Legend and a Symbol is an epic poem in blank verse by Sri Aurobindo, based upon the theology from the Mahabharata. Its central theme revolves around the transcendence of man as the consummation of terrestrial evolution and the emergence of an immortal supermental gnostic race upon earth. The first known manuscript dates from 1916._________________________________________ 96. My Story is (a) The autobiography of Kamala Das (b) A novel by Arun Joshi (c) A play by Girish Kamad _____(d) A volume of poems by Mahapatra _________ Ans. (a) : My Story is an autobiographical book written by Indian author and poetess Kamala Das (also known as Kamala Surayya or Madhavikutty). The book was originally published in Malayalam, titled Ente Katha. The book evoked violent reactions of admiration and criticism among the regards and critic. It remains the best selling woman's autobiography in India. My Story is a chronologically ordered, linear narrative written in a realist style. In the book Kamala Das recounts the trials of her marriage and her painful self-awakening as a woman and writer. ______________________________ 97. Who is the writer of Inquilab: A Play in Three Acts? (a) Nissim Ezekiel (b) Girish Karnad _____(c) Asif Currimbhoy (d) Gieve Patel ______ Ans. (c) : Inquilab: A Play in Three Acts wrote by Asif Currimbhoy (1928-1994) is an Indian Playwright who wrote in English. He was among the very few Indian dramatist writing play exclusively in English. He wrote and/or produced over thirty play in several genres. _____ 98. Who is the "man-eater of Malgudi" in R.K. Narayan's novel The Man-Eater ofMalgudi? (a) Natraj (b) Muthu _____(c) Vasu ____________(d) Kumar __________ Ans. (c) : Vasu is the man-eater ofMalgudi. The title of the novel The Man-Eater of Malgudi is ironic as maneater in the novel is not tiger, but a mighty man, Vasu, who not only kills a number of wild animals, but also kills himself with a single blow of his hammer-like first. The story is narrated in the first person by its tragiccomic hero, Nataraj a printer ofMalgudi. 99. The Golden Gate by Vikram Seth is a novel (a) of sensibility (b) of character (c) for the downtrodden (d) in verse

80

YCT

Ans. (d) : The Golden Gate (1986) is the first novel by poet and novelist Vikram Seth. The work is a novel in verse composed of 590 'Onegin Stanzas' (sonnets written in iambic tetrameter, with the rhyme scheme following the AbAb CC ddE ffEgg pattern of Eugene Onegin). ________

Ans. (b) : The Serpent and the Rope (1960), a semiautobiographical novel recounting a search for spiritual truth in Europe and India, established him as one of the finest Indian prose stylists and won him the Sahitya Akademi Award in 1964. Raja Rao (8 November 100. Chaman Nahal won the Sahitya Akademi 1908- 8 July 2006) was an Indian writer of English language novels and short stories, whose works are Award in 1977 for his famous work deeply rooted in metaphysics. ______________________ (a) Azadi (b) My True Faces 105. Who is the hero of Raja Rao's Kanthapura? (c) Into Other Dawn (a) Laxman (b) Reddy _____(d) The Crown and the Loincloth _____________ _ _ _ _ _(c) Moorthy ___________(d) Naidu ___________ Ans. (a) : Chaman Nahal commonly known as 'C Ans. (c) : Raja Rao's first and best known novel, Nahal’, also known as Chaman Nahal Azadi, was an Kanthapura (1938), is the story of a South Indian Indian bom writer of English literature. He was widely village named Kanthapura. The main character of the considered one of the best exponents of Indian writing novel, Moorthy, is a young Brahmin who leaves for the in English literature, and he is known for his work, city to study, where he becomes familiar with Gandhian Azadi (1977) which is set on India's Independence and philosophy. Moorthy is a "quiet, generous, serene, its partition. For this novel, Chaman Nahal won the deferent" young Brahmin who rejects the hierarchical Sahitya Akademi Award in 1977. __________________ caste system in favor of social equality. ______________ 101. Who is the narrator here in Rushdie's novel Midnight's Children (1981)? (a) Saleem Sinai (b) Natraj _____(c) RlyaBabu _________(d) Akhtar Sahib _____ Ans. (a) : Midnights children (1981) is a novel by author Salman Rushdie. It portrays India's transition from British colonial rule to independence and the partition of Indian. It is considered as an example of postcolonial, postmodern, and magical realist literature. The story is told by its chief protagonist, 'Saleem Sinai', and is set in the context of actual historical events. 102. Jayant Mahapatra is basically a man of ____. (a) Physics (b) Chemistry _____(c) Psychology ________(d) English __________ Ans. (a) : Jayant Mahapatra (bom 22 October 1928) is an Indian English poet, basically he is a man of Physics. He is the first Indian poet to win a Sahitya Akademi award for English poetry. He is the author of poems such as Indian Summer and Hunger, which are regarded as classics in modem Indian English literature. He was awarded Padma Shri the fourth highest Civilian Honour in India in 2009. He returned the award in 2015 to protest against rising intolerance in India. ____________ 103. Arundhati Roy's debut novel The God of Small Things won Booker Prize in (a) 1995 (b) 1996 (c) 1997 ______________(d) 1998 ____________ Ans. (c) : The God of Small Things is a family drama novel written by Indian writer Arundhati Roy. It is Roy's debut novel. It is a story about the childhood experiences of fraternal twins whose lives are destroyed by the Love Laws that lay down who should be loved, and how and how much. The book explores how the small things affect people behavior and their lives. The book also reflects its irony against casteism, which is a major discrimination that prevails in India. It won the Booker Prize in 1997. ___________________________ 104. Raja Rao got the Sahitya Akademi Award in 1964 for his novel (a) Kanthapura (1938) (b) The Serpent and the Rope (1960) (c) The Cat and Shakespeare: A Tale of India (1965) (d) Comrade Kirillov (1976) RPSC Assistant Professor Exam. 2020

106. Anita Desai's Where Shall We Go this Summer has been compared to Virginia Woolfs ____ (a) To the Lighthouse (b) The Voyage Out _____(c) The Waves ________(d) Night and Day Ans. (a) : To the Lighthouse, a novel by Virginia Woolf, was published in 1927. The work is one of her most successful and accessible experiments in the stream of consciousness style. The three section of the book take place between 1910 and 1920 and revolves around various members of the Ramsay family during visits to their summer residence on the Isle of Skye in Scotland. _____________________________________ 107. Jhumpa Lahiri is a _____ (a) writer who excels in science fiction (b) writer who deals with feminist perspectives in her stories. (c) chronicler of Indian American lives within the multicultural context. (d) professor who teaches Creative Writing in _________Boston. ________________________________ Ans. (c) : Jhumpa Lahiri, an English bom American novelist and short story writer whose works illuminate the immigrant experience, in particular that of East Indians. Jhumpa Lahiri is a chronicler of Indian American lives within the multicultural context. 108. What is the parallelism that exists between America and India? (a) One state one country (b) Immigrants from the West Indies (c) One country one climate _____(d) Of welding together into one body many races Ans. (d) : "The parallelism that exists between America and India is of welding together into one body various races". 109. Which one of the following is an example of Indian Englishism? (a) What is your good name? (b) I will miss you till you are away (c) We are like that only _____(d) All of these _____________________________ Ans. (d) : Indian Englishism : Indianisation of English is called Indian Englishism which is used by people living in India (non standard English) It is an expression of characteristic peculiar to the English. YCT

Following are a few examples of Indian English; [The second sentence is grammatical] Lizard is walking on the wall (Indian English) Lizard is creeping on the wall.

Ans. (d) : The poet sees ashes of dead body floating down the river of his country. The old windows and old men find freedom after death but the young generation 110. The ’’Harp” in a poem by the same name is a thieves to find the freedom to change the world yet they symbol of cannot. The poet wonders of being either of them but (a) the glorious poets of ancient India prefers to be alone. (b) an ancient musical instrument 115. Kamala Surayya’s autobiography is titled (c) a musical tradition (a) Summer in Calcutta (b) Her story (d) a musician who used it (c) My Story (d) The Descendants Ans. (a) : The poem "Harp” of India is written by Ans. (c) : My Story is an autobiographical book written ’Henry Derozio’, who led the Young Bengal Movement by Indian author and poetess Kamala Das (also known and is also known as the first modem English poet of India. The poem Harp of India celebrates the glorious as Kamala Surayya or Madhavikutty). This book was past of India and mourns over its loss because of British originally published in Malayalam, titled Ente Katha. In rule and ends with the hope that she (India) will regain this book, Das recounts the trials of her marriage and its glory. The poet uses ’harp’ as a symbol for the Indian her painful self awakening as a woman and writer. poets who were earlier famous but under the British 116. Renowned writer R. K. Narayan is gifted with Rule, they are suffering. a style that is (a) Simple and humorous (b) Prosaic 111. Ezekiel asks, ’You Want One Glass Lassi?’ What is he trying to say? (c) Rich in vocabulary (d) Complicated (a) He is being patriotic Ans. (a) : R. K. Narayan’s writing style was marked by (b) He is proclaiming that Lassi is better than wine simplicity and subtle humour. He told stories of (c) The Indian ingredients are better than those of ordinary people trying to live their simple lives in a wine. changing world. These include: Sahitya Akademi (d) All of these Award (1960) for The Guide in 1958; Padam Bhushan Ans. (b) : He says that lassi with a little salt added in it in 1964; and the AC Benson Medal from the Royal is very lovely drink. It is very good for digestion. He society of Literature in 1980. says that lassi is better than wine. This line is taken Hence, the correct answer is option (a). from poem The Patriot by Nissim Ezekiel. 117. Chitra Divakaruni is a writer who 112. What does Mahapatra mean by ”my country’s (a) tells the stories with a historical perspective body”? (b) writes about women who are censored by the (a) An individual human being press (b) An entire country (c) tells the stories of immigrants especially women (c) A collection of human beings (d) All of these (d) All of these Ans. (d) : The poet sees ashes of dead body float down Ans. (c) : Chitra Banerjee Divakaruni (bom Chitralekha the river of his country. It seems as though my country Banerjee, July 1929, 1956) is an Indian American body floats down somewhere on the river. The poet author, poet and the Betty and Gene McDavid are watches the bodies (dead bodies) float down the river. professor of writing at the University of Houston This refers to the Hindu tradition, where a body is Creative Writing Program. Divakaruni's works are cremated and ashes are let into the river. This line is largely set in India and the United States and often taken from Freedom. focuses on the experiences of South Asian immigrants. 113. Unstrung for ever, must thou there remain” - She writes for children as well as adults and has published These lines from Henry Derozio’s ’The Harp of novels in multiple genres, including realistic fiction, India’ mean historical fiction, magical realism, myth and fantasy. (a) nobody listens to the poets of the past. Hence, the correct answer is option (c). (b) these poets of the past lost their glory in the 118. Tharoor says ’our matrimonial ads have British Rule created their own cultural tropes’. Which (c) development and modernity have silenced tropes? them (a) 'Wheatish complexion’ (d) All of these (b) 'Traditional with modem outlook’ Ans. (d) : The poem The Harp of India celebrates the (c) 'White bride’ glorious past of India and mourns over its loss because (d) All of these of British Rule and ends with the hope that she (India) will regain its glory. The poet uses Harp as a symbol Ans. (d) : The writer of this article is Shashi Tharoor. for the Indian poets who were earlier famous but under He has been an international diplomat and was the the British Rule, they are suffering. under secretary General of The United Nations. He was 114. Jayanta Mahapatra’s ’Freedom’ sketches out the foreign minister of India under the previous government. Tharoor is an acclaimed writer, having the (a) Indian belief about death as freedom from the authored 1823 bestselling works of fiction and nonfiction since 1981, which are centred on India and its body. (b) Death as freedom from the world by bondage history, culture, film, politics, society, foreign policy and more related themes. (c) Death as freedom from physical life Hence, the correct answer is option (d). (d) All of these RPSC Assistant Professor Exam. 2020 82 YCT

119. The train journey in 'Ladies Coupe' is one of self- 125. The short story collection Maladies, is written by discovery for the women travelling in it. How?

(a) By trying to find answers for their lives from the experiences that others undergo (b) By travelling alone (c) By finding some time for themselves _____(d) By eating food that others undergo _________ Ans. (a) : Ladies Coupe is a novel by Anita Nair published in 2001. The novel follows the journey of a middle aged Indian woman named Akhila as she travels to Kanyakumari in her search for Independence and on the train's ladies coupe, swaps stories with 5 different women who inspired her to live her own life. Hence, the correct answer is option (a). ______________ 120. The poem 'Freedom' is written by (a) Jayanta Mahapatra (b) Damala Das _____(c) Pom Moraes ______(d) R. K. Narayan Ans. (a) : The poem Freedom is written by Jayanta Mahapatra. The poem Freedom portrays the Indian belief system about death as freedom from the body. Freedom includes freedom, death and patriotism. It talks about how India is still dependent on the rulers, ever after Indian Independence. Hence, the correct answer is option (a). ______________ 121. The poem 'An Introduction' is written by (a) Kamala Das (b) Dom Moraes _____(c) R.K. Narayan ______(d) Salman Rushdie Ans. (a) : The poem An Introduction is an autobiographical verse of Kamala Das. This poem first appeared in Kamala Das very first volume of poem which was entitled Summer in Calcutta and which was published in 1965. This poem is wholly autobiographical and may also be labelled as a confessional poem. Hence, the correct answer is option (a). ______________ 122. The poem 'Absences* is written by (a) Dom Moraes (b) R.K. Narayan _____(c) Salman Rushdie (d) Jhumpa Lahiri Ans. (a) : The poem Absences is written by Dom Moraes. For Moraes, Absence is a metaphor for life. In a poem entitled "Absences" which later lends itself to the title of a collection of three overlapping pieces of life writing. A variety of Absences he sees: "a world only held together by its variety of absences". Hence, the correct answer is option (a). ______________ 123. The short story 'Antidote' is written by (a) R.K. Narayan (b) Salman Rushdie _____(c) Jhumpa Lahiri (d) Chitra B, Devakaruni Ans. (a) : The Antidote is a short story written by the Indian author, R.K. Narayan. A script is written to provide what Gopal should do and what the doctor should say to declare him as dead. Hence, the correct answer is option (a). 124. The short story 'The Free Radio' is written by (a) Salman Rushdie (b) Jhumpa Lahiri _____(c) Chitra B. Devakaruni (d) Tagore __________ Ans. (a) : The short story The Free Radio is written by Salman Rushdie about the sterilization campaign launched by India' ruling regime during the emergency period (1975-1977) when president Fakhruddin Ali Ahmed, upon advice by Prime Minister Indira Gandhi, declared a state of emergency under Article 352 of the constitution of India. Hence, the correct answer is option (a). RPSC Assistant Professor Exam. 2020

Interpreter

of

(a) Jhumpa Lahiri (b) Chitra B. Devakaruni (c) Tagore _____(d) Ambedkar ____________________________ Ans. (a) : The short story collection 'Interpreter of Maladies' is a book collection of nine short stories by American author of Indian origin Jhumpa Lahiri published in 1999. Hence, the correct answer is option (a). ______________ 126. The essay 'Back from the West and Unable to Find Loading in Baroda' is written by (a) Ambedkar (b) SathyajitRay _____(c) Shashi Tharoor _____(d) Anita Nair _______ Ans. (a) : The essay Back from the West and Unable to Find Lodging in Baroda is written by Dr. B. R. Ambedker. The statement given by him are: In 1916, I returned to India. I had been sent to America by his Highness the Maharaja of Baroda for higher education. I studied at Columbia University in New York from 1913 to 1917. In 1917 I came to London and joined the post graduate department of the school of Economics of the University of London. In 1918 I was obliged to return to India without completing my studies. Since I had been educated by the Baroda State, I was bound to surve the state. [Note: the dates here appear to be a bit confused] Accordingly, on my arrival I straightway went to Baroda. The reasons why I left Baroda service are quite irrelevant to my present purpose. Therefore I do not wish to enter into them. I am only concerned with my social experiences in Baroda, and I will confine myself to describing them. 127. The first Englishman to write a Pindaric ode was _____. (a) Ben Jonson (b) Abraham Cowley _____(c) Thomas Wyatt _____(d) Thomas Gray _____ Ans. (b) : The first Englishman who wrote, a Pindaric Ode was Abraham Cowley. 'Ode' denotes a long lyric poem that is serious in subject and treatment, elevated in style, and elaborate in its stanzaic structure. These structures are (1) Strophe :- Moving in dance rhythm 'right to left'. (2) Antistrophe:-Moving in a dance rhythm 'left to right' (3) Epode : - Stand still. Note : "The Regular or Pindaric Ode" in English is a close imitation of Pindar's form, with all the strophe and Anti strophe written in one stanza pattern, and all the Epodes in another. This form was introduced into England by Ben Jonson. By-M.H. Abrams 'A Glossary of Literary Terms'. 128. Which playwright gives an honest picture of the underworld in London in The Honest Whore Part 2? (a) John Marston (b) Thomas Dekker _____(c) Thomas Middleton (d) Philip Massinger Ans. (b) : The Honest Whore is an early Jacobean city comedy written in two parts; Part 1 is a collaboration between Thomas Dekker and Thomas Middleton, while part 2 is the work of Thomas Dekker alone. The play were acted by the Admiral's Men. Part 2 was most likely written in 1605 or 1606, and was entered into the stationers' Register on 29 April 1608. Part 2 was not published, however, till 1630, in a quarto printed by Elizabeth Allde for the bookseller Nathaniel Butter.

83

YCT

129.

John Cleveland was a poet. (a) Movement (b) Georgian _ _ _ _ _(c) Romantic __________(d) Metaphysical _____ Ans. (d) : John Cleveland was a Metaphysical poet. John Cleveland carried Metaphysical obscurity and conceit to their limits and many of his poems are merely intellectual gymnastics. John Cleveland’s real achievement lay in his political poems which were mostly written in heroic couplets and satirized contemporary person and issues. ____________________ 130. Who was the first king to be declared the king of Great Britain? (a) James I (b) Charles II (c) Henry VII _________(d) Henry VIII _______ Ans. (a) : Athelstan was king of Wessex and the first king of all England. James VI of Scotland also became James I of England in 1603. Upon accession to the English throne, he styled himself ’’King of Great Britain” and was so proclaimed. 131. A New Way to Pay Old Debts is an English Renaissance play by . (a) Marston (b) Tourneur _ _ _ _ _(c) Massinger _________(d) Dekker __________ Ans. (c) : A New Way to Pay Old Debts is an English Renaissance drama, by Philip Massinger. It’s central character, Sir Giles Over-Reach, became one of the more popular villains on English and American stages through the 19th Century. __________________________ 132. Dame Pliant and Lovewit are characters in . (a) The Merry Wives of Windsor (b) Everyman in his humour (c) Valpone _____(d) The Alchemist __________________________ Ans. (d) : The above mention characters have been described in The Alchemist by Ben Jonson. ’The Alchemist’ is a comedy first performed in 1610 by the ’King’s Men'. In The Alchemist we find Dame Plaint is a young widow and the sister of KastriL Dame Pliant (Sometimes called the widow Pliant) is also rich, which means that just about every dude in the play wants to marry her. Like her name suggests this young widow is always compliant when it comes to the men in her life. 133. Who has been called the "Saint of the Metaphysical School"? (a) John Donne (b) George Herbert _____(c) Henry Vaughan (d) Abraham Cowley Ans. (b) : George Herbert was the saint of the Metaphysical School, a Although the term ’Metaphysical School' was first coined by literary critic and poet Dr. Samuel Johnson in his book Lives of the Most Eminent English Poets (1779-1781). His work is often marked by Metaphysical conceits. He uses many lyrical forms and often shows considerable artistry. The poetry of this school appeals to the heart, intellect, indulging in far-fetched expression and conceited thought. ________________________________________ 134. A postmodern parody of "A Valediction Forbidding Mourning" was written with the same title by . (a) Anne Sexton (b) Megan Terry _____(c) Adrienne Rich ______(d) Sylvia Plath ______ Ans. (c) : Adrienne Rich used the same title as John Donne’s poem written in the early 16 th century. His poem entails the departure of two lovers. In it the male RPSC Assistant Professor Exam. 2020

speaker urges his lover not to mourn over their separation. He tells her that their love does not require the need for the physical presence to be validated. _____ 135. "The Garden" is a famous poem by . (a) Andrew Marvell (b) Richard Crashaw _ _ _ _ _(c) George Herbert (d) Abraham Cowley Ans. (a) : The Garden is a famous lyric poem by Andrew Marvell published in 1681, a couple years after Marvell's death. It captures the themes of natural beauty* poetic imagination, and spirituality. Here, in this poem, Marvell seems to be priest of nature. He finds himself in an ambiance that gives a soothing sensation to his soul. He cherishes each moment in the slow journey through the Garden. _________________________________________ 136. "Now therefore, while the youthful hue // sits on the skin like morning dew ...." These lines show . (a) Circumlocution (b) Carpe diem philosophy (c) Hyperbole _____(d) Both (a) and (b) _______________________ Ans. (b) : The above given lines have been discussed in the lyric poem To His Coy Mistress by Andrew Marvell in (1681). In this poem Andrew Marvell has used 'Carpe Diem philosophy' means 'seize the day'. The speaker in this poem emphasizes that 'Life is short and time is fleeting in order to urge his auditor. "While the youthful hue/sits on the skin like morning dew" restates the speakers desire, with a focus on his mistress body. The "morning dew" is also an effective simile in that dew very quickly disappears as the day advances, just like her youthful appearance. _______________________ 137. Blank Verse was introduced in English literature in Surrey’s translation of (a) Aeneid (b) Metamorphosis _____(c) Iliad ______________(d) Orlando Furioso Ans. (a) : 'Blank Verse' consists of lines of 'Iambic Pentameter' (five-stress iambic verse) which are unrhymed. 'Blank Verse' was introduced by the Earl of Surrey in his translations of Book 2 and 4 of Virgil's The Aeneid (about 1540). It became the standard meter for Elizabethan age. For example :- Milton's Paradise Lost ______________Wordsworth's Prelude, etc. __________ 138. Utopia was first published in English in the year (a) 1551 (b) 1557 (c) 1571 ______________ (d) 1584 ____________ Ans. (a) : Sir Thomas More completed Utopia in 1516. His original work was written in Latin. It was translated into English by Ralph Robinson and first published in English in 1551. This is the third edition of Robinsons version, printed in 1597. 139. Tottel's Miscellany was published in the year (a) 1552 (b) 1557 (c) 1560 ______________(d) 1564 ____________ Ans. (b) : Songes and Sonettes usually called Tottel's Miscellany, was the first printed anthology of English poetry. Tottel's Miscellany was first published by Richard Tottel in 1557 in London, it ran to many editions in the sixteenth century. Richard Tottel was an English publisher with a shop at Temple Bar on fleet street in London. His main business was the publication of law textbooks but his biggest contribution to English literature would come in the form of the anthology poetry.

84

YCT

140. Morte d’ Arthur is a prose romance by (a) Alfred Tennyson (b) John Dryden _____(c) John Lyly _______(d) Thomas Malory Ans. (d) : Le Morte d' Arthur (originally spelled Le Morte D’ Arthur, ungrammatical Middle French for "The Death of Arthur") is a 15th century Middle English prose reworking by Thomas Malory of tales about the legendary king Arthur, Guinevere, Lancelot, Merlin and the Knights of the Round Table along with their respective folklore.______________________________ 141. The first complete English translation of Iliad and Odyssey was made by (a) Robert Henryson (b) George Chapman _____(c) JohnOgilby _______(d) Alexander Pope Ans. (b) : The first complete English translation of Iliad and Odyssey was made by George Chapman. Translators and scholars have translated the main works attributed to Homer, the Iliad and Odyssey from the Homeric Greek into English since the 16th and 17th century. Translation are ordered chronologically by date of first publication, with first line provided to illustrate the style of the translation. 142. Who called Spenser "the poet’s poet"? (a) Charles Lamb (b) Dr. Johnson _____(c) Matthew Arnold (d) T.S. Eliot ________ Ans. (a) : Spenser was called "the poet’s poet" by Charles Lamb and was admired by John Milton, William Blake, William Wordsworth, John Keats, Lord Byron, Alfred Lord Tennyson and others. 143. Bacon’s ’New Atlantis’ is modelled on (a) The Tempest (b) Utopia _____(c) The King's Quair (d) Divine Comedy Ans. (b) : New Atlantis is an incomplete Utopian novel by Sir Frames Bacon, published posthumously in 1626. It appeared unheralded and tucked into the back of a longer work of natural history, Sylva Sylvarum (forest of material). In New Atlantis, Bacon portrayed a vision of the future of human discovery and knowledge, expressing his aspirations and ideals for humankind. The novel depicts the creation of a Utopian land where "generosity and enlightenment, dignity and splendour, piety and public spirit" are the commonly held qualities of the inhabitants of the mythical Bensalem. The New Atlantis is the philosophical and intellectual utopia envisioned by Francis Bacon. 144. The Forest of Arden appears in (a) All’s Well that Ends Well (b) Macbeth (c) A Midsummer Night’s Dream _____(d) As You Like it _____________________ Ans. (d) : 'The Forest of Arden' appears in the famous Pastoral Comedy As You Like It written by William Shakespeare in 1599, first published in the First Folio in 1623. In As You Like It, The Forest of Arden represents the country life and an escape from corruption while the Duke’s court represents corrupted city life._____________ 145. Sidney’s ’’An Apology for Poetry" is a reply to (a) Four Ages of Poetry (b) The School of Abuse (c) A Model Proposal _____(d) Euphues _________________________ Ans. (b) : Sidney's An Apology for Poetry is a reply to, The School of Abuse by Stephen Gosson. It is a work of literary criticism, written is 1580, published in 1595, RPSC Assistant Professor Exam. 2020

after his death. Sidney advocates a place for poetry within the framework of an aristocratic state, while showing concern for both literary and national identity. Sidney responds in Apology to an emerging antipathy to poetry as expressed in Stephen Gosson's The School oj Abuse. 146. Devotions upon Emergent Occasions is a prose work by ___ (a) William Caxton (b) John Wycliffe _____(c) John Lyly _________(d) John Donne ______ Ans. (d) : Devotions Upon Emergent Occasions and Several Steps in My Sickness is a prose work by the English metaphysical poet John Donne, published in 1624. It covers death, rebirth and the Elizabethan concept of sickness as a visit from God, reflecting internal sinfulness. The Devotions is divided into 23 parts, each consisting of 3 sub-section, called the ’meditation’, the 'expostulation', and a prayer. _________ 147. Amoretti contains sonnets. (a) 66 (b) 74 (c) 78 ____________ (d) 89 ______________ Ans. (d) : Amoretti (meaning little love poems) is a sequence of 89 sonnets written by Edmund Spenser in 16th century in the tradition of the Petrarchan sonnets, a popular form for poets of the Renaissance period. The Sonnet Cycle describes his courtship and eventual marriage to Elizabeth Boyle, published in 1595. 148. A.C. Bradley’s Shakespearean Tragedy consists of _ _ _lectures. (a) 23 (b) 16 (c) 12 ___________ (d) 10 ______________ Ans. (d) : Andrew Cecil Bradley was an English literary scholar, best remembered, for his work Shakespearean Tragedy (1904) consist often lectures' A.C. Bradley has provided a study of the four great tragedies Hamlet, Othello, King Lear and Macbeth which reveals a deep understanding of Shakespearean thought and art. Note: RPSC has considered option (a) as the correct answer but in reality, option (d) will be correct answer. 149. The first pastoral romance in English is (a) The Romance of the Rose (b) Sidney's Arcadia (c) The Shepheardes Calendar _____(d) As You like it _________________________ Ans. (b) : The first Pastoral romance in English is Arcadia The original title of Arcadia is The Countess of Pembroke’s Arcadia written by Sir Philip Sidney in 16th century, is a pastoral romance in which courtiers disguised as Amazons and Shepherds make love and sing delicate experimental verses. 150. The first blank verse tragedy in English is (a) Titus Andronicus (b) Catiline _____(c) Gorboduc _______(d) The Duchess of Malfi Ans. (c) : 'Gorboduc' (Ferrex and Porrex) is the earliest English tragedy in 'blank verse' by Thomas Norton and Thomas Sackville in 1561. The writers took the story of the play from Geoffrey of Monmouth's History of the kings of Britain (1138). The play premiered before Queen Elizabeth I on 18 January 1561. 'Blank Verse' was introduced into England by Earl of Surrey in about 1540. It is the principal metre of Shakespeare's play and the metre of Milton's Epic poems, as well as many other major works of poetry.

85

YCT

Rajasthan Public Service Commission College Lecture Exam. 2014

ENGLISH-I (Solved Paper)

[Exam. Date : 28 June 2016]

And their children, so intent on/ Finding more Read the following poem and answer the unripe acorns,/ Expect to be taken home,/ Their question given below by choosing from the beauty has thickened./ Something is pushing given options : them/ To the side of their own lives,/ AfternoonsFrom the following statements, pick out the one Summer is fading :/ The Leaves fall in ones and that is true with reference to the poem. twos/ From trees bordering The new recreation (a) Young mothers enjoy their lives immensely. ground./ In the hollows of afternoons/ Young (b) Young mothers engage in interesting mothers assemble/ At swing and sandpit/ Setting activities. free their children./ Behind them at intervals,/ (c) Young mothers are trapped in daily routines. Stand husbands in skilled trades,/ An estateful of (d) Love and romance are a part of the lives of washing,/ And the albums, lettered/ Our wedding, young mothers. lying/ Near the television :/ Before them, the wind/ is ruining their courting-places./ That are Ans. (c) The poem mentions here that young mothers still courting-places (But lovers are all in school),/ are trapped in daily routines. And their children, so intent on/ Finding more 3. Read the following poem and answer the unripe acorns,/ Expect to be taken home,/ Their question given below by choosing from the beauty has thickened./ Something is pushing given options : them/ To the side of their own lives,/ AfternoonWhich of the following is a major theme of the Summer is fading :/ The Leaves fall in ones and poem? twos/ From trees bordering The new recreation ground./ In the hollows of afternoons/ Young (a) Housekeeping mothers assemble/ At swing and sandpit/ Setting (b) Mothers losing their identity free their children./ Behind them at intervals,/ (c) Enduring love Stand husbands in skilled trades,/ An estateful of (d) Poverty washing,/ And the albums, lettered/ Our wedding, Ans. (b) The above cited poem mentions the young lying/ Near the television :/ Before them, the mothers who are losing their identity in order to bring wind/ is ruining their courting-places./ That are up their children. still courting-places (But lovers are all in school),/ Therefore, option (b) ’’Mothers losing their identity" is And their children, so intent on/ Finding more the correct option. unripe acorns,/ Expect to be taken home,/ Their 2. Read the following poem and answer the beauty has thickened./ Something is pushing them/ To the side of their own lives,/ question given below by choosing from the given options : The albums lettered ”Our Wedding” areAfternoons (a) a prized possession. Summer is fading :/ The Leaves fall in ones and (b) just a piece of clutter. twos/ From trees bordering The new recreation (c) a testiomny to the continuing romantic ground./ In the hollows of afternoons/ Young relationship mothers assemble/ At swing and sandpit/ Setting (d) a collection of family photos. free their children./ Behind them at intervals,/ Ans. (b) In the given poem "the albums lettered "Our Stand husbands in skilled trades,/ An estateful of Wedding’ gives an unromantic depiction of married life, washing,/ And the albums, lettered/ Our wedding, because the couples do not stand together, they are lying/ Near the television :/ Before them, the together only in photo-frame. Hence option (b) wind/ is ruining their courting-places./ That are describes these albums just a piece of clutter. still courting-places (But lovers are all in school),/

1.

Rajasthan PSC College Lecture-I Exam. 2014

86

YCT

4.

Read the following poem and answer the question given below by choosing from the given options : Afternoons Summer is fading :/ The Leaves fall in ones and twos/ From trees bordering The new recreation ground./ In the hollows of afternoons/ Young mothers assemble/ At swing and sandpit/ Setting free their children./ Behind them at intervals,/ Stand husbands in skilled trades,/ An estateful of washing,/ And the albums, lettered/ Our wedding, lying/ Near the television :/ Before them, the wind/ is ruining their courting-places./ That are still courting-places (But lovers are all in school),/ And their children, so intent on/ Finding more unripe acorns,/ Expect to be taken home,/ Their beauty has thickened./ Something is pushing them/ To the side of their own lives,/ The poem views marriage with(a) Cynicism (b) Optismism (c) Nostalgia (d) Approbation Ans. (c) In the given poem marriage is viewed as ’nostalgia'. It presents an unromantic picture of marriage. The women are trapped in daily routine while the husbands provide them only economic support and there is a lack of emotional support.

5.

Read the following poem and answer the question given below by choosing from the given options : Afternoons Summer is fading :/ The Leaves fall in ones and twos/ From trees bordering The new recreation ground./ In the hollows of afternoons/ Young mothers assemble/ At swing and sandpit/ Setting free their children./ Behind them at intervals,/ Stand husbands in skilled trades,/ An estateful of washing,/ And the albums, lettered/ Our wedding, lying/ Near the television :/ Before them, the wind/ is ruining their courting-places./ That are still courting-places (But lovers are all in school),/ And their children, so intent on/ Finding more unripe acorns,/ Expect to be taken home,/ Their beauty has thickened./ Something is pushing them/ To the side of their own lives,/ Which of the following expressions best evokes the theme of emptiness? (a) hollows of the afternoons (b) an estateful of washing (c) the albums lettered "Our Wedding" (d) the wind is ruining their courting - places

Rajasthan PSC College Lecture-I Exam. 2014

Ans. (c) The albums lettered 'Our Wedding' best evokes the theme of emptiness. The wormth of husband-wife relation is no more. The mothers are busied in their daily routine while husband are engaged in their job. 6. ’’Invention, Nature’s Child, field stepdame study’s blows” The figure of speech used in the above line is (a) metaphor (b) personification (c) oxymoron (d) transferred epithet Ans. (b) Personification is a figure of speech where non living objects are described to seem like people. 'Invention', which is referred to here as if it were a very young person, is personified in the given line. 7. Which of the following is not an instance of inversion? (a) "Down comes the winter rain ...... " (b) "the body electric" (c) "Thus ceased she" (d) "plods his weary way" Ans. (d) Inversion, also known as 'anastrophe' is a literary technique in which the normal order of words is reversed, in order to achieve a particular effect of emphasis or meter. 'Down Comes', 'body electric' and 'Ceased She' are examples of inversion while 'weary way' is not an instance of inversion. 8. The term ’’Connotation” refers to the(a) close repetition of identical consonant sounds (b) straight forward dictionary meaning of word (c) implications or suggestions which are evoked by a word (d) juxtaposition of images or ideas Ans. (c) Connotation refers to the implications or suggestions which are evoked by a word. Example- "He is such a dog", here the word dog connotes shamelessness or ugliness. close repetition of identical consonant sounds Alliteration straight forward dictionary meaning of word Denotation juxtaposition of images or ideas - Contrast 9. A figure of speech in which a part represents the whole object or idea is called(a) synecdoche (b) symbol (c) conceit (d) metonymy Ans. (a) Synecdoche is a figure of speech in which a part of something is used to refer to its whole. For example, "The captain commands one hundred sails" is a synecdoche that uses 'sails' to refer to ships.

87

YCT

10.

”A chief and imperative task of those who wish to regain for reason the no-man’s land left barren by the abdication of the modern state from its essential function, is to challenge the deceptive fallacy that between right and wrong, between truth and error, between justice and injustice, the whole duty of man is to be impartial.” The language in the above sentence is(a) pompous and loaded with abstractions (b) poetic and loaded with abstractions (c) full of politeness and humility (d) full of irony and satire Ans. (a) The given speech reflects a pompous language loaded with abstractions. Pompous language having or showing the attitude of people who speak and behave in a very formal and serious way because they believe that they are better, smarter, or more important than other people. Abstraction refers to a general idea not based on any particular real person, thing or situation.

11.

Choose the most appropriate determiner to fill in the following sentence. Is there ........... reason why we should not start early? (a) much (b) the (c) any (d) more Ans. (c) Is there any reason why we should not start early? ’Any’ is used in interrogative and negative sentences.

12.

Choose the most appropriate Determiner to fill in the blank in the following sentence. I am giving you responsibility for providing the drinks. (a) any (b) a (c) most (d) the Ans. (d) I am giving you the responsibility for providing the drinks. Here ’the’ is used to emphasis ’responsibility’ and make it specific. 13. Choose the most appropriate determiner to fill in the blank in the following sentence. Big hotels all over the world are very the same. (a) little (b) much (c) few (d) many Ans. (c) Big hotels are very much the same. ’Much’ is used with ’very’ to express the sense of 'to a great extent’. 14. Choose the most appropriate determiner to fill in the blank in the following sentence. There are children in this area, so your child will have company. (a) a little (b) most (c) a few (d) all Rajasthan PSC College Lecture-I Exam. 2014

Ans. (c) 'A few’ is a quantifier meaning ’some’. ’Few’ has a negative meaning - ’hardly any'. 'A few’ has a positive meaning - 'some'. 'The few’ means 'not many, but all there are’. _________ 15. Choose the most appropriate determiner to fill in the blank in the following sentence. He has good knowledge of Indian classical music. (a) a (b) the (c) much (d) some Ans. (a) He has a good knowledge of Indian classical music. A, An and The are types of determiner called articles, which introduce and provide context to a noun or noun phrase. 16. Choose the most appropriate determiner to fill in the blank in the following sentence. I cannot judge accuracy of your calculations: (a) an (b) any (c) the (d) most Ans. (c) I cannot judge the accuracy of your calculations. 'The' is used here to emphasize 'accuracy' and make it specific. 17. Choose the most appropriate determiner to fill in the blank in the following sentence paint helps to protect metal from rust. (a) All (b) A (c) The ______________(d) no article _______ Ans. (d) Paint helps to protect metal from rust. No article is used when a noncountable noun is generic or nonspecific. ___________________________________ 18. Choose the most appropriate determiner to fill in the blank in the following sentence member of the team has to undergo a fitness test before every match. (a) Each (b) All (c) Any _____________(d) The ____________ Ans. (a) Each member of the team has to undergo a fitness test before every match. We use each to refer to individual things in a group or a list of two or more things. 19. Fill in the blank in the following sentence with the correct modal. The keys be in my coat pocket. (Possibility) (a) would (b) shall _____(c) ought ____________(d) may ___________ Ans. (d) The keys may be in my coat pocket. May is used to express possibility. _______________________ 20. Fill in the blank in the following sentence with the correct modal. Mr Sharma is not at home. He be working. (Logical necessity) (a) must (b) will (c) may (d) might

88

YCT

Ans. (a) Mr. Sharma is not at home. He must be Ans. (d) Will you lend me some money ? working. Must be is used to show something logically Will is used for (willingness). inferred or supposed to. 27. Identify the sentence pattern of the following 21. Fill in the blank in the following sentence with sentence from the options given below. I the correct modal. I............... play the piano promised the children a trip to the zoo. when I was five. (Ability in the past) (a) S + V (a) would (b) should (b) S + V + DO (c) could (d) used to (c) S + V + IO + DO Ans. (c) I could play the piano when I was five. r (d) S + V + Adv + DO Could is used in Past to show (ability). 22. Fill in the blank in the following sentence with Ans. (c) I promised the children a trip to the zoo. the correct modal. You ......... take me to the The correct structure is- "S + V + IO + DO". IO = Indirect object station. I can walk (No necessity) (a) mustn't (b) needn't Do = Direct object (c) won't (d) couldn't 28. Identify the sentence pattern of the following Ans. (b) You needn't take me to the station. I can walk sentence from the options given below, Paul Needn't is used to show something that is not essential hired a cab. for the use. (a) S + V 23. Choose the correct meaning of the underlined (b) S + V + DO modal in the following sentence. (c) S + V + DO + IO You should go for a walk every morning. (d) S + V + Adj (a) permission Ans. (b) Paul hired of cab. (b) advice The correct structure is- S + V + DO (c) wish _____(d) warning ______________________________ 29. Identify the sentence pattern of the following sentence from the options given below. The Ans. (b) You should go for a walk every morning. Aravallis are beautiful. Should is used for (Advice). ______________________ (a) S + V + Sub Compliment 24. Pick out of the correct meaning of the underlined modal in the following sentence(b) S + V in those days people would make their own (c) S + V + DO entertainment. (d) S + V +Obj Compliment (a) necessity Ans. (a) The Aravallis are beautiful (b) habit in the past The correct structure is- S + V + Sub. Compliment (c) obligation Here beautiful, Aravallis are subject compliment. (d) probability 30. Identify the sentence pattern of the following Ans. (b) In those days people would make their own sentence from the options given below. The entertainment. teacher called the pupil a fool. Would is used to show the Past habitual action. (a) S + V 25. Select the correct meaning of the underlined (b) S + V + DO modal in the following sentence. (c) S + V + DO + Obj. Compliment Can you draw a perfect circle? (d) S + V + DO + Sub. Compliment (a) permission (b) request Ans. (c) The teacher called the pupil a fool. (c) ability (d) advice The correct structure is- S + V + DO + Obj. Ans. (c) Can you draw a perfect circle? compliment Can is used to show ability. 26.

Which of the following options expresses the 31. meaning of the underlined modal? Will you lend me some money? (a) habit (b) necessity (c) obligation (d) willingness

Rajasthan PSC College Lecture-I Exam. 2014

89

Identify the sentence pattern of the following sentence from the options given below. He painted the door green. (a) S + V (b) S + V + DO + Obj. Compliment (c) S + V + DO (d) S + V + DO + IO YCT

Ans. (b) He painted the door green The correct structure is- S + V + DO + Obj. compliment the door is (DO) while green is objective compliment. 32.

Identify the sentence pattern of the following sentence from the options given below. He put the book on the table. (a) S + V + DO + Adv.P (b) S + V (c) S + V + DO (d) S + V + DO + IO

Ans. (d) He put the book on the table The correct order is- S + V + DO + IO 33.

(a) A B C D P N O M (b) A B C D N M P O (c) A B C D O P N M (d) A B C D M P N O Ans. (b) The correct structure isCarry on (A) - Continue doing (N) Cut up (B) - Angry/upset about (M) Go through (C) - Experience something erp. unpleasant (P) Come round (D) - Recover consciousness (O) 37. Fill in the blank in the following sentence will the most appropriate phrasal verb. I was taught geography at school but never it. (a) spoke for (b) saw through (c) took to (d) ran after

Identify the sentence pattern of the following sentence from the option given below. The driver appeared intoxicated. Ans. (c) Take to means like. So the correct sentence - I (a) S + V was taught Geography at school but never took to it. (b) S + V + DO 38. Complete the phrasal verb keeping in mind the (c) S + V + Sub. Compliment meaning given in brackets. I’ve given (d) S + V + Obj. Compliment smoking. (Stopped) Ans. (c) The driver appeared intoxicated (a) up (b) down (c) out (d) into The correct order is- S + V + Sub. Compliment intoxicated is the subject compliment of Ans. (a) Give up means ’’cease making an effort.” So driver. the correct sentence- I’ve given up smoking. Pick out the correct particle to complete the 34. Identify the sentence pattern of the following 39. phrasal verb in the following sentence sentence from the options given below. The expressing the meaning given in brackets. child was sleeping on the bed. They’ve put the match until next (a) S + V + Adv week. (Postponed) (b) S + Adv + V (a) in (b) off (c) S + V + DO (c) out (d) on (d) S + V + Obj. Compliment Ans. (b) Put off means ’postpone’. The correct sentence Ans. (c) The child was sleeping on the bed. is- They’ve put off the match until next week. The correct structure is-S + V + DO 40. Fill in the blank in the following sentence with 35. Choose the correct meaning of the phrasal verb the correct idiom. There is no way of coping ’Take Up’ from the options given below. with orders that come in throughout the year. (a) Start doing an activity (a) piping hot (b) To buy something (b) part and parcel (c) Begin to like (c) fits and starts (d) To resemble (d) hue and cry Ans. (a) ’Take up’ means regular. Ans. (c) Fits and starts means ’ which is not regular’. So 36. Match List I with List II and select the correct the correct sentence is- There is no way of coping with answer by using the code given below, List I orders that come in fits and starts throughout the year. List II 41. Choose the correct meaning of the idiom List-I List-II ’storm in a tea cup’ from the options given M. angry/upset about A. Carry on below. B. Cut up N. continue doing (a) Eager expectation O. recover consciousness C. Go through (b) Careless behaviour D. Come round P. experience something esp. (c) Fuss about something unimportant unpleasant. (d) An easy task Rajasthan PSC College Lecture-I Exam. 2014

90

YCT

Ans. (c) ’’Storm in a tea cup” means ’Fuss about something unimportant’. 42. Fill the blank in the following sentence with the appropriate idiom. She is so unhappy that she is ready with anyone who contradicts her. (a) to play up (b) to pick a quarrel (c) go along (d) to play down Ans. (b) Here correct option (b) ’to pick a quarrel’ is used. So the correct sentence is- She is so unhappy that she is ready to pick a quarrel with anyone who contradicts her. 43. Fill in the blank in the following sentence with the most appriate tense from the given options.l to work everyday last year. (a) am walking (b) walk (c) walked (d) had walked Ans. (c) I walked to work everyday last year. In the given sentence past word last year is used. So walked V2 is used for denoting past indefinite tense. 44. Fill in the blank in the following sentence with the most appropriate tense from the given options. Even today, a notice at the end of the road people not to go any further. (a) had warned (b) is warning (c) has warned (d) warns Ans. (d) Even today, a notice at the end of road warns people not to go any further. ’Even today’ points out that the sentence is in Present Tense. Since the ’notice’ is still relevant, it is an instance of simple present tense. We use the simple present tense when an action is happening right now, or when it happens regularly. 45.

Fill in the blank in the following sentence with the most appropriate tense from the given options. The girl on the piano and was singing softly to herself. (a) played (b) was playing (c) had been playing (d) has been playing Ans. (b) The girl was playing on the piano and was singing softly to herself. The second part of the sentence hints past continuity of the action, so first part will also be in past continuous tense.

Rajasthan PSC College Lecture-I Exam. 2014

46.

Fill in the blank in the following sentence with the most appropriate tense from the given options. I the instructions but I don’t understand them. (a) have read (b) will read (c) am read (d) was reading Ans. (a) I have read the instructions but I don’t understand them. The sentence indicates the completion of the action, so the first part will be in present perfect tense. 47. Fill in the blank in the following sentence with the most appropriate tense from the given options. He what you told him. (a) is believing (b) was believing (c) was believed (d) believes Ans. (d) He believes what you told him. Option (d) is the most appropriate among the options. The first clause is based on the second clause which is in simple past tense. 48. Fill in the blank in the following sentence with the most appropriate tense from the given options. Soon after he had left, he realized he his wallet. (a) was forget (b) has forgotten (c) had forgotten (d) forget Ans. (c) When two actions take place in past tense, the action which takes place before is considered in past perfect tense (had + V3 ) and which one takes place later is considered in past indefinite tense (V2). ’He realized that he had forgotten his wallet’, in this sentence the action of forgetting takes place first and later he realized. So ’had forgotten’ is the appropriate option here. 49. Fill in the blank in the following sentence with the most appropriate tense from the given options. Who was that girl you to when I passed you in the street ? (a) was talking (b) were talking (c) talked (d) had talked Ans. (b) who was that girl you were talking to when I passed you in the street? This sentence is indicating the continuity of action, and the structure of the sentence is in ’past. Hence, ’were talking’ is the correct option.

91

YCT

50.

Fill in the blank in the following sentence will the most appropriate tense from the given options. He ................. in this colony since 1980. (a) had lived (b) was living (c) has been living (d) is living Ans. (c) He has been living in this colony since 1980. This sentence is showing the continuity of action and time is also given, these conditions indicate to Present Perfect Continuous tense (has been + Vi + ing), hence, option (c) is the correct answer.

51.

Which of the following is NOT enumerated as a constituent element of tragedy by Aristotle ? (a) Imagery (b) Diction (c) Character (d) Plot Ans. (a) According to Aristotle, six constituent parts of tragedy arePlot, character, thought, diction, song and spectacle. The plot is the most important part of a tragedy. Therefore, it is clear that Aristotle does not enumerate imagery as a constituent element of tragedy. 52. Pick out the attribute not mentioned by Aristotle as a requirement of Character(a) good (b) lifelike (c) heroic (d) consistent Ans. (c) According to Aristotle, the tragic hero (character) must be of high rank, relatively virtuous (good), true to life (lifelike) and consistent. He suffers a tragic flaw, which causes his downfall. 53. Select the statement that is FALSE with reference to Aristotle’s conception of tragedy. (a) The change of fortune should follow the probable and necessary sequence of events (b) The plot should be simple, not complex (c) It must imitate actions arousing pity and fear (d) The plot must have multiple issues to enrich its dramatic texture Ans. (d) Aristotle put forward the concept of ’unity of action’ that means tragedy should deal with only one issue. Hence, Aristotle rules out that the plot must have multiple issues to enrich its dramatic texture. 54. In the essay "On the Art of Poetry”, Horace states that poems should (a) focus only on beauty (b) have both beauty and charm (c) focus only on charm (d) focus neither on beauty nor on charm Rajasthan PSC College Lecture-I Exam. 2014

Ans. (b) Horace in his ”On the Art of Poetry” states that poetry should comprise both beauty and charm. According to him, consistency is a special characteristic of poem. He asserts that the poet should not compose more poems beyond his capacity. 55. Which one of the following is not included among the sources of Sublimity by Longinus ? (a) the ability to form grand conceptions (b) the stimulus of powerful and inspired emotions (c) bombastic style (d) the creation of a noble diction Ans. (c) Referring to ’Sublimity’, Longinus discusses how writers seldom use bombastic style and language in their attempt to writing sublime passage. Hence, he does not include bombastic style among the sources of sublimity. 56. Imitation, for Aristotle, is a (a) concept of mimicry (b) reductive concept (c) creative act (d) fictitious concept Ans. (c) In Aristotle’s view, poetic imitation is an act of imaginative creation by which the poet draws his poetic material from the phenomenal world, and makes something new out of it. 57. ’’You must give your days and nights to the study of Greek models”. Who was the first critic to lay down this doctrine with regard to poetry ? (a) Virgil (b) Horace (c) Longinus (d) Aristotle Ans. (b) Horace, renowned for his ’Ars Poetica’ states that ’’you must give your days and nights to the study of Greek models”. When Plato and Aristotle refer to ’mimesis’ it means imitation of nature; to Horace it means imitation of other writers, particularly the old Greek writers. 58. With reference to ’auchitya’ (decorum) in the Natyasastra, which of the following statements is FALSE. (a) A figure should be so devised as to be concerned with a ’Rasa’ (b) Plot, character, and style make a drama an organic whole (c) ’’Rasa’ should be the primary focus of the writer (d) Every element in the drama must be in consonance with ’Rasa’ Ans. (b) The Natya Sastra is a Sanskrit treatise on the performing arts. Auchitya is described as using the elements of a poem such that they deliver essence in their appropriate place. Auchitya includes Guna, Dosha, Alankara, Dhvani, Rasa, Vakrata etc. It has nothing to do with plot, character and style.

92

YCT

59.

Which of the following is NOT one of the eight ’Rasas’ mentioned by Bharat ? (a) Shringara (b) Hasya (c) Santarasa (d) Vira Ans. (c) Bharat Muni mentions eight Rasas(1) Shringara (love) (2) Hasya (humor) (3) Raudra (anger) (4) Karuna (compassion) (5) Bibhatsa (disgust) (6) Bhayanak (horror) (7) Vira (courage) (8) Adbhuta (wonder). He does not mention Santarasa. 60. The theory of ’Dhvani’ school can be condensed into three seminal statements. Which of the following is NOT one of them ? (a) ’Rasa’ is the soul of poetry (b) ’Alankara’ has an intrinsic beauty which is not dependent on ’Rasa’ (c) The mode by which poetry reveals ’Rasa’ is ’Dhvani’ (sugestion) (d) ’Auchitya’ (decorum) is the harmonious accordance of the body to the soul Ans. (b) Anand Vardhan expounded the theory of dhvani. The following statements(1) ’Rasa’ is the soul of poetry (2) The mode by which poetry reveals ’Rasa’ is ’Dhvani’ (3) Auchitya (Decorum) is the harmonious accordance of body to the soul are related to Dhavani but ’’AlanKara’ has an intrinsic beauty which is not dependent on ’Rasa’ is not related to Dhvani. 61. The function of poetry, according to Sidney, is to (a) teach and delight (b) amuse and delight (c) teach and instruct (d) teach like a philosopher Ans. (a) According to Sidney poetry, is an art of imitation, a representing, counterfeiting, or figuring forth, to speak metaphorically, a speaking picture, with this end, to teach and delight. 62. Sidney describes the poet as ’’the right popular philosopher” because(a) only the learned can understand the poet (b) people in general can understand the poet (c) the poet teaches in an obscure manner (d) philosophy is more popular than history Ans. (b) Sidney describes the poet as ’’the right popular philosopher” because the philosopher teaches in obscure manner so the learned only can understand him. It means he teaches them that are already taught. But people in general can understand the poet. Rajasthan PSC College Lecture-I Exam. 2014

63.

In the essay ”An Apology for Poetry”, Sidney states that for the Greeks the poet was a (a) philosopher (b) prophet (c) versifier (d) maker Ans. (d) An Apology for Poetry is a work of literary criticism by Philip Sidney. In this book Sidney states that for the Greeks the poet was a maker. 64. The work ”A Defence of Rhyme”, which answered the attacks on the use of rhyme in English verse during the Elizabethan Period was written by(a) Thomas Compion (b) Francis Bacon (c) Samuel Daniel (d) Edmund Spenser Ans. (c) Samuel Daniel (1562-1619) was an English poet and historian. His work ’ A Defence of Rhyme’ is a response on the use of rhyme in English verse. 65.

George Puttenham’s ’’Art of English Poesie” promotes the idea of(a) Latin poetry (b) vernacular poetry (c) imitation of Greek models (d) poetry as instruction Ans. (b) George Puttenham (1529-1590) was an English writer and literary critic. His book ’’Art of English Poesie” promotes the Vemaculer Poetry.

66.

Ben Jonson believed that comic and satiric literature had a(a) moral purpose (b) libellous intent (c) slanderous objective (d) kathartic effect

Ans. (a) Ben Jonson believed that comic and satiric literature had a moral purpose. 67. In Ben Jonson’s view, an important requirement in a poet is(a) an ability to compose rhymes (b) an adequate amount of metaphysical wit (c) a goodness of natural wit (d) a goodness of supernatural wit Ans. (c) In Ben Jonson’s view, an important requirement in a poet is a goodness of natural wit. According to him ”We require a poet or maker ...... a goodness of natural wit. For whereas all other Arts consist of Doctrine, and Percepts : the poet must be able by nature and instinct, to pour out the Treasure of his mind........ ”

93

YCT

68.

Which English literary critic said, "It is not enough that Aristotle said so, for Aristole drew his models of tragedy from Sophocles and Euripides : and he had seen ours, might have changed his mind”. (a) Ben Jonson (b) Dryden (c) Sidney (d) Pope Ans. (b) The above cited statement is given by John Dryden. Dryden stated this in his ’’Essay of Dramatic Poesie”, published in 1668. 69. Dryden undertakes to vindicate the honour of our English writers from the censure of those who unjustly prefer French in the spirited defence of(a) Lisdeius (b) Neander (c) Crites (d) Eugenius Ans. (b) Dryden undertakes to vindicate the honour of our English writers from the censure of those who unjustly prefer French in the spirited defence of Neander. Lisideius favours French drama over English drama, then Neander defends the latter in Dryden’s ’An Essay of Dramatic Poesie’. 70. Pope’s advice to critics is to(a) acquire a little learning (b) follow nature (c) focus on minor faults _____(d) work with a sense of pride _______________ Ans. (b) Pope’s advice to critics is to follow nature. Alexander Pope gives this advice in his book ’Essay on Criticism’ (1711). He asserts that art should be derived from Nature, should seek to replicate Nature and can be tested against the alternating standard of Nature. 71. For Johnson, the artistic justification for Shakespeare’s neglect of the unities of time and place in his plays is that (a) these unities make the drama credible (b) Shakespeare was a learned man (c) delusion, if delusion be admitted, has no certain limitation (d) it is not possible to observe these unities in a _________play_________________________________ Ans. (c) For Johnson, the artistic justification for Shakespeare’s neglect of the unities of time and place in his plays is that delusion, if delusion be admitted, has no certain limitation. 72. In the preface to Joseph Andrews, Henry fielding describes the novel in theoretical terms as a (a) comic epic in a poetic form (b) comic epic poem in prose (c) tragic poem in prose (d) classical epic in prose Rajasthan PSC College Lecture-I Exam. 2014

Ans. (b) Henry Fielding termed his novel ’Joseph Andrews’ as a ’comic epic in prose’ in the ’Preface to Joseph Andrews’. Fielding has combined the ideal of ’comic epic’ and ’prose epic’ to produce what he termed as ’comic epic in prose’. 73. Henry fielding is of the view that the only source of the ’’true Ridiculous” is (a) affectation (b) irony (c) humour (d) mockery Ans. (a) Henry Fielding in the ’Preface of Joseph Andrews’ states that ’’the only source of true ridiculous is affectation”, to which Fielding assigns two possible causes, ’’Vanity or hypocrisy”. 74. In his discussion of the pleasures of the imagination, Addison limits himself to the pleasures that come from (a) true wit (b) false wit (c) psychology (d) sight ________________________________ Ans. (d) Addison, discussing the ’’Pleasure of the Imagination” states that ”Our sight is the most perfect and most delightful of our all senses”. 75. ’’The language of the age is never the language of poetry”. The above statemetns was made by(a) John Dryden (b) Thomas Gray (c) Alexander Pope _____(d) William Wordsworth ___________________ Ans. (b) Thomas Gray, in a letter to Richard West claimed that ’’the language of the age is never the language of poetry.” Dr. Samuel Johnson criticized his views on poetry. He wrote ’’Gray thought his language more poetical as it was more remote from common use.” _________________________________________ 76. One of the reasons why Wordsworth chose to write humble and rustic life was that (a) it is easy to find people leading such a life (b) green surroundings are always more inspiring (c) the essential passions of the heart mature better in such a condition (d) the feelings of people leading such a life are more inspiring Ans. (c) Wordsworth chose to write humble and rustic life because the essential passions of the heart mature better in such a condition. 77. Wordsworth believes that as compared with common people, a poet is endowed with(a) tenderness which is infectious (b) more livery sensibility (c) superficial knowledge of human nature (d) a less comprehensive soul

94

YCT

Ans. (b) In the ’Preface to Lyrical Ballads’, Wordsworth discusses the definition, qualities and function of a poet as - ”he is a man speaking to men : a man, it is true, endowed with more lively sensibility, more enthusiasm and tenderness, who has a greater knowledge of human nature and a more comprehensive soul, than are supposed to common among mankind.” 78. For S.T. Coleridge, poetic faith is constituted of a/an .................. suspension of disbelief for the moment. Pick out the correct word. (a) permanent (b) unwilling (c) absolute (d) willing Ans. (d) For S.T. Coleridge, poetic faith is constituted of a willing suspension of disbelief for the moment. Coleridge put forth this phrase in his ’Biographia Literaria’ (1817) chapter XIV. _____________________ 79. The purpose of introducing metre, according to Coleridge, is to blend(a) delight with emotion (b) delight with instruction (c) emotions with feelings (d) delight with pleasure Ans. (a) The purpose of introducing metre, according to Coleridge is to blend delight with emotion. 80. ’’Poetry turns all things to loveliness ; it exalts the beauty of that which is most beautiful, and its adds beauty to that which is deformed.” The author of th above statement is(a) P.B. Shelley (b) William Blake (c) Wilpliam Wordsworth (d) Lord Byron Ans. (a) Poetry turns all things to loveliness : it exalts the beauty of that which is most beautiful, and it adds beauty to that which is deformed”. This statement is given by P.B. Shelley in his ’A Defence of Poetry’, it was written in 1821. 81. A man who has the ability to live in uncertainties, mysteries, doubts, without any irritable reaching after fact and reason, according to Keats possesses(a) literary sensibility (b) healthy skepticism (c) negative capability (d) spiritual propensity Ans. (c) A man who has the ability to live in uncertainties, mysteries, doubts, without any irritable reaching after fact and reason, according to Keats possesses Negative Capability. The term was coined by Keats in his 22 Dec. 1817 letter to his brothers George and Thomas. Rajasthan PSC College Lecture-I Exam. 2014

82.

The immediate object of a poem proposed by Coleridge is(a) either truth or pleasure (b) neither truth nor pleasure (c) pleasure, not truth (d) truth, not pleasure Ans. (c) ’’The immediate object of a poem is pleasure, not truth” is told by Coleridge. The statement is extracted from Coleridge’s ’Biographia Literaria’.

83.

Keats valued a life of(a) thoughts rather than of sensations (b) sensations rather than of thoughts (c) spiritualism rather than of materialism (d) rationality rather than of mysticism Ans. (b) Keats valued a life of sensations rather than of thought.’ By sensation Keats means the direct intuitions of the imagination. 84. According to Hazlitt, imitation in a work of art pleases by(a) representing the same idea (b) suggesting new ideas (c) detecting already known properties (d) excluding shades of difference Ans. (b) According to Hazlitt, Imitation in a work of art pleases by suggesting new ideas. 85.

In Arnold’s view, which one of the following poets lacked high seriousness? (a) Dante (b) Homer (c) Shakespeare (d) Chaucer Ans. (d) According to Matthew Arnold, Chaucer fails to maintain high seriousness in his poetry. He regards Chaucer as an excellent poet. But he never recommends him as a great classic. Arnold says that Chaucer was more advanced than his time. His views on Chaucer are cited from his ’The Study of Poetry’.

86.

95

Mathew Arnold’s touchstone method’ of critical evaluation uses which of the following tools? (a) Description (b) Analysis (c) Comparison (d) Explanation Ans. (c) Arnold ’touchstone method’ is a comparative method of criticism. According to this method, in order to judge a poet’s work properly, a critic should prepare it to passage taken from works of great masters of poetry and that these passages should be applied as touchstone to other poetry. YCT

87.

Arnold speaks of different kinds of estimates for poetry. Choose the one he does NOT mention. (a) Historical estimate (b) Social estimate (c) Personal estimate (d) Real estimate Ans. (b) Arnold speaks of three kinds of estimates in his book ’The Study of Poetry’. The three estimates related to poetry are(i) historic estimate (ii) personal estimate (iii) real estimate. It is evident that he does not mention social estimate for poetry. _____________________________ 88. ’’Pathetic Fallacy” is a phrase coined by(a) Coleridge (b) Arnold (c) Ruskin (d) Gray Ans. (c) ’’Pathetic fallacy” is a phrase invented by John Ruskin in 1856 to signify any representation of inanimate natural objects that ascribe to them human capabilities, sensations and emotions. 89. In Britain, the doctrine of ’are for art’s sake’ was advocated by(a) Philip Sidney (b) Walter Pater (c) T.S. Eliot (d) Mathew Arnold Ans. (b) ’Art for Art’s Sake’ emphasizes that the value of art is to serve some moral or didactic purpose. In Britain, the term was advocated by Walter Pater. 90. One of the common aspirations of the group known as the Pre-Raphaelite Brotherhood was(a) fidelity to nature (b) fidelity to the ugliness of modem life (c) desire for material prosperity (d) search for new values Ans. (a) Pre-Raphaelite Brotherhood was a group of English painters, poets and art critics, founded in 1848. Some common aspirations of the group were- to have genuine ideas to express; - to study Nature attentively; - to sympathize with what is direct and serious and heartfelt in previous art - to produce thoroughly good pictures and statues. Therefore, fidelity to nature was one among the aspirations. ____________________________________ 91. ’’All art is quite useless”. The author of the above statement is(a) Oscar Wilde (b) Walter Pater (c) Dante Gabriel Rossetti (d) John Carlyle Rajasthan PSC College Lecture-I Exam. 2014

Ans. (a) Oscar Wilde stated that ’’All art is quite useless”. In its explanation he stated that ’art is useless because its aim is simply to create a mood. It is not meant to instruct, or to influence action in any way. 92.

As a critic, Arnold’s approach to poetry is(a) historical (b) scientific (c) pragmatic (d) conservative

Ans. (c) As a critic, Arnold’s approach to poetry is pragmatic. To Arnold poetry is the criticism of life. In his essay ’The Study of Poetry’ he says that poetry alone can be our sustenance and stay in an era where religious beliefs are fast losing their hold. 93.

One of the elements the Pre-Raphaelites were interested in was (a) medievalism (b) rationalism (c) classicism (d) metaphysical conceits

Ans. (a) Pre-Raphaelite group founded in 1848 in leadership of D. G. Rossetti, was a revolt and reaction against the conventionality of poetry represented by Tennyson. One of its element was that the PreRaphaelite poets escaped from the darkness and ugliness of contemporary society, turned their eyes to the good old days of medievalism. 94.

The essay ’’Modern Fiction” was written by(a) D.H. Lawrence (b) Virginia Woolf (c) E.M. Forster (d) Percy Lubbock

Ans. (b) ’Modem Fiction’ is an essay by Virginia Woolf. The essay published in 1919 is a criticism of writers and literature from the previous generation. 95.

96

T.S. Eliot’s concept of tradition does NOT include which of the following : (a) Tradition consists in following the ways of the immediate generation before us (b) Tradition cannot be inherited, it has to be obtained. (c) Tradition involves the historical sense. (d) Tradition involves the preception of the timeless and the temporal together

Ans. (a) ’Tradition and the Individual Talent’ (1919) is an essay written by poet and literary critic T.S. Eliot. The essay was first published in The Egoist and later in Eliot’s book of criticism - ’The Sacred Word’. YCT

96.

According to T.S. Eliot, poetry is not the expression of personality, but escape from (a) exprssion (b) impersonality (c) personality (d) criticism Ans. (c) T.S. Eliot expresses in his essay ’Tradition and the Individual Talent’ that ............. ’Tradition and the Individual Talent’ is one of the most well known works that Eliot produced in his critic capacity. 97. The historical sense, for T.S. Eliot, involves a perception of(a) the historical estimate of the past authors (b) the historical influence of hte past authors (c) not only the pastness of the past, but of its presence (d) not only the presence of the past, but also its future Ans. (c) T.S. Eliot expresses in his essay ’Tradition and the Individual Talent’ that ................... In it he defines Tradition as it helps the Individual talent produce good poetry. Both of them are closely interrelated and inter dependent. 98. In his book Principles of Literary Criticism, I.A. Richards drew special attention to the (a) dictionary meaning of words (b) arbitriness of words (c) classical and modem meaning of words (d) imaginative and emotional impact of words Ans. (d) Ivor Armstrong Richards was an English Educator, known as I.A. Richards. He wrote ’Principles of Literary Criticism’ in 1924. In this book, he drew special attention to the imaginative and emotional impact of words. 99. E.M. Forster made an important distinction between(a) simple adn episodic plots (b) fancy and imagination (c) flat and round character (d) tragic and comic stories Ans. (c) Edward Morgan Forster was an English Fiction writer, essayist and librettist. Many of his novels examine class difference and hypocrisy. He made an important distinction between flat and round characters in his book ’Aspects of Novel’ (1927). 100.

Ans. (c) I.A. Richards used the term ’’synaesthesis” in his book ’The Foundation of Aesthetic’ (1922). Synaethesis means ’Peaceful and tranquil state of mind’. 101. The book The New Criticism was written by(a) Robert Penn Warren (b) W.K. Wimsatt (c) I.A. Richards (d) John Crowe Ransom Ans. (d) John Crowe Ransom was an American Educator, scholar, literary critic poet, essayist and editor. He wrote the book ’The New Criticism’ in 1941. 102. Who among the following was not associated with New Criticism ? (a) Walter Pater (b) Cleanth Brooks (c) Allen Tate (d) Monroe C. Beardsley Ans. (a) Walter Horatio Pater (1839-1894) was English essayist, literary and art critic and fiction writer but he was belonged to The New Criticism. Main writers of New Criticism. Main writers of New Criticism are Allen Tote, Cleanth Brooks, Monroe C. Beardsley, Robert Penn Warren etc. 103.

New Criticism emphasized(a) the use of everyday language (b) the figures of spech in a text (c) the autonomy of the text (d) the influences on the author Ans. (c) New Criticism was a formalist movement in literary theory that dominated American literary criticism in the middle decades of the 20th century. It emphasized close reading, particularly of poetry to discover how a work of literature functions as a selfcontained, self referential aesthetic object.

104. The primary focus of New Criticism is(a) language and creation of meaning through it (b) literary background (c) biographical details (d) author’s intention Ans. (a) New Criticism was a formalist movement in literary theory that dominated American literary criticism in the middle decades of the 20th century. It emphasized close reading, particularly of poetry to discover how a work of literature functions as a selfcontained, self referential aesthetic object.

I.A. Richards has used the term "synaesthesis" 105. For the New Critics, the poem was to denote(a) an organic system of relationships (a) a harmony between body and soul (b) a network of universal structure (b) units of words and music (c) a series of related metaphors (c) a harmony and equilibrium of our impulses (d) a system of cultural relations (d) equilibrium of sense and rhythm

Rajasthan PSC College Lecture-I Exam. 2014

97

YCT

Ans. (a) New Criticism was a formalist movement in literary theory that dominated American literary criticism in the middle decades of the 20th century. It emphasized close reading, particularly of poetry to discover how a work of literature functions as a selfcontained, self referential aesthetic object. 106. Which of the following reading strategies did the New Critics emphasize? (a) slow reading (b) close reading (c) close-range reading (d) skimming Ans. (b) New Criticism was a formalist movement in literary theory that dominated American literary criticism in the middle decades of the 20th century. It emphasized close reading, particularly of poetry to discover how a work of literature functions as a selfcontained, self referential aesthetic object. 107. Wimsatt and Beardsley in their essays ’’Intentional Fallacy” and ’’Affective Fallacy” argued that in the formation of literary judgement we must take into account (a) the author’s intention (b) the effect on the reader (c) both the author’s intention and the effect on the reader (d) neither the author’s intention nor the effect on the reader Ans. (d) Wimsatt and Beardsley in their essays ’’Intentional Fallacy” and ’’Affective Fallacy” argued that in the formation of literary judgement we must take into account neither the author’s intention nor the effect on the reader. When we concentrate on author intention then it is Intentional Fallacy and when we concentrate on reader effect on it, it is affective fallacy. 108. Match List I with List II and select the correct answer by using the code given below : List I List II A. F. de Saussure M. Of Grammatorlogy B. J. Culler N. Mythologies C. R. Barthes O. Course in General Linguistics D. J. Derrida P. Structuuaralist Poetics A B C D (a) O P N M (b) N M O P (c) N O P M (d) M N P O Ans. (a) Course in General Linguistics is a book by Ferdinand de Saussure which comprises the lectures given by him in University of Geneva in 1906 and 1911. Rajasthan PSC College Lecture-I Exam. 2014

Structuralist Poetics (1975), is a book by Jonathan D. Cullar. Mythologies (1957) is a critical book written by Roland Barthes. Of Grammatology (1967), is written by philosopher Jacques Derrida. 109. The origin of structuralism lies in the work of(a) Noam Chomsky (b) Ferdinand de Saussure (c) Roland Barthes (d) Levi Strauss Ans. (b) The origin of structuralism lies in the work of Ferdinand de Saussure. ’Course in General Linguistics’ is a book of Ferdinand de Saussure which comprises the lectures given by him in University of Geneva in 1906 and 1911. 110. Structuralists believed that the world is(a) divided into structures (b) progressing towards a structure (c) organised as structures (d) structured around meaning Ans. (c) Structuralists believed that the world is organised as structures. The origin of structuralism lies in the work of Ferdinand de Saussure. 111. The essay ”The Death of the Author” was written by(a) Sigmund Freud (b) Friedrich Nietzsche (c) Roland Barthes (d) Harold Bloom Ans. (c) The Death of the Author was written by French literary critic and theorist Roland Barthes. 112. For Roland Barthes, the text is a linguistic/literary object which is(a) given to us by the author (b) found on the shelves of a library (c) stable, fixed and enclosed within one discipline (d) fluid, has many levels of meaning and is interdisciplinary Ans. (d) For Roland Barthes, the text is a linguistic /literary object which is fluid, has many levels of meaning and is interdisciplinary. 113.

98

Derrida questions(a) certain assumptions underlying western metaphysics (b) assumptions underlying deconstructive readings (c) the worth of litgerary woks of the West (d) the distinction between speech and writing YCT

Ans. (a) Jacques Derrida (1930-2004) questions certain assumptions underlying western metaphysics. Derrida developed Deconstruction theory in 1968. 114.

For Deconstructionists, the reversal hierarchy in a binary pair of opposition is(a) the final stage (b) the first stage (c) an unnecessary stage (d) an undesirable stage

of

Ans. (b) For Deconstructionists, the reversal of hierarchy in a binary pair of opposition is the first stage. 115. Which of the following phrases captures the defining characteristic of Modernism(a) ’’Make it new” (b) ’’Read the classics” (c) ’’Follow nature” (d) ’’Work harder” Ans. (a) Modernism is a philosophical movement. According to that movement traditional forms of art, architecture, literature, religious faith and philosophy became old. Therefore, Ezra Pound’s ’’Make it New’ policy was invocated. 116.

Heteroglossia was a concept introduced in literary theory by(a) Henry James (b) Mikhail Bakhtin (c) E.M. Forster (d) Irving Babbitt

Ans. (b) The word ’Heteroglossia’ was used by Mikhail Bakhtin in his paper ’Discourse in the Novel’ in 1934. 117. Carnivalesque is used in literary theory as a mode that(a) associates distinct phonological systems (b) subverts the assumptions of the dominant style through humour and chaos (c) configures the representation of space and time (d) includes a diversity of philosophical backgrounds Ans. (b) ’Carnivalesque’ is a term used by ’Mikhail Bakhtin’ that subverts the assumptions of the dominant style through honour and chaos. 118. Modernism with reference to literature reveals which of the following tendencies ? (a) Adherence to established rules (b) Conformity in form (c) Experiments in form and style (d) Character as a coherent and unified being Rajasthan PSC College Lecture-I Exam. 2014

Ans. (c) Modernism gives importance to experiments in form and style. It talks about newness by removing the traditional art and forms. 119. Which of the following authors would NOT be included among the great literary Modernists? (a) T.S. Eliot (b) Bernard Shaw (c) James Joyce (d) Samuel Beckett Ans. (b) Samuel Beckett, T. S. Eliot, James Joyce, D. H. Lawrence, Forster etc are included in ’Great literary Modernists’. G. B. Shaw is not considered among the great literary Modernists. 120. The important characteristics of Postmodernism do NOT include(a) fragmented forms (b) discontinuous narratives (c) blurring of distinction between genres (d) emphasis on objectivity Ans. (d) Post Modernism is developed in the middle to last of the twentieth century. It shows distrust in objective reality, grand narrations and ideologies etc. 121. By whom was the term ’’intertextuality” first used and where? (a) Ihab Hassan The Postmodern Turn (b) Julia Kristeva Word, Dialogue and Novel (c) Michel Foucault The Order of Things (d) Roland Barthes Writing Zero Degree Ans. (b) Term ’Intertextuality’ was used by Julia Kristeva in her book ’Word, Dialogue and Novel’. 122. William H. Gass coined the term............ for postmodernist novels that deliberately reflect upon themselves as an artifact. (a) pastiche (b) microfiction (c) metafiction (d) metanarrative Ans. (c) William H. Gass coined the term ’Metafiction’ for post modernist novels that deliberately reflect upon themselves as an artifact. It motivates to the Metafiction readers that they should be aware that they are reading a fictional work. 123. In which work of his does Derrida state that ’’There is nothing outside of the text”? (a) Of Grammatology (b) Speech and Phenomena (c) Margins of Philosophy (d) Dissemination Ans. (a) ’Of Grammatology’ (1967) is a book written by French philosopher Jacques Derrida. In the same book he states that ’There is nothing outside of the text’.

99

YCT

124.

What was the title of Frantz Fanon’s seminal book defining the attributes of postcolonialism? (a) Concerning Violence (b) A Dying Colonjalism (c) The Wretched of the Earth (d) Towards the African Revoution Ans. (c) ’The Wretched of the Earth’ (1961) is a book written by Frantz Fanon. In this book he defines the attributes of postcolonialism and dehumanizing effect of colonization.

125.

According to Edward Said the European West perceived ’Orientalism’ as a homogeneous form of(a) The Asians (b) The Africans (c) The Other (d) The New Continents Ans. (c) Said, in his book ’Orientalism’ talks about ’The Other'. ’The Other’ is a homogeneous form of Perceived Orientalism by colonizing countries.

126.

Discourse on Colonialism is a work by(a) Homi K. Bhabha (b) Harish Trivedi (c) Aime Cesaire (d) Bill Ashcroft

Ans. (c) ’Discourse on Colonialism’, is a work by ’Aime Cesaire’, published in 1950. 127. ’’The effect of mimicry on authority of colonial discourse is profound and disturbing” is a quote from(a) Homi K. Bhabha - The Location of Culture (b) Chinua Achebe - An image of Africa (c) Zadie Smith - White Teeth (d) Salman Rushdie - Midnight’s Children Ans. (a) Homi K. Bhabha (bom 1949), wrote the article ’Of Mimicry and Man’ (1992). In this article he discusses post colonialism. According to Bhabha to follow the manners and values of colonizers by colonized is mimicry. ’The effect of mimicry on authority of colonial discourse is profound and disturbing’ is taken from the same book. 128. Match List I with List II. Choose the correct answer from the code given belowList I List II A. Edward Said M. The Wretched of the Earth B. Gayati Spivak N. Orientalism C. Frantz Fanon O. Can the Subaltern Speak Rajasthan PSC College Lecture-I Exam. 2014

A B C (a) O N M (b) M O N (c) N O M (d) M N O ____________________________ Ans. (c) ’The Wretched of the Earth’ is a book written by Frantz Fanon in 1961. ’Orientalism’ (1978) is a book written by Edward W. Said. ’Can the Subaltern Speak’ is an essay written by Gayatri Chakravorty Spivak, (bom 1942). The three pair of option (c) are correct. 129. Which of the following statements does NOT reflect the critical purpose of Postcolonial studies ? (a) To understand and value the modernising effect of colonial literature (b) To account for and combat the residual effects of colonialism upon colonised people (c) Creation of social and cultural spaces for subaltern people (d) To counter the dominant ideologies of the _________west _________________________________ Ans. (a) Post Colonialism or Post colonial studies talks about dehumanizing effect of colonization. Hence, option (a) to understand and value the modernizing effect of colonial literature is not the feature of Postcolonialism. 130. ’’Imagined Communities : Reflection on the Origin and Spread of Nationalism” is a book by (a) Salman Rushide (b) Benedict Anderson (c) Partha Chatterji (d) Stuart Hall ___________________________ Ans. (b) ’Imagined Communities’ : Reflections on the origin and spread of Nationalism’, is a book written by Benedict Anderson, very first it was published in 1988. 131. Complete the following statement. Virginia Woolf situates literature within a ........... context. (a) historical (b) spiritual (c) material (d) philosophical _________________________ Ans. (c) Virginia Woolf situates literature within a material content. Virginia Woolf (1882-1941) was an English writer and modernist of twentieth century. 132. ’’One is not born, but rather becomes a woman.” Who has made the above asseration ? (a) Simone de Beauvoir (b) Betty Friedan (c) Germaine Greer (d) Elaine Showalter

100

YCT

Ans. (a) ’One is not bom, but rather becomes a woman’ 137. Who is the eponymous madwoman in Sandra Gilbert and Susan Gabar’s The Madwoman in is a statement by ’Simone de Beauvoir’ in her book ’The the Attic ? Second Sex’ (1949). (a) Jennie in Charotte Perkins Gilman’s ’’The 133. The key characteristics of Luce Irigaray’s Yellow Wallpaper” oeuvre is the focus on(b) Antoinette in Jean Rhy’s Wide Sargasso Sea (a) Marxian economics (c) Bertha in Charotte Bronte’s Jane Eyre (b) Sexual difference (d) Bea Stephens in Chrissy Breen Keffer’s An (c) Empirical statistics American Heir (d) Cultural archetypes Ans. (c) The Maduoman in the Attic (1979), is the book Ans. (b) The key characteristic of Luce Irigaray’s of Sandra Gilbert and Susan Gabar, which examines the Victorian literature by feminist perspective. Gilbert and Oeuvre is the focus on Sexual difference. Gabar take its title from Charlotte Bronte’s ’Jane Eyre’. Luce Irigaray is a Belgian bom French Feminist. There In which Rochester’s wife Bertha Mason is hidden in a are two characteristics of her Oeuvre - sexual difference attic. and disruption of Western Metaphysics. 138. Which feminist wrote A Manifesto for Cyborgs 134. "Woman must write herself : must write about Science Technology and Socialist Feminism ? women and bring women to writing woman (a) Judith Butler must put herself into the text ” is a quote (b) Betty Friedan from(c) Donna Haraway (a) Helene Cixous (d) Mary Daly (b) Luce Irigaray Ans. (c) A Cyborg Manifesto, is written by Donna (c) Elaine Showalter Haraway. Donna Haraway (bom 1944), is a famous (d) Judith Butler feminist and Professor in University of California. Ans. (a) The above statement is stated by Helen Cixous 139. Which Psychoanalytic theory is associated with Algerian, a French feminist writer, in her essay ’The Jacques Lacan ? Laugh of the Medusa’ (1975). (a) Springboard theory (b) Group behaviour theory 135. Elaine Showalter puts the phases of women’s (c) Mirror stage theory writing in the following order(d) Archetype theory (a) feminine, feminist, female Ans. (c) Jacques Lacan (13 April 1901-9 Sept. 1981), (b) female, feminine, feminist was a French Psychoanalyst and psychiatrist. ’Mirror (c) feminist, female, feminine Stage theory’ is a concept given by Jacques Lacon. (d) female, feminist, feminine According to this theory the kids of about 6 months Ans. (a) Elaine Showalter (bom 21 January, 1941) is an recognize themselves by seeing in the mirror, which American literary critic. She put the sequence of enhance the apperception. women’s writing as given in option (a) (Feminine, 140 is a term coined by Carl Jung to refer feminist and female). to structure shared among beings of the same 136. Luce Irigaray begins her famous book ’’The species. sex which is not one” with a quote and an (a) Animus and Anima Principle episode from (b) Collective unconscious (c) Dominant Patterns (a) Miguel Cervantes Don Quixote (d) Maximal effect (b) Louisa M. Alcott’s Little Women Ans. (b) Collective Unconscious is a term coined by (c) Lewis Carroll’s Through the Looking Glass (d) Charlotte Perkins Gilman’s ’’The Yellow Carl Jung to refer to structures shared among beings of the same species. Wallpaper” Collective unconscious refers the mind of unconscious Ans. (c) Luce Irigaray begins her famous book ’The sex structure, that is shared among beings of the same which is not one’ from Lewis Carroll’s book ’Through species. the Looking Glass’. 141. Harold Bloom’s book The Anxiety of Influence ” she suddenly began again I know it sees the identity crisis in each generation of begins with L” Through the looking Glass. poets as an enactment of Freud’sRajasthan PSC College Lecture-I Exam. 2014

101

YCT

(a) Dependency Complex (b) The Oedipus Complex (c) Martyr Complex (d) Persecution Complex Ans. (b) Harold Bloom’s book ’The Anxiety of Influence’ (1973) sees the identity crisis in each generation of poets as an enactment of Freud’s Oedipus Complex. 142. All of Sigmund Freud’s work depends on the notion of(a) Madness (b) Unconscious (c) Transparence (d) Projection Ans. (b) All of Sigmund Freud’s work depends on the notion of Unconscious. Freud (1856-1939) was on Austrian Neurologist and founder of Psychoanalysis. By the reference of Psychoanalysis he told that Unconscious controls One’s behavior to a great extent. 143. Psychoanalytic criticism uses : (a) literary theories in techniques of pshychoanalysis (b) historical data for the construction of the unconscious (c) some of the techniques of psychoanalysis in the interpretation of literature (d) some of the techniques of psychoanalysis in the treatment of neurotic patients Ans. (c) Psychoanalytic Criticism is based on Freud's Psychoanalysis theory. It interprets the literature according to Psychoanalysis theory. 144. Freud divided human psyche into : (a) unconscious, anxiety and repression (b) desires and dreams (c) oedips and electra complex (d) id, ego and super ego Ans. (d) Sigmund Freud divided human Psyche in three parts - Id, ego and super-ego. It is a part of our personality that exists by birth. Super-ego is a part that senses the morality and ego makes a balance between the two. 145. Parallel reading of literary and non-literary texts usually of the same historical period is called (a) Socio-historical criticism (b) Socio- cultural criticism (c) New Historicism (d) Historiography Ans. (c) New Historicism was developed in 1980s. It is developed by Stephen Greenbait. New Historicists talk about the parallel reading and non-literary texts, that were written in the same historical period. Rajasthan PSC College Lecture-I Exam. 2014

146. Which book is regarded as the beginning of New Historicism ? (a) H. Aram Veeser’s The New Beginning (b) Michel Foucault’s Discipline and Punish (c) D.G. Myer’s The New Historicism (d) Stephen Greenblatt’s Renaissance SelfFashioning : From Moore to Shakespeare Ans. (d) Stephen Greenbait book Renaissance selffinancing : from Moore to Shakespeare (1980) is regarded as the beginning of New Historicism. 147. Besides the English Renaissance Literature, New Historicists also focus on (a) Romantic poets (b) Victorian critics (c) Colonial novelists (d) Diaspora writers Ans. (a) Besides the English Renaissance Literature, New Historicists also focus on Romantic Poets. 148. "The notion of culture as text has a further major attraction : it vastly expands the range of objects available to be read and interpreted" is said by(a) Jerome Me Gann (b) Catherine Gallangher and Stephen Greenblatt (c) Michel Foucault (d) Clifford Geertz Ans. (b) ’’The notion of culture as text has a further major attraction : it vastly expands the range of objects available to be read and interpreted" The above statement is said by Catherine Gallangher and Stephen Greenblatt. 149. Complete the following statement. For New Historicism ............. are agents of history. (a) cultural texts (b) scientific texts (c) futuristic texts (d) pre-historic texts Ans. (a) The complete statement is"For New Historicism Cultural texts are agents of history." 150.

New Historicists firmly believe that literary texts can in fact tell us(a) something about the world outside of the text (b) nothing about the world outside the text (c) little about the world outside of the text (d) something about the inner world of the author Ans. (a) New Historicists firmly believe that literary texts tell us many things from the outside world of text.

102

YCT

Rajasthan Public Service Commission College Lecture Exam. 2014

ENGLISH-II (Solved Paper) 1. The legend of King Arthur Round Table is celebrated in (a) Reginal Peacock (b) (c) Sir Thomas Malory (d)

and Knights of the Morte D Arthur by William Tyndale Thomas Moore

Ans: (c) The legend of King Arthur and Knights of the round table is celebrated in Morte D Arthur by Sir Thomas Malory Le Morted. Arthur is a reworking of traditional tales by Sir Thomas Malory about the legendary king Arthur, Guinevere, Lancelot, Merlin and the Knights of the Round Table. Malory interprets existing French and English stories about these figures and adds original matter. _________________________ 2. The book Euphues is generally called the precursor of the English novel. Who is the author of this book? (a) George Gascoigne (b) John Lyly (c) Thomas Nashe (d) Thomas Delany Ans: (b) ‘Euphues: The Anatomy of Wyt’, a did actic romance written by John Lyly, was entered in the stationer’s Register 2 December 1578 and published that same year. The name Euphues is derived from Greek meaning “graceful, witty”. Lyly adopted the name from Roger Ascham’s The Schoolmaster, which describes Euphues as a type of student who is apte by goodness of witte, and applicable by readiness of wit, to learning, having all other qualities of the mind and parts of the bodie, that trobled, mangled, and halfed, but sound, whole, full & hable to do their office (194) Lyly’s mannered style is characterized by parallel arrangements and periphrases. _____________________ 3. Thomas Moore’s Utopia is set in (a) “Extropia” (b) “The Neverland” (c) “The New Atlantis” (d) “The New World” Ans: (d) Thomas Moore’s Utopia is set in ‘The New World’ Utopia is a work of fiction and political philosophy published in 1516 in England. The book is a frame narrative primarily depicting a fictional Island society and its religious, social and political customs many aspects of More’s description of Utopia are reminiscent of life in monasteries. __________________ 4. Identify the play by Christopher Marlowe in which these lines occur- 66Was this the face that launch’d a thousand ships....”? (a) The Jew of Malta (b) Edward II (c) The Massacre of Paris (d) Doctor Faustus Rajasthan PSC College Lecturer-II Exam. 2014

[Exam. Date : 28 June 2016]

Ans: (d) “Was this the face that launc’d a thousand ships”. These line occurs in Christopher Marlowe’s, Doctor Faustus. Referring to Helen of Troy, or as Marlowe had it Helen of Greece her (Helen) abduction by Paris was said to be the reason for a fleet of a thousand ships to be launched into battle initiating the Trojan Wars. __________________________________ 5. Identify the dramatist who does NOT belong to the group of dramatists known as “University Wits”. (a) Thomas Nashe (b) Robert Green (c) Thomas Kyd _______(d) Cyril Tourneur Ans: (d) The University wits is a phrase used to name a group of late 16th century english playwrights and pamphleteers who were educated at the Universities and who became popular secular writers prominent members of this group were: ♦ Christopher Marlowe ♦ Robert Greene ♦ Thomas Nashe ♦ John Lyly ♦ Thomas Lodge ♦ George Peele ♦ Thomas Kyd Whereas, Cyril Tourneur was an English soldier diplomat and dramatist who wrote. The Atheist’s Tragedy, the Revenger’s Tragedy. __________________ 6. Who among the following is called the “poet’s poet”? (a) Geoffrey Chaucer (b) William Shakespeare (c) Edmund Spenser (d) John Milton Ans: (c) Edmund Spenser is called the ‘poet’s poet’ Edmund Spenser (1552/1553-13 January 1599) was an english poet best known for the Faerie Queen, an epic poem and fantastical allegory celebrating the Tudor dynasty and Elizabeth I. He is recognized as one of the premier craftsmen of nascent modern English verse, and is often considered of the greatest poets in the English language. _____________________________________ 7. The Redcross Knight in The Faerie Oueene represents. (a) Truth (b) Holiness (c) Temperance (d) Courtsey

103

YCT

Ans: (b) The Redcross Knight, hero of Book I. Introduced in the first canto of the poem, he bears the emblem of saint George, patron saint of England; a redcross on a white background is still the flag of England. The Redcross knight is declared the real saint George in Canto X. He also learns that he is of English ancestry, having been stolen by a Fay and raised in Faerieland in the climatic battle of Book I, Redcross slays the dragon that has laid waste to Eden. He marries Una at the end of Book I, but brief appearances in Books. II and III show Redcross still questing through the world. He represents still questing through the world. He represents Holiness. ____________________ 8. Which poem pronounces the poet’s aim as, “Fierce Warres and faithful loves shall moralize my song”? (a) The Faerie Queene (b) “A Hymn to God the Father” (c) “A Parting” (d) “Christ’s Victory and Triumph” Ans: (a) The Faerie Queene pronounces the poet’s aim as, Tierce Warres and faithful loves shall moralize my song”. The Faerie Queene is an incomplete English epic poem by Edmund Spenser Books I to III were first published in 1590, and then republished in 1596 together with books IV to VI. The Faerie Queene is notble for its form; it is one of the longest poems in the English language and the origin of a verse form that came to be known as Spenserian stanza. _____________ 9. Who is the author of the book The Spanish Tragedy? (a) John Fletcher (b) Ben Jonson (c) Thomas Kyd _______(d) Christopher Marlowe Ans: (c) The Spanish Tragedy or Hieronimo is Mad Again is an Elizabethan Tragedy written by Thomas Kyd between 1582 and 1592. Highly popular and invluential in its time the Spanish Tragedy established a new genre in English theatre, the revenge play or revenge tragedy. Its plot contains several violent muders and includes as one of its character a personification of Revenge. It is a play within -a-play. ________________ 10. Who wrote The Praise of Folly? (a) Phillilp Massinger (b) Francis Beaumont (c) Desiderius Erasmus (d) James Shirley ______ Ans: (c) In Praise of Folly is an essay written in Latin in 1509 by Desiderius Erasmus of Rotterdam and first printed in 1511. Inspired by Italian humanist Faustino Peisauli’s De Triumpho stultitiae, it is a satirical attack on superstitious and other traditions of European society as well as on the Western Church. In praise of folly is considered one of the most notable works of the Renaissance and played an important role in the beginnings of the protestant Reformation.____________ 11. Of what does Bacon say that it does, “offend the law .....putheth the law out of office”? (a) Truth (b) Revenge (c) Envy _____________(d) Simulation ________ Ans: (b) Revenge is the correct option. Rajasthan PSC College Lecturer-II Exam. 2014

12. Ben Jonson’s The Alchemist presents tricksters and gulls of whom ........is the most masterly. (a) Sir Epicure Mammon (b) Morose (c) Subtle ____________(d) Dapper ___________ Ans: (a) In ‘The Alchemist’, Jonson unashamedly satiries the follies, vanities and vices of mankind, most notably greed-induced credulity people of all social classes are subject to Jonson’s ruthless, satirical wit. He mocks human weakness and gullibility to advertising and to ‘miracle cures’ with the character of Sir Epicure Mammon, who dreams of drinking elixir of yoth and enjoying fantastic sexual conquests. ________________ 13. Who were called the “Tribe of Ben”? (a) poets who followed Ben Jonson (b) dramatists who followed Ben Jonson (c) courtiers who followed ben Jonson (d) admirers of Ben Jonson ___________________ Ans: (a) The Term, ‘Tribe of Ben’ was employed as self-descriptrion by some of the cavalier poets who admired, followed and were influenced by Jonson’s poetry,including Robert Herrick, Richard Lovelacew, Sir John Suckling and Thomas carew. _______________ 14. “I am a man more sinned against than sinning”, is an excerpt from the play ............ (a) King Lear (b) Romeo and Juliet (c) Julius Caesar _______(d) Othello ___________ Ans: (a) These lines “I am a man more sinned against than sinning”, is an excerpt from the play King Lear. A king is supposed to have all the that he needs without having to worry about anything in his late years. Yet King Lear, in Act 3, Scene 2, Cried out in pitifully an a man more sinned against than sinning. ______________ 15. The character Bottom appears in the following Shakespearean comedy: (a) As You Like It (b) Twelfth Night (c) A Midsummer Night’s Dream (d) A Comedy of Errors ______________________ Ans: (c) Nick Bottom is a character in Shakespeare’s A Midsummer Night’s Dream who provides comic relief throughout the play. A weaver by trade he is famously known for getting his head transformed into that of a donkey by the elusive Puck._______________________ 16. Which of the following translated Homer’s “The Illiad” and “The Odyssey”? (a) Ben Jonson (b) George Chapman (c) Edmund Spenser (d) Philip Sidney ______ Ans: (b) George Chapman translated Homer’s “The Illiad” and ‘The Odyssey’. George Chapman (15591634) was an English dramatist, translator, and poet. He was a classical scholar whose work shows the influence of stoicism Chapman has been identified as the Rival poet of Shakespeare’s Sonnet by William Minto and as an anticipator of the Metaphysical Poets of the 17th century Chapman is best remembered for his translations of Homer’s Illiad and Odyssey, and the Homeric Batrachomyomachia.

104

YCT

17. The formal structure of a typical Shakespearean Sonnet is: (a) An octave and a sester (b) Three quatrains and a couplet (c) An octave, a quatrain and a couplet (d) A couplet followed by three quatrains Ans: (b) The Shakespearean Sonnets are constructed from three quatrains, which are four line stanzas, and a final couplet composed in iambic pentameter. The rhyme scheme is a abab cdcd efef gg. The Sonnets are mostly dedicated to ‘Mr. W.H. and the Dark Lady’. 18. The lines “stone walls do not a prison make, nor iron bars a cage”, are from? (a) Philip Sidney’s “Astrophel an Stella” (b) Edmund Spenser’s “Epithalamion” (c) Richard Lovelace’s “To Althea from Prison” Ans: (c) To Althea, from Prison is a poem written by Richard Lovelace in 1642. The poem is one of Lovelace’s best-known works, and its final stanza’s first line (“stone walls do not a prison make nor iron bars a cage”) is often quoted. Lovelace wrote the poem while imprisoned in gate house prison to encourage the lergy Act 1640 to be annulled. _________________________ 19. Which of Andrew Marvell’s poems deals with the theme of “Carpe Diem” (seize the day)? (a) “Thoughts in a Garden” (b) “To his Coy Mistress” (c) “An Horatian Ode upon Cromwell’s Return from Ireland” (d) “Upon Appleton House” ___________________ Ans: (b) “To his Coy Mistress’ is a sublime example of ‘Carpe Diem’ poem, a Latin phrase meaning ‘seize the day’. This poem is to the genre what the lion is to the animal kingdom or the oak to the vegetable kingdom the poem’s depth. Seduction is the theme of practically every Carpe Diem poem, and each uses the theme of times as a means to an end. _______________________ 20. Who says of Donne: “He affects the metaphysics”? (a) Dr Samuel Johnson (b) T.S. Eliot (c) John Dryden (d) H.J. C Grierson Ans: (c) It was John Dryden who said about John Donne that “He affects the metaphysics’ not only in his satires, but in his amorous verses, where nature only should region.__________________________________ 21. Philip Sidney’s Acradia is ............ (a) an epic poem (b) a verse drama (c) a prose romance (d) a ballad __________ Ans: (c) The countess of Pembroke’s Arcadia, also known simply as the ‘Acradia’, is a long prose work by Sir Philip Sidney written towards the end of the 16th century. Having finished one version of his text, Sidney later significantly expanded and revised his work scholars today often refer to these two major versions as the old Arcadia and the New Arcadia. Rajasthan PSC College Lecturer-II Exam. 2014

22. Which of the following plays resulted in the close of all theaters? (a) All fools (b) The Isle of Doqs (c) The White Devil (d) Poetaster Ans: (c) ‘The Isle of Dogs’ is a play by Thomas Nashe and Ben Jonson which performed in 1597. It was immediately suppressed, and no copy of it is known to exist. _________________________________________ 23. Shakespeare incorporates Livy’s account of Tarquin’s lust in a narrative work called. (a) Cymbeline (b) “venus and Adonis” (c) “The Rape of Lucrece” (d) Titus Andronicus Ans: (c) Shakespeare incorporates Livy’s account of Tarquin’s lust in a narrative work called ‘The Rape of Lucrece’ (1594). It is a narrative poem about legendary Lucretia. The poem contains 1,855 lines, divided into 265 stanzas of seven lines each. The rhythm of each line is iambic pentameter. The rhyme scheme for each stanza is ABABBCC, a format known as ‘rhyme royal’. 24. The work the Anatomy of Melancholy was written by. (a) Francis Bacon (b) Thomas Lodge (c) Richard Booker (d) Robert Burton Ans: (d) ‘The Anatomy of Melancholy’ is a book by Robert Burton, first published in 1621, but republished four more times over the next seventeen years with massive alterations and expansions._________________ 25. Bacon describes his essay as: (a) random thoughts (b) dispersed meditations (c) set treatises (d) focussed descriptions Ans: (b) Bacon describes his essays as dispersed meditations. Essays Religious Meditations Places of Perswasion and Disswasion. Scene and Allowed (1597) was the first published book by the Philosopher Statesman and Jurist Francis Bacon. The Essays are written in a wide range of styles, from the plain and unadorned to the epigrammatic they cover topics drawn from both public and private life, and in each case the essays cover their topics systematically from a number of different angles weighing one argument against another. ________________________________ 26. Which of the following plays is NOT written by Thomas Middleton? (a) The Spanish Gipsy (b) The Merry Wives of Windsor (c) The Changeling (d) Women Beware Women ___________________ Ans: (b) ‘The Merry Wives of Windsor’ is comedy by William Shakespeare, First published in 1602. ‘The Spanish Gipsy’ is an English Jacobean Tragedy, dating from 1621-22. The play was likely a collaboration

105

YCT

between several dramatists, including Thomas Middleton, William Rowley, Thomas Dekker, and John Ford. ‘The Changeling’ is a Jacobean tragedy written by Thomas Middleton and William Rowley. Women Beware women is a Jacobean tragedy written by Thomas Middleton, and first published in 1657. _______ 27. The White Devil, by John Webster, depicts Italian society as: (a) highly progressive (b) religious and refined (c) corrupt and evil (d) honest and ethical _________________________

Ans: (b) In this ‘Areopagitica’ Milton said “He who destroys a good book, kills reason itself, kills the image of god as it were in the eye”. ‘Areopagitica’ is a 1644 prose polemic opposing licensing and censorship. Areopagitica is among history’s most influential and impassioned philosophical defences of the principle of a right to freedom of speech and expression. ____________ 31. Which of the following poems was written by John Dryden in praise of the Restoration and Monarchy? (a) “The Hind and the Panther” (b) “Religio Laici” Ans: (c) ‘The white Devil’, by John Webster, depicts (c) “Astraea Redux” Italiam society as corrupt and evil,. It is a revenge (d) “Mac Flecknoe” __________________________ tragedy the story is loosely based on event in Italy thirty Ans: (c) ‘Astraea Redux’, written by John Dryden in years prior to the plays ’s composition the murder of 1660, is a royalist panegytic in which Dryden vittoria Accoramboni in Padua on 22 December 1585 welcomes the new regime of king Charles II. It is a Webster’s dramatization of this event turned Italian vivid emotional display that overshadows the cautious corruption into a vehicle for depicting “the political and Heroique Stanzas that Dryden composed for Oliver moral State of England in his own day”, particularly the Cromwells ’s death. In the fromer, Dryden apologizes corruption in the royal court. The play explores the for his allegiance with the Cromwellian government, differences between the reality of people and the way and Dryden was later exclused by Samuel Johnson for they depict themselves as good, ‘white’, or pure. ______ his change in allegiance when he wrote, ‘if he changed 28. John Milton’s ‘L Allegro’ invokes which grace he changed with the nation’. The period between with whom he means to live? Cromewell and the Restoration is illustrated as a time (a) Mirth (b) Joy of chaos in ‘Astraea’, and Charles is greeted as a (c) Happiness (d) Love restorer of peace. ________________________________ Ans: (a) ‘L Allegro’ is a pastoral poem by John Milton 32. Pope in his “Letter to Arabella”, in “The Rape of published in his 1645 poems. L Allegro means ‘the the Lock”, states that he borrowed his happy man’). The poem invokes mirth and other supernatural machinery fromallegorical figures of joy and merriment, and extols the (a) the Kabbals (b) the Rosicrucians active and cheerful life, while depicting a day in the (c) Temple Orientis (d) Thelemicism countryside according to this philosophy. Mirth, as one of the graces, is connected with poetry within Renaissance Ans: (b) Pope in his “Letter to Arabella”, in ‘The Rape literature, and the poem, in its form and content, is of the Lock’, states that he borrowed his supernatural similar to dithrambs to Bacchus or hymns to venus. _____ machinery from the Rosicrucians the Rape of the Lock 29. In which of the following does Manoah report the is a mock heroic narrative poem written by Alexander end of the protagonist in Milton’s Samson Pope the poem satirises a minor incident by Aqonistes? comparising it to the epic world of the gods. __________ (a) “Our cure, to be no cure, sad cure” 33. In Jonathan Swift’s The Battle of the Books: (b) “This horror will grow mild, this darkness light” (a) both sweetness and light are associated with the (c) “Calm of mind, all passion spent” Ancients (d) “Above the smoke and stir of this dim spot (b) both sweetness and light are associated with the which men call earth” Modems Ans: (c) In the line “Calm of mind, all passion spent”, (c) sweetness is associated with the Ancients and Manoah report the end of the protagonist in Milton’s light with the Modems Samson Agonistes. Samson Agonistes is a tragic closet (d) light is associated with the Ancients and drama. __________________________________________ sweetness with the Modems 30. In which of his tracts does Milton say “He who Ans: (a) In Jonathan Swift’s The Battle of the Books: destroys a good book, kills reason itself, kills the both sweetness and light are associated with the image god as it were in the eye”? Ancients. It depicts a literal battle between books in the (a) Tetrachordon king’s Library, as ides and authors struggle for (b) Areopagitica supremacy. Because of the satire, “The Battle of the (c) Colasterion Books” has become of term for the quarrel of the (d) De Doctrina Christiana Ancients and the Modems. Rajasthan PSC College Lecturer-II Exam. 2014

106

YCT

34. The surface narrative of Dryden’s Absalom and 39. What do the novels Pamela, Clarissa, Shamela and Lady Susan share with each other? Achitophel” is taken from: (a) They are all picaresque novels (a) English History (b) Plutarch’s Lives (b) They are all travelogues (c) Dutch Legends (d) The Bible (c) They are all dramatic monologues Ans: (d) The Surface narrative of Dryden’s ‘Absalom (d) They are all epistolary and Achitophel’ is taken from the Bible. It is a political satire the poem exists in two parts. The first part, of 1681, is undoubtedly by Dryden. The second part, of 1682, was written by another hand, most likely Nahum Tate, except for a few passages.____________________ 35. The Spectator by Addison and Steele was published in the ........... decade of the Eighteenth century. (a) first (b) second (c) third (d) fourth Ans: (b) The Spectator by Addison and Steele was published in the second decade of the Eighteen century (lasting from 1711 to 1712). ______________________ 36. 44Know then thyself, presume not God to scan, The proper study of mankind is Man”. Where do these lines occur? (a) “The Rape of the Lock” (b) “An Essay on Criticism” (c) “An Essay on Man” (d) “Windsor Forest” Ans: (c) “Know then thyself, presume not God to scan, The proper study of mankind is Man”. These lines occur in “An Essay on Criticism”. It is a poem published by Alexander Pope in 1733 -1734. It is an effort to rationalize or rather ‘vindicate the ways of God to Man’. The essay, written in heroic couplets, comprises four epistles. ___________________________________ 37. Who is Thomas Gray referring to in his lines, “Nor second He, that rode sublime, upon the seraph wings of Ecstasy” from “The Progress of Poesy”: (a) Shakespeare (b) Milton (c) Spenser (d) Donne Ans: (b) It was John Milton to whom Thomas Gray referring in the lines, “Nor second He, that rode sublime, upon the seraph wings of Ecstasy”. From “The Progress of Poesy”. It is Thomas Gray’s grand Pindaric ode, written between 1751 and 1754, is one of the few really successful essays in this difficult form. _________ 38. Goldsmith’s She Stoops to conquer owes its origin to (a) Farquar’s Beaux Stratagem (b) Congreve’s The Way of the World (c) Sheridan’s The Rivals (d) Steels’s The Tender Husband Ans: (a) Goldsmith’s 'She Stoops' to conquer owes its origin to Farquar’s Beaux Strataqem. She stoops to conquer is a comedy first performed in London in 1773. Rajasthan PSC College Lecturer-II Exam. 2014

Ans: (d) Samuel Richardson’s ‘Pamela’ (1740), Henry Fielding’s ‘Shamela’ (1741) Samuel Richardson’s ‘Clarissa, or, the History of a young Lady’ (1748 published) and Jane Austen’s ‘Lady Susan’ (1794) all are epistolary novel. An epistolary novel is a novel written as a series of documents. The usual form is letters, although diary entries, newspaper clippings and other documents are sometimes used. _______________ 40. In which of William Congreve do the characters Millamant and Mirabel appear? (a) The Old Bachelor (b) Love for Love (c) The Way of the World (d) The Double Dealer _______________________ Ans: (c) The way of the world is a William Congreve play. The play is centred on the two lovers Mirabel and Millamant. In order for them to marry and receive Millamant’s full dowry, Mirabel must receive the blessing of Millamant’s aunt, Lady wishfort unfortunately Lady, wishfort is a very bitter lady who despises Mirabel and wants her own nephew Sir Wilfull, to Wed Millament. ____________________ 41. Which of the following is NOT an elegy? (a) “Lycidas” (b) “Thyrsis” (c) “Endymion”________(d) “In Memorium” Ans: (c) ‘Endymion’ is a poem by John Keats published in 1818. It begins with the line ‘A thing of beauty is a joy for ever’. Whereas, ‘Lycidas’ (is a poem by John Milton, written in 1637 as a pastoral elegy dedicated to Edward King). ‘Thyrsis is a poem written by Mathew Arnold in December 1865 to commemorate his friend, the poet Arthur Hugh (laugh), and Tn Memoriam A. H.H.’ is a poem by the British poet Alfred, Lord Tennyson, completed in 1849. It is a requiem for the poet’s beloved Cambridge friend Arthur Henry Hallam), all are elegy. ___________________________ 42. “The still sad music of humanity”. In which poem of William Wordsworth does this line occur? (a) “Ode on the Intimations of Immortality” (b) “Ode on Duty” (c) “Lines Written a few miles above Tintem Abbey” (d) “The Solitary Reaper” ____________________ Ans: (c) ‘The still sad music of humanity’, this line occurs in William Wordsworth ‘Lines written a few miles above Tintem Abbey, (1798) the poem has its roots in Wordsworth’s personal history. The poem is written in tightly-structured decasyllabic blank verse and comprises verses paragraphs rather than stanzas.

107

YCT

43. “Adonais” by P.B. Shelley is an elegy on the death of (a) Byron (b) Keats (c) Leigh Hunt (d) Robert Southey

Ans: (d) In S.T. Coleridge’s poem ‘The Rime of the Ancient Mariner’ (1797-1798) a curse falls on the ship because the mariner shoots an Albatross the Rime of the Ancient Mariner relates the experiences of a sailor who Ans: (b) ‘Adonais: An elegy on the death of John Keats has returned from a long sea voyage. The Mariner stops a man who is on the way to a wedding ceremony and (1821) Author of Endymion, Hperion, etc, is a pastoral elegy written by percy Bysshe Shelley for John Keats in begins to narrate a story. The wedding guest’s reaction from bemusement to impatience to fear to 1821. The poem is in Spenserian Stanza. It was turns published by Charles Ollier in July 1821. _____________ fascination as the mariner’s story progresses, as can be 44. William Blake’s Songs of Innocence contains seen in the language style Coleridge uses narrative techniques such as personification and repetition to short lyrics embodying his views oncreate a sense of danger , the supernatural, or serenity, (a) the contemporary life of London depending on the mood in different parts of the poem. (b) the pleasures of the Garden of Eden 48. Which one of the following poets is attacked by (c) the original state of human society Byron in his poem “The Vision of Judgment”? (d) the hard life of children (a) William Blake (b) Robert Southey Ans: (c) William Blake’s ‘Songs of Innocence’ contains (c) P.B. Shelley (d) John Keats short lyrics embodying his views on the original state of Ans: (b) ‘The Vision of Judgement’ (1822) is a satirical human society. It was first printed in 1789. It is a conceptual collection of 19 poems, engraved with poem in Ottava rima by Lory Byron, Which depicts in artwork. This collection mainly shows, happy, innocent Heaven over the fate of George W’s soul. It was written perception in pastoral harmony, pastoral harmony but at in response to the poet Laureate Robert Southey’s ‘A’ times, such as in the Chimney Sweeper and the little, vision of Judgment (1821), which had imagined the sould Black Boy, subtly shows the dangers of this naive and of King George triumphantly entering to receive his due. vulnerable state. __________________________________ Byron was provoked by the Hig Tory point of view from which the poem was written, and he took personally 45. Which of the following is NOT a Gothic novel? southey’s preface which had attacked those Men of (a) Clara Reeve ’ s The English Baron diseased hearts and deproved imaginations, who had set (b) Jane Austen’s Pride and Prejudice up a ‘satanic school’ of poetry. He mischievously (c) Mathew Gregory’s The Italian referred to Southey as ‘the author of Wat Tyler’. ________ (d) Mary Shelley’s Frankenstein ________________ 49. Jane Austen’s novels are characterised by: Ans: (b) Jane Austen’s Pride and Prejudice (1813 (a) an element of chance published) is not a Gothic Novel. Gothic fiction, which (b) philosophic reflections is largely known by the subgeure of Gothic horror, is (c) social propaganda genre or made of literature and film that combines (d) perfect picture of provincial life fiction and horror, death, and at times romance. _______ 46. Which poem of John Keats has the theme of a Ans: (d) Jane Austen;s novels are characterised by fatal, magical love luring man to his doom? perfect of provincial life. Jane Austen (1775-1817) was (a) “The Eve of St. Agnes” an English novelist known primarily for her six major (b) “La Belle Dame sans Merci” novels, which interpret, critique and comment upon the (c) “Ode to Psyche” British landed gentry at the end of the 18 th century. (d) “ Ode on a Grecian Um” ___________________ Austen’s plots often explore the dependence of women Ans: (b) Ballad ‘La Belle Dame Sans Merci’ (1819) by on marriage in the pursuit of favourable social standing John Keats has the theme of a fatal, magical love luring and economic security. Her works critique the novels of a man to his doom. The poem is considered on English sensibility of the second half of the 18th century and are classic, stereothpical of other of Keats works. It avoids part of the transition to 19th century literary realism. simplicity in interpretation despite simplicity of 50. In the title “The Essays of Elia”, the name Elia structure. At only a short twelve stanzas, of only four stands for: lines each, with a simple ABCB rhyme scheme, the (a) John Carlyle (b) Walter Scott poem is monthless full of enigmas, and has been the (c) Charles Lamb (d) William Hazlitt subject of numerous interpretations. _________________ 47. In S.T. Coleridge’s poem “The Rime of the Ans: (c) Essays of Elia is a collection of essays written Ancient Mariner”, a curse falls on the ship by Charles Lamb; it was first published in book form in because: 1823, with a second volume, Last Essays of Elia, issued in 1833 by the publisher Edward Moxon. The essays in (a) all the sailors hate the innocent mariner the collection first began appearing in the London (b) philosophic reflections Magazine in 1820 and continued to 1825. Lamb’s (c) the mariner shoots one of the sailors essays were very popular and were printed in many (d) the mariner shoots an Albatross 108 Rajasthan PSC College Lecturer-II Exam. 2014 YCT

subsequent editions throughout the nineteenth century. The personal and conversational tone of the essays has charmed many readers; the essays established Lamb in the little he now holds, that of the most delightful of English essayists. Lamb himself is the Elia of the collection and his sister Mary is “Cousin Bridget” Charles first used the Pseudonym Elia for an essay on the South Sea House, where he had worked decades earlier, Elia was the last name of an Italian man who worked there at the some time as Charles, and after that essay the name stuck. ____________________________ 51. From which poem of Robert Browning have these lines been taken-“So, one day more am ideified. Who knows but the world may end to night”? (a) “Porphyria’s Lover” (b) “The Last Ride Together” (c) “Andrea Del Sarto” (d) “The Grammarian’s Funeral” Ans: (b) The above lines have been taken from Robert Browning’s Poem “The Last Ride Together”. The poem is full of romantic overtones. It describes the romantic feelings of the lover towards his Lady love. Therefore, ‘The Last Ride Together’ can be aptly described as a love poem. ____________________________________ 52. “Theirs not to make reply, Theirs not to reason why, Theirs but to do or die”. These lines are from Tennyson’s poem(a) “In Memorium AHH” (b) “Locksley Hall Sixty Years after” (c) “Ullyses” (d) “The Charge of the Light Brigade”

‘flood’, and the ocean that lies, beyond (death), the boundless deep, to which we return. Tennyson is believed to have written the poem while on the sea, crossing the solent from Aidworth to Farringford on the Isle of wight. __________________________________ 54. “this strange disease of modern life, with its sick hurry and divided aim”, Mathew Arnold speaks of this malaise in his poem. (a) “The Scholar Gypsy” (b) “Dover Beach” (c) “Thyrists” (d) “Stanzas from the Grande Chartreuse” _______ Ans: (a) Arnold speaks of the above malaise in this poem ‘The Scholar Gypsy’. In the above lines Arnold implores the scholar gypsy to avoid all who suffer from it, in case he too should be infected and die Arnold ends with an extended simile of a Tyrian merchant seaman who flees from the irruption of Greek competitors to seek a new world in liberia. ‘The Scholar Gypsy’ (1853) is a poem based on a 17th century Oxford Story found in Joseph Glanvill’s the vanity of Dogmatizing. _________ 55. The narrative of Thomas Hardy’s The Return of the Native beings on the evening of(a) May Dance Day (b) Guy Fawke’s Night (c) The Skimmington Ride (d) Maumbury Rings

Ans: (b) ‘The Return of the Native begins on the evening of Guy Fawke’s Night as Diggory Venn is slowly crossing the heath with his van, which is being drawn by ponies. The novel take place entirely in the environs of Egdon Heath, and, with the exception of the Ans: (d) The above lines have been taken from Alfred epilogue, Aftercourses, covers exactly a year and a day. Tennyson’s poem “The charge of the Light Brigade. It ‘The Return of the native’ is Thomas Hardy’s sixth is an 1854 narrative poem about the charge of the Light published novel. It first appeared in the Magazine Brigade at the Battle of Balaclava during the Crimean Belgravia, a publication known for its sensationalism, War. He was the Poet Laureate of the United Kingdom and was presented in twelve monthly installments from at the time he wrote the poem. It was published in ‘The January to December 1878. _______________________ Examiner’ in 1854.______________________________ 56. The pioneering classic of outdoor literature, Travels with a Donkey in Cevennes is the work 53. For what does Tennyson use the extended ofmetaphor of “crossing the bar” in his poem by (a) George Moore (b) George Gissing the same title? (c) Rudyard Kipling (d) R.L. Stevenson (a) setting forth in the unknown world (b) travelling serenely from life through death Ans: (d) The pioneering classic of outdor literature (c) defying conventions and leaning towards ‘Travels with a Donkey in Cevennes’ (1879) is the work Pandeism of Robert Louis Stevenson. Travels with a Donkey (d) commenting on the ill-advised, pre-conceptions describes strevenson’s hilcing trip in the Cevennes, in of human kind. South -Central France.___________________________ Ans: (b) Travelling serenely from life through death is 57. Whose essay does Walter Pater-re-work for the conclusion of his book The Renaissance Studies the extended metaphor used by Tennyson in his poem in Art and Poetry? “crossing the bar”. ‘Crossing the Bar’ is an 1889 poem. It is considered that Tennyson wrote it in elegy; the (a) S.T. Coleridge poem has a tone of finality and the narrator uses an (b) Dante Gabriel Rossetti extended metaphor to compare death with crossing the (c) Johann Winckelmann ‘sandbar’ between river of life, with its outgoing (d) William Morris 109 Rajasthan PSC College Lecturer-II Exam. 2014 YCT

Ans: (d) In was William Morris essay whom Walter 60. In which novel two young girls leave Miss Pikerton’s Academy and one of them throws out Pater rework for the conclusion of his book ‘The of the coach window Johnson’s “Dictionary”, her Renaissance studies in Art and Poetry’ Walter Horatio farewell gift? Pater (1839-1894) was an English essayist, literary and (a) Jude the Obscure (b) Yeast art critic, and fiction writer, regarded as one of the great (c) Vanity Fair ________(d) Foul Play _________ stylists. _______________________________________ Ans: (c) It is William Makepeace Thackeray’s novel 58. Match the opening lines with their texts‘Vanity Fair’ in which two young girls leave Miss A B Pikerton’s Academy and one of them throws out of the (A) “Now what I want is facts” (P) Oliver Twist coach window Johnson’s ‘Dictionary’ her farewell gift. Vanity Fair is a novel which follows the lives of Becky (B) “It was the best of times it sharp and Emmy sedley amid their friends and families (Q) Hard Times was the worst of times during and after the Napoleonic wars. It was first (C) “Among other public published as a 19 volume monthly serial form 1847 to building there is one 1848, carrying the subtitle Pen and Pencil Sketches of (R) David Copperfield a work house” English Life, reflecting both its satisfaction of early 19 th century British society and the many illustrations drawn (D) “Whether I shall turn out by Thackery to accompy the text. __________________ to be the hero of my own 61. Between which two ‘dark places’ does Josepch life these pages must Conrad, in his Heart of Darkness, create a show” (S) A Tale of Two Cities. parallel? (a) A B C D P Q R S (a) Poland and London (b) A B C D S R P Q (b) Africa and Costaguana (c) A B C D Q S P R (c) London and Africa (d) A B C D R P Q S (d) Costaguana and Geneva Ans: (c) All of the above lines are taken from Charles Ans: (c) ‘Heart of Darkness’ (1899) is a novella by Dicken’s novels i.e. Oliver Twist. Hard Times, David Polish-British novelist Joseph Conrad, about a Voyage up the up the Congo River into the Congo Free State, in Copperfield, A Tale of two cities. the heart of Africa, by the Story’s narrot Marlow. ♦ Oliver twist - “Among other public buildings there Marlow tells his story to friends about a boat anchored is one a work house”. Oliver twist is Dickens on the River Thomas, London, England. This setting second novel published as a serial in 1837-39. provides the frame for Marlow’s story of his obsession ♦ Hard Times - “Now, what I want is facts”. It is the with the invory trader Kurtz. Which enables Conrad to create a parallel between between London and Africa as Tenth novel of Dicken published in 1854. ♦ David Copperfield - “Whether I shall turn out to be places of darkness. ______________________________ the hero of my own life these pages must 62. In Nineteen Eighty-Four, George Orwell’s novel, the location is and its protagonist is show”. It was Dicken eighth novel published as a (a) Eurasia; Big Brother serial in 1849-50. (b) Airstrip one; Winston Smith ♦ A Tale of two cities, (1859) -“It was the best of (c) Ministry of Truth; Doublethink time it was the worst of times”. Dickens famous (d) Eustasia; Emmanuel Goldstein ______________ opening sentence introduces the universal approach Ans: (b) Nineteen Eighty -Four, is a dystopian novel by of the book the French Revolution, and the dram English author George Orwell published in 1949. The depicted within._____________________________ novel is set in Airstrip one a province of the superstate 59. The continued popularity of Oscar Wilde’s plays Oceania in a world of perpetual war, and its protagonist depends on: is Dublethink. __________________________________ (a) The polish and scintillating wit of his dialogue 63. In which novel does Aldous Huxley satirise the idea of progress by creating a fantasy of the (b) A sentimental portrayal of the elite of his times future world? (c) A realistic portrayal of the middle class (a) Point Counter Point (b) Eyeless Gaza (d) The slapstick mode of humour (c) Antic Hay _________(d) Brave New World Ans: (a) The continued popularity of Oscar Wilde’s Ans: (d) ‘Brave New World’ is a novel written in 1931 plays depends on the polish and scintillating wit of his by Aldous Huxley, and published in 1932. Set in dialogue. Oscar Fingal O Flahertie Wills Wilde (18 London in the year AD 2540 (632 A.F. ‘Afta Ford’-in October 1854-30 November 1900) was, an Irish the book), the novel anticipates development in playwright, novelist, essayist, and poet. After writing in reproductive technology, sleep-learning, psychological different forms throughout the 1880s, he became one of manipulation, and classical conditioning that combine profoundly to change society. London’s most popular playwrights in the early 1890s. Rajasthan PSC College Lecturer-II Exam. 2014

110

YCT

64. D.H. Lawrence wrote a sequel to his 1915 novel, 68. “Tomorrow, when I wake, or think I do, what The Rainbow entitled ........which continues with shall say of today?” These lines are from ........... the lives of the .......sisters. (a) T.S. Eliot’s “The Love Song of Alfred Prufork” (a) Sons and Lovers; Morel (b) Virginia Woolfs To the Lighthouse (b) Women in Love; Brangwen (c) Samuel Beckett’s Waiting for Godot (c) Lady Chatterley’s Lover, Mellor (d) Wilfred Owen’s “Anthem for Doomed Youth” (d) John Thomas and Lady Jane; Cresswell Ans: (c) The above lines have been taken from Samuel Beckett’s ‘Waiting for Godot’ spoken by Valdimir. Ans: (b) ‘Women in Love’ is a novel by British author Valdimir (affectionately known as Didi, a small boy D.H. Lawrence, published in 1920. It is a sequel to his calls him Mr. Albert) is optimist and represents the earlier novel ‘The Rainbow’ (1915), and follows the intellectual side of the two man character ‘Waiting for continuing loves and lives of the Brangwen Sisters, Godot’ is a play in which two characters, Vladimir and Gudrun and Ursula. Ursula is a teacher and Gudrun is Estragon, wait for the arrival of someone name Godot an artist. They meet Rupert Birkin and Coal-Mine heir who never arrives and while waiting they engage in a Gerald Crich, and the four become friends. ___________ variety of discussions and encounter three other 65. Major Scobie is a character in which novel by character. _____________________________________ 69. Yeats poem “The Second Coming” prophesises Graham Green? the coming of : (a) The End of an Affair (a) God (b) The Quiet American (b) Jesus Christ (c) The Power and The Glory (c) The Holy Spirit (d) The Heart of the Matter ___________________ (d) A new Destructive God ___________________ Ans: (d) ‘The Heart of the matter’ (1948) is a novel by English author Graham Greene, a British intelligence Ans: (d) Yeat’s poem “The Second Coming’ officer in freetown, Sierra Leone, drew on his prophesises the coming of ‘A new Destructive God’. experience there. Greene illustrated by describing ‘The Second Coming’ (Sometimes called the second Scobie, the main character of the book, as “a weak man Advent or the Parousia) is a Christian concept regarding the future return of Jesus to Earth after his ‘First with good intentions doomed by his big sense of pity”. Coming’ and ascension to heaven about two thousand 66. In which of her works does Virginia Woolf invent years ago. The belief is based on messianic prophesises the fictional character of Shakespeare’s sister found in the Canonical gospels and is part of most naming her .........? Christian eschatologies. __________________________ (a) Three Guineas; Roxana 70. In Bernard Shaw’s St Joan the independence of (b) Flush: A Biography: Dorothy Joan is seen in opposition to the forces of: (c) The Waves: Rhoda (a) organised society (b) materialist ideas (d) A Room of One’s Own: Judith ______________ (c) materialist ideas (d) anarchy Ans: (d) In ‘A Room of One’s Own’ (published on 24 October 1929), Virginia Woolf invented the fictional Ans: (a) In Bernard Shaw’s St Joan the independence character of Shakespeare’s sister naming her ‘Judith’. In of Joan is seen in opposition to the forces of organised his Essay Woolf fries to illustrate that a woman with society. _______________________________________ Shakespeare’s gifts would have been denied the same 71. The story of Washington Irvin’s Rip Van Winkle is about. opportunities to develop them because of the doors that (a) The life in the newly formed New England were closed to women.___________________________ (b) Life in the Frontier West 67. Harold Pinter in The Birthday Party makes (c) How life changed after the Civil War Petey remark as Stanley is being taken away: (d) Life of a Southern Slave (a) “You’re dead, you can’t live, you can’t think, you can’t love. You’re dead”. Ans: (c) The story of Washington Irvin’s Rip Van (b) “Don’t let them tell you what to do. I’ve lived Winkle is about How life changed after the Civil War. It that line all my damn life. Never more than was published in 1819. The story of Rip Van Winkle is now”. set in the years before and after the American (c) “Let’s finish and go. Let get it over and go. Get Revolutionary War. _____________________________ the thing done”. 72. James Fennimore Cooper’s The Last of the (d) “Play up, play up and play the game!” Mohicans belongs to the: (a) Twice Told Tales Ans: (b) Harold Pinter in ‘The Birthday Party’ (1957) (b) Tales from Here and There makes Petey remark as Stanley is being taken away. (c) Leather Stocking Tales Don’t let them tell you what to do. I’ve lived that line all my damn life. Never more than now”. ____________ (d) Trader Tales Rajasthan PSC College Lecturer-II Exam. 2014

1

YCT

Ans: (c) ‘The Last of the Mohicans A Narrative of 1757’ (1826) is a historical novel by James Fennimore Cooper’s. It is the second book of the Leather Stocking Tales and the best known to contemporary audiences. It is set in 1757, during the French and Indian War, when France and Great Britain battled for control of North America. The novel is primarily set in the upper New York wilderness, detailing the transport of the two daughters of Colonel, Munro, Alice and Cora, to a safe destination at Fort William Henry. _________________ 73. In Hawthorne’s The Scarlet Letter, the letter “A”stands for: (a) Abducted (b) Abandoned (c) Adultress __________(d) Amoral___________ Ans: (c) In Hawthorne’s ‘The Scarlet Letter’, the letter “A” stands for Adultress. ‘The Scarlet Letter A Romance’ is a 1850 work of fiction in a historical setting, written by American author Nathaniel Hawthorne. The book is considered to be his “masterwork”. Set in 17th century puritan Boston, Massachuselts, during the years 1642 to 1649, it tells the story of Hester Prynne, who conceives a daughter through an affair and struggles to create a new life of repentance and dignity. Throughout the book, Hawthorne explores themes of themes of legalism, sin, and guilt. _____________________________________ 74. Edger Allan Poe’s Raven, in the poem by the same name, repeats the word .........to aggravate his distress. (a) Nothing more (b) Nevermore (c) Never again ________(d) Nathemore ________ Ans: (b) The Raven” is a narrative poem by American writer Edger Allan Poe First published in January 1845, the poem is often noted for its musicality, stylized language, and supernatural atmosphere. It tells of a talking Raven’s mysterious visit to a distraught lover, tracing the main’s slow fall into madness. The lover, often identified as being a student, is lamenting the loss of his love, Lenore. Sitting on a bust of Pallas, the raven seems to further instigate his distress with its constant repetition of the word “Never more”. The poem makes use of a number of folk, mythological, religious, and classical references. _____________________________ 75. Mark Twin’s Adventures of Huckleberry Finn explores the notions of: (a) superstion and magic (b) character and sin (c) race and identity (d) exile and terrorism Ans: (c) ‘Adventure of Huckleberry Finn’ is a novel by Mark Twain, first published in the United Kingdom in December 1884 & in United States in 1885. The book is noted for its colourful description of people and places along the Missippi River set in Southern antebellum society that had creased to exist about 20 years before the work was published. It is often scathing satire on entrenched attitudes, particularly racism. Rajasthan PSC College Lecturer-II Exam. 2014

76. “I celebrate myself, and sing myself’ is a line from Walt Whitman’s ........... (a) “One’s Self I Sing” (b) “ I hear America Singing” (c) “A Passage to India” (d) “Song of Myself’ Ans: (d) “I celebrate myself, and sing myself’, is a line from Walt Whitman’s ‘Song of Myself. It was first published in 1855 in the edition of Leaves of Grass. I is written in free verse style. ________________________ 77. “It‘s when I’m weary of consideration ...I’d like to get away from earth a while And then come back to it and begin over”. These lines are from Robert Frost’s poem .............. (a) “Birches” (b) “After Apple Picking” (c) “Two Tramps at Mud Time” (d) “Mending Walls” ________________________ Ans: (a) The above lines have been taken from Robert Frost’s poem ‘Birches’. It was collected in Frost’s third collection of poetry ‘Mountain Interval’ that was published in 1916 ‘Briches’ along with other poems deal with rural landscape and wildlife shows Frost as a nature poet. The poem is written in blank verse with a particular emphasis on the ‘Sound of Sense’. _________ 78. Ernest Hemingway’s For Whom the Bell Tolls taken its title from: (a) John Donne’s “Devotions upon Emergent Occasions, Meditation XVII” (b) 1558 talk by George Peel (c) A verse in The Bible (d) A speech by U.S. War Confederate, Thomas J. Jackson Ans: (a) ‘For Whom the Bell Tolls’ is a novel by Ernest Hemingway published in 1940. The book’s take is taken from the metaphysical poet John Donne’s series of Meditation and prayers on health, pain, and sickness published in 1624 as ‘Devotions upon Emergent Occassions’, Meditation XVII. ‘For whom the bell Tolls’ tells the story of Robert Jordan, a young American in the international Brigades attached to a republican guerrilla until during the Spanish Civil War. As a dynamiter, he is assigned to blow up a bridge during an attack on the city of Segoria. ______________ 79. John Steinbeck’s The Grapes of Wrath, a novel set in the Great Depression, is about: (a) The anger of God at Man’s disobedience (b) A bank robbery and its after effects (c) Oklahoma farmers driven from home in search of jobs (d) Californian grape growers facing a water crisis

112

YCT

Ans: (c) ‘The Grape of Wrath’ is an American realist novel written by John Steinbeck and published in 1939. The book won the National Book Award and Pulitzer Prize for fiction, and it was cited prominently when Steinbeck was awarded the Nobel Prize in 1962. Set during the Great Depression, the novel focuses on the Joads, a poor family of tenant farmers driven from their Oklahoma home by drought, economic hardship, agricultural industry changes and bank foreclosures forcing tenant farmers out of work._______ 80. The Hairy Ape by Eugene O Neill can be categorized as a/an .........play* (a) Expressionist (b) Symbolist (c) Absurdist __________(d) Realistic __________ Ans: (a) ‘The Hairy Ape’ is a 1992 expressionist playing American playwright Eugene O Neill. It is about a brutisn, unthinking laborer known as yank, the antagonist of the play, as he searches for a sense of belonging in a world controlled by the rich. At first, Yank feels secure as he stokes the engines of an Ocean liner, and is highly confident in his physical power over the ship’s engines and his men. ‘The Hairy Ape’ displays oppression of the industrial working class. 81. Nick Carraway, the narrator of F. Scott Fitzerald’s The Great Gatsby is: (a) the romantic hereo of the novel (b) an outsider who observes the action from a distance (c) the one who brings about the downfall of the protagonist (d) the one who nurtures a destructive passion for the heroine Ans: (b) ‘The Great Gatsby’ is a 1925 novel written by American author F. Scott Fitzgerald that follows a cast of characters living in the fictional town of West Egg on prosperous Long Island in the Summer of 1922. Nick Carraway a Yale graduate originating from the Midwest, a World War I veteran, and, at the start of the plot, a newly arrived resident of West Egg, who is aged 29 was an outfider who observes the action from a distance. ______________________________________ 82. The title of Edward Albee’s Who’s Afraid of Virginia Woolf is a rewriting of a children’s song that begins with: (a) The bear went over the mountain (b) When you wish upon a star (c) Who is afraid of the bad, bad woolf? (d) There’s hole in the bucket

normally taking a little less than three hours to perform, with two 10 minute intermissions. The title is a pun on the song “Who’s Afraid of the Big Bad, Bad Woolf?” From Walt Disney’s Three Little Pigs’ (1933), Substituting the name of the celebrated English author Virgina Woolf. _________________________________ 83. Kurt Vonnegut’s Slaughter House-Five uses a refrain 106 times. What is it? (a) Do not go gentle into the good night (b) Something there is that doesn’t love a wall (c) And miles to go before I sleep (d) So it goes Ans: (d) Vonnegut frequently uses the phrase, “So it ageds”, as a refrain when events of death, dying and mortality occur, as a narrative transition to another subject, as a memento mori, as comic relief, and to explain the unexplained it appears 106 times. The book has been classified as a post modem, metafictional novel. ________________________________________ 84. Which of the following poems is NOT by Emily Dickinson? (a) “Because I stopped for Death” (b) “I taste a liquor never brewed” (c) “Success is counted Sweetest” (d) “Hope is a thing with feathers” Ans: (a) “Because I stopped for Death” is a poem which is not written by Emily Dickinson, further the original title is “Because I could not stop for ‘Death’, which is lyrical poem first published posthumously in poems: series 1 in 1890. The persona of Dickinson’s poem meets personified deth. Emily Elizabeth Dickinson (1830-1886) was an American Poet, bom in Amherst, Massachusetts. _________________________ 85. Sylvia Plath’s novel The Bell Jar is: (a) written in the epistolary form (b) a journal of her final yer in college (c) highly autobiographical (d) a reminiscence of her happy childhood

Ans: (c) ‘The Bell Jar’ is the only novel written by the American writer and poet Sylvia Plath. Originally published under the pseudonym “Victoria Lucas” in 1963, the novel is autobiographical, with the names of places and people changed. The book is often regarded as a roman a clef since the protagonist’s descent into mental illness parallels Plath’s own experiences with what may have been clinical depression. _____________ 86. Langston Hughes, Zora Neale Hurston, Countee Ans: (c) ‘Who’s Afraid of Virginia Woolf? Is a 1962 Cullen, among others, belonged to a Movement play by Edward Albee. It examines the breakdown of popularly known as : the marriage of a middle aged couple, Martha and (a) Progresssive Politics George Late one evening, after a university faculty party, they receive an unwritting younger couple Nick (b) Civil Rights Agitation and Honey, as guests, and draw them into their bitter (c) Black Identity Crisis and Frustrated relationship. The play is in three acts, (d) Harlem Renaissance 113 Rajasthan PSC College Lecturer-II Exam. 2014 YCT

Ans: (d) Langston Hughes, Zora Neale Hurston, Countee Cullen, Nella Larsen all belong to a Movement popularly known as Harlem Renaissance Harlem Renaissance was a cultural, social, and artistic explosion that took place in Harlem, New York, spanning the 1920s. During the time, it was known as the ‘New Negro Movement’, named after the 1925 anthology by Alain Locke. 87. Richard Wright’s novel, Native Son, tells the story of ......... (a) Bigger Thomas (b) Shug Avery (c) Mr Bellmont (d) Jamie Cranford Ans: (a) “Native Son” (1940) is a novel written by African-American author Richard Wright. The novel tells the story of 20 years old Bigger Thomas an African youth living in utter poverty in a poor area on Chicago’s South side in the 1930s. 88. “Womanist is to feminism, as purple is to lavender” is a phrase used by: (a) Toni Morrison (b) Alice Walker (c) Rita Dove (d) Gloria Naylor Ans: (b) “Womanist is to feminism, as purple is to lavender” is a phrase used by Alice Walker. Alice Malsenior Walker (bom February 9, 1944) is an American novelist, short story writer, poet, and activist. She wrote the critically acclaimed novel the color purple (1982) for which she won the National Book Award and the Pulitzer Prize for fiction. 89. The latest Toni Morrison novel is (a) A Mercy (b) God Help the Child (c) Home (d) Jazz

Ans: (a) Maya Angelou’s (bom Marguerite Annie Johnson, April 4, 1928 May 28, 2014) was an American poet, memoirist, and civil rights activist. She published seven autobiographies, three books of essays, several books of poetry of she received dozens of awards and and more than 50 honorary degrees. Angeloue is best known for her series of seven autobiographies, which focus on her childhood early adult experiences. The first, ‘I know why the Caged Birds sings’(1969), tells of her life up to the age of 17 brought her international recognition and acclaim. 92. The Play A Raisin in the Sun is written by: (a) Gwendolyn Brooks (b) Lorraine Hansberry (c) Dorothy West (d) Octavia E. Butler Ans: (b) ‘A Raisin in the Sun’ is a play by Lorraine Hansberry that debuted on Broadway in 1959. The title comes from the poem “Harlem” (also known as ‘A Dream Deferred’) by Langston Hughes the story tells of a black family’s experiences in the Washington Park Subdivision of Chicago’s Woodlawn neighbourhood as they attempt to “better” themselves with an insurance payout following the death of the father. The New York Drama critics circle named it the best play of 1959. 93. The ‘Beat’ poet Le Roi Jones later renamed himself: (a) Claude Mckay (b) Alex Haley (c) Amiri Baraka (d) Nikki Giovanni

Ans: (b) ‘God Help the Child’ is a latest novel by Ans: (c) The ‘Beat’ poet Le Roi Jones later renamed American Writer Toni Morrison. The book is her 11th himself Amiri Baraka. Amiri Baraka (bom Everett Le novel News of the book, as well as the title and opening Roi Jones; 1934 -2014), previously known as Le Roi line were released in December 2014. The novel’s Jones and Imamu Amiri Baraka, was an African-Ameri original title, preferred by Morrison herself, is ‘The can writer of poetry, drama, fiction, says and music criticism. He received the PEN open Book Award, Wrath of Children’. 90. Go Tell it on the Mountain is a semi- previously known as the Beyond Margins Award, in 2008 for ‘Tales of the out and the Gone’. autobiographical novel written by ............ (a) Toni Cade Bambara (b) Amiri Baraka 94. The Autobiography of Malcolmx is a ............ (c) Ralph Ellison (d) James Baldwin (a) recounting of residual effects of slavery Ans: (d) ‘Go Tell it on the Mountain’ is a 1953 semi (b) vision document of the Harlem Renaissance autobiographical novel by James Baldwin. It tells the (c) narrative about the Black Arts Movement story of John Grimes, an intelligent teenager in 1930s (d) spiritual conversion narrative Harlem, and his relationship to his family and his church. The novel also reveals the back stories of John’s Ans: (d) ‘The Autobiography of Malcolm X’ was mother, his biological father, and his violent, religious published in 1965, the result of a collaboration between human rights activist Malcolm X and journalist Alex fantic stop-father, Gabriel Grimes. 91. Maya Angelou’s first in the series of seven Haley Coauthored the autobiography based on a series of in-depth interviews he conducted between 1963 and autobiographies is: Malcolm X’s 1965 assassination. The Autobiography is (a) I Known Why the Caged Bird Sings (b) Singin and Swingn and Getting Merry like a spiritual conversion narrative that outlines Malcolm X’s philosophy of black pride, black nationalism, and Christmas pan-Africanism. After the leader their collaborative (c) The Heart of a Woman process and the events at the end of Malcolm X’s life. (d) All God’s Children Need Travelling Shoes 114 Rajasthan PSC College Lecturer-II Exam. 2014 YCT

95. Thomas and Buelah, a book of poems that won Set in the fictional free Republic of Aburiria, Wizard of the 1987 Pulitzer Prize for poetry, is by: the crows dramatizes with corrosive humor and Keenness of observation a battle for the souls of the (a) Wanda Coleman (b) Lawrence Dunbar Aburirian people, between a megalomaniac dictator (c) AudreLorde (d) Rita Dove and an unemployed young man who embraces the Ans: (d) ‘Thomas and Beulah’ is a book of poems by African American poet Rita Dove that tells the Semi- mantle of a magician. ____________________________ fictionalized chronological story of her grandfather in 100. In his collection of essays, The Pleasures of Exile, George Lamming reinterprets: the first half and her grandmother (named Beulah in the (a) Milton’s Paradise Lost book, although her real name was Georgianna) in the second. It won the 1987 Pulitzer Prize for Poetry. _____ (b) The Babylonian Captivity (c) The Book of Jeremiah 96. Wole Soyinka’s play Death and the King’s Horseman is based on a real incident focusing on (d) Shakespeare’s The Tempest ________________ the Yoruba tradition of: Ans: (d) George Lamming In his 1960 collection of (a) taking a new wife essays ‘The Pleasure of Exile’, is a pioneering work (b) ritual suicide that attempts to define the place of the West Indian in (c) ploughing the land the post-colonial world, re-interpreting Shakespeare’s (d) cremating the dead The Tempest and the characters of prospero and caliban Ans: (b) ‘Death and the King’s Horseman’ is a play by in terms of personal identity and the history of the Wole Soyinka based on a real incident that took place in Caribbean. ____________________________________ Nigeria during British colonial rule the horseman of an 101. Margaret Atwood’s novel The Blind Assassin important chief was prevented from committing ritual uses a novel within a novel technique. This third suicide by the colonial authorities.__________________ tale is called. 97. The title of Chinua Achebe’s novel No Longer at (a) “His Fair Assassin” Ease is taken from: (b) “Blind Assassin” (a) W.B. Yeat’s “Second Coming” (c) “Assassin’s Apprentice” (b) an Igbo proverb (d) “Assassin’s Assassin” (c) a Nigerian naming ceremony Ans:(b) Margaret Atwood’s novel The Blind Assassin (d) T.S. Eliot’s “The Journey of the Magi” uses a ‘novel with a novel’ technique. This third tale is Ans: (d) ‘No Longer at Ease’ is a 1960 novel by called “Blind Assassin”. Nigerian author Chinua Achebe. It is the story of an ‘The Blind Assassin’ was first published by McClelland Igbo man. Obiu Okonkwo, who leaves his village of for and Stewart in 2000. Set in Canada, it is narrated from a British education and a job in the Nigerian colonial the present day, referring to previous events that span civil service, but struggles to adapt to a Western the twentieth century. The work was awarded the Man lifestyle and ends up taking a bribe. The book’s title Booker Prize in 2000 and the Hammett Prize in 2001. It comes from the closing lines if T.S. Eliot’s poem, the was also nominated for Governor General’s Award in Journey of the Magi. ____________________________ 2000, orange prize for fiction, and the international 98. Buchi Emecheta’s autobiography is called: Dublin Literary Award in 2002. Time Magazine named (a) The New Tribe it the best novel of 2000 and included it in its list of the (b) Our Own Freedom 100 greatest English language novels since 1923. ______ (c) Head Above Water 102. Which one of the following novels is written by (d) Second Class Citizen _____________________ Susanna Moodie? (a) The Stone Diaries Ans: (c) Buchi Emecheta’s autobiography is called Head Above Water. Emercheta’s autobiography spans (b) Surfacing the transition from a tribal childhood in the African (c) Roughing it in the Bush bush to life in North London as an as an internationally (d) No Great Mischief acclaimed writer. _______________________________ Ans: (c) ‘Roughing it in the Bush’ is an account of life 99. Ngugi wa Thiongo’s Wizard of Crow (2006), a as a Canadian Settler by Susanna Moodie. Moodie fantasy novel, is set in: immigrated to Upper Canada, near modem day (a) the fantasy “Always Land” Peterborough, Ontario during the 1830s. At the (b) the imaginary “Free Republic of Aburiria” suggestion of her editor, she wrote a “guide” to settle life for British subjects considering coming to Canada. (c) the utopian “Freelgood Country” (d) the Kingdom of “Tahtifantasia” _____________ Roughing it in the Bush was first published in London in 1852. It was Moodie’s most successful literary work. Ans:(b) ‘Wizard of the Crow’ (2006) is an ambitious, The work is part memoir, part novelization of her magisterial, comic novel from the acclaimed Kenyan experiences, and is structured as a chronological series novelist, playwright, poet and critic Naugi wa Thiongo. of sketches. Rajasthan PSC College Lecturer-II Exam. 2014

115

YCT

103. A very characteristic strategy in the stories of Men” is a novel by V.S. Naipaul first published by Andre Deutsch in the UK in 1967. ‘Mystic Masseur’ is Alice Munro is to: a comic novel by V.S. Naipaul. It is set in colonial (a) draw imaginary characters Trinidad and was published in London in 1957. (b) recreate futuristic metropolises ‘A House for Mr. Biswas’ is a 1961 novel by (c) capture old-age perceptions V.S.Naipal, significant as Naipaul’s first work to (d) move forward and backward in time achieve acclaim worldwide. _______________________ Ans: (d) A very characteristic strategy in the stories 107. Derek Walcott’s epic poem “Omeros” loosely stories of Alice Munro is to move forward and echoes and refers to characters from : backward in the time. Aice Munro (1931) is a Canadian (a) Dante’s Divine Comedy short story writer and Nobel Prize winner. Munro’s (b) Homer’s The Iliad work has been described as having revolutionized the (c) Shakespeare’s Othello architecture of short stories, especially in its tendency to (d) Ovid’s Metamorphosis move forward and backward in time. Her stories have been said to embed more than announce, reveal more than parade. ___________________________________ 104. Which novel of Sally Morgan is considered to be a breakthrough memoir about indigeneous life in in Australia? (a) My Place (b) In Your Dreams (c) Cruel Wild Woman and David Milroy (d) Wanamurraganya Ans: (a) Novel ‘My Place’ by Sally Morgan is considered to be a breakthrough memoir about indigeneous life in Australia. ‘My Place’ is an autobiography written in 1987. It is about Morgan’s quest for knowledge of her family’s past and the fact that the she has grown up under false pretences. _______ 105. One of Les Murry’s verse novels is entitled: (a) Fredy Neptune (b) Blacks and Tackles (c) An Absolutely Ordinary Rainbow (d) Flowering Eucalypt in Autumn Ans: (a) ‘Fredy Neptune: A novel in verse’ (1998) is a novel written in verse by the Australian poet Les Murray. Told in eight-line Stanzas, Fredy Neptune describes the experiences of Fred Boetteher, an Australian of German Parentage, dowing the years between the world wars. The work was described by the British Poet Ruth Padel as a haunting, loving, fiercely democratic epic by a master poet. It won the 1998 over Queensland Premier’s Literary Awards fiction book award. ________________ 106. Which one of the following novels is NOT by V.S. Naipaul? (a) Across the Black Waters (b) The Mimic Men (c) Mystic Masseur (d) A House for Mr. Biswas

Ans: (b) “Omeros’ is an epic poem by Caribbean writer Derek Walcott, first published in 1990. The work is divided into seven “books” containing a total of sixty four chapters. The book won the W.H. Smith Literary Award in 1991. Walcott was also awarded the Nobel Prize for Literature. The poem very lossely echoes and refremces Homer and some of his Major characters from The Iliad through the vast majority of the poem, Walcott uses a three line form that is reminiscent of the terza rima form that Dante used for the Divine Comedy. _____________ 108. The novel by Patrick White, Memoirs of Many in One was published under the pen-name ......... (a) Diedrich Knickerbocker (b) Komilworthy Whisp (c) Licsan-Antti ja Jussi Porilainen (d) Alex Xenophon Demirjian Gray Ans: (d) The novel by Partick white, ‘Memoirs of Many in one’ was published under the pen name Alex Xenophon Demirjian Gray. It is a 1986 novel in which white is taken to be editing the papers of a fictional Alex Gray. ____________________________________ 109. The Play The Department by David Williamson is concerned with: (a) an academic department (b) a commercial department (c) a marketing department (d) a sales department

Ans: (a) ‘The Department’ is a 1957 play by David Williamson about political intrigue at a University department. David Keith Williamson, AO, bor 24 February, 1942 is one of Australia’s best-known dramatists and playwrights. He has also written screeplay and teleplays. __________________________ 110. Which of these works is NOT by Arnold Wesker? (a) Chicken Soup with Barley (b) Chicken Soup for the soul Ans: (a) ‘Across the Black Waters’ is an English novel (c) Chips with Everything by the Indian writer Mulk Raj Anand first published in 1939. It is not written by V.S. Naipaul. ‘The Mimic (d) I’m Talking about Jerusalem 116 Rajasthan PSC College Lecturer-II Exam. 2014 YCT

Ans: (b) ‘Chicken Soup for the Soul’ is a publishing. 115. Matangini in Bankim Chandra Chatterji’s novel, Rajmohan’s Wife, can be taken as a Consumer goods and media company based in Coscob, representative of women who: CT. It is known for the ‘Chicken Soup for the (a) willingly submit to their husband’s wishes Soul’series of books. Works by Arnold Wesker are: (b) lead a purely spiritual life ♦ Chicken Soup with Barley’ (1956) (c) believe in the value of selfless service ♦ ‘Chips with Everything’ (1962) (d) remain strong in the face of brutality__________ ♦ I’m Talking about Jerusalem. _____________________ Ans: (d) Matangini in Bankim Chandra Chatterji’s 111. A novel written by Mulk Raj Anand is entitled: novel, Rajmohan’s wife, can be taken as a (a) The Untouchable (b) An Untouchable representative of women who believe in the value of (c) Untouchable (d) Untouchables selfless service ‘Rajmohan’s wife’, it seems is the first Ans: (c) ‘Untouchable’ is a novel by Mui Raj Anand published English novel by an Indian. It has also the distinction of being Bankim Chandra Chatterjee’s debut published in 1935. The novel established Anand as one novel as a writer. _________________________________ of India’s leading English authors. The book was 116. Harindranath Chattopadhyay’s play The inspired by his aunt’s experience when she had a meal Window deals with: with a Muslim women and was treated as on outcast by (a) the need for better communication with people his family. (b) the needs of the intellectuals 112. Which one of the following poets was advised in (c) the exploitation of the textile workers in England by Sir Edmund Gosse to seek Bombay inspiration from India and not from England? (d) the welfare programmes for slum dwellers in (a) SarojiniNaidu (b) Torn Dutt _______Bombay _________________________________ (c) Rabindra Nath Tagore (d) W.B. Yeats Ans: (b) Harindranath Chattopadhyay’s play The Ans: (a) Sarojini Naidu (1879-1949), also known by the Window’ deals with the needs of the intellectuals. Sebriquet Nightingale of India, was a poet and Harindranath Chattopadhyay (2 April, 1898-23 June politician Naidu served as the first governor of the 1990) was a multitalented personality as an Indian an actork, a musician and a United Provinces of Agra and Oudh from 1947 to 1949. English poet, a dramatis, st member of the 1 Lok Sabha from Vijayawada She was advised in England by Sir Constituency. He had been awarded with Padma Edmund Gosse to seek inspiration from India and not Bhushan in 1973. _________________________________ from England. ___________________________________ 117. Sir Aurobindo’s play Perseus the Deliverer is 113. The goddess who protected the people of the based on a: villages in the novel Kanthapura is: (a) Persian Romance (b) Greek Legend (a) Narsamma (b) Kenchamma (c) Punjabi Folktale (d) French Novel (c) Ratna (d) Rangamma Ans: (b) Sir Aurobindo’s play ‘Perseus the Deliverer’, Ans: (b) The goddess who protected the people of the is based, on a Greek Legend Sri villages in the novel Kanthapura (1938) is Kenchamma. Aurobindo has derived the source from Perseus Kenchamma is the mother of Himavathy. She came Andromeda myth in the Greek mythology. But from the Heavens, it was the sage Tripura who had Sri Aurobindo has turned the myth into a real made penances to bring her down-and she wages such a happening. ‘Perseus the Deliverer’ is a romantic The play has been written during battle and she fought so many a night that the blood comedy. Soaked and Soaked into the earth, and that is why the Sri Aurobindo’s stay at Baroda, a life no less busier than at Calcutta ‘Perseus the Deliverer’ is a sketch for Kenchamma Hill is all red Kenchamma is the Goddess supermanhood. The task high-lighted in the play is a for the people of Kanthapura. _______________________ problem of present day life of the world. And the purpose 114. The Poem “Our Casuarina Tree” is authored by: of the playwright is to obtain nobility of existence. _______ (a) SarojiniNaidu (b) Kamala Markandaya 118. “To India-My Native Land” is a poem by: (c) Torn Dutt (d) A.K. Ramanujan (a) Michael Madhusudan Dutt (b) Harindranath Chattopadhyay Ans: (c) ‘Our Casuarina Tree’ is a poem published in 1881 by Torn Dutt, an Indian poet. It is a perfect (c) Sri Aurobindo example of craftsmanship. In this poem Torn Dutt (d) Henry Louis Vivian Derozio celebrates the majesty of the Casuarina Tree and Ans: (d) “To India-My Native Land” is a poem remembers her happy childhood days spent under it and composed by Indian poet Henry Louis Vivian Derozio revives her memories with her beloved siblings it still in 1828. It is one the most notable works by the Indian remains one of the more popular poems in modem poet. The poet was very pained at the fact that India was Indian literature. under British rule and laments that fact in this poem. Rajasthan PSC College Lecturer-II Exam. 2014

117

YCT

119. The first book written by an Indian in English Ans: (c) was: Kamala Das - My Story (a) Travels of Dean Mahomet Nirad Chaudhary - The Autobiography of an unknown (b) Gitanjali Indian. Shobha De Selective Memory (c) Untouchable Nayantara Sehgal Prison and Chocolate Cake. (d) Autumn Leaves Above are the correctly matched pairs ‘My Story’ Ans: (a) The first book written by an Indian in English by Kamala Das is to date the best selling women’s was ‘Travels of Dean Mahomet’, a travel narrative by autobiography in post Independence India ‘My sake Dean Mahomet published in England in 1794. The Story’ is a Chronologically ordered, linear narrative book begins with the praise of Genghis Khan, Timur in a realist style. ‘The Autobiobraphy of an and particularly the first Mughal Emperor Babur and unknown India’ is the 1951 Autobiography of Nirad describes several important cities in India, and describes C. Chaudhary. Written when he was around 50, it a series of military conflicts with local Indian records his life from his birth in 1897 in Kishorganj, a small town in present day Bangladesh. Written in principalities. __________________________________ voice that is consistently confident and candid, 120. “Purdah is a kind of safety” is aline taken from” ‘Selective Memory Stories from My life’ is (a) Saroj ini Naidu’s “Purdah Nashin” remarkable for the honesty with which with which it (b) Imtaiz Dharker’s Purdah (I)” captures the essence of a fascinating woman who (c) Sylvia Plath’s “Purdah” has become a legend in her own time. ‘Prison and (d) Fiona Pearse’s “It’s the Right thing to do” Chocolate Cake’, is a soft memoir of MS. Nayantara Sahegal of her childhood. The book Ans: (b) “Purdah is a kind of safety” is a line taken without much political bragging of Nehru dynasty from Imtiaz Dharker’s Purdah (I)’. takes you to era of pre-independence through the 121. Put the following novels by R.K. Narayan in eyes of a sweet little girl. ______________________ their correct chronological order. 123. Difficult Daughters by Manju Kapoor is a story (A) The Eglish Teacher of three generations of women.The are: (B) The Guide (a) Leelavati, Virmati, Aastha (C) The Man-Eater of Malgudi (b) Harjeet, Virmati, Aastha (D) The Vendor of Sweets (c) Saguna, Karuna, Rea (a) ABCD (b) BACD (d) Kasturi, Virmati, Ida (c) CABD ____________(d) DABC ___________ Ans: (d) Difficult Daughters by Manju Kapoor is a Ans: (a) R.K. Narayan (10 October 1906-13 May 2001) story of three generations of women. They are Kasturi, full name Rasipuram Krishnaswami Iyer Virmati and Ida. ________________________________ Narayanswami, was an Indian writer, he was known for 124. Which prize did Arundhati Roy win for her novel The God of Small Things. his works set in the fictional South Indian town of (a) Nobel Prize (b) Pultizer Prize Malgudi. He was a leading author of early Indian (c) British Book Award (d) Booker Prize ______ literature in English. Chronology of R.K. Narayan Novels is as: Ans: (d) ‘The God of Small Things’ (1997) is the debut novel of Indian writer Arundhati Roy. It is a story about a) The English Teacher (1945, Eyre) the childhood experiences of fraternal twins whose lives b) The Guide (1958, Methuen) are destroyed by the “Love Laws” that lay down “who c) The Man-Eater of Malgudi (1961, Viking) should be loved, And how much”. The book explores d) The Vendor of Sweets (1967. The Boodley Head) how the small things affect people’s behavior and their 122. Pick out the option that correctly matches the lives. It won the Booker Prize in 1997. ______________ writer with the book125. Manohar Malgonkar’s novel A Bend in the Writer Book Ganges is about: A. Nayantara Sehgal p. The Autobiography (a) positive value of the philosophy of non-violence of an Unknown India (b) the violent aftermath of the Partition B. Shobha De (c) the partition as a positive experience Q My Story (d) the effects of the partition of states in India C._Nirda Chaudhary R. Prison and Chocolate Cake Ans: (b) ‘A Bend in the Ganges’ (1964) is a novel by D. Kamala Das S. Selective Memory Indian author Manohar Malgonkar. The novel opens with the civil disobedience movement of the early (a) D C B A P Q R S (b) D C B A S R P Q 1930s and ends with the partition riots in Punjab. It (c) D C B A Q P S R (d) D C B A R S Q P 118 Rajasthan PSC College Lecturer-II Exam. 2014 YCT

encompasses the Swadeshi movement, the activities of the freedom fighters the autbreak of the second World War, the British retreat from Rangoon, the Bombay dock explosion, and the division of India in 1947. _____ 126. In Khushwant Singh’s novel “Train to Pakistan”, Juggat Singh redeems himself by: (a) marrying the girl he loved (b) preventing the train from going to Pakistan (c) refusing to board the train which was going to _______Pakistan ________________________________ Ans: (c) In Khushwant Singh’s novel “Train to Pakistan”, Juggat Singh redeems himself by saving many lives at the end of the story ‘Train to Pakistan’ is a historical novel by Khushwant Singh, published in 1956. It recounts the partition of India in August 1947. 127. The title of one of Kamala Markandaya’s novels is drawn from S.T. Coleridge’s poem “Work Without Hope”. Identify the novel. (a) Nectar in a Sieve (b) A Silence of Desire (c) A Handful of Rice (d) Possession ______________________________ Ans: (a) ‘Nector in a Sieve’ is a 1954 novel by Kamala Markandaya. The novel is set in India during a period of intense urban development and is the chronicle of the marriage between Rukmani, youngest daughter of a village headman, and Nathan, a tenant farmer. The story is told in the first person by Rukmani, beginning from her arranged marriage to Nathan at the age of 12 to his death many years later. The title of the novel is taken from the 1825 poem ‘Work Without Hope’, by Samuel Taylor Coleridge. 128. Rama Mehta’s Inside the Haveli is the story of a Bombay girl who when married into a conservation family in Rajasthan (a) gladly accepts the traditional values (b) decides to go back to her own family in Bombay (c) fights to maintain her modem values

Ans: (b) ‘The Shadow Lines; (1988) is a Sahitya Akademi Award winning novel of Amitav Ghosh. This novel gives us the story of three generations of a family against the background of the growth of India as a nation. It is a book that captures perspective of time and events, of lines that bring people together and hold then apart; lines that are clearly visible from one perspective and none-existent from another; lines that exist in the memory of one, and therefore, in another’s imagination. The novel is set against he backdrop of historical events like the Swadeshi movement, Second World War, Partition of India and communal riots of 1963-64 in Dhaka and Calcutta.____________________ 130. Who among the following Indian poets gives Indian English its special flavour largely through error? (a) Dom Moraes (b) P.Lal (c) Nassim Ezekiel (d) KekiDaruwala Ans: (c) Famous Indian Poet Nissim Ezekiel gave Indian English its special flavour largely through error. Sahitya Akademi Awardi (1983) Nissim Ezekiel (16 December 1924-9 January 2004) was an Indian Jewish poet, actor, playwright, editor and artcritic. He was foundational figure in post colonial India’s literary history, specifically for Indian writing in English. ______________________________________ 131. Pick out the poet who wrote the poems “The Logic”, “Indian Summer Poem”, “The Whorehouse in a Calcutta Street” and “A Missing Person”. (a) Jayant Mahapatra (b) Rukmani Bhaya Nair (c) Gieve Patel (d) A.K. Ramanujan _________________________ Ans: (a) The poet who wrote the poems ‘The Logic’, ‘Indian Summer Poem’ ‘The Whorehouse in a Calcutta Street’ and ‘A Missing Person’, is Jayant Mahapatra (bom 22 October 1928). He is the first ever Indian poet to win Sahitya Akademi Award for English poetry. He is also a winner of Padma Shri. He returned his Padma Shri to protest against the rising intolerance in India. 132. R. Parthasarathy’s poem “Exile” examines the question of the: (a) advantages of writing in one’s own language (b) advantages of living in a new culture (c) loss of identity with one’own culture (d) disadvantages of living in one’s own culture Ans: (c) R Parthasarathy’s poem “Exile” examines the question of the loss of identity with one’s own culture. 133. Arun Kolatkar’s poem “Jejuri” oscillates between: (a) urban and rural life (b) fancy and imagination (c) love and hate (d) faith and scepticism

Ans: (d) Rama Mehta’s ‘Inside the Haveli’ (1977) presents the story of the protagonist Geeta’s identity crisis in the cross-cultural content. Brought up in Bombay, Geeta the independent young woman struggles to maintain her modem identity in a traditional world of the haveli of Udaipur, where she is married. Therefore, the novel projects a journey to find woman space in the society and to search one’s own authentic identity. The novel presents mind-boggling variety of theme as well as well as style. _____________________ 129. Which of Amitav Ghosh’s novels gives us the story of the generation of a family against the background of the growth of India as a nation? (a) Sea of Poppies (b) The Shadow Lines (c) The Glass Palace (d) River of Smoke 119 Rajasthan PSC College Lecturer-II Exam. 2014

YCT

Ans: (d) Arun Kolatlar’s poem “Jejuri” oscillates between faith and skepticism. ‘Jejuri’ was first published in 1976. ‘Jejuri’ describes a day trip to the town of Jejuri, a pilgrimage town in Maharastra. Both devotion and commercialism populate the town, and the man we follow doesn’t search for englightenment, he’s sight seeing. 134. Asif Currimbhoy’s play Goa is about: (a) Racial discrimination of Goa (b) Goa as a tourist destination (c) the Portuguese people living in Goa (d) the traditional practices of the Portuguese people Ans: (a) Asif Currimbhoy’s first play ‘Goa’ written in the 1964 deals with racial discrimination as a paradigm of post colonialism. 135. When Mahesh Dattani’s play Dance like a Man ends, the young couple, Lata and Viswas: (a) are hoping to get married (b) are already married (c) are disappointed that they can’t get married (d) are waiting for the approval of their parents to get married Ans: (b) When Mahesh Dattani’s play ‘Dance like a man ends’, the young couple Lata and Viswas are already married. Mahesh Dattani’s (bom 7 August 1958) play “Dance Like a Man” (1989) opens with a young dancer, Lata, bringing her fiance, Viswas, home to meet her parents. Their hopes and the family histories that emerge from their conversation, are quietly amusing, and the missteps and misunderstandings that trip up everyone when Lata’s parent return to the house are very funny. 136. Girish Ka mad’s Tughalq: (a) is concerned with history, with no contemporary relevance (b) has contemporary relevance, but no connection with history (c) is concerned with history and also has contemporary relevance (d) is neither concerned with history nor has contemporary relevancy

Ans: (d) Ved Parkash Mehta (21 March 1934-9 January 2021) is a writer who was bom in Lahore, British India (now in Pakistan city) to a Hindu family. He lost his sight at the age of four to cerebrospinal meningitis. He was the Author of ‘Fee to Face: An Autobiography’ (1957) ‘Vedi and John is Easy to Please : Encounters with the written and the Spoken Word’ (1971). 138. The book the helped popularize Dalit Literature throughout India in recent times is: (a) Janani Navaratna Manjari- Shri Narayana Gum (b) Poisoned Bread: Translations From Modem Marathi Dalit Literature- Arjun Dangle (c) Bahishkrit Bharat- B.R. Ambedkar (d) Asprashyanthi Kaifiyat - Jyoti Rao Phoole Ans: (b) ‘The Poisoned Bread translations From Marathi Dalit Literature’ by Arjun Dangle is impetuously revives the prevalent slavery in precolonial and colonial times, the continuous pattern peasants, women, scheduled tribes and scheduled caste in post colonial India. Who remained ethically, economically and politically deprived of their rights and privileges. This book helped in popularizing Dalit Literature throughout India in recent times. 139. On the publication of which of her novels was Bama ostracised from her village for portraying it in a poor light? (a) Kusumbukkaran (b) Sangati (c) Karakku (d) Vamnan Ans: (c) On the publication of ‘Karakku’ (1992) was Bama ostracised from her village for portraying it in a poor light. Bama (bom 1958), also known as Bama Faustina Soosairaj, is a Tamil, Dalit Feminist and novelist. She rose to fame with her autobiographical novel ‘Karakku’ (1992), which chronicles the joys and sorrows experienced by Dalit Christian women in Tamil Nadu. She subsequently wrote two more novels, ‘Sangati’ (1994) and ‘Vamnan’ (2002) along with two collections of short stories ‘Kusumbukkaran’ (1996) and ‘Oru Tattvum Erumaiyun’ (2003). 140. Which of the following is the autobiography of Om Prakash Valamiki? (a) Safai Devats (b) Ab Aur Nahin (c) Bas I Bahut Ho Chuka (d) Joothan

Ans:(c) Girish Kamad’s (bom 19 May 1938) ‘Tughalq (1964) is concerned with history and also has contemporary relevance. It is about a rashly idealist Ans: (d) Om Prakash Valmiki (30 June, 1950-17 14th-Century Sultan of Delhi, Muhammad Bin Tughalq, November 2013) was an Indian Dalit writer poet well and allegory on the Nehruvian era which started with known for his autobiography, ‘Joothan’ considered as ambitious idealism and ended up in disillusionment. milestone in Dalit literature in his novel ‘Joothan’ he 137. Who is the author of the books (I) Face to Face: talked about the discrimination they had to face in the An Autobiography, Vedi and ohn is Easy to school at different points. He says during the please: Encounters with the Written and the examinations we could not drink water from the glass Spoken Word? when thirsty. To drink water, we had to cup our hands. The peon would pour water from way high up lest our (a) Ruskin Bond (b) Amit Chaudhari hands touch the glass. (c) Chaman Nahal (d) Ved Mehta 120 Rajasthan PSC College Lecturer-II Exam. 2014 YCT

(c) Tales from Firozsha Bagh 141. Who wrote that “Dalit Literture is not a literature of vengeance Dalit Sahitya is not a (d) Family Matters literature which spreads hatred”? Ans: (c) ‘Tales from Firozsha Bagh’ is a collection of (a) Namdeo Dhasal (b) Baburao Bagul 1 1 short stories by Rohinton Mistry about the residents (c) DayaPawar (d) ArjunKamble of Firozsha Bagh, a Parsi dominated apartment complex Ans: (b) Baburao Baghul (1930-2008) wrote that ‘Dalit in Mumbai. ____________________________________ Literature is not a literature of vengeance Dalit Sahitya 146. Salman Rushdie’s novel The Ground Beneath Her Feet is variation on the myth of: is not a literature which spreads hatred’. (a) Pallas and Centaur Baburao was a Marathi writer from Maharastra India a pioneer of modem literature in Marathi and an (b) Oedipus and Iphigeneia important figure in the Indian short story during the (c) Troilus and Cressida late 20th century. When it experienced a radical (d) Orpheus and Eurydice departure from the past, with the advent of Dalit Ans: (d) ‘The Ground Beneath Her Feet’ is Salman writers such as him. _____________________________ Rushdie’s sixth novel. Published in 1999, it is a 142. “Ms Militancy”is a collection of poetry of : variation on the Orpheus/ Eurydice myth with rock (a) BabyKamble (b) UrmilaPawar music replacing orpheus’s lyre . The myth works as a (c) Meena Kandasamy (d) AshaThorat red thread from which the author sometimes strays, but Ans: (c) “Ms Militancy” (2010) is a collection of poetry to which he attaches an endless series of references. 64 by Meena Kandasamy (1984) is an Indian poet, fiction 147. But that I am writing about the territory, about writer, translator and activist who is based in Chennai, the feelings, of a new kind of pioneers here in America”. This has been said by: Tamil Nadu India. Most of her works are centered on feminism and anti-caste Annihilation movement of the (a) Jhumpa Lahiri (b) Sujata Bhatt contemporary Indian milieu. ______________________ (c) Amitav Ghosh (d) Bharati Mukherjee 143. The Prisons We Broke is the first Ans: (d) The above line had been said by autobiographical narrative by a Dalit woman Bharati Mukherjee (July 27, 1940-January 28, 2017). writer. Who is she? She was an American writer and Professor Emerita in (a) Jyoti Lanjewar (b) Maya Pandit the department of English at the University of (c) Shantabai Kamble (d) KumudPawade California, Berkeley. ____________________________ Ans: (c) ‘The Prisons We Broke’ is the first 148. What was Always Hers is a collection of stories by: autobiographical narrative by a Dalit Woman writer (a) Shuana Singh Baldwin (b) Sujata Bhatt Shantabai Kamble (bom 1 March 1923). She is Marathi writer and Dalit activist. (c) Uma Parameswaran (d) Until Namjoshi “The Prisons We Broke” deals with two major Ans: (c) ‘What was Always Hers’ is a collection of problems of the society firstly, the oppression and stories by Uma Parameswaran. ____________________ exploitation of the Dalit by the upper class secondaly, 149. In An Equal Music Vikram Seth tells the story the discrimination towards women in a patriarchal of Michael and Julia are ...........respectively. society. _______________________________________ (a) singer, dancer 144. “Human beings do not perceive things whole, (b) guitarist, vocalist we not gods but wounded creatures, cracked (c) violinist, pianist lenses, capable of fractured perceptions”. Who (d) orchestrator, solo ballet dancer says this and where? Ans: (c) In ‘An Equal Music’ (1999) Vikram Seth tells (a) V.S. Naipaul -India A Wounded Civilization the story of Michal and Julia who are violinist, pianist (b) Salman Rushdie -Imaginary Homelands respectively. ___________________________________ (c) Homi K. Bhabha- Nation and Narration 150. Which of the following is NOT a novel by Anita Ans : (b) Above lines had been said by Salman Rushdie Rau Badami? in his Imaginary Homelands ‘Imaginary Homelands’ is (a) Tamarind Mem (b) The Hero’s Walks (c) Tell it to the Trees (d) A Cycle of the Moon a collections of essays. 145. Rohinton Mistry’s short story colletion about Ans: (d) ‘A Cycle of the Moon’ is a novel by Uma the residents of a Parsi dominated apartment Parameswaran and not by Ania Rau Badami. complex is entitled: ‘Tamarind Mem’ is Anita’s first novel. ‘The Hero’s (a) Searching for Stevenson Walk’ is the second novel by Anita And ‘Tell it to the Trees’ is also by Anita Rau Badami. (b) The Screm Rajasthan PSC College Lecturer-II Exam. 2014

121

YCT

Uttrakhand Public Service Commission Asst. Prof. (GDC) Exam. 2017

ENGLISH (Solved Paper) 1.

Whan that Aprill with his shoures soote The droghte of March hath perced to the roote And bathed every veyne in switch licour Of which vertu engendered is the flour.” Who wrote these lines? (a) Chaucer (b) Langland (c) Gower (d) None of these Ans. (a) : These lines- Whan that Aprill ......... the flour are written by Geoffrey Chaucer. These lines appear in ’The Canterbury Tales’, it is a collection of 24 stories that runs to over 17000 lines written between-1387 and 1400. ________________________________________ 2. In Anglo-Saxon Christian poems, Christ, Juliana, Elene and The Fates of the Apostles are ascribed to (a) Bede (b) Dunstan (c) Cynewulf (d) Junius Ans. (c) : In Anglo-Saxon Christian poems, Christ, Juliana, Elene and The Fates of the Apostles are ascribed to Cynewulf author of four old English poems preserved in late to century manuscripts. 3. A group of 17th century English poets whose work is notable for ingenious use of intellectual and theological concepts in surprising conceits, strange paradoxes and far fetched imagery is known as (a) The Soldier Poets (b) The Humanists (c) The Metaphysical Poets (d) The Cavalier Poets Ans. (c) : A group of 17th ........ is known as the metaphysical poets. The term metaphysical poets was coined by critic-Samuel Johnson. 4. The first long narrative poem written by Chaucer is (a) The Book of the Duchess (b) The House of Fame (c) The Parliament of Foules (d) Troylus and Criseyde Ans. (a) : The first long narrative poem written by Chaucer is- The Book of the Duchess, this book is also known as The Deth of Blaunche which is the earliest of Chaucer’s major poems. UKPSC Asst. Prof. (GDC) Exam. 2017

[Exam. Date : 29 Oct 2017]

5.

The oldest of the Anglo-Saxon Biblical poems which dates from the beginning of the eighth century is (a) Daniel (b) Exodus (c) Vulgate (d) The Dream of the Road Ans. (b) : The oldest of the Anglo-Saxon Biblical poems which dates from the beginning of the eighth century is- Exodus.

6.

’’Genesis” the longest poem based on Old Testament Story, contains nearly (a) One thousand lines (b) Two thousand lines (c) Four thousand lines (d) Three thousand lines Ans. (d) : ’’Genesis” the longest poem (2936 lines) based on Old Testament Story, contains nearly Three thousand lines. 7. Choose the playwright who was killed in a brawl. (a) Thomas Nash (b) Thomas Watson (c) Christopher Marlowe (d) Edward Moore Ans. (c) : Christopher Marlowe was an English playwright of Elizabethan period. He was the leader of University wits. His notable works areHero and Leander, Tamburlaine the great, Edward second, Dido, Queen of Carthage etc. He was killed in a brawl (1593).

8.

Among the following works which one is not written by Ben Jonson? (a) Volpone or The Fox (b) Epicoene or The Silent Woman (c) Pericles (d) The Alchemist Ans. (c) : Pericles, Prince of Tyre is a Jacobean play written by William Shakespeare. Volpone or the Fox, Epicoene or The Silent Woman, The Alchemist are written by Ben Jonson. Ben Jonson was Contemporary to William Shakespeare.

122

YCT

9.

Frankenstein or The Modern Prometheus written by Mary Shelley is a (a) Picaresque novel (b) Gothic tale _____(c) Tragedy __________(d) Murder Mystery Ans. (b) : Frankenstein or The Modem Prometheus is an 1818 novel written by English author Mary Shelley. This gothic novel or gothic tale that tells the story of victor Frankenstein, a young scientist who creats a hideous sapient creature in an unorthodox scientific experiment. 10. What is the subtitle of Goldsmith’s semiautobiographical poem, The Traveller? (a) A Song to David (b) The Citizen of the World (c) A Prospect of Society _____(d) The Prince ____________________________ Ans. (c) : ’The Traveller’ is a philosophical poem by Oliver Goldsmith. The subtitle of this poem is A Prospect of Society. This poem was published in 1764. In heroic verse of an Augustan style it discusses the cause of happiness and unhappiness in nations. _______ 11. Which of the following poems was completed by Tennyson in seventeen years? (a) The Lotus Eaters (b) In Memoriam _____(c) The Princess _______(d) The Lady of Shalott Ans. (b) : ’In Memoriam’ A.H.H is a poem by the British poet Alfred Lord Tennyson, published in 1850. It is a requiem for the poet's beloved Cambridge friend Arthur Henry Hallam, who died suddenly of a cerebral hemorrhage in Vienna in-1833. This poem was completed by Tennyson in seventeen years. __________ 12. Identify the members of the Cockney School of Poetry. (a) Leight Hunt, John Keats, William Wordsworth (b) John Keats, P.B. Shelley, Lord Byron (c) Leigh Hunt, John Keats, Wilfred Owen _____(d) Leigh Hunt, John Keats __________________ Ans. (d) : 'The Cockney School' refers to a group of poets and essayists writing in England in the second and third decades of the 19th century. The term came in the form of hostile reviews in Blackwood's Magazine in 1817. Its primary target Leigh hunt but john keats and William Hazlitt were also included. 13. In which of Hardy’s novel does the character sorrow appear? (a) A Pair of Blue Eyes (b) Tess of the D'urbervilles (c) Under the Greenwood Tree _____(d) Jude the Obscure _______________________ Ans. (b) : Tess of the D’urbervilles : A Pure Woman Faithfully Presented is a novel by Thomas Hardy This novel was published in-1891. The Character sorrow is Tess’s son with Alec d urbervilles. Sorrow dies in his early infancy, after Tess Christens him herself. She later buries him herself as well and decorates his grave.

UKPSC Asst. Prof. (GDC) Exam. 2017

14.

Which one out of the following is not a dystopian novel? (a) When the Sleeper Wakes (b) Nineteen Eighty Four (c) Riddley Walker (d) Animal Farm Ans. (d) : Animal Farm is an allegorical novella by George Orwell, first published in England on 17th August 1945. This novel is not a dystopian novel. 15. The first secular usage of the term ’’Epiphany” was done by (a) Oscar Wilde (b) Shakespeare _____(c) James Joyce _______(d) Marlowe _________ Ans. (c) : The first secular usage of the term ’’Epiphany" was done by Irish novelist James Joyce. A Portrait of the Artist as a Young Man is the first novel of Irish fames Joyce. 16. ’’The pen is mightier than the sword.” This is an example of (a) Apostrophe (b) Metathesis _____(c) Metonymy ________(d) Archaism ________ Ans. (c) : "The pen is mightier than the sword." This is an example of Metonymy. In metonymy the literal term for one thing is applied to another with which it has become closely associated because of a recurrent relation in common experience. 17. In which of the following texts does the character Madame Sosostris appear? (a) The Waste Land (b) Ulysses (c) Dubliners (d) The Rainbow Ans. (a) : 'The Waste Land' is a poem by T.S. Eloit. This poem was published in-1922. The character Madame Sosostris, famous clairvoyante appear in first canto of 'The Waste Land' - The Burial of the Dead. 'The Waste Land' has five cantosi. The Burial of the dead ii. A Game of Chess iii. The Fire Sermon iv. Death by water v. What the Thunder said. 18. Which one out of the following novels by Virginia Woolf was published posthumously? (a) To The Lighthouse (b) Between the Acts _____(c) Orlando___________(d) The Years _______ Ans. (b) : 'Between the Acts' is the final novel by Virginia woolf. It was published shortly after her death in 1941. At the time of her death Woolf had yet to correct the type script of the novel and a number of aritics consider it to be an unfinished novel. 19. The Grapes of Wrath is written by (a) John Steinbeck (b) F. Scott Fitzgerald (c) Gertrude Stein (d) Nathaniel Hawthorne

123

YCT

Ans. (a) : ’The Grapes of Wrath’ is an American realist novel written by John Steinbeck. This novel was originally published in 1939. The book won the National Book Award and Pulitzer Prize for fiction, and it was cited prominently when Steinbeck was awarded the Nobel Prize in 1962. 20. Who, out of the following literary artists, declared himself "a classicist in literature, an Anglo-Catholic in religion and a royalist in politics”? (a) F.R. Leavis (b) James Joyce (c) William Empson (d) T.S. Eliot Ans. (d) : T.S. Eliot declared himself ”a classicist in literature, an Anglo-Catholic in religion and a royalist in politics”. T.S. Eliot was an American British poet, essayist, playwright, literary artifice and editor. He was awarded Nobel prize in literature in 1948. 21. Two prominent figures of post-structuralism are : (a) Roland Barthes and Edward Said (b) Roland Barthes and Jacques Derrida (c) Jacques Derrida and Ferdinand de Saussure (d) Ferdinand de Saussure and Freud Ans. (b) : Post-structuralism, is a movement in Literary Criticism and philosophy began in France in the late 1960s and the writers associated with this movement are - Roland Barthes and Jacques Derrida. So the option (b) is correct. 22. ’’Implied Reader” is a term introduced by (a) Wolfgang Iser (b) Cleanth Brooks (c) Ferdinand de Saussure (d) Roland Barthes Ans. (a) : "Implied Reader” is a term introduced by Wolfgang Iser. Wolfgang Iser was a German. He is known for his reader-response criticism in literary theory. _______________________________________ 23. Which element, out of the following, is not essential for accurate pronunciation? (a) Stress (b) Fluency (c) Intonation (d) Semantics Ans. (d) : Semantics is the study of the meaning of words and of the combination of words in phrase, sentence, and larger linguistic unit. Semantics is not essential for accurate pronunciation. 24. The existence of conflicting discourses within any field of linguistic activity, such as a national language, a novel, or a specific conversation is called (a) heteroglossia (b) hermeneutics (c) denotation (d) circumlocution UKPSC Asst. Prof. (GDC) Exam. 2017

Ans. (a) : The existence of conflicting discourses within any field of linguistic activity, such as a national language, a novel, or a specific conversation is calledheteroglossia. 25. Which critic out of the following has expressed the theory, that ”the technique of analysis each teacher will, for the most part, have to develop for himself’? (a) Cleanth Brooks (b) T.S. Eliot (c) LA. Richards (d) F.R. Leavis Ans. (d) : F.R. Leavis was a British literary critic of the early mid twentieth century. He has expressed the theory that "the technique of analysis each teacher will, for the most part, have to develop for himself.” 26. Milton’s first work is (a) Paradise Lost (b) On the Morning of Christ’s Nativity (c) Samson Agonistes (d) Defensio Ans. (b) : John Milton was English poet, pamphleteer, and historian considered the most significant author after William Shakespeare. Milton’s first work is-"0n the Morning of Christ’s Nativity" is a nativity ode written by John Milton in 1629 and published in his Poems of Mr. John Milton. His other works- Paradise Lost (1667) Samson Agonists (1671) Defensio Secunda (1654). 27. Poetry was composed in the East Midland dialect by (a) Skelton (b) Stephen Hawes (c) Chaucer (d) Thomas Wyatt Ans. (c) : Geoffrey Chaucer was the outstanding English poet before Shakespeare and ’the first finder of our language’. Poetry was composed in the East Midland dialect by Chaucer. 28. In which century were the two parts of Cervantes’s Don Quixote published? (a) Fourteenth century (b) Early sixteenth century (c) Seventeenth century (d) Early eighteenth century Ans. (c) : ’Don Quixote’ is a novel published in two parts part 1-1605 and part-II-1615 by Spanish writer Miguel de Cervantes in seventeenth century. 29. ”As flies to wanton boys, are we to the Gods; They kill us for their sport” - Who uttered these lines? (a) Gloucester in Kign Lear (b) King Lear in King Lear (c) Othello in Othello (d) Hamlet in Hamlet

124

YCT

Ans. (a) : Earl of Gloucester uttered these lines, As Flies to wanton boys; are we to Gods They kill us for their sport, This famous lines occurs in Act-IV scene I in famous tragedy-King Lear by William Shakespeare. 30. Who wrote these lines and in which poem do these lines appear? ’’Eyes I dare not meet in dreams In death's dream kingdom.” (a) W.B. Yeats in Leda and the Swan (b) T.S. Eliot in The Hollows Man (c) T.S. Eliot in The Waste Land (d) Rudyard Kipling in Recessional Ans. (b) : ’The Hollow Man’ is a poem by T.S. Eliot. The poem was written in 1925. The famous line of this poem is: Eyes I dare not meet in dreams In death’s dream kingdom. 31. ’’Poetry is simply the most delightful and perfect form of utterance that human words can reach.” The above definition has been given by (a) Francis Bacon (b) Edgar Allan Poe (c) LeightHunt (d) Matthew Arnold Ans. (d) : Matthew Arnold was an English poet and cultural critic. He was given the definition of poetry as’’Poetry is simply the most delightful and perfect form of utterance that human word can reach. He has also said about poetry: It is a criticism of life under the Conditions fixed for such a criticism by the laws of poetic truth and poetic beauty.’’ 32. In which year was the Tatler first issued? (a) 1611 (b) 1711 (c) 1609 (d) 1709 Ans. (d) : ’The Tatler’ was a British literary and society journal begun by Richard Stele in 1709 and published for two years. 33. Who is Achitophel in Dryden's Absalom and Achitophel? (a) Earl of Shaftesburg (b) Earl of Monmouth (c) James (d) None of the above Ans. (a) : ’Absalom and Achitophel’ is generally acknowledge as the finest political satire in the English language. It is also described as an allegory regarding contemporary political events and a mock heroic epic. Achitophel is Earl of shaftesburg in John Dryden’s Absalom and Achitophel. 34. A blending or confusion of different kinds of sense-impression, in which one type of sensation is referred to in terms more appropriate to another is called (a) Morpheme (b) Symbolist (c) Synaesthesia (d) Synchronic UKPSC Asst. Prof. (GDC) Exam. 2017

Ans. (c) : Synaesthesia, in literature the term is applied to descriptions of one mode of sensation in terms of another; Color is attributed to sounds, odor to colors, sound to odors, and so on. 35. "A kind of criticism that restricts itself to describing the critics' own subjective response to a literary work, rather than ascribing intrinsic qualities to it" is called (a) Legislative Criticism (b) Impressionistic Criticism (c) Comparative Criticism (d) New Criticism Ans. (b) : "A kind of criticism that restricts itself to describing the critics’ own subjective response to a literary work, rather than ascribing intrinsic qualities to it” is called impressionistic criticism. 36. A kind of literary interpretation that regards literary works as expressions or embodiments of recurrent patterns or of timeless archetypes is known as (a) Formalism (b) Expressionism (c) Myth Critiscism (d) Dadaism Ans. (c) : Myth Critiscism is known as a kind of literary interpretation that regards literary works as expressions or embodiments of recurrent patterns or of timeless archetype. 37. Match the following authors/poets from List 'A' to the titles of their works/poems in List 'B': A B A. Gower (i) Heroides B. Robert (ii) The Romount of the Manning Rose C. Geoffrey (iii) Vox Clamantis Chaucer D. Ovid (iv) Handlyng Synne ABCD ABCD (a) iii iv ii i (b) ii iii i iv (c) ii iv iii i (d) i iii ii iv Ans. (a) : Gower’s poem is-Vox clamantis, Robert Mannings- Hadlyng synne, Geoffrey Chaucer-The Romance of the rose and ovid’s-Heroides. So option (a) is correctly matched. 38. The story of Robinson Gusoe was inspired by the real life adventures of a sailor who spent four years on a deserted island. Name the sailor. (a) Juan Fernandez (b) Borneo (c) Squire Lagoon (d) Alexander Selkirk Ans. (d) : ’Robinson Crusoe’ is a novel by Daniel Defoe first published on 25th April 1719. The book is presented as an autobiography of the title character whose birth name is Robinson Kreutznaer, a castaway who spends 28 years on a remote tropical desert island near the coast of Venezuela and Trinidad. The story has been thought to be based on the life of Alexander Selkirk.

125

YCT

39.

Charles Lamb characterises her manner as ’’innocence resembling boldness*' and William Hazlitt found in her "that forced and practised presence of mind". Identify that character. (a) Vittoria in The White Devil (b) Duchess in The Duchess of Malfi (c) Miranda in The Tempest (d) Isabella in The White Devil Ans. (a) : Vittoria Corombona-appears in ’The White Devil’. The White Devil is a tragedy by English playwright John Webster. She is a Venetian lady, sister of Flamineo, first married to camillo afterwards to Brachiano. Charlse Lamb Characterised her manner as-’innocence resembling boldness’ and William Hazlitt found in her’that forced and practised presence of mind’. 40. Paston Letters is the collection of the descriptions of the three generations of (a) Edward family (b) Norfolk family (c) Georges (d) Olivers Ans. (b) : ’Paston Letters’ is a collection of correspondence between members of the paston family of Narfolk gentry and others connected with them in England between the years 1422 and 1509. Paston Letters is the Collection of the descriptions of the three generation. 41. Which among the following is not a play be William Congreve? (a) The Double Dealer (b) The Mourning Bride (c) The Way of the World (d) All for Love Ans. (d) : ’All for Love’ is a heroic drama by John Dryden. The sub title of the play is- The world Well Lost. The play was first performed in-1677. It is a tragedy in blank verse. 42. The letters of which English poet have been described by T.S. Eliot as "certainly the most notable and most important ever written by any English poet"? (a) P.B. Shelley (b) John Keats _____(c) Lord Byron ________(d) S.T, Coleridge Ans. (b) : Keats’ letters are admired almost as much as his poetry and were described by T.S. Eliot as”Certainly the most notable and most important ever written by any English poet” They provided the best biographical detail available and shed invaluable light on Keats poems. 43. Arrange the following works in chronological order : (i) Almayer’s Folly (ii) The Niger of Narcissus (iii) An Outcast of the Islands (iv) Under Western Eye UKPSC Asst. Prof. (GDC) Exam. 2017

(a) i, ii, iv, iii (b) i, iv, ii, iii (c) i, iii, ii, iv (d) i, iii, iv, ii Ans. (c) : All the given novels are written by Joseph Conrad. The correct chronological order of the novel isAlmayer’s Folly - 1895 An Outcast of the Island 1896. The Niger of Narcissus 1897 Under Western Eye 1911. 44. The Wind Among the Reeds is a collection of lyrics composed by (a) William Wordsworth (b) T.S. Eliot (c) W.B. Yeats (d) P.B. Shelley __________________________ Ans. (c) : ’The Wind Among the Reeds’ is a collection of lyrics composed by William Butler Yeats. This collection was published in-1899. 45. Which of the following is not written by Anita Desai? (a) Fasting, Feasting (b) Games at Twilight (c) The Inheritance of Loss (d) Diamond Dust Ans. (c) : ’The Inheritance of Loss’ is the second novel by Indian author Kiran Desai. It was first published in2006. It won a number of award including the Man Booker Prize for that year. The story centres around the lives of Biju and Sai. Biju is an Indian living in the United State illegally. Sai is a girl living in mountainous kalimpong with her maternal grandfather. 46. 'Peripeteia' is generally explained as (a) a reversal of situation (b) the conclusion (c) the conflict in the play (d) the change from ignorance to knowledge Ans. (a) : ’Peripeteia’ means reversal in fortune from happiness to disaster. Aristotle introduces the concept of peripeteia in his discussion of simple and complex plots in 'Poetics'. 47. Who considered Dryden "the father of English Criticism"? (a) William Hazlitt (b) Charles Lamb (c) T.S. Eliot (d) Samuel Johnson Ans. (d) : Dr. Samuel Johnson was an English writer who made lasting contributions to English literature as a poet, playwright, moralist and literary critic. He considered John Dryden ’The father of English Criticism’. 48. Anatomy of Criticism is a work by (a) Northrop Frye (b) LA. Richards (c) George Saints bury (d) Raymond Williams

126

YCT

Ans. (a) : ’Anatomy of Criticism’ : Fours essays is a book by Canadian literary critic and theorist Northrop Frye. This book was published in-1957. This book presents principles of literary criticism by drawing upon a wide survey of western literature from Homer to joyace. 49. For which book did Ruskin Bond get Sahitya Akademy Award? (a) A Handful of Nuts (b) Our Trees Still Grow in Dehra (c) The Room on the Roof (d) Vagrants in the Valley Ans. (b) : ’Our Trees Still Grow in Dehra’ is a collection of short stories by Ruskin Bond, with each story tracing his life from childhood to adulthood. The writers journey begins from Java and reaches India with cities like Bombey, Delhi and finally cradling in the Himalayas. He was awarded the sahitya Academy Award in-1992 for Our Tress Still Grow in Dehra. 50. In which of the following works of Ruskin Bond Mariani Labadoor is found as a character? (a) When Darkness Falls (b) Strangers in the Night (c) A Flight of Pigeons (d) Rain in the Mountains Ans. (c) : ’A Flight of Pigeons’ is Ruskin BondsClassical novella about the twists of fate, history and the human heart. This novella is set in Shahjahanpur during the revolt of 1857. The novel begins with the death of Ruth Labadoor is father before her in St Mary’s Church, by the Indian rebels who want to eliminate the existence of the Europeans in their town. Mariam Labadoor stealthily escapes to a loyal Hindu friend Ramjimal who provide them with, food and shelter until Javed Khan-a pathan and male lead hopes into action and plans to kill them. 51. ’’Tyger Tyger, burning bright, In the forests of the night” Who wrote these lines? (a) Blake (b) Hardy (c) Shakespeare (d) None of these Ans. (a) : ’The Tyger’ is a poem by the English poet William Blake, published in-1794 as part of his songs of experience collection. The opening line of the poem isTyger, Tyger, burning bright, In the forests of the night; What immortal hand or eye Could frame thy fearful Symmetry. 52. Which one out of the following is not a characteristic feature of Ruskin Bond's short stories? UKPSC Asst. Prof. (GDC) Exam. 2017

(a) They are children stories. (b) His heroes have an openness of mind. (c) They spring up from the Himalayan soil. _____(d) They are totally objective in nature. ________ Ans. (d) : Ruskin Bond is an Indian author of British descent. He is a famous short story writer and novelist Major Characteristics feature of his stories are as following: They are children story. His heros’ have an openness of mind. They spring up from the Himalayan soil. 53. When was Irwin Allan Sealy’s first novel The Trotter Nama published? (a) 1990 (b) 1986 (c) 1988 _____________(d) 1987 ____________ Ans. (c) : ’The Trotter Nama’ is the first novel of Irwin Allan Sealy. He is an Indian Writer. This novel was published in 1988. His novel The Everest Hotel : Calendar was short listed for the 1998 Booker Prize. He was awarded the Padam Shree. 54. Which one out of the following is not found in the fiction of Ruskin Bond? (a) It appeals to eradicate the roots of evil and violence against man and animal. (b) It wants to keep this earth beautiful and healthy. (c) It interrelates liberty and dreams. (d) It intellectualises nature. Ans. (d) : Ruskin Bond is an Indian author of British descent. He is a famous short story writer and novelist. Major Characteristic feature of his fiction are as follows: It appeals to eradicate the roots of evil and voilence against man and animal. It wants to keep this earth beautiful and healthy. It interrelates liberty and dreams. 55. Who out of the following Indian playwrights has started the theatre group play pen? (a) Badal Sircar (b) Mohan Rakesh _____(c) Vijay Tendulkar (d) Mahesh Pattani Ans. (d) : Mahesh Dattani has started the theatre group play pen. Mahesh Dattani is an Indian director, actor, playwright and writers. He wrote such plays as final solution, Dance like a man, Thirty days in September etc. 56. Which one out of the following is the first novel of V.S. Naipaul? (a) A House for Mr. Biswas (b) The Mimic Men (c) In a Free State (d) Mystic Masseur Ans. (d) : Sir Vidiadhar Surajprasad Naipal, was a Trinidad and Tobago bom British writer of works of fiction and non-fiction in English. The Mystic Marseur is the first novel of V.S. Naipal. The Mystic Marseur is a comic play. It is set in Colonial Trinidad and was published in London in-1957.

127

YCT

57.

Which of the following novels of R.K. Narayan has Nataraj as narrator? (a) The Financial Expert (b) The Bachelor of Arts (c) The Man-eater of Malgudi (d) The Guide Ans. (c) : ’The Man-eater of Malgudi’ is an Indian novel written by R.K. Narayan. This novel was published in1961. In this novel R.K. Narayan uses the myth of Bhasmasura. Nataraj is a printing press owner. He is the narrator of the novel. 58. Which of the following novels of Raja Rao has the description of the freedom struggle of the 1930’s? (a) The Cat and Shakespeare (b) The Serpent and the Rope (c) Kanthapura (d) None of the above Ans. (c) : Raja Rao’s first and best-known novel Kanthapura (1938) is the story of south Indian village named Kanthapura. The novel is narrated in the form of a Sthala Purana by an old woman of the village, Achakka. This novel is the description of the freedom struggle of the 1930’s. 59. Which one out of the following is not written by Jim Corbett? (a) Man-Eaters of Kumaon (b) The Jungle Book (c) Jungle Lore (d) My India Ans. (b) : The Jungle Book is a collection of short stories by English author Rudyard Kipling. The stories were first published in magazines in 1893-1894. 60. Which one out of the following is not written by Nissim Ezekiel ? (a) Night of the Scorpion (b) A Time to Change (c) The Professor (d) Ways of Belonging Ans. (d) : Nissim Ezekiel was an Indian Jewish poet and art critic. He was a foundational figure in post colonial India’s literary history specifically for Indian poetry in English. Night of the scorpion, A time to change, the professor is written by Nissim Ezekiel. Ways of Belonging is not written by Nissim Ezekiel. 61. Which of the following is not a language activity? (a) Listening (b) Learning (c) Reading (d) Writing Ans. (b) : Learning is not a language activity. 62. Who is the father of American lexicography ? (a) Noah Webster (b) Merriam Webster (c) Charles Random (d) Wifred Funk UKPSC Asst. Prof. (GDC) Exam. 2017

Ans. (a) : Noah Webster is the father of American lexicography. 63. Who is the lord of the flies in Golding’s novel Lord of the Flies? (a) The Pig (b) Satan (c) Beelzebub (d) Belial Ans. (c) : ’Lord of the Flies’ is a 1954 novel by Nobel Prize winning British author William Golding. The book focuses on a group of British boys stranded on an uninhabited island and their disastrous attempt to govern themselves. Beelzebub is the lord of the flies in this novel. 64. Who wrote The Buddha of Suburbia (1990) and Intimacy (1998)? (a) Caryl Phillips (b) Buchi Emcheta (c) A.S. Byatt (d) Hanif Kureishi Ans. (d) : Hanif Kureishi, is a British playwright and novelist of Pakistani and English descent. He wrote The Buddha of Subrbia (1990) and Intimacy (1998). The Times included kureishi in its list of the 50 greatest British writer since 1945. 65. Which poet out of the following wrote the lines: ’’Was it a vision, or a waking dream? Feed is that music : do I wake or sleep?” (a) P.B. Shelley (b) Robert Browning (c) William Wordsworth (d) John Keats Ans. (d) : John Keats was the famous poet of romantic age-The following linesIn the next valley - glades was it a vision, or a walking dream? Fled in the music : do I woke or sleep Appears in-Ode to a Nightingale. 66. ’The Touchstone method’ of literary criticism has been propounded by (a) T.S. Eliot (b) Mathew Arnold (c) A.C. Bradley (d) Homi Bhabha Ans. (b) : Touchstone Method is a term coined by Matthew Arnold, the famous Victorian poet and critic. He introduced the term in his essay ’Study of Poetry’ to denote short but distinctive passage, selected from the writing of great poets which he used to determine the excellence of passage or poems which are compared to them. 67. In which play of Shakespeare is there a tribute to Marlowe as the ’dead shepherd’? (a) Twelfth Night (b) Measure for Measure (c) As You Like It (d) The Tempest

128

YCT

Ans. (c) : ’As You Like It’ is a pastoral comedy by William Shakespeare believed to have been written in 1599 and first published in the first folio in-1623. In this play of Shakespeare there is a tribute to Marlowe as the ’dead shepherd'. ________________________________ 68. Who wrote the novel Waverley? (a) Maria Edgeworth (b) Mrs. Radcliff (c) Walter Scott (d) Lawrence Sterne Ans. (c) : Waverely or ’Tis Sixty Years Since’ is a historical novel by Sir Walter Scott published anonymously in 1814 as Scott's first venture into prose fiction, it is often regarded as one of the first historical novels in the western tradition. 69. Which chapter of Coleridge's Biographia Literaria contains his famous distinction between Fancy and Imagination? (a) Chapter I (b) Chapter X (c) Chapter XIII _______(d) Chapter XV Ans. (c) : Biographia Literaria is an autobiography in discourse by Samuel Taylor Coleridge, which he published in 1817, in two volumes of twenty-three chapters. Chapter XIII contains his famous distinction between Fancy and Imagination. 70. Charles Dickens was the first editor of a newspaper which started in 1846. Which was it? (a) The Morning Chronicle (b) Daily News (c) Morning Post (d) Daily Chronicle Ans. (b) : The Daily News was a national daily newspaper in the united kingdom. The News Paper was founded in 1846 by Charlse Dickens. He was the first editor of this news paper. _________________________ 71. Which American critic applied the term ’New Criticism’ in 1910 to a range of critical methods that were developing in Europe? (a) Cleanth Brooks (b) T.S. Eliot _____(c) William Empson (d) Joel Spingam _____ Ans. (d) : Joel Elias Spingam was an American educator, literary critic, and civil rights activists. He applied the term ’’New Criticism” in 1910 to a range of critical methods that were developing in Europe. The New Criticism : A lecture delivered at Columbia University, March 9, 1910 (1911). 72. Who wrote All About H. Hatterr (1948)? (a) M.G. Vassanji (b) G.V. Desani (c) Shauna Singh Baldwin (d) Rohinton Mistry Ans. (b) : All About H. Hatter (1948) is a novel by G.V. Desani chronicling the adventures of an AngloMalay man in search of wisdom and enlightment. "As for back as in 1951 Desani later wrote, ”1 said H. Hatter was a portrait of a man, the Common vulgar species found everywhere, both in the east and in the west". UKPSC Asst. Prof. (GDC) Exam. 2017

73.

’’Its characters, instead of being taken from sacred narratives, or legends of the saints, were personified abstractions.” Which dramatic form does this statement refer to? (a) Miracle Play (b) Morality Play (c) Interlude (d) Romantic Comedy Ans. (b) : Morality Play were dramatized allegories of a representative Christian life in the plat form of a quest for salvation, in which the crucial events are temptations, sinning, and the climatic confrontation with death. 74. Which of the following plays has not been written by Christopher Marlowe? (a) Dr. Faustus (b) The Spanish Tragedy (c) Edward II (d) The Jew of Malta Ans. (b) : The Spanish Tragedy or Hieronimo is Mad Again is an Elizabethan tragedy written by Thomas kyd between 1582 and 1592. The Spanish Tragedy established a new genre in English theatre, the revenge play or revenge Tragedy. The play was first performed in 1587. ______________________________________ 75. Which of the following novels has not been written by Thomas Hardy? (a) Desperate Remedies (b) Under the Greenwood Tree (c) Vanity Fair (d) A Pair of Blue Eyes Ans. (c) : Vanity Fair is an English novel by William Makepeace Thackeray, which follows the lives of Beckysharp and Amelia Sedley amid their friends and families during and after the Napoleonic wars. The novel was originally published in 1847-48. 76. Who made the following comment on the first publication of Leaves of Grass - ’’The most extraordinary piece of wit and wisdom that America has yet contributed”? (a) Robert Frost (b) R.W. Emerson (c) H.D. Thoreau (d) Ernest Hemingway Ans. (b) : The statement- "The most extraordinary piece of wit and wisdom that America has yet contributed" appeared in letter to Whitman from Ralph Waldo Emerson on 21st July 1855. "Dear sir, I am not blind to the worth of the wonderful gift of "Leaves of Grass" I find the most .......... contributed. 77. Which play has been approved by Sir Philip Sidney in his Defence of Poesie? (a) Gorboduc (b) The Spanish Tragedy (c) Hamlet (d) Tamburlaine

129

YCT

Ans. (a) : Sidney began writing poetry in 1578, and his writing career only lasted 7-8 years. His works are- The Defence of Poesie and An Apologie for Poetrie. The play Gorboduc has been approved by Sir Philip Sidney. ”Our tragedies and comedies not without cause cried out against, observing rules neither of honest civility nor of skilful poetry, excepting Gorboduc, - again I say of............ 78. Whose lines are these? ’’What we call the beginning is often the end, And to make an end is to make a beginning.” (a) W.B. Yeats (b) T.S. Eliot (c) D.H. Lawrence (d) William Empson Ans. (b) : The lines-”What we call the beginning is often the end, And to make an end is to make a beginning.’ appear in ’Little Gidding’ by T.S. Eliot. Little Gidding is the fourth and final poem of T.S. Eliot’s Four Quartets, a series of poems that discuss time, perspective, humanity and salvation. 79. ’’For art’s sake alone I would not face the toil of writing a single sentence.” Who made the above statement? (a) John Gabworthy (b) G.B. Shaw (c) Stephen Phillips (d) Oscar Wilde Ans. (b) : George Bernard Shaw was an English dramatist who rejected the Conventional romantic theme of his age and founded the theatre of idea. Shaw used drama as a vehicle of Idea. He said drama is ’nothing but an explanation’. In preface to Man and Superman he notes-”For art’s sake alone I would not face the toil of writing a single sentence. 80. Which of the following is the first novel by Tobias Smollett? (a) Roderick Random (b) Peregrine Pickle (c) Ferdinand Count Fathom (d) Humphry Clinker Ans. (a) : The Adventure of Roderick Random is a picaresque novel by Tobias Smollett. The novel was first published in-1748. It is partially based on Smollett’s experience as a naval surgeon’s mate in the Royal navy especially during the Battle of Cartagena de Indias in-1741. 81. Who out of the following does not belong to ’’Scottish Chaucerians”? (a) Gavin Douglas (b) Cynewulf (c) William Dunbar (d) Robert Henryson Ans. (b) : Chaucer’s influence on 15th century Scottish literature began towards the beginning of the century with James I of Scotland. This first phrase of Scottish ’’Chaucerianism” was followed by a second phase, comprising the works of Robert Henryson, William Dunbar and Gavin Douglas. UKPSC Asst. Prof. (GDC) Exam. 2017

82.

Meaning of Meanings is a book by I.A. Richards and (a) C.K. Ogden (b) J.C. Ransom (c) William Empson (d) F.R. Leavis Ans. (a) : ’The Meaning of Meaning : A study of the Influence of Language upon Thought and of the Science of Symbolism’ (1923) is a book by C.K. Ogden and LA. Richards. It is accompanied by two supplementary essays by Bronislaw Malinowski and F.G. Crook Shank. 83. Who launched the ’’Criterion” in 1922? (a) T.S. Eliot (b) F.R. Leavis (c) Northrope Frye (d) J.C. Ransom Ans. (a) : The Criterion was a British literary magazine published from October 1922 to January 1939. The Criterion was, for most of its run, a quarterly journal, although for a period in 1927-1928. It was published monthly. It was created by the poet, dramatist, and literary critic. T.S. Eliot who served as its editor for its entire run. 84. Which of the following is the first book printed in English by Caxton? (a) Beowulf (b) Morte D. Arthur (c) The Canterbury Tales (d) The Recuyell of the Histories of Troye Ans. (d) : William Caxton was an English merchant, diplomat and writer. He is thought to be the first person to introduce a printing press in England in-1476. The first book printed in English by Caxton is-The Recuyell of the Histories of Troye. 85. Which of the following theories establishes the critic not as a mere interpreter of the text but as a co-creator of the text? (a) Marxist Criticism (b) Structuralism (c) Deconstruction (d) Post-colonial criticism Ans. (c) : Deconstruction theory establishes the critic not as a mere interpreter of the text but as a Co-creator of the text. Deconstruction, as applied in the criticism of literature designate a theory and practice of reading that question and claims to ’subvert’ and undermine the assumption that the system of language provides ground that are adequate to establish the boundaries and the determinate meaning of a literary text. 86. Match the following writers with their works : A B A. Nayantara Sahgal (i) In Custody B. Anita Desai (ii) Mistaken Identity C. Upamanyu (iii) Pleasure City Chatterjee D. Kamala (iv) The Last Burden Markandaya

130

YCT

ABCD ABCD (a) i ii iv iii (b) ii iii i iv (c) iv iii ii i (d) ii i iv iii Ans, (d) : _____________________________________ Nayantara Sahgal - Mistaken Identity (Novel) (1988) Anita Desai - In custody - (Novel) 1984 Upamanyu Chatterjee - The Last Burden (Novel) 1993 Kamala Markandaya - Pleasure city (Fiction) (1982) 87. Match the Playwrights with their Plays : A B A. Pratap Sharma (i) The Miracle Seed B. Asif Currimbhoy (ii) A Touch of Brightness (iii) The Fire and The C. Mahasweta Devi Rain D. Girish Kamad (iv) Mother of 1084 ABCD ABCD (b) ii i iv iii (a) i iii ii iv (d) ii iii iv i (c) iii iv ii i Ans. (b) : The correctly matched the playwright with their plays are as followsPratap Sharma - A Touch of Brightness. Asif Currimbhoy- The Miracle Seed Mahasweta Devi - Mother of 1084 Girish Kamad - The Fire and The Rain. 88. Match the following novels of E.M. Forster with their year of publication: A B A. A Passage to India (i) 1907 B. The Longest Journey (ii) 1910 C. Howard’s End (iii) 1905 D. Where Angels Fear to (iv) 1924 Tread D A B C D ABC ii iv iii (a) iv i ii iii (b) i (c) ii iii iv i (d) None of the above Ans. (a) : The correctly matched novels of E.M. Forster with their years of publication are as follows. A Passage to India - 1924 The Longest Journey - 1907 Howard’s End - 1910 Where Angels Fear to Tread - 1905 89. Match the following critics with their works : A B A. Derrida (i) Of Grammatology B. Homi Bhabha (ii) The Wretched of the Earth (iii) Outside in the C. Frantz Fanon Teaching Machine D. Gayatri Spivak (iv) Nation and Narration UKPSC Asst. Prof. (GDC) Exam. 2017

ABCD (a) iii iv ii i (b) (c) i iv ii iii (d) Ans. (c) : Option (c) is correctly with their works. Derrida Homi Bhabha Frantz Fanon Gayatri Spivak

ABCD iv iii ii i ii iii iv i matched the critics

-

Of Grammatology (1967) Nation and Narration (1990) The Wretched of the Earth (1961) Outside in the Teaching Machine (1993) 90. Match the following novels with the characters found in them : A B A. Nina Sibal’s Yatra (i) Sarala B. Deshpande’s Small Remedies (ii) Mini C. Suma Josson’s (iii) Madhu Circumferences D. Sunny Singh’s Nani's Book of (iv) Krishna Suicide ABCD ABCD (a) iv iii ii i (b) iv iii i ii (c) iii iv ii i (d) ii i iii iv Ans. (b) : The correctly matched novels with the characters found in them are as followsNina Sibal’s Yatra - Krishna Deshpande’s Small Remedies - Madhu Suma Josson’s Circumferences - Sarala Sunny Singh’s Nani’s Book of - Mini Suicide 91. It has been said that for the success or failure of a teaching programme, three factors are mainly responsible. Which one out of the following is not included in them? (a) Classroom (b) Men (c) Material (d) Methods Ans. (a) : For the success or failure of a teaching programme, three factors are mainly responsible. They are-Men, Material and Method. But classroom is not included in them. 92. Who wrote The Chess master and his Moves (1988)? (a) Salman Rushdie (b) Anita Desai (c) Rohinton Mistry (d) Raja Rao Ans. (d) : ’The Chess master and his Moves' is a most ambitions novel, and like most of Raja Rao’s writing, rooted in Indian tradition, thought and sensibility. The novel was published in-1988. The Chessmaster is the story of an impossible love between Shivarama Sastri, an Indian mathematician working in Paris, and a married women which can only end in sorrow and despair.

131

YCT

Direction : Question No. 93 to 95 have sentences which are divided and marked (a) to (d). The part containing the error has to be marked as your answer. 9$ If I would have been there (a) (b) I certainly would have taken care of the problem (C) ' (d) Ans. (a) : Conditional sentences are used to speculate about what could happen, what might have happened, and what we wish would happen. In English, most sentences using the conditional contain the word ’if. ’If + past perfect’+ perfect conditional or perfect continuous conditional (OR) | If + had.......... would have....... | Correct sentenceIf had been there, I certainly would have taken care of the problem. 9

The principal of the college, together with some (a)

'

(b)

other teachers, are (c) planning to roganize an on - the - spot painting competition (d)

Ans. (c) : When two nouns are joined by-as well as along with, together with some one etc. verb is used in singular number. According to first subject. Correct sentenceThe principal of the college, together with some other, teachers is planning to organize an on-the-spot painting competition. 93

As there are only one taxi available (a) I have no other alternative but to pay the fare (b)

(c)

demanded by the driver (d) Ans. (a) : In the given sentence, part (a) is erroneous. Singular verb ’is’ will be used because noun- ’One taxi’ is singular number. So, the correct sentence will beAs there is only one taxi available, I have no. other alternative but to pay the fare demanded by the driver. 96. ’’What but design of darkness of appall? If design govern in a thing so small.” In which poem do these lines occur? (a) ’’Design” (b) ’’Mending Wall” (c) ’’Time to Change” (d) None of the above UKPSC Asst. Prof. (GDC) Exam. 2017

Ans. (a) : These lines-”What but design of darkness of appal ........ so small” have been taken from the poem ’Design’ by Robert Lee Frost. It is a fourteen-line sonnet that is separated into the stanza. Frost published the definitive version of ’Design’ in-1936 in-A Further Range. 97. "Creeds and schools in abeyance Retiring back a while sufficed at what they are, but never forgotten." In which poem do these lines occur ? (a) "Dover Beach” (b) The Waste Land (c) Song of Myself (d) None of these Ans. (c) : The lines-”Creeds and schools in abeyance ........ never forgotten” have been taken from the poem’Song of Myself. Song of Myself is a poem by Walt Whitman that is included in his work-Leaves of Grass. It has been credited as-”representing the core of Whiteman’s poetic version." 98. "At the start of the winter came the permanent rain and with the rain cause the cholera. But it was checked and in the end only seven thousand died of it in the army." In which novel do these lines occur ? (a) A Farewell to Arms (b) Torrents of Spring (c) For Whom the Bell Tolls (d) None of these Ans. (a) : The lines-’ At the start of ........ the army’ have been taken from the novel-A Farewell to Arms. This novel was written by Ernest Hemingway. These lines are spoken by-Lieutenant Frederic Henry. Rain is a recurring symbol of death in Farwell to Arms. 99. Which of the following is not written by Tennessee Williams? (a) Fugitive Kind (b) A Streetcar Named Desire (c) The Refugees (d) The Rose Tattoo Ans. (c) : 'The Refugees’ is a short story collection by Viet Thanh Nguyen. It is Nguyen's first published short story collection in-2017 and his first book after winning the Pulitzer Prize for the Sympathizer. 100. Dickinson's poetry is not characterised by (a) extensive use of dashes (b) idiosyncratic vocabulary (c) unconventional capitalisation (d) robust nationalism Ans. (d) : Emily Dickinson wrote about what she knew and about what intrigued her. A keen observer she uses extensive use of dashes, unconventional capitalisation, idiosyncratic vocabulary etc. She used images from nature, religion, laws music, medicine, fashion and domestic activities to probe Universal theme. Her poetry is not characterised by robust nationalism.

132

YCT

MP Public Service Commission

Asst. Prof. Exam. 2017

ENGLISH (Solved Paper) 1.

What does the term Mythos mean in Aristotle’s poetics? (a) Plot (b) Action _____(c) Serious ___________(d) Character ________ Ans. (a) : The plot (which Aristotle termed the ’mythos’) in a dramatic or narrative work is constituted by its events and actions, as these are rendered and ordered toward achieving particular artistic and emotional effect. _______________________________ 2. In which Shakespearean play, ’’Oedipus Complex” is reflected? (a) Othello (b) King Lear _____(c) Hamlet ___________(d) Macbeth _________ Ans. (c) : The Oedipus complex is a term used by Sigmund Freud in his theory of psychosexual stages of development to describe a boy’s feeling of desire for his mother and jealously and anger toward his father. In Shakespeare’s famous tragedy ’’Hamlet” this term is reflected.______________________________________ 3. What best describes Bunyan’s Pilgrim’s Progress? (a) a travelogue (b) an allegory (c) a picaresque novel _____(d) a long sermon _________________________ Ans. (b) : Pilgrim’s Progress is an allegory by John Bunyan that was first published in - 1678. The Pilgrim Progress represent the selfish bend of puritan theology. The main character Christian, lives in the city of Destruction, with his wife and children, symbolizing his own state of domination. _________________________ 4. Who wrote ’The Way of The World’? (a) John Donne (b) John Dryden (c) John Bunyan (d) William Congreve Ans. (d) : ’The Way of the World’ is a play written by the English Playwright William Congreve. It was first performed in early March 1700, in the theatre in Lincoln’s inn Fields in London. It is widely regarded as one of the best Restoration comedy and is still occasionally performed. __________________________ 5. ’Heroic couplet’ consists of(a) lambic Tetrameter (b) lambic Hexameter (c) lambic Pentameter (d) None of these MPPSC Asst. Prof. Exam. 2017

[Exam. Date : 26 March 2017]

Ans. (c) : ’Heroic couplet’ consists of lambic Pentameter so that in each line an unstressed syllable is followed by a stressed syllable which rhyme in pairs: aa, bb, cc and so on. 6. In which novel, character ’’Lalitha” and ’’Saroja” are found? (a) The Nowhere Man (b) Two Virgins (c) Open Seasons (d) Confession of a Lover Ans. (b) : ’Two Virgins’ is eight novel of Kamala Markandaya, published in-1974. The story set in a village in India, is timeless and universal one, of growing up, and of love, and conflict between parents and children, and of the lure of the big city. ’Lalitha and Saroja’ are the character in this novel. 7. The Neo-classical age in English literature is also called as(a) The Jacobean age (b) The Restoration age (c) The Puritan age (d) The Augustan age Ans. (d) : The Neo-Classical period (1660-1785) is known as the Augustan age in the history of English literature. 8. Gothic Novels deal with(a) Adventure (b) Comedy (c) Humour _____(d) Horror and Suspense ____________________ Ans. (d) : The Gothic novel was the pre-romantic novel of terror, the supernatural, great sensation of horror wonder, witchcrafts etc. The setting used to be in the medieval period often in gloomy ruined buildings or abandoned castles. ______________________________ 9. Who is the poet of this famous line-”Did he who made the lamb make thee”(a) William Blake (b) William Wordsworth (c) John Keats _____(d) Leigh Hunt ___________________________ Ans. (a) : The famous line ’’Did he who made the lamb make thee” appear in the poem- "The Tyger” a poem by William Blake published in 1794 as part of his ’Songs of experience’ collection.

133

YCT

10.

To which poet does the following phrase apply- Ans. (b) : The representation of the people act 1832 ”Willing suspension of disbelief*? was an act of parliament of the United Kingdom that (a) Wordsworth introduced major changes to the electoral system of (b) Coleridge England and Wales. This act marked the beginning of (c) Keats The Victorian Era. (d) Shelley 16. In Tn Memoriam’ Tennyson mourns the death Ans. (b) : ’Willing suspension of disbelief' is a term ofcoined by S.T. Coleridge in-1817 who suggested that if (a) John Keats a writer infuse a human interest and a semblance of (b) Hugh Clough truth into a fantastic tale, the reader would suspend (c) Lord Byron judgment concerning the implausibility of the narrative. (d) Arthur Hallam 11. Who is the protagonist of Rama Mehta’s novel. Ans. (d) : 'In Memorian' A.H.H is Tennyson's poem. It Inside the Haveli? takes 17 year to complete. It is completed in 1894. It is (a) Sudha (b) Rani about Arthur Henry Hallam who dies suddenly of a (c) Geeta (d) Kamla cerebral hemorrhage in Vienna in 1833. Ans. (c) : 'Inside the Haveli' is an English language 17. In which of Hardy’s Novels does the scene of novel written by Rama Mehta. For this novel Mehta was wife’s auction take place? conferred Sahitya Akadami Award in 1979. The story (a) TessoftheD’Urbervillies of the novel revolved around a young girl Geeta from (b) Far from the Madding Crowd mumbai, India. She marries a son of a farmer Indian (c) The Mayor of Casterbridge prince and post marriage she relocates to Udipur Rajasthan. (d) The Return of the Native 12. Adonais is a pastoral elegy written on the death Ans. (c) : The Mayor of casterbridge (1886) is Hardy's ofnovel. Its sub title is-The Life and Death of a man of (a) Byron (b) Keats character. It is set in the fictional town of casterbridge. (c) Scott (d) Coleridge In this novel Michael Henchard auctions his wife Susan. So the scene of wife's auction takes place in the present Ans. (b) : Adonais is Shelley's pastoral elegy on the novel. death of Keats. It is written in 1821. It has 55 Spenserian stanza of 9 line each. It has 495 lines. 18. "My Last Duchess" is a/an13. Which poem of P.B. Shelley contains this line (a) Ode "Our sweetest songs are those that tell of (b) Ballad saddest thought"? (c) Mock-epic (a) "Ode to the West Wind" (d) Dramatic Monologue (b) "Song to the Men of England" Ans. (d) : "My Last Duchess" is a poem by Robert (c) "Ozymandias" Brwoning frequently anthologised as an example of the (d) "To the Skylark" _______________________ dramatic monologue. It first appear in 1842 in Ans. (d) : "To the Skylark" is Shelley's poem. It is Browning's dramatic lyric. The poem is written in 28 written in the spring of 1820. It represents Shelley's rhyming couplets of iambic pentameter. thought and style. This poem contains the famous line"Our sweetest songs are those that tell of saddest 19. Who was the leader of the Pre-Raphaelite group of artists in England? thought".______________________________________ (a) Swinburne 14. Essays of Elia is written by(b) Morris (a) De Quincey (c) D.G. Rossetti (b) Charles Lamb (d) Christina Rossetti (c) William Hazlitt _____(d) Samuel Rogers ________________________ Ans. (c) : Pre-Raphaelite group of artists in England Ans. (b) : Essay of Elia is Charles Lamb Collection of was a group of young British painters who banded essay. It is published in 1823. Lamb's essay, first appear together in 1848 in reaction against what they conceived to be the unimaginative and artificial historical painting in the London Magazine. of the Royal academy. It was farmed in 1848 by three 15. The "Great Reform Act" which marked the beginning of ’The Victorian Era’ was passed by Royal academy students-Dante Gabriel, Rossetti, who was a gifted poet as well as a painter, William Holman British Parliament in Hunt and John Everett Millais, all under 25 years of (a) 1830 (b) 1832 age. D.G. Rossetti was the leader of this group. (c) 1837 (d) 1835 134 YCT MPPSC Asst. Prof. Exam. 2017

20.

Who is the author of the popular tragic play Riders To The Sea? (a) J.M Synge (b) Lady Gregory (c) J.M Barrie (d) Sean O’ Casey Ans. (a) : Riders to the sea, one act play by John Millington synge published in 1903 and produced in 1904. Riders to the sea is set in the Aran Islands of the west coast of Ireland and is based on a tale synge heart there._________________________________________ 21. In how many parts is The Waste Land divided? (a) Two (b) Three _____(c) Four _____________(d) Five ____________ Ans. (d) : ’The Waste Land’ is a long poem by the American author T.S. Eliot. This poem is of 433/434 lines in free verse. It is published in 1922. It has five sections namely - The Burial of dead, A game of chess, Fire seranon, Death by water and what the thunder said. 22. Which narrative technique did James Joyce use in his writing? (a) Surrealist Technique (b) Episodic Technique (c) Stream of Consciousness Technique _____(d) Imaginative Technique __________________ Ans. (c) : James Joyce used stream of consciousness Technique in his writing. Stream of consciousness is a literary technique which was pioneered by Dorothy Richardson, Virginia Woolf, and James Joyce. Stream of consciousness is characterized by a flow of thoughts and image, which may not always appear to have a coherent structure or cohesion. 23. In which novel of Virginia Woolf, a character changes sex? (a) Orlando (b) Mrs. Dalloway (c) To the Light house _____(d) Jacob's Room _________________________ Ans. (a) : ’’Orlando" is a novel by Virginia Woolf that was published in 1928. Orlando is first introduced as a 16 year old by living in the 16th century during the reign of Queen Elizabeth I. The Queen falls in love with Orlando and he is thrower into 1 lavish, life surrounded by women who fallen over him. Since Orlando lines for 400 years he mysteriously transform into a women. 24. What is the central theme of Bernard Shaw's Mrs. Warren's Profession? (a) Social Service (b) Prostitution (c) Education (d) Adultery Ans. (b) : Mrs. Warren's Profession is a play written by George Bernard Shaw in 1893 and first performed in London in 1902. The play is about a farmer prostitute, now a madam, who attends to come to terms with her disappearing daughter. It is a problem play. MPPSC Asst. Prof. Exam. 2017

25.

Aldous Huxley's Brave New World is(a) A novel of adventure (b) A prophetic novel (c) A picaresque novel (d) A psychological novel Ans. (b) : 'Brave New world' a dystopian novel published in 1932, is perhaps Aldous Hyxley's most famous and enduring work, consistently ranked among the top 100 English language novels by entities such as modem library, B.B.C. and the observer. This is a prophetic novel. 26. Who were the founders of Bloomsbury Group, a literacy group of England? (a) Virgina Woolf, E.M. Forster and Lytton Strachey (b) Christina Rossetti, Virgina Woolf and Swinburne (c) Edith Sitwell, W.B. Yeats and Virgina Woolf (d) Katherine Mansfield, Graham Greene and Virginia Woolf Ans. (a) : The Bloomsbury group was a group of associated English writers, intellectuals, philosopher and artists in the first half of the 20th century including Virginia Woolf, E. M. Forster and Lytton Strachey. 27. Who was believed to be a "Classicist in Literature, Royalist in politics and Anglo Catholic in religion"? (a) Samuel Butler (b) Aldous Huxley (c) Matthew Arnold (d) T.S. Eliot _____________________________ Ans. (d) : T.S. Eliot was an American bom poet, dramatist and literary critic whose works- 'The Waste Land' and ’Four Quartets’ are considered defining achievements of 20th century modernist poetry. Eliot pronounced himself a classicist in literature, Royalist in politics and Anglo Catholic in religion. 28. Which of the following critics preferred Shakespeare's comedies to his tragedies? (a) Dr. Johnson (b) Addison (c) Dryden (d) Pope Ans. (a) : Samuel Johnson was and English writer who made lasting contribution to English literature as a poet, Playwright, essayist, moralist, literary critic, biography editor and lexicographer. He preferred Shakespeare's comedies to his tragedies. 29. Who wrote the novel Lord of the Flies? (a) Graham Greene (b) George Orwell (c) William Somerset Maugham (d) William Golding 135 YCT

Ans. (d) : ’Lort of the Flies’ 1954 is a novel from the pen of William Golding. It is an adventurous story which has become a kind of allegory. It is termed as the loss of innocence fiction. 30. Kate Millet’s book Sexual Politics is associated with (a) Orientalism (b) Feminist Criticism (c) Post-colonial Criticism (d) New Historicism Ans. (b) : ’Sexual Politics’ is a 1970 book by Kate Millet based on her P.H.D dissertation. It is regarded as a classic of feminism and one of radical feminism’s key text. 31. Principles of Literary Criticism is written by(a) Kenneth Burke (b) LA Richards (c) R.S. Crane _____(d) Northrop Frye _________________________ Ans. (b) : ’Principles of Literary Criticism’ is written by LA. Richards originally published in 1924. This book developed a closely reasoned theory of the mind’s response to rhythm and meter. His theory is organic and contextual; the sound effects of prosody have little psychologic effect by themselves. 32. Ars Poetica is the most important critical work of— (a) Ovid (b) Virgil _____(c) Horace ___________(d) Longinus ________ Ans. (c) : ’Ars Poetica' or 'The art of Poetry' is a poem written by Horace in 19 B.C, in which he advises poets on the art of writing poetry and drama. The Ars Poetica has exercised a great influence in later ages on European literature. _____________________________ 33. What is the meaning of the term ’Hamartia’ as used by Aristotle in his ’Theory of Tragedy’? (a) Tragic end of the tragedy (b) Role of fate against the hero (c) A strong quality in the character of the hero (d) The fatal flaw in the character of the hero Ans. (d) : The term Hamarita means the act of the hero which initiates the fatal process or the fatal flaw in the character of the hero, suggests a basis for a more developed theory of tragedy. ______________________ 34. Who is the author of the novel A Train to Pakistan? (a) R.K. Narayan (b) Mulk Raj Anand (c) Khushwant Singh (d) Shashi Tharoor Ans. (c) : 'Train to Pakistan’ is a 1956 historical novel by Indian novelist Khushwant Singh set during the 1947 partition of India that create the nations of Pakistan and India. MPPSC Asst. Prof. Exam. 2017

35.

Who is the author of A House for Mr. Biswas? (a) Nirad C. Chaudhuri (b) V.S. Naipaul (c) Amitav Ghosh (d) R.K. Narayan Ans. (b) : 'A House for Mr. Biswas’ is a 1961 novel by V.S. Naipaul. A House far Mr. Biswas follows the life of Mr. Mohan Biswas, a protagonist inspired by Naipaul's father, as the struggle to find his freedom and a house of his own. 36. Who is the author of the novel Disgrace? (a) ChinuaAchebe (b) Ngugi wa Thiong'o (c) Charles Manjua (d) J.M. Coetzee Ans. (d) : 'Disgrace' is a novel by J.M. Coetzee, published in 1999. It won the booker prize in literature. The writer was also awarded the nobel prize in literature four years after its publication. 37. Who wrote these lines: "Into that heaven of freedom, my father, Let My Country awake"? (a) Sarojini Naidu (b) R.K. Ramanujan (c) Rabindranath Tagore (d) Mahadevi Verma Ans. (c) : These lines - "Into that haven of freedom, my father, Let any country awake" are written by Rabindranath Tagore in his poem- My Heaven. This poem appear in Gitanjali poem no. 35. ______________ 38. Cupid in Greek Mythology is the God of: (a) Wealth (b) War (c) Love (d) Hate Ans. (c) : Cupid in Greek mythology is the God of Love. According to myth, Cupid was the son of Mercury, the winged messenger of Gods. 39. The novels of Ashok K. Banker are the retelling of (a) The Ramayana and The Mahabharata (b) The Mahabharata and The Shivapurana (c) The Shivpurana and The Ramayana (d) The Mahabhagavad Purana Ans. (a) : Ashok Kumar Banker is an author and screen writer. He became widely known for his retelling of Indian mythological epics. 40. Who propounded the "The theory of Dhavani" in his treatise Dhvanyaloka? (a) Bhamah (b) Bharata (c) Anandavardhana (d) Kuntaka Ans. (c) : Anandavardhana was the author of Dhvanyaloka, or, a light on suggestion (dhvani) a work articulating the philosophy of aesthetic suggestion.

136

YCT

41.

Vibhava, Anubhava and Vyabhicharibhavas are the constituents of (a) Vakrokti (b) Rasa (c) Alamkara (d) Riti _________________________________ Ans. (b) : Vibhava, Anubhava and Vyabhicharibhavas are the constituents of - Rasa. 42. What is meant by inflectional language? (a) Change in language from time to time (b) Change in the sound of a word (c) Change in the linguistics of the word (d) Change in the form of a word as grammatical function Ans. (d) : Change in the form of a word as grammatical function is nearest by inflectional language. 43. What is target language? (a) A language which is acquired first (b) A language which is learnt after mother tongue (c) A language pertaining to meaning (d) The language into which text is translated Ans. (d) : The language into which text is translated is called target language. 44. A theory of grammar which was proposed by the American linguist Noam Chomsky in 1957 is called(a) Modem Generative Grammar (b) Traditional Generative Grammar (c) Transformational Generative Grammar (d) Linguistic Generative Grammar Ans. (c) : Transformational Generative Grammar, a theory of Grammar was proposed by the American linguist Noam Chomsky in 1957. 45. What is intonation? (a) An ancient usage of language (b) Nasal letters used in a language (c) Word borrowed from another dialect (d) The way the voice rises and falls Ans. (d) : The way the voice rises and falls is intonation. Intonation, in phonetics, the melodic pattern of an utterance. Intonation is primarily a matter of variation in the pitch level of the voice, but in such languages as English stress and rhythm are also involved. 46. The literary term used to indicate the particular type of writing is called: (a) Metaphor (b) Symbolism (c) Genre (d) Cliche _______________________________ Ans. (c) : The literary term used to indicate the partition type of writing is called-Genre. MPPSC Asst. Prof. Exam. 2017

47.

What is Personification? (a) a comparison between two objects (b) contradictory statement of a character (c) Giving human attributes to inanimate objects (d) When a human being fails to understand himself Ans. (c) : Personification, in Greek term prosopopeia, in which either an inanimate object or an abstract concept is spoken of as though it were endowed with life or with human attributes or feeling. 48. The first eight lines of a sonnet are known as(a) Sestet (b) Quartet (c) Octave (d) Tercet Ans. (c) : Sonnet is a lyric poem consisting of a single stanza of fourteen iambic pentameter lines linked by an intricate rhyme scheme. The Italian sonnet falls into two main part an octave : eight lines a sestet : six lines _______________________________ 49. Who is the exponent of the view the poetry is ’’Criticism of Life”? (a) R.L. Stevenson (b) W.H. Auden (c) John Ruskin _____(d) Matthew Arnold _______________________ Ans. (d) : Mathew Arnold's importance in the history of English literary criticism is acknowledged by one and all. His greatness lies in the fact that he had a define aim in writing poetry. He clearly stated this aim and tied to conform to this aim. It was' a criticism of life. ________ 50. In the Prologue to the Canterbury Tales the pilgrimage was undertaken in which month? (a) June (b) April (c) February (d) December Ans. (b) : In the Prologue to the Canterbury Tales the pilgrimages was undertaken in April month. The poem's opening lineWhen that April with his shower soote The droghte of march hath perced to the roote. 51. "Frailty, thy name is woman!" is a famous from the playta) King Lear (b) Hamlet (c) Macbeth (d) As You Like It ________________________ Ans. (b) : Hamlet, the hero of Shakespeare's Masterpiece Hamlet, utters this famous phrase "Frailty, thy name is women" in Act I Scene II. In fact he is, recalling the beautiful memories of his mother and deceased father. 52. Name the autobiography of Kamala Das: (a) My Experiments with Truth (b) Love and Death (c) My Story (d) Summer in Calcutta

137

YCT

Ans. (c) : ’My story’ is an autobiographical book written by Indian author and poet Kamala Das. The book was originally published in Malayalam, titled-Ente Katha. This book was originally published in 1973. 53. What did the Pre-Raphaelites aim at? (a) Showing interest in medievalism (b) Pictorial realism with symbolic overtones (c) Union of the flesh and the spirit (d) All of these Ans. (d) : Pre-Raphaelite brotherhood, group of young British painters who banded together in 1848 in reaction against what they conceived to be the unimaginative and artificial historical painting of the Royal Academy and who purportedly sought to express a new moral seriousness an sincerity in their work This aim was showing interest in medievalism, Pictorial realism with symbolic overtone, union of the flesh and the spirit etc. 54. In which of his essays, T.S. Eliot introduced the term "objective correlative"? (a) Hamlet and his Problems (b) Tradition and the Individual Talent (c) The Perfect Critic (d) Religion and Literature Ans. (a) : In his famous essay ’’Hamlet and his problems” T.S. Eliot introduced the term objectivecorrelative. Eliot wrote this essay in 1919. 55. Who is the person that appears throughout The Waste Land? (a) Marie Larisch (b) Tiresias (c) Madame Sosostris (d) Thomas Becket Ans. (b) : ’The waste land’ a long poem by the American bom poet T. S. Eliot. The Waste Land was first published in 1922 in criterion a magazine edited by T.S. Eliot then a few days later in the magazine ’The Dial’, ’Tiresias’, the central figure, according to Eliot is the central figure in the waste land, an interested spectator of the modem waste land and what Tiresias see in the substance of the whole poem. 56. Which of the following plays begins with the lines: "If music be the food of love, play on"? (a) The Tempest (b) Twelfth Night (c) As You Like It (d) Romeo and Juliet Ans. (b) : William Shakespeare’s ’Twelfth Night’ or ’What You Will’ is a comedy about the cross dressing, ship wreck surviving, poetry living girl who finds herself at the center of a non so average love triangle. The opening line of Twelfth Night, in which a moping Orsino, attended by his servant and musician says - ”If music be the food of love, play on”. MPPSC Asst. Prof. Exam. 2017

57.

The Twice-Born Fiction A Critical Study of Indian Fiction, is written by (a) M.K. Naik (b) Meenakshi Mukherji (c) K.R.S Iyengar (d) K.A. Abbas ___________________________ Ans. (b) : The Twice-Born Fiction A critical study of Indian fiction is written by- Meenakshi Mukherji. 58. Kurtz, a powerful figure appears in the novel: (a) Heart of Darkness (b) The Secret Agent (c) Nostromo - A Tale of Seaboard _____(d) The Nigger of the Narcissus ______________ Ans. (a) : Heart of Darkness, novella by Joseph Conrad that was first published in 1899. ’Heart of Darkness’ examines the horrors of western colonialism, depicting it as a phenomenon that tarnishes not only the lands and people it exploits but also those in the west who advance it. Kurtz, a colonial agent who is supposedly unmatched in his ability to procure ivory from the continent’s interior a powerful figure appears in the novel. The essay "Can the Subaltern Speak?" was written by (a) Krishna Rayan (b) A.K. Ramanujan (c) AniaLoomba (d) Gayatri Chakravorty Spivak Ans. (d) : The essay ’’Can the Subaltern Speak” was written by Gayatri Chakravorty Spivak. Gayatri Chakravorty Spivak is an Indian scholar literary theorist and feminist critic. Sea of Poppies has the theme of: (a) Linguistic conflict (b) Historical Strife (c) Migration and displacement (d) Domestic Problems _____________________ Ans. (c) : ”Sea of Poppies” is a novel by Indian novelist Amitav Ghosh which was short listed for the Man Booker Prize in 2008. Sea of Poppies is a story of indentured laborers Theme of this novel is-migration and displacement. 61. In which year the theatres were closed in England? (a) 1642 (b) 1650 (c) 1660 _____________(d) 1665 ____________ Ans. (a) : The theatre were closed in England in 1642. In this period England was ruled by Parliament under the puritan leader Oliver Cromwell. Drama almost disappeared for eighteen years after the puritan closed the public theatre in September 1642. 62. Which of the following works belongs to Thomas Carlyle? (a) The American Scholar (b) Sartor Resartus (c) The Shepherd’s Calendar (d) Plato and Platonism

138

YCT

Ans. (b) : ’Sartor Resartus’ is an 1836 novel by Thomas Carlyle, first published as a serial in Fraser’s Magazine in November 1833 - August 1834. It is a comic novel. 63. Who is the most notable figure in the Oxford Movement originated in 1833? (a) Keble (b) Newman _____(c) Freud ____________(d) All of these ______ Ans. (b) : John Henry Newman was the most notable figure in the oxford movement originated in 1833, who was an English theologian and poet. He became one of the more notable leader of the oxford movement. 64. Pantheism is best associated with which of the following Poets? (a) Wordsworth (b) Browning (c) Scott (d) George Herbert Ans. (a) : Pantheism refers the belief that God exists in all natural things is best associated with William Wordsworth who is called the worshipper of nature. 65. In Chaucer’s The Canterbury Tales which tale deals with two young Theban Warriors? (a) The Knight’s Tale (b) The Squire’s Tale (c) The Miller’s Tale (d) The Friar’s Tale Ans. (a) : The Canterbury Tales is a collection of 24 stories written in middle English poet by Geoffrey Chaucer. The Knights Tale deals with two young Theban warriors Polomon and Arcite. 66. Name the period of Literature in which Wycherley, Congreve, and Dryden developed comedy of manners. (a) Jacobean Period (b) Neo-Classical Period (c) Romantic Period (d) Restoration Period Ans. (d) : The restoration period of English literature in which wycherley, Congreve, and Dryden develop the comedy of manners. 67. Wordsworth published Lyrical Ballads in collaboration with (a) Lord Byron (b) S.T. Coleridge _____(c) John Keats ________(d) P.B. Shelley ______ Ans. (b) : Lyrical Ballads, collection of poems first published in 1798 by Samuel Taylor Coleridge and William Wordsworth, the appearance of which is often designated by scholars as a signal of the beginning of English Romanticism. 68. Who among the following poets does not belong to the Victorian Period? (a) A. Tennyson (b) R. Browning (c) Thomas Gray (d) Matthew Arnold MPPSC Asst. Prof. Exam. 2017

Ans. (c) : Thomas Grey does not belong to the Victorian period. His ’An Elegy Written in a Country Churchyard’ is one of the best known of English lyric poem. Although his literary output was slight, he was the dominant poetic figure in the mid 18th century and precursor of the Romantic movement. 69. Who wrote "It is strange but true, for truth is always strange, stranger than fiction."? (a) John Keats (b) P.B. Shelley (c) William Blake (d) Lord Byron Ans. (d) : In English literature ’Don Juan’ by Lord Byron is a satirical, epic poem that portrays Don Juan not as a womaniser but as a man easily seduced by women. Stanza 101 of Don Juan begins. It is strange but true; for truth is always strange; stranger than fiction.” 70. Essay on Criticism in verse was written by (a) William Wordsworth (b) S.T. Coleridge (c) Alexander Pope (d) Matthew Arnold Ans. (c) : ”An Essay on criticism” is one of the first major poems written by the English writer Alexander Pope, published in 1711. 71. Coleridge deals with the allegory of Sin, Punishment and Redemption in his poem (a) Christabel (b) KublaKhan (c) The Rime of Ancient Mariner (d) The Frost at Midnight Ans. (c) : Coleridge deals with the allegory of Sin, Punishment and Redemption in his poem-The Rime of Ancient Mariner. This is the longest major poem by the English poet Samuel Taylor Coleridge written in 17971798 and published in 1798 in the first edition of Lyrical Ballads. 72. Which play of Shakespeare has a character named Banquo? (a) King Lear (b) Romeo and Juliet (c) Hamlet (d) Macbeth Ans. (d) : ’Macbeth’ a famous tragedy by William Shakespeare it was written in 1605/06 and published in 1623 by Hemming and condell. Banquo was the ’Thane of Lochaber’ is a character in play Macbeth. 73. The Advancement of Learning is written by (a) Roger Bacon (b) Francis Bacon (c) Thomas Hobbes (d) Thomas More 139 YCT

Ans. (b) : ’The Advancement of learning’ is written by Francis Bacon. He was a 16th and 17th century specific/Prolific writer. He is known as father of English Essay. 74. Which one of the following poems was not written by Wordsworth? (a) Daffodils (b) The Solitary Reaper (c) Tintem Abbey (d) Love’s Philosophy Ans. (d) : ’Love’s Philosophy' poem was not written by William Wordsworth. It was written by P.B. Shelley published in 1819 by Leigh Hunt. 75. Expressionism was a revolt against (a) Naturalism (b) Dadaism (c) Futurism (d) Surrealism Ans. (a) : ’Expressionism’ was a revolt against Naturalism. Naturalism is the belief, refers to a realistic approach to art that rejects idealized experiments. 76. Who propounded the mimetic theory of art? (a) Plato (b) Aristotle (c) Horace (d) Socrates Ans. (a) : Plato propounded the ’Mimetic theory’ of art. The ’Mimetic theory’ of desire is an explanation of human behaviour and culture which originated with the French historian and ploy myth Rene Girard. 77. The Mystery plays were based on (a) Stories from the Bible (b) Stories from the Vedas (c) Stories from the Lives of the Saints (d) Stories from the Greek writers Ans. (a) : The mystery plays were based on ’stories from the Bible’. Mystery and Miracles plays are among the earliest formally developed plays in medieval Europe. It focused on the representation of Biblian stories. 78. Which one of the following works of Wordsworth is known for his theory of poetic diction? (a) Prelude Book V (b) Prelude Book IV (c) Ode on the Intimation of Immorality (d) Preface to Lyrical Ballads 1798 Ans. (d) : ’’Preface to lyrical ballad 1798”, work by Wordsworth is known for his theory for poetic diction. Poetic diction is the term used to refer to linguistic style, the vocabulary, and the metaphors used in the writing of poetry. MPPSC Asst. Prof. Exam. 2017

79.

"I am a part of that I have met/Yet all experience is an arch where this/Gleams that untraveled world whose margin fades/For ever and for ever when I move” These lines occur in Tennyson’s following poem(a) Crossing the Bar (b) Ulysses (c) Elegy _____(d) Lotus Eaters ___________________________ Ans. (b) : ’Ulysses’ a poem by Tennyson in Blank verse, written in 1833 and published in 1842 in his second volume of poetry. In which we find the line. I am a part of that I have met/yet all experience is an arch where this ____ 80. Duchess of Malfi is a tragic play by (a) Christopher Marlowe (b) John Webster (c) Bertolt Bracht (d) W.H. Auden __________________________ Ans. (b) : ’Duchess of malfi’ is a tragic play by John Webster. He was an English Jacobean dramatist best known for tragedies-The white Devil and The Duchess of Malfi. 81. Who among the following has not written on the life of Rama? (a) Kalhan (b) Kamban (c) Valmiki (d) Tulsidas Ans. (a) : ’Kalhan’ has not written in the life of Rama. Kalhan was the Kashmiri author of ’Rajatamgini’ on account of the history of kashmir. 82. What is the Biblical Source for the theme of Creation and the Fall of Man? (a) The Book of Luke (b) The Book of Exodus (c) The Book of Genesis _____(d) The Book of Chronicles _________________ Ans. (c) : ’The Book of the Genesis’ is the Biblical source for the theme of ’Creation and the Fall of Man’. The book of the Genesis, the first book of the Hebrew bible and the Christian old Testament. 83. Which one of the following is a Pastoral elegy written by Milton? (a) Lycidas (b) Adonais (c) Elegy Written in A Country Church Yard (d) Elegy to the Memory of an Unfortunate Lady Ans. (a) : ’Lycidas’ is a pastoral elegy on the death of 'Edward King’ written by John Milton in 1637. It first appeared in 1638 collection of elegies. 84. Who is one of the ’University Wits’? (a) Shakespeare (b) Wyatt (c) Herbert (d) Marlowe

140

YCT

Ans. (d) : Marlowe (Christopher Marlowe) was the university wits. The university wits in a phrase used to name a group of late 16th century English Playwrights, and pamphleteers’ who were educated at the university of Oxford and Cambridge. ________________________ 85. Who is the author of Epistle to Dr. Arbuthnot? (a) Pope (b) Chaucer (c) Dryden (d) Spenser Ans. (a) : ’Alexander Pope’ is author of the poem ’Epistle’ to Dr. Arbuthnot. It was written in 1734 and published in January 1735. Addressed to Pope’s friend John Arbuthnot the epistle is an apology in which Pope defends his works against the attacks of his detractors. 86. Who is at once the child of the Renaissance and the Reformation? (a) Milton (b) Sidney (c) Kyd (d) Nashe Ans. (a) : ’John Milton’ is known as the child of Renaissance and Reformation. He was bom in 1608 and died in 1674. He has written many things in English literature as-on the morning of Christ Nativity, On His Blindness, Lycidas and Comas, etc._________________ 87. The term ’diction* signifies (a) The Choice and arrangement of words (b) The Choice and arrangement of the Figures of Speech (c) The Choice of intonation (d) The Choice of Meters Ans. (a) : The term 'diction’ signifies. The choice and arrangement of words. Style of speaking or writing as dependent upon choice of word. 88. Eric Blair is the original name of which of the following writers? (a) James Joyce (b) George Orwell _____(c) Henry James _______(d) D.H. Lawrence Ans. (b) : ’Eric Blair’ is the original name of George Orwell. He was an English novelist, essayist, Journalist and critic; His work is characterised by lucid prose, biting social criticism. 89. "All animals are equal but some animals are more equal than others". This quotation appears in which novel? (a) Nineteen Eighty Four (b) Animal Farm (c) Of Human Bondage (d) The Heart of the Matter Ans. (b) : ’’All animals are equal but some animals are more equal than others”. This quotation appears in ’Animal Farm’. It is an allegorical novel published in 1945. The book tells the story of a group of farm animals, who rebel against their human farmer. 90. Hemingway’s code hero Santiago appears in (a) The Old Man and the Sea (b) For Whom the Bell Tolls (c) The Sun Also Rises (d) Farewell to Arms MPPSC Asst. Prof. Exam. 2017

Ans. (a) : Ernest Hemingway’s code hero Santiago appears in ’The Old Man and the Sea’. He won the Nobel Prize in Literature in 1954. The Old Man and the Sea is a short novel, written by American Author. 91. What does the term ’Prosody’ signify? (a) The Study of showing the relation between words (b) Dictionary marking (c) Sense drawn (d) The systematic study of versification Ans. (d) : The term ’Prosody Signify’ the systematic study of verification. Prosody conveys attitude, for example, in credulity, disdain, sadness or anger....... 92. Who is the author of Utopia? (a) Pindar (b) Thomson (c) Thomas More (d) Robert Boyle Ans. (c) : ’Thomas More’ is the author of the novel ’utopia’ it was published in 1516. ’’Utopia” is derived from the Greek prefix hence the name literally means ’nowhere’. 93. Authorized (King James) Version of the Bible was published in (a) 1609 (b) 1608 (c) 1611 _____________(d) 1616 ____________ Ans. (c) : Authorized (King James) Version of the Bible was published in 1611. 94. Which one of the following is a famous work of Longinus? (a) On the Sublime (b) Hyperion (c) Apology for Poetry (d) On Beauty Ans. (a) : ’On the Sublime’ Greek Peri hypsous’ on literary criticism by Longinus dating to about the 1st century. The Earliest surviving manuscript, from the 10th century, was printed in 1554. 95. To which school do John Donne, George Herbert and Andrew Marvell belong? (a) Romantic (b) Victorian (c) Metaphysical (d) Renaissance Ans. (c) : The term metaphysical poet was coined by the critic Samuel Johnson to describe a loose group of 17th century English poets as-John Donne, George Herbert and Andrew Marvel. 96. The Spectator was a joint enterprise of these two authors (a) Addison and Sterne (b) Steele and Addison (c) Steels and Sterne (d) Defoe and Steels Ans. (b) : ’The Spectator’ was a daily publication founded by Joseph Addison and Richard Steel. In England from 1711 to 1712. The Spectator was a joint enterprise of these two authors.

? i

YCT

97.

’When You Are Old’ was written by (a) Keats (b) Eliot (c) Browning (d) W.B. Yeats Ans. (d) : ’When you are old’ is a poem written by W.B. Yeats. This poem is directly addressed to the speakers lover. 98. Who is the Protagonist in Raja Rao’s Kanthapura? (a) Bhakha (b) Raka (c) Moorthy (d) Kanchamma Ans. (c) : Raja Rao's first and best known novel Kanthapura (1938) is the story of a south Indian village named Kanthapura. The novel is narrated in the form of a Sthala Purna by an old woman of the village Achakka. Moorthy is the protagonist of the novel. 99. The term ’Ecocriticism’ was first used by (a) H.D. Theoreau (b) William Rueckert (c) Lawrance Coupe (d) Grey Grarrard Ans. (b) : The term Eco-criticism was first used by William Rueckert. Since the publication of William Rueckert Literature and ecology : An experiment in Ecocriticism in 1978, the field of ecocriticism has boomed within the field of literature studies establishing itself as an integral part of the environmental humanities. 100. ’Dissociation of Sensibility’ was introduced into literary criticism by T.S. Eliot in his essay on (a) Milton (b) Metaphysical Poets (c) Poetry and Poets (d) Hamlet Ans. (b) : 'Dissociation of sensibility is a literary term first used by T.S. Eliot in his essay-The metaphysical poets. It refers to the way in which intellectual thought was separated for the experience of feeling in seventeenth century poetry. _______________________ 101. Which one of the following novels, deals with the story of Indian partition? (a) Riots (b) Azadi (c) Kanthapura _____(d) The Serpent and the Rope ________________ Ans. (b) : 'Azadi' is a novel about India's freedom movement and the partition of India by-Chaman Nahal. The novel was published in 1975. Chaman Nahal commonly known as C Nahal also known as Chaman Nahal Azadi. 102. Which British Playwright wrote a play based on two minor characters in Hamlet called Rosencrantz and Guildenstern? (a) Ted Hughes (b) Tom Stoppard (c) Edward Bond (d) David Hare Ans. (b) : Tom Stoppard wrote a play based on two minor character in Hamlet called Rosencrantz Guildenstern. "Rosencrantz and Guildenstern are dead" is an absurdist, existential tragicomedy by Tom Stoppard. _____________________________________ MPPSC Asst. Prof. Exam. 2017

103. What is the title of Wordsworth’s autobiographical poem relating to his childhood? (a) Ode: Intimation Immortality (b) Tintem Abbey (c) The Prelude (d) Ode on Duty Ans. (a) : Intimation of Immortality is the title of Wordsworth's autobiographical poem related to his childhood. The poem was completed in 1805 and published in 'Poems, in two volume' in 1807. 104. ’’Beat Generation” is (a) A Group of Indigenous Australian authors (b) A Group of authors from Africa renouncing their colonial past (c) A Group of authors who migrated from Asian countries (d) A Group of non-conformist authors the emerged in America in 1950's Ans. (d) : 'Beat movement', also called Beat Generation, American social and literacy movement originating in the 1950s and centered in the bohemian artist communities of San Francisco's North Beach, Los Angeles' Venice west and Newyork city's Greenwich village. 105. The Mother is a well known work of (a) Sri Aurobindo (b) Sarojini Naidu (c) Mahatma Gandhi (d) Manmohan Ghosh Ans. (a) : 'The mother' is a well known work of sri Aurobindo 'The Mother' consists of six chapters, all of them written in 1927. The first chapter was originally written as a message, the second to fifth chapter as letter. The six and longest chapter was written for inclusion in a booklet that eventually comprised the message, the letters, and chapter six. This booklet was first published under the title 'The Mother' in 1928. 106. ’’Milton was of the Devil’s party without knowing it”. Who said this? (a) William Blake (b) P.B. Shelley (c) Dryden (d) Dr. Johnson Ans. (a) : "Milton was of the devil's party without knowing it" is said by William Blake in his poem. The Marriage of Heaven and Hell (1790). 107. Which school of English poets does Richard Crashaw belong to? (a) Romantic (b) Metaphysical _____(c) Neo-Classic _______(d) Pre-romantic _____ Ans. (b) : Richard Crashaw was an English poet, who was among the major figures associated with the metaphysical poets in seventeenth century English literature. _____________________________________ 108. ’’She dwells with beauty-beauty that must die”, who said these lines in his poem? (a) William Wordsworth (b) S.T. Coleridge (c) P.B. Shelley (d) John Keats

? 12

YCT

Ans. (d) : ’Ode on Melancholy” is one of the five odes composed by English poet John Keats in the spring of 1819. Famous line of this ode is ’’She dwells with beauty-beauty that must die”. _____________________ 109. The play St. Joan was written by (a) T.S. Eliot (b) W.B. Yeats (c) George Orwell _____(d) George Bernard Shaw ___________________ Ans. (d) : ’St. Joan’ is a play by George Bernard shaw about 15th century French military figure John of Arc. Premiering in 1923, three years after her com canonization by the Roman Catholic Church. The play is set between 1429 and 1431 with an epilogue set in 1456. ________________________________________ 110. The Playboy of the Western World was written by (a) Meredith (b) Jonathan Swift _____(c) J.M. Synge ________(d) J.M. Barrie _______ Ans. (c) : ’The Playboy of the Western World’ is a three act play written by Irish Playwright John Millington Synge and first performed at the Abbey theater, Dublic, on 26 January 1907. _____________________________ 111. "The Lady of Shalott" was written by (a) Alfred Tennyson (b) Walter Scott _____(c) Wordsworth _______(d) Browning ________ Ans. (a) : ’The Lady of Shallot” is a lyric ballad by the English poet Alfred Lord Tennyson. Based on the medieval La Damigella di Scalot. It tells a tragic story of Elaine of Astolat, a young Nobel women imprisoned in a tower up the river from Camelot. _______________ 112. Jacques Derrida is the originator of a mode of reading known as (a) Deconstruction (b) Modernism _____(c) Post-Modernism (d) Realism _________ Ans. (a) : Jacques Derrida is the originator of a made of reading known as deconstruction. According to Derrida ’’Deconstruction seems to center around the idea that language and meaning are after inadequate in trying to convey the message or idea a communicator is trying to express”. ______________________________________ 113. The term grammatology - which deonotes "a general science of writing" - was used by (a) Jacques Derrida (b) St. Augustina _____(c) Thomas Hobbes (d) Benedits Spinoza Ans. (a) : The term grammatology - which denotes ”a general science of writing” was used by Jacques Derrida. ______________________________________ 114. The term ’Cultural Materialism’ is first associated with (a) Hegel (b) Roland Barthes (c) Raymond Williams (d) Terry Eagleton Ans. (c) : The term ’Cultural Materialism’ is first associated with Raymond William Cultural materialism makes analysis based in critical theory, in the tradition of Frankfurt school. MPPSC Asst. Prof. Exam. 2017

115. What was the tragic flaw in the character of Macbeth? (a) Over thinking (b) Insanity (c) Inaction (d) Over ambition Ans. (d) : ’Macbeth’ is regard of the famous tragedy among the four great tragedies of William Shakespeare. Over Ambition was the tragic flow in the character of Macbeth. 116. Who said, "God’s in His Heaven and all’s right with the world"? (a) Robert Browning (b) Alfred Tennyson (c) John Keats (d) Matthew Arnold Ans. (a) : Pippa Passes’ is a verse drama by Robert Browning. It was published in 1841 as the first volume of his Bells and Pomegranates series. Famous lines ’God’s in his heaven and all’s right with the world’ appears in this verse drama. 117. Who is the author of these lines "In vain your bangles cast/charmed circles at may feet/I am Abiku, calling for the first/And the repeated time.’’? (a) ChinuaAchebe (b) Ngugi wa Thiong’o (c) J.P. Clark (d) Wole Soyinka Ans. (d) : These lines are written by Wole Soyinka in his famous work - ’Abiku’. He is a Nigerian playwright, novelist, poet and essayist in English Language. 118. The Play Murder at the Prayer Meeting which echoes the title of Eliot’s Murder in the Cathedral has been written by (a) GirishKarnad (b) GurcharanDas (c) Lakhan Deb (d) Gieve Patel Ans. (c) : The murder at the prayer meeting is a play in two acts and a prologue by Lakhan Deb. The title of the play echos the title of Eliot, "Murder in the Cathedral.” 119. What does 'Wessex’ signify? (a) The region in which Hardy’s novels are set (b) The region in which R.K. Narayan’s novels are set (c) The region in which Bronte Sisters lived (d) The hometown of George Eliot Ans. (a) : Thomas Hardy is famous for his novels of 19th century rural life. Rich in description and dialect, they are written museums of a vanished culture. Hardy set them in Wessex, an imaginary region mapped on to the geography of south and south west England. 120. Which one of the following books of J.L Nehru is the exquisite projection of his sympathy with Indian Tradition? (a) The Discovery of India (b) The Autobiography (c) The Glimpses of World History (d) Letters to My Daughter

? 13

YCT

Ans. (a) : ’The Discovery of India’ was written by India’s first Prime-Minster Pt. Jawahar Lal Nehru during his imprisonment in 1942-1945 at Ahmednagar fort in Maharashtra, India by the British during the British Raj. This book of Jawaharlal Nehru is the exquisite projection of his sympathy with Indian tradition. 121. Which author argued that ’symbols’ evoke the ’’Great Mind” and ’’Great Memory”? (a) W.B. Yeats (b) Baudelaire (c) Ezra Pound (d) Mallarme Ans. (a) : Yeats has formulated the concept of the Great memory as a result of his search for a repository of symbols. He has postulated three assumption regarding this unique feature, in his essay ’Magic’. Thirdly-”that this great mind and great memory can be evoked by symbols”. 122. Select the book which is not authored by Shashi Deshpande. (a) That Long Silence (b) Agnigarbha (c) Small Remedies (d) The Binding Vine Ans. (b) : ’Agnigarbha’ is a short story collection in 1978 by Mahasweta Devi. She was an Indian writer in Begali and activist. 123. ’’Grunge Literature” is a genre of literature that has emerged in (a) South Africa (b) Australia (c) Canada (d) New Zealand Ans. (b) : ’Grunge Literature’ is a genre of literature that has emerged in Australia. Grunge fiction mainly deals with dissatisfaction urbanties who endeavor to fill the vacuity and spleen of their existence with music, drugs, sexual delight and intoxication. 124. The ’’Hundred Years War” was fought between England and (a) Germany (b) Spain (c) Italy (d) France Ans. (d) : The Hundred years’ war was a series of conflicts in Western Europe from 1337 to 1453 waged between the House of Plantagenet and its cadet House of Lancaster, rules of the Kingdom of England and the House of Valois over the right to rule the Kingdom of France. 125. Sue Bridehead, a character created by Thomas Hardy figures in (a) Tess of D’Urbervilles (b) The Mayor of Casterbridge (c) The Return of the Native (d) Jude the Obscure Ans. (d) : ’Jude the obscure’ is a novel by Thomas Hardy published in 1845. It was Hardy’s last completed novel. Its protagonist Jude Fawley is a working class men, a stone-mason who dreams of becoming a scholar. The other main character is his cousin-sue Bridehead, who is also his central love interest. MPPSC Asst. Prof. Exam. 2017

126. Which of the following essays by Virginia Woolf is on ’’Feminism”? (a) A Room of One’s Own (b) The Death of the Moth (c) The Moment (d) None of these Ans. (a) : ’ A Room of One’s Own’ is an extended essay of Virginia woolf, first published in September 1929. This essay is based on ’Feminism’. 127. Who first used the phrase ’Stream of Consciousness’ in the book Principles of Psychology? (a) Sigmond Freud (b) William James (c) C.G. Jung (d) James Joyce Ans. (b) : Principles of psychology is an 1890 book about psychology by William James, a American philosopher and psychologist who trained to be a physician before 'stream of consciousness' in his book, 'Principle of psychology. 128. Who is the author of the famous book The Bluest Eye? (a) Toni Morrison (b) Margaret Mitchell (c) Eidth Wharton (d) Daphne De Maurier Ans. (a) : ’The Blust eyes’ published in 1970, is the first novel written by Toni Morrison. The novel takes place in Lorain, ohio, and tells the story of a young African American girl named-Pecala uh. grows up during the year followings the great depression. 129. In the novel Sons and Lovers, D.H. Lawrence himself is represented by (a) William Morel (b) Walter Morel (c) Paul Morel (d) Arthur Morel Ans. (c) : ’Sons and Lover's is novel by the English writer D.H. Lawrence. Originally published in 1913. In this novel D.H. Lawrence himself is represented by Paul Morel. 130. What was the name of the poetess who was married to Ted Hughes? (a) Ezra Pound (b) Ivy Compton Burnett (c) Sylvia Plath (d) Elizabeth Jenkins Ans. (c) : Sylvia Plath was an American poetess, novelist and short story writer. She was married to Ted Huges. 131. Noun, Verb, Adjective, Adverb, etc., are components of (a) Figures of Speech (b) Parts of Speech (c) Reported Speech (d) None of these Ans. (b) : Noun, Verb, Adjective, Adverb, etc are components of parts of speech. 14

YCT

132. Who among the following writers does not 138. Who is the author of ’’Final Solutions and Other Plays”? subscribe to the ideas of existentialism? (a) Vijay Tendulkar (a) Soren Kierkegaard (b) Jean Paul Sartre (b) Mahesh Dattani (c) A.G. Gardiner (d) John Osborne (c) GirishKarnad Ans. (c) : A.G. Gardiner was an English journalist, (d) U.R. Ananthamurthy editor and author. He does not subscribe the ideas of Ans. (b) : ’Final Solutions and Other Plays’ are written existentialism. by Mahesh Dattani’ who is an Indian director, actor, 133. ’Phonetics’ deals with playwright and writer. He wrote such plays as final (a) Speech Sounds solutions, Dance like a man, Tara etc. (b) Lexicography 139. Samuel Langhorne Clemens is the real name of (c) Syntax (a) Eugene O’Neill (b) O.Henry (d) Manuscriptology (c) Mark Twain (d) Maxim Gorky Ans. (a) : Phonetics is the study of speech sounds and Ans. (c) : ’Samuel Langhorne Clemens’ known by his how they are produced. pen name-Mark Twain, was an American writer, 134. Who has written the play A Street Car Named publisher and lecturer. Desire? 140. Gora by Rabindra Nath Tagore is a/an (a) Eugena O’Neill (a) Play (b) Poem (b) Tennessee Williams (c) Novel (d) Essay (c) Arthur Miler Ans. (c) : ’Gora’ is a novel by Rabindra Nath Tagore set (d) Edward Albee in Calcutta in the 1880s during the British Raj. It is the Ans. (b) : ’A street car named desire’ is a play written fifth in order of writing and the longest of Tagore’s by Tennessee Williams that was first performed on twelve novels. Broadway on December 3, 1947. The play dramatizes 141. The theoretical concept of ’Ideological State the experiences of Blanche Dubois, is a farmer southern Apparatus’ was propounded by: belle. (a) Althusser 135. Who wrote the novel English August : An (b) Antonio Gamsci Indian Story? (c) Michel Focault (a) Vikram Seth (d) Fredric Jameson (b) AnuragMathur Ans. (a) : Louis Pierre Althusser was a French Marxist (c) Upamanyu Chatterjee philosopher. His notable work is ’’Ideology and Ideological state apparatus in 1970. (d) Bhabani Battacharya Ans. (c) : ’English August : An Indian story is a novel 142. ’’The curfew tolls the knell of parting day”, in which poem of Thomas Gray the quoted line by Indian author Upamanyu Chatterjee written in occurs? English, first published in 1988. It was adopted into a (a) Hymn to Adversity film of the same name in 1994. (b) Ode on a Distant Prospect of Eton College 136. Who was the first Australian to be awarded (c) Elegy Written in a Country Churchyard Nobel Prize in Literature? (d) None of these (a) Patrick White (b) David Marr Ans. (c) : Elegy written in a Country Churchyard is a (c) J.M. Coetzee (d) James Clavell poem by Thomas Gray completed in 1750 and first Ans. (a) : ’Patrick white’ was the first Australian who published in 1751. Opening line of the poem is ’’The was awarded Nobel prize in literature in 1973. He was a curfew tolls the knell of parting day”. novelist, playwright, poet, short story writer and 143. A.K. Ramanujan has constantly translated essayist. from 137. Which ’Bhakti’ poet’s poems were translated (a) English and Bengali into Sanskrit into English by Rabindranath Tagore? (b) English and Tamil into Bengali (a) Surdas (c) Bengali and Sanskrit into English (b) Tulsidas (d) Tamil and Kannada into English (c) Chaitanya Mahaprabhu Ans. (d) : Attipate krishnaswami Ramanujan was an (d) kabirDas Indian poet and scholar of Indian literature who wrote Ans. (d) : Bhakti poet Kabir Das’s poems were in both Kannada and English. He was constantly translated into English by Rabindra Nath Tagore. translated from Tamil and Kannada into English. MPPSC Asst. Prof. Exam. 2017

145

YCT

144. T.S. Eliot’s The Waste Land concludes with a phrase from which language? (a) Latin (b) French (c) German (d) Sanskrit Ans. (d) : ’The Waste Land’ is a poem by T.S. Eliot widely regarded as one of the most important poem of 20th century and a central work of modem poetry published in 1922. The waste land concludes with a phrase from Sanskrit language as followingDatta.Dayadhvam.Damyata Shantih, Shantih, Shantih 145. Charles Lamb belongs to (a) Romantic Age (b) Victorian (c) Elizabethan Age (d) Neo-Classical Age Ans. (a) : Charles Lamb belongs to Romantic Age (1785-1832). He was bom on 10 Feb 1775 in London and died on Dec, 27, 1834. He was English author critic and minor poet and is best known for his essays named essay of elia. 146. Who wrote the famous book Modern Painters? (a) John Ruskin (b) Matthew Arnold (c) D.G. Rossetti (d) Christina Rossetti Ans. (a) : 'Modem Painters' is a five volume work by the eminent Victorian art critic John Ruskin. 147. T.S. Eliot’s The Waste Land was dedicated to (a) Shakespere (b) Charles Lamb (c) Ezra Pound (d) Milton Ans. (c) : T.S. Eliot dedicated his poem 'The Waste Land’ to Ezra Pound. The dedication is "Far Ezra Pound : il migliar fabbro". It means "the better craftsman. 148. English tragedy followed the model provided by: (a) Seneca (b) Webstar (c) Malowe (d) Shakespeare Ans. (a) : English tragedy followed the model provided by Seneca. He was a Roman stoic philosopher statesman and dramatist. 149. From whom do we get the phrase, ’’negative capability”? (a) William Hazlitt (b) P.B. Shelley (c) William Wordsworth _____(d) John Keats ____________________________ Ans. (d) : Negative capability is a phrase attributes to the English poet John Keats. Negative capability, that is, when a man is capable of being in uncertainties, mysteries, doubt, without any irritable reaching after fact and reason. MPPSC Asst. Prof. Exam. 2017

150. Which of the following books was translated into English by Wyciffe? (a) Alliterative Verses (b) Bible _____(c) A King and No King (d) Gorbuduc ________ Ans. (b) : John Wyciffe was an English theologian philosopher, church reformer, and promoter of the first complete translation of the Bible in to English. He was one of the reformers of the protestant reformation. _____ 151. The attribute for which the Romantic poems are known for is: (a) Satire (b) Lyrical Quality (c) Wit (d) Didacticism Ans. (b) : The attribute for Romantic poems are known for ’Lyrical Quality'. Lyrical quality is a formal type of poetry which expresses personal emotions or feelings, typically spoken in the first person. _________________ 152. Who was Ulysses? (a) The Duke of Savoy (b) The King of Spain (c) The King of Ithaca (d) The King of Holland Ans. (c) : 'Ulysses' was the king of Ithaca. He was most valiant character in the poem Ulysses (dramaticmonologue) by Tennyson. It was written in 1833, and published in 1842 in his second volume of poetry. _____ 153. The line ”1 am not Prince Hamlet, nor was meant to be” is taken from which poem of T.S. Eliot? (a) Gerontion (b) Preludes (c) The Love Song of J. Alfred Prufrock (d) The Wast Land ________________________ Ans. (c) : 'The Love song of J. Alfred Prufrock' is a poem (Interior monologue) by Thomas Stem Eliot, in it we find this line "I am not prince Hamlet, nor was meant to be". It was first published in 1915 and later on it was published in 1917. _________________________ 154. Vikram Seth has written a novel in verse entitled . (a) The Golden Breath (b) The Golden Bough (c) The Golden Gate (d) The Golden Threshold Ans. (c) : 'The Golden Gate' is the first novel by poet and novelist Vikram seth. It was published march 12, 1986. The novel brought its author Sahitya Akademi Award for English by the Sahitya Akademi in 1988. 155. A Grammarian’s Funeral was written by (a) Dylan Thomas (b) Robert Browning (c) Thomas Hardy (d) W. H. Auden Ans. (b) : "A Grammarian's Funeral" is a dramatic monologue set in Renaissance in Europe. Composed by English poet 'Robert Browning'. Notable worksMen and Woman The Ring and the Book, etc.

146

YCT

156. When did the ’’Movement Poetry” come into Ans. (c) : "To His Coy Mistress” is a Cavalier poem existence? written by English author 'Andrew Marvell' it was (a) After 1902 (b) After 1885 published posthumously in 1681. He was an Meta (c) After 1914 ________(d) After 1942 _______ physical poet, satirist and politician. He was a lecturer Ans. (d) : After 1942, ’’Movement Poetry” came into at Holy Trinity Church. the existence. The Movement Poetry refers to the 162. The First Folio edition of Shakespeare’s plays literary group of poets of 1950’s. This group was was printed in: formed in 1950's which consisted of renown poets of (a) 1627 (b) 1628 that era. (c) 1623 _____________(d) 1621 ____________ 157. Who wrote the novel Kangaroo? Ans. (c) : In T623’ The First Folio edition of William (a) Virginia Woolf Shakespeare’s plays was printed. English author (b) James Joyce Shakespeare has written 154 sonnets and 37 plays in his life career. There are Thirty Six (36) plays were being (c) D.H. Lawrence published in First Folio by Heminges and Condell. (d) John Mansfield Ans. (c) : The novel ’Kangaroo’ 1923 by D.H. 163. Which was the period when Dr. Samuel Johnson was at work on the Dictionary? Lawrence. It was set in Australia. Kangaroo is an account of a visit to ’New South Wales’. It was adopted (a) 1746-55 (b) 1750-55 as film, also called kangaroo in 1987. (c) 1748-50 __________(d) 1762-65 _________ 158. Sohrab and Rustum by Matthew Arnold is Ans. (a) : Since 1746-1755 Dr. Samuel Johnson was at inspired by work on the dictionary. He was famous poet, (a) Arabian Nights playwright, essayist, moralist, literary, critic, biographer, editor and lexicographer. 'A Dictionary of (b) rubaiyat of Omar Khayyam the English Language' written by Samuel Johnson, (c) ShahNama _____(d) Babur Nama __________________________ Published in 1755. Ans. (c) : ’Sohrab and Rustum’ is a narrative poem with 164. ’Rime Royal’ is used in: (a) The Reaper strong tragic themes first published in 1853 by Matthew (b) The Faerie Queen Arnold. He was inspired by Ferdowsils ’Shah Nama’. It is a long epic poem published in 1832. ______________ (c) The Canterbury Tales (d) World is Too Much With us 159. Which of the following books is written by Edward W. Said? Ans. (c) : 'The Canterbury Tales' is a collection of 24 (a) Post-colonial Studies stories that runs to over 17,000 lines written by 'Geoffrey Chaucer' between 1387 and 1400. It was (b) Towards a New Psychology of Women written in the form of 'Rime Royal'. It was coined by (c) Orientalism _____(d) Beyond the Pleasure Principle ____________ Chaucer, seven-line iambic pentameter, Rhyming scheme-ababbcc. _______________________________ Ans. (c) : ’Orientalism’ is a book written by Edward W. 165. Who is the author of the play, Waiting for said in 1978. ’’orientalism” as a critical concept to Godot? describe the west’s common contemptuous-depiction (a) Samuel Beckett and portrayal of "The East” i.e. the orient. (b) T.S. Eliot 160. Which of the following is not written by Arun (c) W.B. Yeats Joshi? (d) Tennessee Williams (a) Combat of Shadows Ans. (a) : 'Waiting for Godot' is a play by Samuel (b) The Apprentice Beckett in which two characters, Vladimir (Didi) and (c) The Last Labyrinth Estragon (Gogo) engage in a variety of discussion and (d) The Foreigner encounters while awaiting Godot, who never arrives. Ans. (a) : ’Combat of shadows' is not written by Arun 166. Who has written Survival, the celebrated Joshi. He was an Indian writer. He is known for his commentary on Canadian Literature? novels. He won the Sahitya Akademi Award in 1982. (a) F.R. Scott (b) A. J. M. Smith Combat of Shadow's is a novel by Manohar Malgonkar. (c) Norhrop Frye (d) Margaret Atwood 161. Who has written the poem, ”To His Coy Ans. (d) : 'Margaret Atwood' has written ’survival’, the Mistress”? celebrated commentary on Canadian literature. She has (a) John Donne published 18 books of poetry, 18 novels, 11 books non(b) Robert Herrick fiction, 9 collections of short function 8 children’s books (c) Andrew Marvell and two graphic novels etc. (d) Richard Crashaw 147 YCT MPPSC Asst. Prof. Exam. 2017

167. The one which is not a famous twentieth century novel is: (a) Nineteen Eighty Four (b) The Lord of Flies (c) The Talisman (d) Lucky Jim Ans. (c) : ’The Talisman’ is not a famous novel of Twentieth century. It is a novel by American writers Stephen king and Peter struab in 1984. 168. Who is the writer of American novel, Grapes of Wrath? (a) Hemingway (b) Steinback (c) Faulkner (d) Farrell Ans. (b) : ’Grapes of wrath’ is novel by American author John Steinbeck. It was published in 1939. He was awarded the Nobel Prize in 1962. 169. Mark the Anglo-Saxon period in English History: (a) 500-1070 AD (b) 450-1066 AD (c) 1400-1480 AD (d) 923-1473 AD Ans. (b) : 450-1066 AD is regarded as the Anglo-Saxon period in English literature. 170. Shydney’s Arcadia is a/an (a) Elegy (b) Memoir (c) Prose Romance (d) Travelogue Ans. (c) : ’Philip Sydney’ was a famous author and he has written Arcadia, it is a heroic prose romance. 171. How many pilgrims were there in the Canterbury Tales? (a) Thirty Two (b) Thirty Five (c) Fifty (d) Fifty Five Ans. (a) : There are Thirty two pilgrims were there in the Canterbury Tales, written by Geoffrey Chaucer in 1385-1400. It is collection of 24 stories in all. ________ 172. What does the term ’’Cacophony” mean? (a) Pleasant and musical (b) Powerful and strong (c) Harsh and rough _____(d) Young an beautiful _____________________ Ans. (c) : The term ’Cacophony’ means language which is perceive as harsh, rough and unmusical sound. ______ 173. Who is the writer of the book Practical Criticism, which is a teaching manual for the study of poetry? (a) John Rich (b) LA. Richards (c) Samuel Richardson (d) H.H. Richardson _______________________ Ans. (b) : The book ’’Practical Criticism” is written by LA. Richards. Practical criticism is, like the formal study of English literature, a relatively young discipline. It began in 1920 with a series of experiments by the Cambridge critic LA. Richard. MPPSC Asst. Prof. Exam. 2017

174. Which one of the following is not an epistolary novel? (a) Pamela (b) The Colour Purple (c) Heart of Darkness (d) Wake Up, Stupid _______________________ Ans. (c) : ’Heart of Darkness’ is a novella by Polish English novelist Joseph Conrad about a narrative voyage up the Cango river into the Congo free state in the heart of Africa. This novel is not an epistolary novel. 175. Who has written the poem ’’Poor Women in a City Church? (a) T.S. Eliot (b) D.H. Lawrence (c) W.B. Yeats (d) Seamus Heaney Ans. (d) : ’Poor women in a city church’ by Seamus Heaney is a poem first published in 1966 in ’’Death of a Naturalist” dealing with women praying in a church. 176. Which of the following is called the Magna Carta of Romanticism? (a) Biograhia Literaria (b) Defence of Poetry (c) Lyrical Ballads (d) The Spirit of the Age Ans. (c) : ’Lyrical Ballads’ is called the Magna Carta of Romanticism. Lyrical Ballads, with a few other poems is a collection of poems by William Wordsworth and S.T. Coleridge first published in 1798. 177. Willy Loman is the Protagonist of which of the following plays? (a) The Importance Of Being Earnest (b) The Glass Managerie (c) Death of a Salesman (d) Loyalties Ans. (c) : ’Death of a salesman’ is a 1949 stage play written by American playwright Arthur Miller. It won the 1949 Pulitzer Prize for drama and Tony award for best play. Willy Loman, Biff Loman, Happy Loman, Linda Loman are the major characters of this play. 178. Which is not essentially a skill in English Language teaching? (a) Reading (b) Scaling (c) Writing (d) Listening Ans. (b) : ’Scaling’ is not essentially a skill in English language teaching. 179. To ask a student to read poem means to test this skill: (a) Comprehension (b) Writing _____(c) Reporting _________(d) Memorizing ______ Ans. (a) : ’Comprehension’ is an exercise that tests how well, you understand spoken or written language and comprehension is skill to ask a student to read poem means to test this skill.

? 18

YCT

180. Which of the following is not a novel by Rhinton Mistry? (a) Such A Long Journey (b) Family Matters (c) A Fine Balance (d) The Victim Ans. (d) : ’The Victim' is a novel by Saul Bellow, published in 1947 in Vangword Press. The story explore the men’s evolving relationship, all while Lenthal is struggling to deal with his own family problems. 181. Which of these poems is not written in dramatic monologue? (a) Ulysses (b) The Last Ride Together (c) Morte d' Arthur (d) Evelyn Hope Ans. (c) : 'Morte d' Arthur' is a poem by Lord Alfred Tennyson. It is in form of 'Threnody' it came in the volume "Idylls of the king" a cycle of Twelve narrative poems published between 1859 and 1885. 182. Which of John Dryden’s work is written on the life of Antony and Cleopatra? (a) All for Love (b) Fables (c) Adsalom and Architophel (d) Antony and Cleopatra Ans. (a) : 'All for Love' is a famous work by John Dryden. It is written on the life of Antony and Cleopatra (by Shakespeare) It was also known as 'The world well lost' it is a tragedy written in Blank-verse in 1677. _____ 183. This is considered to be the first play of Shakespeare by general consent: (a) Romeo and Juliet (b) Henry VI, Part I (c) Two Gentlemen of Verona (d) Love's Labour Lost Ans. (b) : 'Henry VI, Part IP is a history play written by William Shakespeare, possibility in collaboration with C. Marlowe and Thomas Nashe in 1591. Some important characters are asKing Henry VI- King of Scotland Duke of Bedford, Humphery, Duke of Gloucester. 184. The God of Small Things by Arundhati Roy has following character: (a) Mammachi (b) Yayati _____(c) Bakha ____________(d) Bhagvata ________ Ans. (a) : 'The God of Small Things" is the debut novel of Indian Author 'Arundhati Roy'. It was written in 1997, for this she got Booker prize awarded in 1987. In this novel we find a famous character; Mammachi. 185. The Twelve Knights in 'The Faerie Queen' represent twelve: (a) Journeys (b) Sins (c) Decisions (d) Virtues MPPSC Asst. Prof. Exam. 2017

Ans. (d) : 'The Faerie Queen' is an English epic poem by Edmond Spenser. Book I-III were published in 1590 and then republished in 1596 with books IX-XI. In this epic Twelve Knight presents as virtues. 186. L2 in English Language teaching stands for: (a) Second language acquired after mothertongue as English (b) Acquired foreign language (c) Any acquired language after mother tongue (d) Any language Ans. (c) : L2 is a language that is not the native language of the speaker, but is learned later and L2 in A person's second language) in English language teaching stands for any acquired language after mother tongue. 187. Blending, Back Formation, Compounding and Clipping are forms of: (a) Hybrid Language (b) Creolisation (c) Word formation (d) Intonation Ans. (c) : Blending, Back Formation, Compounding and clipping are forms of 'Word formation'. In linguistics, 'Word formation is the creation of a new word'. There are a number of methods of word formation, Borrowing, Derivations, Compounding, Blending etc. 188. What do you call the minimal unit of meaning or grammatical function? (a) Phoneme (b) Morpheme (c) Synonym (d) Register Ans. (b) : "Morpheme" is the smallest (minimal) meaningful unit in a language. A morpheme is not necessarily the same as a word. Roots are composed of only one morpheme. 189. "The Padma Awards were announced by the Rashtrapathi Bhavan", is an example of: (a) Synecdoche (b) Metaphor (c) Metonymy (d) Collocation Ans. (c) : "Metonymy" is a figure of speech in which a thing or concept is referred to by the name of something closely associated with that thing or concept as- "The Padma awards were announced by the Rastrapati Bhawan". 190. The study of the intended meaning of the speaker is: (a) Semantics (b) Comprehension (c) Etymology (d) Pragmatics Ans. (d) : The study of the intended meaning of the speaker is Pragmatics. It means dealing with problems in a practical way rather than by following ideas or principles.

? 19

YCT

191. The Language which is the product of a multilingual situation who no native speaker is called: (a) Pidgin (b) Li (c) L2 (d) Pastiche Ans. (a) : The language which is the product of a multilingual situation with no native speaker is calledPidgin. It is a simple form of a language, especially English, Portuguese or Dutch, used with words from a local language as means of communication by individuals who do not speak a common language. 192. According to Saussure, the specific acts of speech or utterance based on the rules of language is called: (a) Langue (b) Duree (c) Parole (d) Sign Ans. (c) : ’Parole’ (speech) is a French term refers to the concrete instances of the use of language, including texts which provide the ordinary research material for linguistics by saussure the specific acts of speech or utterance based on the rule of language is called ’Parole’._______________________________________ 193. Jakobson described the vertical relation between any single word in a sentence and other words which can be substituted for it as relation: (a) Syntagmatic (b) Paradigmatic _____(c) Generator _________(d) Syntactic ________ Ans. (b) : ' Paradigmatic’ relating to the way different words or language items can be chosen to play a particular part in a language structure. By Jakobson vertical relation between any single word in a sentence and other words which can be substituted for it as ’ Paradigmatic'. 194. Bakhtin’s argument that no living word relates to its object in a ’singular' way is the basis of: (a) Dialogism (b) Polyphony (c) Camivalesque (d) Morphology Ans. (a) : Bakhtin's argument that no living word relates to its object in a singular way is the basis of 'Dialogism'. Dialogism refers to the use of conversation or shared dialogue to explore the meaning of something. 195. Which poem of Nissim Ezekiel deals with the sentiments of a mother? (a) Island (b) Poet, Lover and Bird Watcher (c) Patriot _____(d) The Night of Scorpion __________________ Ans. (d) : The poem ’Night of Scorpion' composed by Nissim Ezekiel in his book 'The Exact Name' in 1695. It deals with the sentiments of a mother. This poem is about how the Scorpion stung the poets mother and mother's love for her children. MPPSC Asst. Prof. Exam. 2017

196. Namdeo Dhasal, Laxman Gaikwad, Arjun Dangle and Hira Bansode are representative writers of: (a) Diaspora Literature (b) Dalit Literature (c) Children’s Literature (d) Young Adult Fiction Ans. (b) : Namdeo Dhosal, Laxman Gaikwad, Arjun Dangle and Hira Bansode are representative writers of Dalit Literature. Dalit Literature is literature written by Dalits about Dalit literature. It emerged in the 1960s in the Marathi language. 197. Name of the autobiography of Nayantara Sahgal is ____ (a) My Story (b) Joothan (c) Prison and Chocolate Cake (d) Prison Days Ans. (c) : 'Prison and Chocolate Cake' is the autobiography of Nayantara Sahgal, an Indian writer. She is a member of the Nehru-Gandhi family. She was awarded Sahitya Akademi Award for her English Novel Rich Lime Us 1985. _____________________________ 198. Who is the author of Ghachar-Ghochar? (a) Amish Tripathi (b) Vivek Shanbag (c) Meena Kandasamy (d) Manjura Padmanabhan Ans. (b) : Vivek Shanbag is the author of 'GhacharGhochar', is a psychological drama written by Kannada author and was translated into English by 'Srinath Perur'. Original published in 10th December 2015. 199. Who is the presiding deity of 'Hasya Rasa'? (a) Rudra (b) Kaal (c) Brahma (d) Pramath Ans. (d) : 'Pramath' is the presiding deity of 'Hasya Rasa'. Hasya is Sanskrit word for one of the nine rasas a bhava, means 'flavour' the theory of rasa is the primary concept behind classical Indian arts. 200. How many chapters are there in Natyasastra? (a) Thirty Six (b) Twenty Four (c) Thirty Three (d) Eleven Ans. (a) : The 'Natyasastra' consists 36 chapter with a cumulative total of 6000 poetic verses describing performance arts was written by sage Bharata Muni. It is a notable as an ancient encyclopedic treatise on the arts, one which has influenced, dance, music and literary traditions in India.

150

YCT

Chhattisgarh Public Service Commission Asst. Prof. Exam. 2019

ENGLISH (Solved Paper) 1.

Tempe in the ’Grecian Urn’ alludes to a valley in (a) Thessaly (b) Italy (c) Alps (d) Sicily Ans: (a) John Keats here expresses his desire to know the exact location of the scenes depicted on the urn. Whether it is set in Tempe, a beautiful valley in Thessaly, on the Easter coast of the Northern Greece, or in Arcadia a mountainous country in the centre of the Peloponnesus. ______________________________ 2. Andrea del Sarto was a real artist who lived in (a) 17th Century (b) 13th Century (c) 16th Century (d) 14th Century Ans: (c) ’Andrea del Sarto’ was a real artist who lived in 16th Century. Andrea del Sarto, original name Andrea d'Angolo, Italian painter and draftsman whose works of exquisite composition and craftsmanship were instrumental in the development of Florentine Mannerism. Robert Browning composed a dramatic monologue ’Andrea del Sarto’. 3. In November 1791, Wordsworth visited France and fell in love with (a) Annette Vallon (b) Mary Hutchinson (c) Dorothy __________(d) Lucy_____________ Ans: (a) In November 1791, Wordsworth visited Revolutionary France and became enchanted by the Republic movement. He fell in love with a French woman, Annette Vallon, who in 1792 gave birth to their daughter Caroline. 4. Which one of the following is a play by Wordsworth? (a) Dove Cottage (b) Nutting (c) The Borderers _____(d) The Prelude _______ Ans: (c) ’The Borderers’, Wordsworth wrote his only play. The Borderers, a verse tragedy set during the reign of King Henry III of England, when Englishmen in the North Country came into conflict with Scottish border rivers. 5. ’’The homely nurse doth all she can To make her foster child her inmate man.” Who is the ’nurse’? (a) Sky (b) Earth (c) Dorothy __________(d) Lake District ______ Ans: (b) ’’The homely nurse doth all she can To make her foster child her inmate man” lines of William Wordsworth’s ”Ode : Intimations of Immortality Recollections of Early Childhood”. Here Nurse is Earth. It was published in 1807. CGPSC Asst. Prof. Exam. 2019

[Exam. Date : 06 Nov 2020]

6.

’’Charmed magic , opening on the foam.” Fill in the blank. (a) wands (b) balls (c) books ____________(d) casements _________ Ans: (d) Casement means a part of window set on a vertical hinge. These lines belong to John Keats’ ’Ode to a Nightingale’. 7. Which of the following words does not mean indolence? (a) Sloth (b) Inertness (c) Languor __________(d) Agility ___________ Ans: (d) Agility is the ability to move quickly and easily in response to change. 8. Tennyson’s poetry is considered as: (a) an escape from life (b) humour of the folly of man (c) an attack on his age (d) a mirror of his age _______________________ Ans: (d) • Tennyson poetry is remarkable for its Metrical Variety, rich imagery and Verbal Melodies. Alfred Lord Tennyson is significant Victorian poet. He was appointed poet laureate in 1850, in succession to Wordsworth. • He won the Chancellor’s Medal for a poem on ’Timbuctoo’ (1829). • ’Chiefly Lyrical (1830) published while he was an undergraduate. • ’In Memoriam’ (1850) is a pastoral elegy on the dead of his friend Arthur Henry Hallam. 9. The heroic deeds and adventures of Ulysses are also sung by (a) Gray’s 'Elegy' (b) Dryden’s All for Love (c) Milton’s Paradise Lost (d) Homer’s Odyssey________________________ Ans: (d) The heroic deeds and adventures of Ulysses are also sung by Homer’s ’Odyssey’. The Greek hero Odysseus king of Ithaca and his journey to home after the Trojan war are full of adventures. 10. Which is the three-volume epic closet drama by Hardy? (a) The Dynasts (b) The Wessex (c) Two on a Tower (d) The Poor Man and the Lady

151

YCT

Ans: (a) The Dynasts is an English language closet drama in verse by Thomas Hardy. Hardy himself described this work as ”an epic drama of the war with Napoleon in three parts, nineteen acts and one hundred and thirty scenes. 11. Oliver Twist’s aunt in the novel is finally discovered as: (a) Rose (b) Nancy (c) Estella ___________(d) Edda _____________ Ans: (a) • The subtitle of Oliver Twist is ’’The Parish Boys Progress”. ’Oliver Twist’ (published serially from 1837 to 1839) has written by Victorian novelist Charles Dickens. It was appeared piecemeal in Bentley’s Miscellany. Rose Maylie is sister of Oliver’s mother Agnes, thus Oliver’s aunt. 12. Most of Browning’s works are: (a) Epics (b) Tragedies (c) Dramatic Monologue (d) Dramas _________ Ans: (c) Dramatic Monologue is a poem form. It is a poem written in the form of a speech of an individual character, it compresses into a single scene. 13. The opening line of ’’Rabbi Ben Ezra” is: (a) Let age approve of youth (b) Grow old along with me (c) Look not thou down but up (d) Youth ended I shall try ___________________ Ans: (b) "Rabbi Ben Ezra” is poem of Robert Browning. In this poem compare of Old and New age. In this poem comparison between old and new age is made. 14. Arnold’s term for ’meanness of spirit’ is” (a) Anarchy (b) Lack of culture (c) Americanism ______(d) Philistinism _______ Ans: (c) Arnold’s term for ’meanness of spirit’ is Americanism. Americanism ideology is a set of United States patriotic values aimed at creating a collective American identity and can be defined as "an articulation of the nation’s rightful place in the world, a set of traditions, a political language and a cultural style imbued with political meaning". 15. Charles Dickens as a child had to work at a place which made a lasting impression on him and later influenced his fiction. The place was (a) Portsea Island (b) Navy Pay Office (c) Warren’s Blacking Warehouse (d) Marshalsea Debtors' Prison _______________ Ans: (c) Charles Dickens had begun working ten hours days at Warren's Blacking Warehouse, on Hungerford stairs. Dickens relates the misery he felt during this time in the fictionalized account of David Copperfield working at Murdstone and Grinby’s warehouse. CGPSC Asst. Prof. Exam. 2019

16. Identify the writer who proclaimed the novelist to be superior to the saint, the scientist, the philosopher and the poet: (a) Thomas Hardy (b) Charles Dickens (c) E. M. Forster ______(d) D. H. Lawrence Ans: (d) D. H. Lawrence is a British novelist, establishes the superiority of novel and novelists criticizing the Parson the philosopher and the scientist in his great essay "why the novel matters". • D.H. Lawrence proclaimed the novelist to be superior to the saint, the scientist, the philosopher and the poet. 17. Identify the writer who also wrote under the pseudonym Michael Angelo Titmarsh: (a) Thomas Hardy (b) Charles Dickens (c) William Makepeace Thackeray (d) E. M. Forster ___________________________ Ans: (c) In the late 1830s Thackeray became a notable contributor of article on varied topics to Fraser’s Magazine, The New Monthly Magazine, his work was unsigned or written under such pen names as Mr. Michael Angelo Titmarsh. • ’The Rose and the Ring' an excellent example of his love of parody. 18. What is the subtitle of Oliver Twist! (a) The Pickwick Papers (b) Chimney Sweep (c) The Parish Boy's Progress (d) The Artful Dodger ______________________ Ans: (c) Charles Dicken's novel ’Oliver Twist' subtitle is 'The Parish Boy's Progress'. The novel first published serially in Bentley's Miscellany between 1837 to 1839. _________________________________ 19. Who among the following did not take part in Dryden’s An Essay of Dramatic Poesie! (a) Thomas Sackville (b) Sir Robert Howard (c) Sir Charles Sedley (d) Dryden ___________ Ans: (a) Thomas Sackville was an English statesman, poet and dramatist. He was collaborated with Thomas Norton and wrote first Tragedy in English in blank verse ’Gorboduc’. • John Dryden was appointed England's first poet laureate in 1668. • 'All for Love' subtitled 'The World Well Lost’ is heroic drama by John Dryden. 20. Coleridge pays his tribute to Herbert’s Temple in chapter of Biographia Literaria. (a) 14 (b) 17 (c) 18 _______________(d) 19 _______________ Ans: (d) George Herbert is a Metaphysical poet. His significant poem ’The Temple’ published in 1633 shows his zeal for the church of England and concern with practical theology.

152

YCT

Coleridge in the 19th chapter of his ’Biographia Literaria’ speaks about Herbert’s ’The Temple’ - ”a sonnet, equally admirable for the weight, number and expression of the thoughts and for the simple dignity of the language”. 21. Which is not a characteristic of Mrs. d’Urberville, to whom Joan sends her daughter Tess to work for? (a) A rich relation (b) A character of false respectability (c) A blind woman (d) None of the above _______________________ Ans: (a) Thomas Hardy’s greatest novels ’Tess of the D’Urbervilles’ (1881), subtitle is ’A Pure Woman faithfully presented’. • This novel was rejected by taro publishers. • Thomas Hardy’s last novel ’Jude The Obscure’. • ’’That happiness was but the occasional episode in a general drama of pain” taken from significant novel ’The Mayor of Casterbridge’ (1886). 22. What was the name of Aristotle’s father? (a) Alexander (b) Nicomachus (c) Amyntas (d) Stagira Ans: (b) Aristotle is a Greek philosopher. He wrote significant book ’Politics’. The title of politics literally means ’’The things concerning the polis”. He also coined the term ’Entelechy’ that means anything which is currently happening. His father’s name was Nicomachus. 23. Longinus, while giving examples of sublime, quotes from: (a) Plato (b) Sophocles (c) Sappho (d) Aristotle Ans: (c) Sublime means ’loftiness and excellence in language’. • Longinus is well known for the aesthetic treatise ’On the Sublime’, a work which focuses on the effect of good writing. • ’On the Sublime’ is both the treatise and the work of literary criticism. • It is written in an epistolary form. • This treatise is dedicated to ’Posthumous Terentianus’. • He promotes ’Elevation of Style’. Longinus gives an example of sublime from Sappho’s poem. 24. The term ’Katharsis’ or ’Catharsis’ is derived from: (a) Technological world (b) Engineering world (c) Mechanical world (d) None of the above Ans: (d) The term ’Katharsis’ or ’Catharsis’ is derived from the medical term Katharsis which means in Grek ’Purgation’ or ’Purification’ or ’Clarification’. CGPSC Asst. Prof. Exam. 2019

Aristotle defined tragedy in poetics Ch-VI ’’Tragedy is an imitation of an action that is serious magnitude, complete in itself ------- ” • Aristotle was the disciple of Plato. 25. Longinus, the great critic, was a/an: (a) French (b) Roman (c) Greek ____________(d) Englishman _______ Ans: (c) Longinus is the greatest critic of Greek. - Scott James called him ’First Romantic Critic’. - Quotation by Longinus”Sublimity is the echo of the noble mind”. ’’Literature is the nature of revelation”. ’’Poetry has the character of oracle”. 26. Which critical work declares criticism as a disinterested and flexible mode of thought whose application extends far beyond literature? (a) The Function of Criticism at the Present Time (b) Tradition and the Individual Talent (c) Biographia Literaria (d) An Essay of Dramatic Poesie ______________ Ans: (a) Matthew Arnold’s essay ’The Function of Criticism at the Present Time’, published in ’’The National Review” in 1864 - In ’Function of Criticism’ Arnold says that ’’for good literature to flourish two powers are necessary the creative and the critical”. - Arnold called about Shelley”A beautiful and ineffectual angel beating in the void his luminous wings in vain”. - His famous phrase is - ’’Literature is a Criticism of Life”. 27. Identify the writer who produced many of the finest court masques in collaboration with Inigo Jones: (a) Philip Massinger (b) Congreve (c) Thomas Dekker (d) Ben Jonson ________ Ans: (d) Ben Jonson was an English playwright and poet. He popularized the ’Comedy of Humours’. He is best known for the satirical plays ’Every man in His Humour’ (1598), ’Volpone or The Fox’ (1606) and ’The Alchemist’. Inigo Jones was an architect and as a theatrical designer he collaborated with Ben Jonson in several court masques. 28. Who among the following is not a writer of the Restoration Comedy? (a) Congreve (b) Etherege (c) Marlowe _________(d) Wycherley ________ Ans: (c) Restoration comedy was written and performed about 1660 to 1700. George Etherage (The Man of Mode, 1676), William Congreve (The Way of the World, 1700) and William Wycherley (The Country Wife, 1675) are famous Restoration comedy dramatists.

153

YCT

Christopher Marlowe was a great Renaissance playwright, famous for his tragedies ’The Jew of Malta’ and ’Dr. Faustus’. 29. Who among the following is credited with the invention of sonnet? (a) Shakespeare (b) Milton (c) Petrarch __________(d) Giacomo da Lentini Ans: (d) Sonnet : A lyric poem consisting of a single stanza of fourteen iambic pentameter lines linked by an intricate rhyme scheme. It is divided followings: - Italian or Petrarchan sonnet. - English sonnet or Shakespearean sonnet. - Spenserian sonnet. - For example - Wordsworth’s ’The River Duddon’. Giacomo da Lentini was an Italian poet of the 13th century. He is credited with the invention of the sonnet. 30. In English, both English type and Italian type sonnets are traditionally written in: (a) Volta (b) Quartrain (c) Heroic Couplet (d) lambic Pentameter Ans: (d) lambic Pentameter is a unstressed syllable followed by a stress syllable in ten syllable. - Volta is a turning point of poem. - Quartrain means stanza of four lines with rhyme. - Heroic couplet - iambic pentameter which rhyme in pairs: aa, bb, cc so on. Such couplet used to in heroic poem and heroic drama. - For example - Geoffery Chaucer’s ’The Legend of Good Women’ and ’The Canterbury Tales’. 31. The purpose of satire in literature is: (a) political revolution (b) insulting people (c) making people laugh at others (d) None of the above _______________________ Ans: (d) Satire can be described as the literary art of diminishing or derogating a subject by making it ridiculous and evoking toward it attitudes of amusement, contempt, scorn or indignation. Satire has usually been justified by those who practice it as a corrective of human vice and folly, therefore its purpose is making social commentary. 32. Which of the following terms used by Aristotle refers to ’overweening pride or confidence’? (a) Hubris (b) Catharsis (c) Peripeteia _________(d) Mimesis __________ Ans: (a) Hubris describes a personality quality of foolish pride. - Catharsis which in Greek signifies ’’Purgation” or ’Purification’ of pity and fear. - Peripeteia means reversal of fortune to misfortunes. Mimesis means imitation or re-presentation. 33. Gorboduc is a good example of a: (a) Farce (b) Restoration comedy (c) One-act tragedy (d) Senecan tragedy CGPSC Asst. Prof. Exam. 2019

Ans: (d) Gorboduc is first English tragedy written by Thomas Sackville and Thomas Norton, published in 1561. - Senecan tragedy was written to be recited rather than acted, but to English playwrights who thought that these tragedies had been intended to the stage. - Restoration comedy: The Restoration period started when Charles II come throne of England in 1660, at the end of the commonwealth. - Restoration comedy is also known as ’Comedy of Manners’. 34. Which of the following is an unfinished epic satire in ottava rima? (a) The Faerie Queene (b) Don Juan (c) Paradise Lost (d) Gondibert Ans: (b) ’Don Juan’ is an unfinished epic satire in ottava rima by Romantic poet Lord Byron. - ’The Vision of Judgment’ (1822) is one of the finest of English political satire. Underlying the attack on Southey. 35. Which of the following is a one-act tragedy? (a) Riders to the Sea (b) King Lear (c) The Duchess of Malfi (d) The Spanish Tragedy Ans: (a) ’Riders to the Sea’ is a one-act tragedy written by John Millington Synge. - ’King Lear’ is a tragedy of William Shakespeare. - ’The Duchess of Malfi’ is a tragedy of John Webster. - ’The Spanish Tragedy’ is a tragedy of Thomas Kyd. 36. Which among the following is not an epic poem? (a) Ramayana (b) Odyssey (c) Paradise Lost (d) None of the above Ans: (d) ’Ramayana’ is an epic of Valmiki. ’Odyssey’ is an epic of Homer. It epic has divided 24 books. ’Paradise Lost’ is an epic written by John Milton. 37. The theory of the Comedy of Humours was developed by: (a) Shakespeare (b) John Fletcher (c) Ben Jonson (d) Thomas Dekker Ans: (c) The theory of the Comedy of Humours was developed by Ben Jonson, the Elizabethan playwright, based on the ancient physiological theory of the ’four humours’. The four element of humours(1) Blood (2) Phlegm (3) Choler (4) Melancholy. 38. The theme of an elegy is: (a) celebration of love (b) praise of war (c) lamentation for something/someone loved and lost (d) praise of God 154 YCT

Who speaks the following in Hamleft ’’Let the doors be shut upon him, that he may play the fool nowhere but in’s own house, Farewell.” (a) Hamlet (b) Claudius (c) Horatio ___________(d) Polonius __________ Ans: (a) The character Hamlet speaks in play 'Hamlet'. "Let the doors be shut upon him, that he may play the fool nowhere but in's own house, Farewell". It is taken from Shakespeare longest tragedy ’Hamlet’ Act 3 Scene 1. 45. A melodrama is: (a) a sentimental comedy (b) a favourite pastime of the early twenty first century (c) one that relies chiefly on verbal wit (d) None of the above Ans: (d) ’Melodrama' was originally applied to all musical plays, including opera. 'Melas' is Greek for 'song'. - 'Sentimental comedy' which were representations of Middle class life that replaced tough amorality and the comic or satiric representation of aristocratic sexual license in Restoration comedy. 46. An epic is: (a) a long narrative poem of rueful mourning's (b) a long narrative poem of heroic exploits of a great character of historical importance (c) a short poem of reflective nature (d) a narrative poem, neither long nor short, of _______psychological broodings __________________ Ans: (b) An epic is a long narrative poem of heroic exploits of a great character of historical importance. For example - Milton’s epic 'Paradise Lost' (1667), Dante's epic poem ’Divine Comedy'. 47. Aristotle defines the dramatic art form in his: (a) Politics (b) Ethics (c) Rhetoric __________(d) Poetics ___________ Ans: (d) Aristotle's 'Poetics'. It has 26 chapters. - He also coined the term 'Entelechy' that means anything which is currently happening. 48. Dante used the term ’comedy’ in the title of his poem: (a) Monarchia (b) Convivo (c) La Commedia (d) None of the above Ans: (c) ’La Commedia’ is an poem in three parts composed by Dante. Its English translation is 'Divine Comedy'. 49. Who remarks that Hamlet himself is ’’dominated by an emotion which is inexpressible, because it is in excess of the facts as they appear”? (a) A. C. Bradly (b) John Dover Wilson (c) T.S. Eliot (d) G. Wilson Knight 155 YCT

Ans: (c) ’Elegy’ is a poem written in elegiac meter (alternating hexameter and pentameter lines). The theme of an elegy is lamentation for something/someone loved and lost. - Chaucer’s ’The Book of Douches’ (elegies in the mode of dream allegory. - The ’dirge’ is also a versified of a grief on the occasion of a particular person’s death but differs from the elegy in that it is short, is less formal and is usually represented as a text to be sung. - John Milton’s ’Lycidas’ (1638). 39. A lyric is a form of poetry which expresses: (a) a note of revenge (b) the history of music (c) a war scene (d) personal emotions Ans: (d) A lyric is a form of poetry which expresses personal emotions. 40. Which of the following lines is not from Gray’s ’Elegy Written in a Country Churchyard’? (a) Haply some hoary headed swain may say (b) Full many a gem of purest ray serene (c) Sunset and evening star and one clear call for me (d) Far from the madding crowd’s ignoble strife Ans: (c) ’Sunset and evening star/and one clear call for me’ these lines are appeared in Alfred Lord Tennyson’s poem ’Crossing the Bar’. The rest three options belong to Thomas Gray’s ’Elegy Written in a Country Churchyard’. 41. Where did the Masque originate? (a) Greece (b) Rome (c) England (d) Italy Ans: (d) Masque dramatic presentation that originated in ’Italy’ but became popular in the England. 42. The three forms of English ode are the Pindaric, the Horatian and ........... (a) the irregular (b) the Miltonic (c) the romantic _______(d) the elegiac ________ Ans: (a) An ode is a type of lyrical stanza. It is Greek origin, it is an elaborately structured poem praising or glorifying an event intellectually as well as emotionally. - There are three typical forms of odes- the Pindaric, Horatian and irregular. 43. Edward Dowden divided Shakespeare’s career into four periods: Tn the Workshop’, Tn the World’, ’Out of Depths’ and ..... (a) ’Romantic Comedy’ (b) ’His Mind and Art’ (c) 'On the Heights' (d) ’Romances’________ Ans: (c) Edward Dowden divided Shakespeare's career into four periods : In the workshop 'In the world, out of Depths and on the Heights'. Edward Dowden is an Irish critics and poet. Edward Dowden's ’Shakespeare A critical study of His mind and Art'. CGPSC Asst. Prof. Exam. 2019

44.

Woolf, her sister Vanessa Bell, Clive Bell, Leonard Woolf, Lytoon Strachey and Saxon Sydney Turner. - Lytton Strachey was an English writer and critic. He is known as a biographer. - Virginia Woolf was an English writer, considered one of the most important modernist 20th century authors and also a pioneer in the use of stream of consciousness as a narrative devices. 54. Aspects of the Novel is written by: (a) W. Cross (b) Percy Lubbock (c) E. M. Forster ______(d) Robert Liddell _____ Ans: (c) ’Aspect of the Novel’ is written by E.M. Forster. It is a collection of literary lectures, published in 1927. ________________________________________ 55. ’Tradition and the Individual Talent’ was published in Eliot’s first book of criticism entitled: (a) The Tradition (b) Impersonal Poetry (c) New Criticism _____(d) The Sacred Wood Ans: (d) The essay ’Tradition and the Individual Talent’ first published in a literary magazine ’The Egoist’ in 1919 and later in Eliot’s first book of criticism, ’’The Sacred Wood” (1920). 56. Who made the remark, ”A poem of any length neither can be, or ought to be, all poetry”? (a) Matthew Arnold (b) Coleridge (c) Sidney ___________(d) Longinus _________ Ans: (b) Samuel Taylor Coleridge, in his famous work ’Biographia Literaria’ distinguishes between ’fancy and imagination’, ’primary and secondary imagination’, ’poem and poetry’ and so on. Differing poetry from poem, he remarks ’a poem of any length neither can be, or ought to be all poetry’. 57. In which place is Dr. Aziz assumed to have attempted to assault Adela? (a) Railway Station (b) Mosque (c) Marabar Caves_____(d) Barabar Caves _____ Ans: (c) Dr. Aziz assumed to have attempted to assault Adela in Marabar Caves, in the significant novel of E. M. Forster’s ’A Passage to India’ (1924). It is set against the backdrop of the British Raj and the Indian Independence movement in the 1920. 58. Whose day of death ’’was a dark cold day”, according to Auden? (a) T.S. Eliot (b) W. B. Yeats (c) Arnold ___________(d) Robert Frost _______ Ans: (b) W. B. Yeats is an Irish poet, dramatist, prose writer. ’’The day of his death have been was a dark cold day” lines have been taken from W.H. Auden’s poem ’In Memory of W.B. Yeats’. 59. Which of the following writers served as an ambulance driver for the Republicans in the Spanish Civil War? 156 YCT

Ans: (c) Thomas Steam Eliot was a poet, essayist, playwright and literary critic. T.S. Eliot in his essay ’Hamlet and His Problems’ has termed it ’an artistic failure’. For Eliot ’’Hamlet is dominated by an emotion which is inexpressible, because it is in excess of the facts as they appear.” - A. C. Bradly is known for his masterpiece work ’Shakespearean Tragedy’ (1904). - G. Wilson Knight’s ’The Wheel of Fire’ a collection of essay on ’Shakespeare’s play. - J.D. Wilson was scholar of Renaissance drama. His famous work is ’The New Shakespeare’. 50. Who, in A Passage to India, is the principal of the government college? (a) Dr. Aziz (b) Mrs. Moore (c) Cyril Fielding (d) Adela Ans: (c) ’A Passage to India’ (1924) is a novel by English author E.M. Forster. The novel is based on Forster’s experiences in India, deriving the title from Walt Whitman’s poem ’Passage to India’ in ’’Leaves of Grass”. Cyril Fielding, in this novel is the principal of the government school, and he is generally sympathetic towards the Indians. 51. Sons and Lovers traces the relationship between: (a) son and mother (b) son and father (c) brother and sister (d) son and nephew Ans: (a) ’Sons and Lovers’ is a 1913 novel by the English writer D.H. Lawrence, originally published by Gerald Duckworth and Company Ltd, London and Mitchell Kennerley publishers, New York. 52. T. S. Eliot’s impersonal theory of poetry is elaborately dealt with in: (a) The Function of Critic (b) The Perfect Critic (c) Tradition and the Individual Talent (d) Hamlet and His Problems _________________ Ans: (c) T.S. Eliot’s ’Tradition and Individual Talent’ (1919), the essay was first published in ’The Egoist’. - ’Tradition and Individual Talent’ essay was divided into three parts1. The concept of tradition. 2. The theory of impersonality. 3. Conclusion. - Eliot defines poetry as’’Poetry is not a turning loose of emotion but an escape from emotion”. - T.S. Eliot has used term ’Objective-correlative’ in essay ’’Hamlet and His Problems”. 53. Who amongst the following was not a member of the Bloomsbury Group? (a) Virginia Woolf (b) T. S. Eliot (c) Lytton Strachey (d) E. M. Forster ______ Ans: (b) ’The Bloomsbury Group’ was a circle of artistic writers and intellectuals including Virginia CGPSC Asst. Prof. Exam. 2019

64. Auden shifted to the United States in: (a) T. S. Eliot (b) W. H. Auden (a) 1930 (b) 1939 (c) W. B. Yeats _______(d) G. B. Shaw _______ (c) 1940 _____________(d) 1945 _____________ Ans: (b) Wystan Haugh Auden was an AngloAns: (b) In 1939, Auden moved to the U.S. and his American poet. He was writer as well as an work became less political as he turned to ambulance driver for the Republicans in the Spanish Christianity. Civil War. - W. H. Auden’s long poem ’The Age of Anxiety : A 65. The word ’Spawned’ in line no. 9 of ’Gerontion’ means: Baroque Edoque’ (1947) in 6 parts. The poem won the (a) produced (b) expanded Pulitzer Prize for poetry in 1948. (c) threw out as larva (d) planted ___________ 60. Joan of Arc belongs to: Ans: (d) ’Gerontion’ is a poem by T.S. Eliot that was (a) 13th Century (b) 15th Century first published in 1920. The title is Greek for 'Little (c) 14th Century ______(d) Early 16th Century Old Man’. The theme of poem is religion, sexuality Ans: (b) ’Joan of Arc’ nickname ’’The Maid of and other general topics of modernist poetry. Orleans”. ’Saint Joan’ is a play by George Bernard The poet narrates - ’’And the Jew squats on the Shaw about 15th century French Military figure Joan window sill, the owner/Spawned in some estaminet of of Arc. Antwerp”. - Michael Holroyd has characterized the play as ”a 66. The Scarlet Letter is set against: tragedy without Villains”. (a) the Catholic background 61. What factor was not responsible for the rift (b) the Puritan background between Morel and Mrs. Morel in Sons and (c) the Protestant background Lovers? (d) the Hebrew background __________________ (a) Morel’s occasional conviviality Ans: (b) ’The Scarlet Letter’ is considered to be (b) Morel’s drinking habit Magnum opus of Nathaniel Hawthorne. It is set in (c) Morel’s irresponsible sensuality 17th century Puritan Boston. It tells the story of (d) Morel's lack of education _________________ Hester Prynne, who conceived a daughter through an affair and struggles to create a new life of repentance Ans: (a) Morel and Mrs. Morel fight and drift apart and dignity. and Walter Morel retreats to the pub after work each day. Gradually Mrs. Morel's affections shift to her 67. The Hairy Ape is cast in: (a) Psychological mode sons beginning with the oldest William. As a boy (b) Expressionistic mode William is so attached to his mother that he does not (c) Realistic mode enjoy the fair without her. (d) Impressionistic mode ____________________ 62. St. Joan is remarkable for its distinctive feature: Ans: (b) ’The Hairy Ape’ is a play of Eugene (a) A chorus Gladstone O’ Neil. The protagonist Yank in the play (b) An interlude searches for a sense of belonging in a world controlled (c) An epilogue by the rich. (d) A stage device involving a mechanical ________contraption ____________________________ - O’ Neil is also famous for his well known comedy ’Ah Wilderness!’. Ans: (c) St. Joan is remarkable for its distinctive - He got four times Pulitzer Prize for his drama. feature an epilogue. - He was the first American dramatist to win Nobel An epilogue or epilog is a piece of writing at the end Prize in literature (1936). of a work of literature. For example Shakespeare uses 68. Thoreau lived on the shores of Walden Pond in epilogue in his last play 'The Tempest’. a cabin for: 63. ’The Shield of Achilles’ recalls: (a) 24 months (b) 3 years (a) Hindu mythology (b) Roman mythology (c) 30 months ________(d) 26 months ________ (c) Greek mythology (d) Irish mythology Ans: (d) Henry David Thoreau lived for two years, Ans: (c) 'The Shield of Achilles’ is a poem by W.H. two months (26 months) and 2 days by Walden Pond Auden first published in 1952. Auden’s response to in Concord, Massachusetts. His time in Walden the detailed description or ekphrasis of the Shield Woods became a model of deliberate and ethical borne by the hero Achilles in Homer’s epic poem the living. Iliad. "The Shield of Achilles” recalls Greek 69. Who among the following writers was born in Mythology. It poem is divided into two different New York? stanza form, one form with shorter lines, the other (a) Hemingway (b) H. D. Thoreau with longer lines. (c) Stephen Crane (d) None of the above CGPSC Asst. Prof. Exam. 2019

157

YCT

Ans: (d) H.D. Thoreau is an American naturalist essayist, poet and philosopher. He is best known for his book Walden. He was bom in Messachusetts. - Ernest Miller Hemingway is an American novelist, short-storywriter and sportsman. He was bom in Illinois. - He founded the term The Iceberg Theory. - Stephen Crane is an American poet, novelist and short story writer. Crane’s first novel was the 1893 Bowery tale Maggie : A Girl of the Streets. He was bom in Newark in New Jersey. 70. The founder editor of The Cornhill Magazine was: (a) W. M. Thackeray (b) T. S. Eliot (c) D. H. Lawrence (d) G. B. Shaw _______ Ans: (a) The founder editor of ’The Comhill Magazine’ was W.M. Thackeray. The ’Comhill Magazine’ (1860-1975) was a monthly Victorian Magazine. 71. Who dies in the arms of Hester towards the end of The Scarlet Letter! (a) Pearl (b) Master Prynne (c) Arthur Dimmesdale (d) Roger Chillingworth _____________________ Ans: (c) Arthur Dimmesdale dies in the arms of Hester towards the end of ’The Scarlet Letter’. The Scarlet Letter opens with a long preamble about how the book came to be written. 72. The unprecedented success of ’The Red Badge of Courage’ made Stephen Crane: (a) editor of a national daily (b) security advisor to US Government (c) a Pulitzer Prize Awardee (d) a recognized war correspondent ____________ Ans: (d) Stephen Crane was one of America’s realistic writers. His Civil War novel ’The Red Badge of Courage’ is a classic of American literature. It won him international fame. His reputation as a war writer and his fascination with death and danger sent him to Greece and then to Cuba as a war correspondent. 73. The subtitle of Thoreau’s Walden (1854) is: (a) A Life in Mediation (b) A Yogin’s Experiments (c) Life in the Woods (d) None of the above _______________________ Ans: (c) The subtitle of Thoreau’s ’Walden’ (1854) is ’Life in the Woods’. It is based on American transcendentalism. The work deals with social experiment, voyage of spiritual discovery, satire and to some degree a manual for self-reliance. 74. Who among the following was not an active member of the Transcendental Club? (a) Emerson (b) Thoreau (c) Bronson Alcott (d) None of the above CGPSC Asst. Prof. Exam. 2019

Ans: (d) The Transcendental Club was inaugurated in 1836 by a Unitarian discussion group. It was a group of New England authors, philosophers, socialists, politicians and intellectuals of the early to mid 19th century which gave rise to Transcendentalism. - Transcendental club used to publish their own periodical ’The Dial’. - Transcendental club author areRalph Waldo Emerson, Bronson Alcott, Thoreau, Nathaniel Hawthorne and so on. 75. Who represents Shakespeare in The Tempest! (a) Caliban (b) Prospero (c) Miranda __________(d) Ariel _____________ Ans: (b) ’The Tempest’ is a last play of William Shakespeare. The sorcerer Prospero a complex and contradictory character, lives with his daughter Miranda and his two servants Caliban, a savage monster figure and ’Ariel’ an airy spirit. - A significant lines in ’The Tempest’, Prospero tells to Ariel”The rarer action is in virtue than in vengeance.” Coleridge has called Prospero ’’the very Shakespeare himself, as it were, of the Tempest." 76. "We are such stuff as dreams are made on”, which of the following is the next line of the above speech? (a) There be some sports (b) And are melted into air, thin air (c) Too light gains make the prize little (d) And our little life is rounded with a sleep _____ Ans: (d) "We are such stuff as dreams are made on And our little life is rounded with a sleep", is taken from Shakespeare play ’The Tempest’ Act IV scene I. Prospero speaks these lines. It means that which ’we imagine or fantasize about. It is that wish that we can only see in our dreams’. 77. Who speaks the following in The Tempest! ’’These be fine things, and if they be not spirits; that’s a brave god and bears celestial liquor. I will kneel to him.” (a) Trinqulo (b) Stephano (c) Caliban __________(d) Ariel _____________ Ans: (c) "These be fine things, and if they be not spirits, that’s a brave god and bears celestial liquor" speak by Caliban "The Tempest", Act II, Scence II (asides), written by Shakespeare. 78. ’’Thus the native hue of resolution is sicklied over with pale cast of thought”, who has said this? (a) Gertrude in Hamlet (b) Macbeth in Macbeth (c) Antony in Antony and Cleopatra (d) Hamlet in Hamlet _______________________ Ans: (d) ’Thus the native hue of resolution is sicklied over with pale cast of thought"

158

YCT

- Hamlet uses a metaphor of sickness to compare ’thought’ and ’resolution’. - Hamlet means that the decision to act is healthy, but thinking too much makes action sick or turns action to inaction. 79. Which of the following characters in The Tempest is associated with the Earth? (a) Ferdinand (b) Ariel (c) Caliban __________(d) Prospero __________ Ans: (c) Caliban son of the witch ’Sycorax’ is an important character in William Shakespeare’s play ’The Tempest’. Caliban both mirrors and contrast with Prospero’s other servant, Ariel. While Ariel is ”an airy spirit”, Caliban is the earth, his speeches turning to ’Springs brine pits’. 80. In which of the following dramas the hero is the villain as well? (a) Hamlet (b) King Lear (c) Macbeth __________(d) None of the above Ans: (c) The hero is the villain as well in Shakespeare’s tragedy ’Macbeth’ in character Macbeth. He was the Thane of Glamis and quickly the Thane of cawdor. - The significant lines from ’Macbeth’”A11 the perfumes of Arabia will not sweeten this little hand”. _______________________________________ 81. Who has said, ’’Indian Empire or no Indian Empire, we cannot do without Shakespeare”? (a) John Ruskin (b) Lord Tennyson (c) John Stuart Mill (d) Thomas Carlyle Ans: (d) Thomas Carlyle has said ’’Indian Empire or no Indian Empire, we cannot do without Shakespeare”. 82. Which character of Shakespeare disguises as Cesario? (a) Viola (b) Maria (c) Olivia ____________(d) Rosalind __________ Ans: (a) Viola is a character of Shakespeare’s comedy ’Twelfth Night’, who disguises as Cesario. Viola is the protagonist of the play. Washed up on the shore of Illyria when her ship is wrecked in a storm. Viola decides to make her own identity in the world. 83. The daughter of Polonius is: (a) Viola (b) Celia (c) Miranda __________(d) Ophelia___________ Ans: (d) Ophelia is a character in William Shakespeare’s drama ’Hamlet’, she is a young noble woman of Denmark, the daughter of Polonius, sister of Laertes and potential wife of Prince Hamlet, who due to Hamlet’s actions ends up in a state of madness that ultimately leads to her drowning. 84. Who survives at the end of Hamlet! (a) Laertes (b) Hamlet (c) Claudius (d) None of the above CGPSC Asst. Prof. Exam. 2019

Ans: (d) Horatio is distressed at the thought of being the last survivor and living in the play ’Hamlet’. Hamlet dies in Horatio’s arms proclaiming ’’The rest is silence”. 85. ’Antic disposition’ means: (a) Fantastic behaviour (b) Idiocy (c) Mysterious conduct (d) Childish department _____________________ Ans: (a) The phrase ’antic disposition’ is used in Shakespeare’s ’Hamlet’. It means fantastic behaviour, Hamlet intends to pretend he is losing his mind. Under the pretence of insanity, he knows he will have license to say and do unexpected and otherwise concerning things. 86. ’’Frailty thy name is woman.” Which of the following words most approximates to the meaning of ’frailty’? (a) Feebleness (b) Arrogance (c) Fashion __________(d) Beauty ___________ Ans: (a) ’’Frailty thy name is woman”. It has been taken from play of Shakespeare’s ’Hamlet’. - Frailty means ’Feebleness’ is a state of being very weak. Hamlet, in his first soliloquy recalls tender scenes between his mother, Queen Gertrude, and her deceased husband. 87. The Tempest is considered a classical play because of the following characteristic: (a) A tragic hero of eminence (b) Cathartic effect (c) Observance of dramatic unities (d) Tragic flaw in the hero ___________________ Ans: (c) ’The Tempest’ is considered a classical play because observance of dramatic unities. The classical unities, Aristotelian unities, or three unities represent a prescriptive theory of dramatic tragedy that was introduced in Italy in the 16th century. The three unities are1. Unity of action. 2. Unity of time. 3. Unity of place. 88. Who says, ’’Something is rotten in the state of Denmark”? (a) Hamlet (b) Marcellus (c) Horatio ___________(d) The Ghost ________ Ans: (b) ’’Something is rotten in the state of Denmark”, this line is spoken by Marcellus in Act I, Scene IV, as he and Horatio debate whether or not to follow Hamlet and the ghost into the dark night. 89. Which of the following plays of Shakespeare is not a romance? (a) A Mid-summer Night’s Dream (b) Cymbeline (c) The Winter’s Tale (d) The Tempest

159

YCT

Ans: (a) ’A Midsummer Night’s Dream’ is a comedy written by William Shakespeare. The play is set in Athens and consists of several subplots that revolve around the marriage of Theseus and Hippolyta. One subplot involves a conflict between four Athenian lovers. 90. ’’The rarer action is in virtue.” Who said this? (a) Oliver in As You Like It (b) Duncan in Macbeth (c) Ferdinand in The Tempest (d) None of the above Ans: (d) Prospero tells Ariel that ’’The rarer action is in virtue than in vengeance”, it means it is better to forgive than to hate one’s enemies. It has been taken from Shakespeare’s play ’The Tempest’. 91. Ophelia succumbs to death by: (a) consuming poison (b) a stab of dagger (c) jumping off the castle’s ramparts (d) drowning Ans: (d) Ophelia attempted to hang one of her garlands on a willow tree, but when the branch snapped, she consequently fell into the river and drowned. 92. Wordsworth became the poet laureate in the year: (a) 1800 (b) 1825 (c) 1843 (d) 1845 Ans: (c) Wordsworth became the poet laureate in the year 1843. Wordsworth was poet laureate from 1843 until his death from pleurisy on 23 April 1850. 93. Complete the sentence : ’’The sounding cataract ...........” (a) admired me with love (b) was treated by a doctor (c) made my heart dance (d) haunted me like a passion Ans: (d) This line has been taken from William Wordsworth ’s poem ’Tintem Abbey’. ’’The sounding cataract haunted me like a passion”. 94. ’’The appreciation of it (his verse) has been called the last reward of consummate scholarship.” Which author is implied in this remark? (a) Shakespeare (b) Milton (c) T. S. Eliot (d) Bacon Ans: (b) John Milton is implied in this remark ’’The appreciation of it (his verse) has been called the last reward of consummate scholarship” Qualified as mark pattison prescribes one may, with Raleigh find that Milton’s style is all substance and weight; that he is almost too packed to be read aloud, and go on to acclaim the top of his skill in choruses of Samson Agonisten. CGPSC Asst. Prof. Exam. 2019

95. Complete the following line: ”God doth not need .........” (a) dark world and wide (b) my true account (c) only stand and wait (d) either man’s work or his own gifts Ans: (d) ’God doth not need either man’s work or his own gifts’. This line has been taken from John Milton sonnet ’When I Consider How My Light is Spent’ also known as ”On His Blindness”. 96. ’An Essay on Man’ by Pope is written in: (a) Personal style (b) Farce (c) Heroic couplet (d) Epic form Ans: (c) ’An Essay on Man’, philosophical essay written in heroic couplets of iambic pentameter by Alexander Pope. It was conceived as part of larger work that pope never completed the poem consists of four epistles. - ’An Essay on Man’ is taken from Epistle II. 97. Which of the following poems invokes goddess Melancholy? (a) IlPenseroso (b) The Vanity of Human Wishes (c) Ulysses (d) Andrea del Sarto Ans: (a) ”11 Penseroso” invokes the goddess Melancholy and describes the satisfactions of solitude, music, epic poetry, tragic drama and the meditative life in general. But in contrast to its companion poem ”L allegro” which celebrates mirth, the beauties of rural scenery and urban vitality. 98. In 1763, Johnson befriended: (a) J. S. Mill (b) James Boswell (c) Joshua Reynolds (d) None of the above Ans: (b) In 1763 Johnson befriended James Boswell, a Scottish lawyer whose ’Life of Johnson’ contributed to Johnson’s fame and reputation when it was published after Johnson’s death. - James Boswell’s ’The Life of Samuel Johnson’ published in 1791. 99. ’’One truth is clear, ’whatever is, is right’.” The source of this line is: (a) An Essay on Man (b) On His Blindness (c) The Vanity of Human Wishes (d) Rabbi Ben Ezra Ans: (a) ’One truth is clear, whatever is right”, this line is taken from Alexander Pope’s poem, ”An Essay On Man”, epistle I. 100. "The Vanity of Human Wishes" is-. (a) an interlude (b) a short novel (c) an epic (d) None of the above Ans: (d) ’The Vanity of Human Wishes’ : The Tenth Satire of Juvenal Imitated is a poem by the English author Samuel Johnson. It was written in late 1748 and published in 1749.

160

YCT

Chhattisgarh Public Service Commission Asst. Prof. Exam. 2017

ENGLISH (Solved Paper)

[Exam. Date : 26 March 2017]

1.

Choose an option that will complete the 3. ’’noble diction”; 4. ’’dignified and elevated composition.” statement correctly: Identify the missing fifth element from among Aristotle, in Poetics contrasts lyric poetry the options given below: with ......... (a) transcend the limits of the sublime (a) ballads and odes (b) the due formation of figures (b) drama and odes (c) emulation of previous great poets (c) drama and epic poetry (d) polyptota and periphrasis (d) discourse and drama _____________________ (e) timidity of bombast of language ____________ Ans: (c) Lyric poetry refers to either poetry that has Ans: (b) Longinus defines sublime as a kind of the form and musical quality of a song, or a usually loftiness and excellence in language raising the style short poem that expresses personal feelings which of the ordinary language. Sublimity springs from a may or may not be set to music. Aristotle in poetics great and lofty soul, thereby becoming "one echo of a contrast lyric poetry with drama and epic poetry. An great soul". It should not only be distinct and excellent example would be a poem that expresses feelings and in composition but also move the readers along with may be a song that could be perform to an audience. the effects of pleasure and persuasion Longinus 2. Which one of the following features is not identifies five element of the sublime. mentioned by Aristotle vis-a-vis the proper 1. Power of forming great conceptions. structure2. Vehement and inspired passion. fa) An imitation of an action that is complete, and 3. The due formation and use of figures of speech. whole, and of a certain magnitude in language 4. Noble diction. embellished with each kind of artistic ornament 5. Dignified and elevated composition. _____________ (b) The sequence of events, according to the law of 4. Between World War I and II many American probability or necessity, will admit of a change writers lived either permanently or temporarily from bad fortune to good or from good fortune Dubbed ’high modernists’, these writers to bad. (Choose the option that completes this MOST (c) A whole is that which has a beginning, a truthfully and appropriately) middle, and an end. (a) were still rooted in the landscape of America (d) It must not only have an orderly arrangement and spread American values abroad. of parts, but must also be of a certain (b) managed to protect the purity of American magnitude for beauty depends on magnitude culture and rejected the influences of Europe. and order. (c) ’came of age’ and brought American literature (e) "Unity of plot does, as some persons think, to the notice of literary critics. ________consist in the Unity of the hero."____________ (d) managed through their aesthetic movement to Ans: (e) "Unity of plot does as some persons think, bring together American and European culture. consist in the Unity of the hero”, this feature is not (e) rejected American values, traditions and values mentioned by Aristotle vis-a-vis (in relation to) the ________and adopted European ones. _______________ proper structure. Ans: (d) High modernism, the predominant artistic ’Tragedy’, says Aristotle is an imitation of an action movement in the early 20th century, was greatly that is serious complete and of a certain magnitude; in influenced by racial transformations of the the language embellished with each kind of artistic understanding of time. Between World War I and II, ornament, the several kinds being found in separate many American writer lived either permanently or parts of the play : in the form of action, not of temporarily Dubbed ’high modernists’, these writers narrative; through pity and fear effecting the proper managed through their aesthetic movement to bring purgation Catharsis of these and similar emotion. _____ together American and European culture. 3. Longinus identifies five elements of the sublime; 5. The Hairy Ape has groups of people chattering four are given here: in many scenes. It helps the audience gain a 1. ’’the power of forming great conceptions”; serene milieu. These ’voices’ also enabled 2. ’’vehement and inspired passion”; O’Neil to: CGPSC Asst. Prof. Exam. 2017

161

YCT

(Identify the option that is not true, as per your 8. Homer described his Heroes men of great understanding of the play): appetites, lovers of beef broiled upon the coals, (a) usher in realistic audible elements along with and contrary to the practice of the French visual props, to be exploited later. Romances, whose Heroes neither eat, nor drink, (b) cameo Yank to stand out aurally and nor sleep, for love. physically, against the voices, revealing Crites makes this observation in An Essay of his Dramatic Poesy to question the view that: (c) introduce the choric element into his play to (a) French romances are better than Greek tales as make it approximate a classic drama the former are more imaginative (d) reflect the polyphonic universe and explore the (b) The French conceived of their heroes tragedy of a forgotten voice, besi to belong differently reflecting their milieu and value (e) make a cynical statement about and ’reject’ the (c) The Ancients perfected the art of composition ________gibberish that we are exposed to ....................... while the Modems only imitated t Ans: (e) ’The Hairy Ape’ is a 1922 expressionist play (d) The Modems have not acquired a new by American playwright Eugene O’Neil. It is about a perfection in writing, but only altered the beasty, unthinking labourer known as yank, the (e) The Modems have enfeebled their heroes and protagonist of the play, as he searches for a sense of _______put them on a diet._______________________ belonging in a world controlled by the rich. The Hairy Ans: (d) In Dryden ’An Essay of Dramatic Poesy’ Ape has groups of people chattering in many scenes. 1668, the four persons who canyon the dialogue are It helps the audience to make a cynical statement Eugenius, Crites, Lisideius and Neander. Malone, the about and ’reject’ the gibberish that we are exposed to. first critical editor of Dryden, pointed out that 6. The English dramatic poets have Eugenius represents the Earl of Dorset, Lisideius, Sir Shakespearized now for two hundred years. Charles Sedley, and Neander, Dryden himself. These This declaration of Emerson substantiates his identification have been accepted by all succeeding thesis that . editors of Dryden; that of Eugenius is supported by (a) artists have long listened ’too long to the the authority of Priar, that of Neander by an elegy on courtly muses of Europe’. Dryden by Mrs. Elizabeth Thomas and that of (b) critics have made Shakespeare the ’canon’ and Lisideius by an obvious anagram from sidleius. Crites stifled creativity. makes this observation in Essay of Dramatic Poesy to (c) the eyes of man are set in his forehead, not in question the view that the modems have not acquired a his ’hind-head’. new perfection in writing but only altered the imitation. (d) instead of Man Thinking, we have the 9. Whom does Dryden focus in these lines? bookworm. If there was any fault in his Language, was that (e) the (creative) genius, too, can suffer from the he weaved it too closely and laboriously in his _______undue influence of books. _________________ comedies, especially; perhaps too, he did a little Ans: (e) In ’The American Scholar’ Emerson declared too much Romanize our Tongue, leaving the ’’The English dramatic poets have Shakespearized words which he translated almost as much Latin now for two hundred years”. But the genius, too, can as he found them: wherein though he learnedly suffer from the undue influence of books. Emerson, followed the Idiom of their language, he did not example of his kind of sufferer are the English enough comply with the idioms of ours. dramatic poets, who he says have been (a) Shakespeare (b) Milton ’’Shakespearized” for two hundred years. Rather than (c) Beaumont and Fletcher (d) Ben Jonson producing new, original texts and thoughts, they (e) Marlowe _____________________________ mimic Shakespeare's writing. _____________________ Ans: (d) These lines - "If there was any fault in his 7. What is the key formal characteristic of high Language, was that he weaved it too closely and modernist works? laboriously in his comedies, especially; perhaps too, (a) Fragmentation he did a little too much Romanize our Tongue, leaving (b) Continuity the words which he translated almost as much Latin as (c) Conformity he found them: wherein though he learnedly followed (d) Reverence for tradition the Idioms of their language, he did not enough (e) Abstractions ___________________________ comply with the idioms of ours” appears in John Ans: (a) Modernism was a cultural wave that Dryden’s work ’An Essay of Dramatic Poesy (1668). originated in Europe and swept the United State John Dryden focuses Ben Jonson in these lines. He during the early 20th century. Modernism impacted writes if I would compare him (Ben Jonson) with music, art and literature by radically undoing Shakespeare, I must acknowledge him the more traditional forms, expressing a sense of modem life is correct poet but Shakespeare the greater wit. a sharp break from the past and its rigid convention. Shakespeare was the Homer or father of our dramatic Fragmentation is the key formal characteristic of high poet; Jonson was the Virgil, the pattern of elaborate modernist work. Fragmentation in modernist literature writing, I admire him but I love Shakespeare. is thematic as well as formal. CGPSC Asst. Prof. Exam. 2017

162

YCT

(d) capturing certain moments or self or others, Name the critic who said this of Shakespeare: reflecting upon them in tranquility, Those who accuse him to have wanted learning, give him the greater commendation: he was ________spontaneously, using the ballad-forms._______ naturally learned, he needed not the spectacles of Ans: (c) Traditionally, lyric poetry which express the books to read nature; he looked inwards, and feelings of the poet and ballads which tell a story found her there. I cannot say he is everywhere distinct forms. Considered against this Lyrical Ballads alike; were he so, I should do him injury to could be an Oxymoron. But it is not even while telling the story of the self or an outcast. The composition of compare him with the greatest of mankind. poetry ballads were lyrical._______________________ (a) Dr. Johnson (b) Dryden (c) T.S. Eliot (d) Hazlitt 13. Who speaks the following line in Hamlet! (e) Coleridge______________________________ Something is rotten in the state of Denmark. (a) Claudio (b) Hamlet Ans: (b) John Dryden said this of Shakespeare in his (c) Marcellus (d) Polonius work ’An Essay of Dramatic Poesy (1668). To begin (e) Horatio _______________________________ then with Shakespeare, he was the man who of all Ans: (c) This line ’’Something is rotten in the state of modern and perhaps ancient poets had the largest and Denmark” appears in ’Hamlet’ in (Act I Scene IV) most comprehensive soul. All the images of nature spoken by Marcellus to Horatio. This line spoken by were still present to him, and he drew them not Marcellus (and not Hamlet as is commonly believed) laboriously but luckily, when he describes anything is one of the most recognizable lines in all of you mare than see it, you feel it too. Those who Shakespeare’s work. ____________________________ accuse him to have wanted learning, give him the greater commendation: he was naturally learned, he 14. Identify the critic who posited this view: The levity of Hamlet, his repetition of phrase, his needed not the spectacles of books to read nature; he puns, are not part of a deliberate of dissimulation, looked inwards, and found her there. I cannot say he is but a form of emotional relief. everywhere alike; were he so, I cannot say he is (a) Coleridge (b) Bradley everywhere alike; were he so, I should do him injury (c) Hazlitt (d) Charles Lamb to compare him with the greatest of mankind. (e) Eliot _________________________________ 11. Thoreau’s Civil Disobedience calls for . Ans: (e) T.S. Eliot posited this view in his critical (a) a better government, in which the elected essay ’’Hamlet and His Problems”. ’’The levity of majority rules Hamlet, his repetition of phrase, his puns, are not part (b) the dissolution of governments that ignore of a deliberate plan of dissimulation, but a form of individuals emotional relief. In the character Hamlet it is the (c) governance that has minimal influence on the buffoonery of an emotion which can find no outlet in rights of people action; in the dramatist it is the buffoonery of an (d) an obligation to abide by the decisions of the emotion which he cannot express in art.” majority in a democracy 15. Identify the element from the options given (e) a conscientious government guaranteeing below that makes Hamlet superior to ________individual liberty________________________ Elizabethan revenge plays: Ans: (c) Resistance to Civil Government, called ’Civil (a) insanity or feigned insanity (b) violent death of hero Disobedience’ for short, is an essay by American (c) apparitions encouraging vengeance transcendentalist Henry David Thoreau, that was first (d) awareness of the moral implications of revenge published in 1849. Thoreau’s ’Civil Disobedience’ (e) the use of a number of soliloquies __________ calls for governance that his minimal influence on the Ans: (d) The Tragedy of Hamlet, Prince of Denemark, rights of people. Opening line of this essay ”1 Heartily often shortened to Hamlet, is a tragedy written by accept the motto, ’’that government is best which William Shakespeare sometime between 1599 and governs least”, and I should like to see it ” 1601, first performed in 1609. Awareness of the moral 12. Choose the most meaningful explanation: Traditionally, lyric poetry which expresses the implications of revenge is the element that makes feelings of the poet and ballads which tell a story Hamlet superior to other Elizabethan revenge plays. distinct forms. Considered against this Lyrical 16. The OED defines a sonnet, as follows: "A piece of verse (properly expressive of one main Ballads could be an oxymoron. But it is not idea) consisting of fourteen decasyllabic lines with (a) Coleridge and Wordsworth, who were great rhymes arranged according to one or other of Romantic poets, used it only as a title of the certain definite schemes." poem. The ’decasyllabic lines’, generally, employed (b) romantic poets captured certain moments and reflected the feelings of person at (a) an iambic meter (b) a trochaic meter (c) even while telling the story of the ’self or an (c) a spondaic meter (d) an anapestic meter ’outcast’, the composition of poetry ballads (e) a dactylic meter were lyrical. 163 YCT CGPSC Asst. Prof. Exam. 2017 10.

Ans: (a) ’Decasyllabic lines’ means - A line of verse 19. Study the lines from Milton’s sonnet addressed to the nightingale: having ten syllables. Decasyllable is a poetic meter of Now timely sing, ere the rude bird of hate 10 syllables used in poetic traditions of syllabic verse. Foretell my hopeless doom in some grove nigh In languages with a stress accent, it is the equivalent What is the ’rude bird of hate’ referred to here? of pentameter iambic pentameter. (a) Nightingale (b) Owl 17. A crucial aspect of Milton’s divergence from the (c) Sparrow (d) Cuckooo Petrarchan sonnet tradition concerns not on (e) Raven ________________________________ employed or the ’occasional’ nature of his Ans: (d) ’O Nightingale’ is also known as Milton’s sonnets, but also . (a) the presence of a slight pause at the end of the sonnet 1. It presents a contrast between two birds, a nightingale and a cuckoo. The theme of the poem is fourth line (b) how the thought rhythm of the octave carried the contrast and strife between love and hate, where love succeeds because it has more strength than hate. on into the sestet without any break (c) the rejection of the idyllic love-poetry Here rude bird of hate referred to cuckoo. traditions of the Elizabethans 20. attempts to ridicule prevailing vices (d) the rejection of the three quatrains and the or follies in society; a , describes the couplet scheme of Shakespeare appearance and character of a particular person (e) the presence of Miltonic similes and images or class of persons. (a) parody; satire (b) satire; lampoon Ans: (c) A crucial aspect of Milton’s divergence from (c) sarcasm; satire (d) irony; satire the Petrarchan sonnet tradition concerns not only he (e) satire; scorn ____________________________ employed or the occasional nature of his sonnet but also the rejection of idyllic love poetry traditions of Ans: (b) Satire attempts to ridicule prevailing vices or the Elizabethan. During Milton’s time England was follies in society; a lampoon describes the appearance changing in 1630s and 1640s was the polarization of and character of a particular person or class of political opinions between those who supported the persons. King Charles I and the puritan parliament led by 21. Identify a work that is not satirical: Oliver Cromwell. A group of monarchist collectively (a) Gulliver’s Travels known as the Cavalier poet, Carew Harrick. (b) Pilgrim’s Progress Lovelace and suckling, Cavalier poetry was flourish (c) The Animal Farm of lyricism but Milton was apart. (d) Utopia (e) The Rape of the Lock ____________________ 18. Dr. Johnson is said to have observed, ’’Milton .... was a genius that could cut a Colossus from Ans: (b) ’The Pilgrim’s Progress’ is not a satirical a rock; but could not carve heads upon cherry- work. It is a religious allegory by the English writer stones.” John Bunyan, published in two parts in 1678 and Which one of the following statements about his 1684. The work is a symbolic vision of the good sonnets contradicts Dr. Johnson’s view? man’s pilgrimage through life. At one time second (a) His personal sonnets have great and solemn only to the Bible in popularity, the Pilgrim’s progress beauty, revealing a great personal spon taneous is the most famous Christian allegory still in print. thought of qedonal life Which one of the following statements (b) In his hands the sonnet was a trumpet, blowing misrepresents the tone and tenor of The Red soul-animating strains (Wordsworth’s Scorn Badge of Courage not the sonnet) (a) The novel deromanticizes the popular notions (c) His deep religious patience is reflected in of courage and manliness associate. ’’They also serve who only stand and wait” (b) The novel captures the real and the metaphoric when I consider how my light is spent. haze - the confusion, the uncertainty (d) All that short compositions can commonly battlefronts and engulf soldiers. attain is neatness and elegance ........... ”he was (c) The novel re-establishes the notion that ’Dulce a ’lion’ that had no skill in dandling the Kid.” et decorum est pro patria mori ......... for one’s (Dr. Johnson). nation. (e) In his lines the old dialect of the sonnet, its (d) The discrepancies between Henry’s thoughts sentiments, conceits, and familiar images and those of the author are mostly involved manner took over. (e) The novel traces the process by which a raw Ans: (b) William Wordsworth’s statement in ’Scorn youth metamorphoses into a tried and succeed. not thee sonnet’ that in Milton’s hand the sonnet was a Ans: (c) ’The Red Badge of Courage’ is a war novel trumpet, blowing soul-animating strains is contradicts by American author Stephen Crane. Taking place to Dr. Johnson’s view that ’’Milton was a genius that during the American civil war, the story is about a could cut a Colossus from a rock, but could not curve young private of the union Army, Henry Fleming, heads upon cherry stone. who flees from the field of battle. The statement - The CGPSC Asst. Prof. Exam. 2017

164

YCT

British imperialism, before its liberation. In the novel novel re-establishes the notion that ’Dulce et decorum At the English Club at Chandrapore where no Indians est pro patria mori -means - It is sweet and fitting to are allowed, it is suggested by Fielding, the principal die for the home land’ misrepresents the tone and of the local government college, that in order to see tenor of ’The Red Badge of Courage’. 23. Which epic convention that Milton followed in real India, one has to ’’try seeing Indian” Fielding is sympathetic to Indians his notion is that the English Paradise Lost is missing in this list? Noble theme, invocation to Muse, deployment of should try seeing Indians as individual instead of as a heroic figures, beginning in the middle, elevated homogenous. style and descriptions, impressive catalogues, war 26. A Passage to India explores various issues which render the events in the novel tragic, under the like speeches, supernatural machinery and use of super natural and allusions. characteristic modern tragedy. Identify the (a) unity of action (b) unity of time critical issue that makes the tragedy, especially (c) unity of place (d) uniform style for Aziz and Fielding’s conversation in the novel (e) unfamiliar dialect ____________________ (a) Religion and faith (b) Colonialism Ans: (a) ’Unity of action’ is missing in epic (c) Racial issues convention that Milton followed in ’Paradise Lost’. Unity of action refers the combination of incidents (d) Limits of comprehension which are the action of the play/poem should be one. (e) Hearing the ”ou-boum” ___________________ The unity of actions limits the supposed action to a Ans: (d) ’Limits of Comprehension’ is the critical single set of incidents which are related as cause and issue that makes the tragedy especially for Aziz and effect having a beginning, middle and the end. No Fielding’s friendship and conversation in the novel. scene is to be included that does not advance the plot Aziz relies heavily on intuition over logic, and he is directly. The ’Paradise Lost’ denotes the action in more emotional than his best friend Fielding. His heaven, hell and earth. __________________________ chief drawback is inability to view a situation without 24. Which one of the following is NOT an elegy (or emotion which Forester suggests is a typical Indian an enology) of a poet? difficulty. (a) Adonais 27. What does Lawrence call ’’the tragedy of (b) Elegy Written In A Country Churchyard thousands of young men in England” vis-a-vis (c) ”0 Captain! My Captain” Sons and Lover (d) In Memoriam A.H.H. (Tennyson) (a) Paul giving his mother an over-dose of (e) Lycidas _______________________________ morphine. Ans: (c) ” 0 Captain! My Captain” is an extended (b) Paul’s ’mother-complex’. metaphor poem written in 1865 by Walt Whitman (c) Paul’s inability to settle down with Miriam. about the death of American President ’Abraham (d) Paul’s inability to prioritize his filial values. Lincoln’. Whether ’Adonais’ is a pastoral elegy on the (e) Paul’s unjustifiable condescending attitude death of John Keats, ’Lycidas’ on the death of Edward ________towards Miriam. ________________________ King, ’In Memoriam’ on the death of Arthur Henry Ans: (b) ’Sons and Lovers’ (1913) is generally Hallam. ’O Captain! My Captain’ is ceremonial poem considered Lawrence’s autobiographical, indeed uttered in a public voice on a public occasion. The ’Oedipal’ novel, in which he describes his youth, poem is lyric genre. dwelling on his relationship with his mother and the 25. In A Passage to India, Forster uses Fielding, an problematic bond she cultivated between herself and English man, to say, ’’Try seeing Indians.” her son. Lawrence started work with the trauma of his Why ..... mother’s death still with him. In late 1912 he finally (a) The English are blind to Indians, as they tend finished the novel and changed the title from Paul to look through them. Morel to the far more significant ’Sons and Lovers’. (b) The English do not distinguish between By then he was well aware that he had written a quasieducated and uneducated Indians. (c) The English need to observe the behavior and autobiographical novel which would have deep and disturbing personal significant for a great many customs of Indians. (d) The English often see Indians but neither people. ”It is the tragedy of thousands of young men in England” he wrote to Edward Garnett. respect nor understand them. 28. Lawrence wanted the episodic narrative of Sons (e) The English should try seeing Indians as and Lovers to be ’organic and open to experience ________individuals instead of as a homogenous ______ different episodes, the three central women Ans: (e) E.M. Forster’s ’A Passage to India’ (1924) react differently to flowers. This shows starts with a rather abominable description of (a) that Lawrence narrated ’felt life’ as it happened Chandrapore. He writes: ’’The very wood seems made (b) that Lawrence carefully organized his episodes of mud, the inhabitants of mud moving. So abased, so (c) that Lawrence imposed an extrinsic structure monotonous is everything that meets the eye...” ’A through realistic events Passage to India’ portrays a colonial India under 165 YCT CGPSC Asst. Prof. Exam. 2017

(d) that Lawrence relied on an underlying pattern to substitute inorganic form (e) that Lawrence innocently employed the ________episodic structure _______________________ Ans: (d) Flowers symbolize femininity and female sexuality in ’Sons and Lovers’ women are referred to as flowers or compared with flower throughout the novel. When William describe, his many female admires to Paul, he describes then as different flowers that live ’’like cut bloom in his heart.” This shows that Lawrence relied on an underlying pattern to substitute inorganic form. 29. Who makes this remark and where is it made? Man is .... Metamorphosed into a thing into many things. (a) Thoreau in ’Civil Disobedience’ (b) Forster in ’A Passage to India’ (c) Emerson in ’The American Scholar’ (d) Ovid in ’Metamorphoses’ (e) Longinus in ’On the Sublime’ ______________ Ans: (c) The American Scholar is a lecture by Ralph Waldo Emerson into an essay. The occasion for the lecture was an address that Emerson gore to the Phi Beta Kappa society at Cambridge, on August 1837. This remark - ’’Man is thus metamorphosed into a thing, into many things. The planter, who is man ........” is made by Emerson in ’The American Scholar’. 30. Choose the best option: (A) Who wrote these lines? (B) Who is the poet referred to in these lines? "By mourning tongues The death of the poet was kept from his poems." (B) Yeats (a) (A) Auden (B) Freud (b) (A) Auden (B) King (c) (A) Milton (B) Yeats (d) (A) Owen (B) Auden (e) (A) Yeats Ans: (a) The lines - ”By mourning tongues, the death of the poet was kept from his poems” appears in ’In Memory of W.B. Yeats’ by W.H. Auden. ’In Memory of W.B. Yeats’ is an elegy written by W.H Auden on the death of W.B. Yeats, whom Auden admired and also deeply influenced by him. ____________________ 31. What impelled Milton the Puritan to employ the 'masque* and compose Comas'! (a) To re-usher in ’masquing’ into a Puritan society. (b) To emphasize the virtue (of an individual) over debauchery (in the court). (c) To showcase his artistic abilities as a dramatist. (d) To celebrate the elevation of an earl to a lord. (e) To prove that his genius included this skill too. Ans: (b) Milton was puritan. The puritans were English protestants in the 16th and 17th century, who sought to purify the Church of England of Roman CGPSC Asst. Prof. Exam. 2017

Catholic Practices. Comus, masque by John Milton, presented on Sept. 29, 1634 before John Egerton, earl of Bridgewater, at Ludlow Castle in Shrophire, and published anonymously in 1637. Comus is a masque against ’’masquing” contrasting a private. 32. Critics consider The Hairy Ape to be an exemplar of: (a) Absurdist drama (b) Agitprop theatre play (c) Expressionist drama (d) Revenge play (e) Morality play __________________________ Ans: (c) ’The Hairy Ape’, drama in eight scene by Eugene O’ Neill, produced in 1922 and published the following year. It is considered one of the prime achievements of expressionism on stage. Expressionism, artistic style in which the artist seek to depict not objective reality but rather than subjective emotion and responses that object and events arouse within a person. 33. Modern tragicomedy is something used synonymously with , which suggest that the laughter is the only response left to man when he is faced with the tragic emptiness and meaninglessness of existence. (a) expressionist drama (b) absurdist drama (c) realistic drama (d) epic theatre (e) social theater ___________________________ Ans: (b) Modem Tragicomedy is sometimes used synonymously with absurdist drama, which suggest that laughter is the only response left to man when he is faced with the tragic emptiness and meaningless of existence. 34. The genre ’Comedy of Manners’ owes its advent (coming into being and development) to (a) the desire to depict and satirize the manners and affectations of the contemporary (b) decadence and cynicism (material prosperity and moral latitude) of the Restoration period (c) the need to reform society during the Reformation by Cromwell and his follower (d) the urge of authors to impress their own coterie or social class. (e) failure of Shakespearean romantic and ’dark’ ________comedies ______________________________ Ans: (b) The genre ’Comedy of Manners’ owes its advent to decadence and cynicism of the restoration period. Comedy of manners, with cerebral form of dramatic comedy that depicts and often satirizes the manners and affections of a contemporary society. 35. Which one of the following 19th century American scholars was not a transcendentalist? (a) Amos Bronson Alcott (b) Henry Thoreau (c) Ralph Waldo Emerson (d) Walt Whitman (e) Henry James

166

YCT

C. man’s pursuit of happiness in this world Ans: (d) Transcendentalism, an idealist philosophical D. the nature of man and with his place in the tendency among writers in and around Baston in the Universe mid 19th century. Growing out of Christian Identify the option that delineates the proper Unitarianism in the 1830s under the influence of sequence: German and British romanticism transcendentalism (a) D,C,B,A (b) D,B,A,C affirmed Kant’s principle of intuitive knowledge not (c) D,A,B,C (d) D,A,C,B derived from the sense, while rejecting organized (d) D,C,A,B __________ religion for an extremely individualistic celebration of Ans: (b) Pope’s ’Essay on Man’ consist of four epistle. the divinity in each human being. Walt Whitman was Correct order is as followingnot associated with this group. R.W. Emerson, D. The nature of man and with his place in the Bronson Alcott, F.H. Hedge, Margret Fuller, George Universe. Ripley, Nathaniel Hawthorne, Henry Thoreau were associated with transcendentalism.__________________ B. Man is an individual. 36. The One-Act Play has a beginning, a middle and A. Man in relation to human society and to the political and social hierarchies. an end and is divided into Exposition, Conflict, C. Man's pursuit of happiness in this world. _________ the climax and the Denouement. Given below are some other features of this 39. Johnson’s The Vanity of Human Wishes is simply genre. Identify the ONE that does not define pessimistic, straining against optimism, against optimism, against the possibility that human life this. (a) limited characters. could get better. It . (b) observance of the unities. (a) argues against the philosophy in Pope’s Essay (c) implies, and not explains, meaning. on Man (d) elaborate stage directions.__________________ (b) depicts the vanity of human wishes as the fountain of political hope Ans: (c) A one act play is a play that has only one act, (c) employs the literary mode of oblique allusion as distinct from plays, that occur over several acts. One to reflect on the British experience act plays may consist of one or more scene. Long (d) shows a deep concern with the processes of dialogue and soliloquies are not the feature of this genre. history 37. Pope’s Essay on Man is neither an essay nor is it (e) examines the way in which all kinds of dreams about Man or even ’man’. ________ and wishes and ambitions fill man ___________ Identify its true nature and aim in the following Ans: (d) ’The Vanity of Human Wishes' is 370 lines options. (a) to show man as the measure of things and in heroic couplet. Johnson had a temperamental affinity with Juvenal's gravity and moral sense, his celebrate his free-will and intellect. (b) to reiterate Leibnizian philosophy that God is remorseless pessimism and Roman stoicism. Vanity looks back on centuries of human struggle. On beneficent and that, in creating the........ (c) to reject the Divine plan in its entirety and to illusory hopes and inevitable fate, on sudden shifts in judge by isolated parts as its laws ........ the political power and the cruel indignity of death. It portrays disillusion from the disgraced Cardinal Wolsey universe (d) to lecture on man, manners and moral in the and the murdered Thomas Wentworth to the young student burning for renown and the poor scholar crushed true Neo-Classical spirit of the England. (e) an attempt to justify the ways of God to Man, and by "Toil, Envy, want the Garret and the Tail." It shows a a warning that man himself is not, as, in his pride, deep concern with the process of history. 40. Identity the poet and the poem. ________he seems to believe, the center of all things. "Our birth is but a sleep and a forgetting:/ The Ans: (e) ’The Essay on Man’ is a philosophical poem, soul that rises with us our life's Star / Hath had written characteristically, in heroic couplets and else where its setting". published between 1732 and 1734. Pope intended it as (a) Wordsworth; Tiintern Abbey the centre piece of a proposed system of ethics to be (b) Wordsworth; Ode on Intimations of put forth in poetic form. It is in fact a fragment of Immortality larger work which pope planned but did live to (c) Shakespeare; The Tempest complete. It is an attempt to justify, as Milton had (d) Coleridge; Kubla Khan attempted to vindicate, the ways of God to man, and a (e) Milton; Paradise Lost _____________________ warning that man himself is not, as in his pride, he Ans: (b) The lines "Our birth is but a sleep and a seems to believe, the centre of all thing. forgetting, The soul that rises with us our life’s 38. Pope’s Essay on Man consists of four epistles; they are summed up below, marked A, B, C and Star/Hath had elsewhere its setting" appears in "Ode on Intimations of Immortality from recollection of D jumbled order below. Early Childhood" is a poem by William Wordsworth A. man in relation to human society and to the completed in 1804 and published in poems in two political and social hierarchies volume (1807). B. man as an individual 167 YCT CGPSC Asst. Prof. Exam. 2017

41. In the options below, which ONE does not delineate the special features of Ode on Intimation of Immortality. (a) When Wordsworth arranged his poems for publication, he placed this Ode at the end as if he regarded it as the crown of his creative life. (b) The Ode is in three parts: a crisis, an explanation, and a consolation; in all three parts Wordsworth speaks of what is most important and most original in his poetry. (c) The Ode’s unusual form is matched by its unusual language: a stately metrical form is matched by a stately use of words. (d) Wordsworth seems to have decided that his subject was so important that it must be treated an usual manner and for it he adopted a classical ’style’. (e) Wordsworth in this Ode discusses the idea of immortality not in mortal, but in spiritual. Ans: (d) Option (d) does not delineate the special feature of ’Ode on Intimation of Immortality”. Correct feature - Wordsworth seems to have decided that his subject was so important that it must be treated in what was for him an unusual manner, and for it he fashioned his own style. 42. Which one of the following statements is untrue or unacceptable about the Custom House in the novel ’The Scarlet Letter’. (a) Hawthorne introduces in the preface the manuscript that is purported to be the or (b) Hawthorne describes in the preface how he discovers the scarlet letter and ’burns (c) Hawthorne uses the preface to explain the circumstances under which Hester co (d) Hawthorne uses the preface to frame his tale, strike the key note and set up expe (e) Hawthorne introduces in the preface the scarlet ________letter as a magical artifact, intert ....................... Ans: (b) Hawthorne describes in the preface how he discovers that scarlet letter and bums is not acceptable about the Custom House in the novel ’The Scarlet Letter’. The primary purpose for the CustomHouse introduction was to give the reader a better understanding of the people living in the Custom House. 43. Identify the person Caliban refers to in this line. What a pied ninny’s thisl-Thou scurvy patch!(a) Alonso (b) Antonio (c) Gonzalo (d) Trinculo (e) Stephano ______________________________ Ans: (d) This line ’’What a pied ninny’s this! Thous scurvy patch!” appear in ’Tempest’ by William Shakespeare, in Act III scene II, spoken by Caliban. Here in this speech Thou refers - Trinculo. Caliban says ’’where the quick freshest are” Stephano says Trinculo, run into no further danger, makes it clear that ’Thou’ refers ’Trinculo’ who is Alonso’s Jester. CGPSC Asst. Prof. Exam. 2017

44.

’The Masque’ (a pageant, actually) in The Tempest differs from typical masques as it

(a) is performed before a king and his court and as the occasion, appropriately, is an (b) provides a bridge transporting the audience from magic to reality (c) gives a momentary vision of a world without problems, immediacy, and drama. (d) gives winter no role because of its associations with abduction and rape. (e) conjures up goddesses and reapers to shower ________eternal spring on the couple in wed................... Ans: (b) ’The Masque’ (a pageant, actually) in ’The Tempest’ differs from typical masques as it provides a bridge transporting the audience from magic to reality. ’The Tempest’ is a play by English playwright William Shakespeare probably written in 1610-1611 and considered as one of the last plays that Shakespeare wrote alone. 45. Identify in the features listed as options, the One feature that does not define Romanticism: (a) An emphasis on a rational and scientific way of understanding the world. (b) An exploration of the relationship between nature and human life. (c) A belief in the power of the imagination, including an interest in mythological, f. themes. (d) An emphasis on the imagination as a positive and creative faculty. (e) A desire to understand what influences the ________human mind. ___________________________ Ans: (a) Romanticism, attitude or intellectual orientation that characterized many works of literature, painting, music in Western civilization over a period from the late 18th to the mid 19th century. Romanticism can be seen as a rejection of precepts of order, calm, harmony, balance, idealization and rationality that typified classicism in general and late 18th century Neo Classicism in particular, An emphasis on rational and scientific way of understanding the world is the characteristic of NeoClassicism. ___________________________________ 46. Explain the significance of these lines from The Shield of Achilles-. As three pale figures were led forth and bound To three posts driven upright in the ground. (a) Auden is able to see reflected in the shield of Achilles the scenes of totalitarianism. (b) The anachronistic image on Achilles’s shield alludes to Crucifixion and renders t (c) Auden deliberately attempts to weave into a Homeric episode future events -wh (d) The images in the lines are not as crucial as the attitude of the spectators and the (e) Thetis sees a scene that seems more like one from the Second World War: barbed 168 YCT

Ans: (b) ’The Shield of Achilles’ is a poem by W.H. Ans: (c) ’Quibble’ in archaic means - a play on words; Auden, first published in 1952 and the tile work of a a pun. Hamlet speaks this line in Gravedigger scene collection of poem by Auden, published in 1955. Act V scene I. Hamlet: They are sheep and calves which seek out These lines are significant that represents the assurance in that. anachronistic image on Achilles’s shield alludes to I will speak to this fellow - whose grave's this, sirrah? crucifixion. 47. One of the features given as option is not true of Gravedigger: Mine, sir ’Modern poetry’ (associated with The Waste (Sings) Land by T.S. Eliot). Oh, a pit of clay for to be made (a) experiments in form and style For such a guest is meet (b) new themes and word-games Hamlet: (c) new modes of expression I think it be thine, for thou liest in it. (d) poets’ own definition of self and placement in In ’liest’ is pun. society Liest - to say or write something that you know is not (e) simple and open-ended nature of meaning true Ans: (e) ’Modernist poetry’ refers to poetry mainly in Lying - to be or to stay at rest in a horizontal position. Europe and North America between 1890 and 1950 in 51. In the Twelfth Night, Malvolio reads aloud, the tradition of modernist literature. while alone, Maria’s forged letter; he is utterly ’Simple and open-ended nature of meaning’ is not the ignore other characters and audience who would features of ’Modernist poetry’. actually engage in some kind of ’’dialogue” with 48. Catharsis is usually associated with tragedy. him. This gives rise toBut, given that laughter can be equally (a) verbal irony cathartic, likely to provide such instances (b) situational irony through reversals of logic and expectation is (c) dramatic irony (d) unintentional irony (a) tragicomedy (b) comedy of humors (e) situational and dramatic ironies ____________ (c) comedy of manners (d) slapstick Ans: (e) Dramatic irony involves a situation in a play (e) farce _________________________________ or a narrative which the audience or reader shares Ans: (e) ’Farce’ was a component in the comic with the author. Knowledge of present and future episodes in medieval miracle plays. Catharsis is circumstances of which a character is ignorant, in this usually associated with tragedy. But, given that situation character unknowingly acts in a way we laughter can be Cathartic likely to provided such recognize to be grossly inappropriate to the actual instances through reversal of logic and expectation is circumstances, or expects the opposite of what we - Farce. know that fate holds in store, or says something that 49. Shakespeare’s soliloquies are defined as ” ......... anticipates the actual outcome but not at all in the way a locution dominating the stage and the attention that the character intends. In this play 'In Twelfth delivered by a speaker who is alone on the stage.” Night’ Malvolio approaches on the path, talking to Identify an exception to this definition in the himself. He speaks of Olivia. Sir Toby and others context of Shakespearean tragedy. can’t help Jeering at Malvolio’s pride from this hiding (a) Hamlet’s Soliloquy, ’To be or not to be’ place, but they do it softly so that he will not over hear (b) Macbeth’s Soliloquy - ’Is this a dagger which I them and realize that they are there. ________________ see before me’ 52. In Eliot’s consideration the poets of the 18th (c) Hamlet’s soliloquy, ’Now might I do it pat’ century were intellectuals, who thought but did (d) Iago’s Soliloquy, ’If it were done when this not feel; the romantics of the 19th century felt ________done....' _______________________________ but did not think; Tennyson and Browning can Ans: (c) Hamlet's Soliloquy "Now might I do it pat; merely reflect or ruminate but cannot express Now he is praying. And now I’ll do it" appears in their experience poetically. Eliot diagnosed this Act III Scene III. This soliloquy is exception in the and coined the phrase . context of the definition of Shakespeare’s Soliloquy (a) objective correlative (b) dissociation of sensibility " ....... who is alone on the stage". In present soliloquy (c) negative capability both Hamlet and Claudius are present. After speaking (d) tradition and individual talent his dialogue, Claudius kneels on the stage, then (e) theory of impersonality ___________________ Hamlet enters on the stage and speaks his soliloquy. 50. The line T think it be thine indeed, for thou liest Ans: (b) Eliot’s theory of the "dissociation of sensibility" may be said to be an attempt to find some in if in Hamlet is an example of: kind of historical explanation to the dissolution of the (a) Retort (b) Aphorism tradition of unified sensibility which found its (c) Quibble (d) Wit perfection in the writings of Dante and Shakespeare. (e) Banter CGPSC Asst. Prof. Exam. 2017

169

YCT

The unified sensibility was a sensibility which was the important for the man may find no place in his poetry, and those which become important in the product of a true synthesis of the individual with the poetry have no significance for the man, traditional, of feeling with thought and of the temporal (a) Classic subjectivism with the eternal. It was not only representative of the (b) Romantic subjectivism mind but also of the traditions of European thought (c) Neo-classic objectivism and culture; but unfortunately according to Eliot the (d) Neo-classic subjectivism traditions of unified sensibility were suddenly (e) Romantic imagination ____________________ disrupted in the 17th century as a result of a spilt in the Ans: (b) There is always a difference between the creative personality of the artist, for which he artistic emotion and the personal emotion of the poet. formulated his famous theory of the ’’dissociation of For example the famous ’Ode to Nightingale’ of Keats sensibility”. 53. Why does Eliot say, ”No poet, no artist of any contains a number of emotions which have nothing to do with the Nightingale ’’the difference between art art, has his complete meaning alone”? (a) Because the past should be altered by the and the event is always absolute; the poet has no personality to express he is merely a medium in which present as much as the present is directed by impressions and experiences combine in peculiar and the past. unexpected ways; impression and experiences which (b) Because as a principle of aesthetic, are important for the man may find no place in his comparisons with the past ones are desirable. poetry, and those which become important in the (c) Because any new poet or artist must inevitably poetry have no significance for the man”. Eliot thus be judged by the standards of the past. rejects romantic subjectivism. ____________________ (d) Because the past cannot be a lump nor could 56. Eliot quotes this sentence in one of his essays: the present help create works on its He was desirous to use English words with a (e) Because in language, the poet’s medium, the foreign idiom. _______past and the present coexist and coa ......... Who originally said this and about whom was Ans: (a) ”No poet, no artist of any art, has his this said? (a) Dryden; Shakespeare (b) Dryden; Sydney complete meaning alone”. His significance, his (c) Johnson; Dryden (d) Dryden; Johnson appreciation is the appreciation of his relation to the (e) Johnson; Milton ________________________ dead poet and artist. You cannot value him alone, you must set him for contrast, and comparison among the Ans: (e) Dr. Johnson in his essay on poetic theory ’’From Lives of the Poet’ stated this line - ’He was dead. Eliot this says because the past should be altered desirous too use English words with a foreign idiom’ by the present as much as the present is directed by the past. ______________________________________ about Milton. This novelty has been, by those who can find nothing wrong in Milton, imputed to his 54. What is Eliot’s sense of tradition? labourious endeavors after words suitable to the (a) Know and understand the past as a whole grandeur of his idea. ”Our languages” says Addison, (b) Admire, know and, perhaps, imitate a few past ’’Sunk under him” but the truth is that both in prose poets. and verse, he had formed his style by a perverse and (c) Learn and study literary history and concurrent pedantic principle. He was desirous to use English socio-political economic movement. words with a foreign idiom. ______________________ (d) Explore, glean and use insights form chosen 57. Comedy is divided into ’high’ and ’low’ comedy; epochs or periods. farce belongs to the latter and, generally, used to (e) Be critically conscious of the main literary provoke the audience to simple jokes, physical ________trends, perhaps enriched by ’smaller’ ________ actions to create laughter. Given below is a list Ans: (e) Tradition according to Eliot, is that part of of plays. Keeping this in view that the play is living culture inherited from the past and functioning farcical or that does not contain any farcical in the formation of the present. Eliot maintains that situation or scene: tradition is bound up with historical sense, which is a (a) The Taming of the Shrew perception that the past is not something last and (b) She Stoops to Conquer invalid. So Eliot’s sense of tradition is to be critically (c) St. Joan conscious of the main literary trends perhaps enriched (d) The Tempest by smaller. (e) Waiting for Godot _______________________ 55. When Eliot makes these observations, what is he Ans: (c) ’Saint Joan’ is a play by George Bernard Shaw, about 15th century French military figure Joan rejecting? The difference between art and the event is always of Arc. The play is set between 1429 and 1431, with absolute; the poet has no personality to express he an epilogue set in 1456. During the time, the 100 is merely a medium in which impressions and years war was raging between England and France. experiences combine in peculiar and unexpected This play does not contain any farcical situation or ways; impression and experiences which are scene. 170 YCT CGPSC Asst. Prof. Exam. 2017

58. One of the following is not a theme related to W.B. Yeat’s most famous poem, written in 1919 soon 'Romantic Imagination' as embodied in 'Tintern after the end of World War 2. It describes a deeply mysterious and powerful alternative to the Christian Abbey' (a) The centrality of subjectivity in imaginative idea of second coming Jesus's prophesied return to the Earth as a savior announcing the kingdom of Heaven. processes. (b) The alchemy of the mind is bound by the Here the Anti-Christ’s stony sleep was vexed to transformation of worldly perceptions into new nightmare. ____________________________________ poetic forms. 61. From where does Eliot borrow this epigraph for (c) The sense of participating in the divine through Gerontion? creativity. Thou hast nor youth nor age (d) The search for meaning in life through But as it were an after dinner sleep exploring the mystery of our being Dreaming of both (e) Nature is the primary source of inspiration. (a) The Tempest Ans: (b) Lines written a few miles above ’Tintern (b) Twelfth Night Abbey’ is a poem by William Wordsworth. The title (c) Measure for Measure Lines written a Few Miles above Tintern Abbey, on (d) Much Ado About Nothing revisiting the Banks of the wye during a Tour, July 13, (e) Cymbeline _____________________________ 1798, is often abbreviated simply to Tintern Abbey; Ans: (c) ’Gerontion’ means "little old man” and comes although that building does not appear with in the from the Greek geron "old man”. The epigraph is from poem. The alchemy of the mind is bound by the Shakespeare’s ’Measure for Measure' (Act III scene I). transformation of worldly perceptions into new poetic The scene is a Vienna prison, the speaker is the states forms is not a theme related to ’’Romantic disguised ruler. Duke Vincentia and the listener a Imagination”. _________________________________ young man, Claudio, who has been condemned to 59. Epistolary novel is a novel told through the death by Vincentio’s temporary stand in Angelo for medium of letters written by one or more of the bedding and getting with child Juliet, Claudio, characters below enumerate its advantages, but betrothed before marriage. Eventually, Vincentio convicts Angelo himself of violating this law and the One of them is not true. Identify it: (a) Hinders objective analyses of characters’ death penalty is revoked both cases so that the actions, thoughts and words owing to r marriage can be made to flourish happily and (b) Presents an intimate view of the character’s fruitfully. The Duke’s speech iis as follows: thoughts and feelings without interfe Be absolute for death, either death or life (c) The method was most often a vehicle for Shall thereby sentimental novels, though it was not li Thou hast nor youth nor age (d) Conveys the shape of events to come with But as it were an after dinner sleep Dreaming of both dramatic immediacy. (e) Lends the story dimension and verisimilitude 62. In these lines from Gerontion the word 'word' ________through a presentation of events fro .................. occurs three times, with different meanings: Ans: (a) Epistolary novel, a novel told through the The word within a word, unable to speak a word, medium of letters written by one or more of the Swaddled with darkness. characters. Originating with Samuel Richardson’s Choose from the options the One meaning Pamela : Or, virtue Rewarded (1740). It presents an 'word' does not denote or connote here: intimate view of the character’s thoughts and feeling (a) God (b) Lexeme without interference from the author and that it (c) Jesus (d) Infant conveys the shape of events to come with dramatic (e) Scripture __________ ____________________ immediacy. In it the presentation of events from Ans: (b) In the line ’The word within a word’ unable several points of view ends the story dimension and to speak a word Swaddled with darkness. From verisimilitude. In the given options Hinders objective ’Gerontion’ the word ’word’ occurs three times with analyses of characters actions, thoughts and words different meaning which are (1) God 'Jesus' (2) Infant owing to ......... is not true. _______________________ (3) Scripture. Word cannot denote here ’Lexeme’. The term lexeme means a language’s most basic unit of 60. In the following lines meaning often also thought of as a word in its most That twenty centuries of stony sleep basic form. ___________________________________ Were vexed to nightmare by a rocking cradle, Whose 'stony sleep' was 'vexed to nightmare'? 63. Disambiguate this line from Gerontion, keeping (a) The poet’s (b) Humanity’s in mind that Christ has been made 'the (c) The political world’s (d) The Antichrist’s tiger of communion means eternal death: (e) History's ______________________________ The tiger springs in the new year. Ans: (d) The following lines "That twenty centuries (a) The tiger comes alive in the new season. of stony sleep, were vexed to nightmare by a rocking (b) The tiger pounces on the sinners (who have cradle” appears in ’The Second Coming”. It is one of knowledge and thus are not sinless). CGPSC Asst. Prof. Exam. 2017

171

YCT

(c) The tiger can rejuvenate like spring, or spring up and devour - ’springs’ is a sylle (d) The tiger is active in spring (regenerative). (e) The tiger unleashes a new era. _____________ Ans: (c) The given lines ’The tiger springs in the new year’ from ’Gerontion’ poem by T.S. Eliot published in 1920. Poet explained his thoughts through the given line which mean ’The tiger can rejuvenate like spring or spring up and devour - ’springs’ is a sylle ’. Keeping in mind that Christ has been made ’’The tiger unleashes a new era. Eliot is able to formulate his own skepticism and states”. __________________________ 64. Explain the significance of Death by Water in The Waste Land*. (a) Instead of the regeneration as a shadow that lurks behind earlier sections, there is profound but has no deeper significance. (b) The section must be read as a parable, reminding the living of death. (c) This section is about death and needs to be contrasted with others that de (d) This section is based on Ferdinand’s supposed/imagined death by water in The Tempest (e) This part fulfills the prediction of Madame ________Sosostris in the poem’s first section ................... Ans: (e) The shortest section of the poem ’’Death by Water” describes a man. Phlebas the Phoenician who had died apparently by drowning. In death he has forgotten his worldly cares as the creatures of the sea have picked his body part. The narrator asks his reader to consider Phlebas and recall his or her own mortality. This part fulfills the prediction of Madame Sosostris in the poem’s first setion 65. What does Yeats tell his daughter here? It's certain that fine women eat A crazy salad with their meat Whereby the Horn of Plenty is undone. (a) Yeats does not want his daughter to be very beautiful as such women lose their fortune. (b) Yeats here is anguishing over the fact he has experienced in his life - Maud Gonre man to marry. (c) Yeats here metaphorizes marriage into ’diet’ he avers that beautiful women are fortune. (d) Yeats here is recalling how Maud Gonne always chose the wrong man to be with (e) The cornucopia or horn of plenty is a symbol of abundance and nourishment; Year women ________lose through bad marriages. _______________ Ans: (a) In his poetry ’A Prayer for My Daughter’ W.B. Yeats wants his daughter to possess some qualities so that he can face the future years independently and with confidence. Let her be given beauty, but a more important thing is that her beauty should not be of a kind which may either make her proud of her beauty or distract a stranger’s mind and eyes. W.B. Yeats does not want his daughter to be very beautiful as such women lose their fortune. CGPSC Asst. Prof. Exam. 2017

66.

What is its significance of these lines from Byzantium? Keep in mind this: Yeats has reached Those images that yet Fresh images beget, That dolphin-torn, that gong-tormented sea. (a) Yeats purges his poetic soul of ennui - the old images of Sailing to Byzantium has (b) Coming back to Byzantium, Yeats can only recall his earlier high and low images reconstructed in his memory - as dolphins transport people. (c) Yeats now apprehends the image of the sea its surface is ’tom’ by leaping-divi toll (perhaps the drowned) agonize the sea. (d) Coming back to Byzantium, Yeats has sensed high and low images; now, they are dolphins transport people. (e) The sea is tormented and ’tom’; but the poet’s _______psyche is calm and whole, since h. . .. ________ Ans: (a) The significance of the given lines ’Those images that yet Fresh images beget, That dolphin-tom, that gong-tormented sea, is W. B. Yeats purges his poetical soul of ennui - the old images of Sailing to Byzantium has ’’Sailing to Byzantium” is a poem by W. B. Yeats, first published in the 1928 collection ’The Tower’. It comprises four stanzas in ottave rima, each made up of eight lines of iambic pentameter. ________________ 67. What does this piece of dialogue imply? WARWICK, I am informed that it is all over, Brother Martin. LADVENU (enigmatically) We do not know, my lord. It may have only just begun. (a) The first talks about the burning, the second about what will happen in the Epilog (b) Warwick announces that Joan has been burned in the stake, but Ladvenu, who en.... and her trust in Heaven, thinks her Eternal Life might have just begun. (c) The dogmatic Warwick senses an end of ’heretical Joan’; the pious Ladvenu app of St joan. (d) The pious Ladvenu knows that Joan’s martyrdom might be similar to Jesus’s, wh. . . (e) The dialogue highlights the contrast between the cold-blooded English assassins Catholics. Ans: (b) The piece of dialogue imply that Warwick announces that Joan has been burned in the stake, but Ladvenu, who en.... and her trust in Heaven, thinks her Eternal Life might have just begun. Saint Jean is a play by G. B. Shaw about 13th century French military figure Joan of Arc. G. B. Shaw characterized Saint Joan as ”A chronicle play in 6 scenes and an Epilogue.” 68. What happens when these lines are spoken: JOAN. Woe unto me when all men praise me! I bid you remember that I am saint, and that saints can work miracle. And now tell me: shall I rise

172

YCT

from the dead, and come back to you a living woman? (a) Since mortal eyes cannot distinguish the saint from the heretic; she gives up the i (b) The men who praised her retract their statements if she became a normal woman. (c) Each of the people who once praised Joan for her actions during her life now des... earth, as living saints are a ’nuisance’. (d) Since Joan is brought back to earth, the force of tragedy conveyed and experience. . . (e) Saints can work miracles, but mortals cannot ________so the assembled discourage and ...................... Ans: (c) ’Saint Joan’ (1923) is a play by G. B. Shaw. ’Saint Joan’ is a tragedy without villains, for everyone, in some way or another, believes he or she is acting for the good. The tragedy lies in human nature itself, which involve us all. The epilogue gives Shaw the chance to step forward and talk the play ever with you the audience. It is a historical play consisting six scenes and an epilogue. When these lines are spoken, Each of the people who once praised Joan for her actions during her life now des .... Earth as living saints are a ’nuisance’. 69. What does Rabbi Ben Ezra communicate through these lines? Ay, note that Potter's wheel, That metaphor! and feel Why time spins fast, why passive lies our clay (a) God shapes us as we are clay - the capital P in ’Potter’s’ stands for God. (b) God controls and offers lessons to us all; suffering is related to growth; God cont (c) We use metaphors to refer to God because we can only feel him. (d) We sense and experience the life on earth only through metaphors and artefacts. (e) God and Time make us and spin our lives. Ans: (b) By the poem ’Rabbi Ben Ezra’ Robert Browning communicates through the given lines that God controls and offers lessons to us all : suffering is related to growth; God cont Rabbi Ben Ezra is a poem by Robert Browning about Abraham Ibn Ezra. It is basically a dramatic monologue. It was first published in 1864. It has 192 lines and is the clearest expression of Browning’s optimistic philosophy of life. _____________________ 70. How does Browning’s Andrea del Sarto differ from his other dramatic monologues? (Choose the most logical option) (a) It is about a painter who failed in life because of his dominating shrew of a wife. (b) It is about a painter who can only regret his lack of opportunities - not - v successful artists. (c) It captures the mood and the attitude of a regretful painter, and lacking any action (d) Is an affirmation of love as the narrator suggests that if he and his wife can sit an ..... CGPSC Asst. Prof. Exam. 2017

rejuvenated and able to produce work with new vigor. (e) It is about a painter who, being under the thumb of his wife, has a resigned, mela at ________the end. _______________________________ Ans: (c) ’’Andrea del Sarto” (also called ’’The Faultless Painter”) is a poem by Robert Browning published in his 1855 poetry collection, Men and Women. It is a dramatic monologue by which he is famous about the Italian painter Andrea del Sarto. It is differ from his other dramatic monologues because it captures the mood and the attitude of a regretful painter and lacking any action It’s incredibly melancholic tone and pessimistic view of art. The voice, as well drawn as usual falls into blank verse, unrhymed, mostly iambic lines, but lacks the charisma of most of Browning's speakers. ___________ 71. Which of the following is NOT a dramatic monologue? (a) Fra Lippo Lippi (b) Kubla Khan (c) The Love Song of J. Alfred Prufrock (d) The Flea (Donne) (e) A Valediction Forbidding Mourning ________ Ans: (b) ’Kubla Khan’ is not a dramatic monologue. It is considered to be one of the greatest poems by the English romantic poet Samuel Taylor Coleridge, who said he wrote the strange and hallucinatory poem shortly after waking up from an opium influenced dream in 1797. Poem begins in a place called Xanadu, the Mongalian leader Kubla Khan ordered his servants to construct an impressive domed building for pleasure and recreation on the blanks of the holy river Alph. 72. Arnold in Function of Criticism holds that the creation of the modern poet implies a great cri exemplify his proposition, he compares Byron and Goethe. The below list enumerates the si The list includes something that Arnold has not expressed at all. Identify it: (a) Both Byron and Goethe had a great productive power, but Goethe’s was nourishe true materials for it, and Byron's was not. (b) Goethe knew life and the world, the poet’s necessary subjects, much more compr (c) Byron’s poetry had so much endurance in it, and Goethe’s so little. (d) Both Byron and Goethe had the creative power of poetry that requires ideas and and achieve success. (e) Byron was so empty of matter, while Goethe ________enriched his thoughts through book _________ Ans: (c) Matthew Arnold in ’Function of Criticism’ compares the emotional state of writing criticism with the emotional state of creative writing. Arnold observes that great writing emerges from great ideas and they are manifested when these ideas reach the masses.

173

YCT

Byron’s poetry had so little endurance in it and (e) because Coleridge really considered Milton the Goethe’s so much : both Byron and Goethe had a great ________epic poet a model to be imitated. ____________ productive power, but Goethe’s was nourished by a Ans: (a) Biographia Literaria was the most important great critical effort providing the free materials for it, work of literary criticism of the English Romantic and Byron’s was not. period, combining philosophy and literary criticism in 73. What does Hawthorne depend on to develop and a new way and it was lastingly influential. In the Biographia Literaria, Coleridge has a chapter titled ’In structure the plot of The Scarlet Letter*! the imagination or essemplastic power'. Essemplastic (a) The scarlet letter is a word he devised himself from Greek and means (b) The scaffold scenes "to shape into one". In the given option, option (a) (c) The Black Man in the forest does not explain why in Biographia Literaria, (d) Crime, remorse and repentance Coleridge disapprove of Milton.____________________ (e) Chillingworth's passion for revenge _________ 76. Which One of the following is not true of Ans: (b) The three scaffold scenes in ’The Scarlet Coleridge’s Biographia Literaria*! Letter’ are integral to the structure and unity of the (a) Is easy to understand and defend as it offers narrative. They are the most dramatic scene at the principles the writer obeyed as a poet beginning, in the middle and at the end of the novel.. (b) Has an imbalanced structure, more due to padding. Artistically and dramatically, these scenes are at the (c) Incorporates too much of material produced by very care of Hawthorne's tale of crime and others. punishment. Each of these scenes brings together the (d) Propounds creative principles, flouted in major characters and forces of the story and each practice by Coleridge himself. scene, rivets our attention to the scarlet letter ’A' on (e) Is full of whimsical and prejudicial ideas Hester Prynne bosom. ________which are not rationally developed. __________ 74. Arnold says in Study of Poetry that his Ans: (a) ’Biographia Literaria’ is an autobiography is admiration of Chaucer does not, however, make discourse by S.T. Coleridge, published in 1817 in two him the greatest classics alongside volumes of twenty three chapters. It is the creative Homer, Dante or Shakespeare. Towards this, he impulse that enables poetry and other art. 'Biographia proposes Literaria' contains the first instance of the phrase (a) the ’Touchstone' method ’suspension of disbelief. Coleridge is concerned (b) the high seriousness criterion throughout Biographia Literaria with exploring the (c) 'fluidity of movement and divine liquidness of relationships between the circumstances and the events of his own life on the one hand and on the diction’ other hand his poetry and its reception by the public. (d) criticism of life - men, mode and manners Biographia Literaria is not easy to understand and defend (e) the harmony between action and expression as it offers principles the writer obeyed as a poet. _______ Ans: (a) The term ’Touchstone’ was coined by TL The 21st century readers of Thackeray would Matthew Arnold in his essay ’’The Study of Poetry", say, ’’Esmond’s memoir is not written like a where he gives Hamlet's dying words to Harotio as an no.... like we know them.” example of a touchstone. Touchstone black siliceous What are the achievements of Thackeray the stone used to ascertain the purity of gold and silver. novelist in this novel? Assaying by "touch" was one of the earliest methods (a) A novel narrated in first person was not in employed to assess the quality of precious metals. The vogue when Esmond was published. metal to be assayed is rubbed on the touchstone, (b) Memoirs, such as Captain Singleton’s had created a traditional space within whi.... adjacent to the rubbing on the touchstone of a sample showcased his genius. of a metal of known purity. He introduced the term (c) Esmond, till when Esmond yields to his ’touchstone' to denote short but distinctive passages Oedipal urge, is a placid recollection of 1 selected from the writing of great poets, which he (d) Esmond captures an era that has gone by used to determine the excellence of passages on through the memories of the central cha. ... poems which are compared to them. (e) Thackeray could not develop men characters 75. Choose the option that does NOT explain why in ________hence his Esmond is static and un ...................... Biographia Literaria, Coleridge disapprove of Ans: (b) ’Esmond memoir' written by Thackeray. This Milton novel is ’The History of Henry Esmond. The name (a) because of the growing distance between Esmond is a boy’s name of English origin meaning Wordsworth and him. ’graceful protection’. Though slightly haughtier and (b) because of the Romantic poets’ disposition to less accessible than Cousin Edmond, Edmond could devalue scholarship and prefer con appeal to some parents seeing a distinguished (c) because of Wordsworth’s criticism of Milton appellation. It began being used (albeit-sparingly) in England in the 19th century, possibly influenced by was not based on the creative princi (d) because it was fashionable to criticize Milton's William Makepeace Thackeray's novel, most parents today would prefer the more popular 'Desmond'. language and Coleridge did not f. 174 YCT CGPSC Asst. Prof. Exam. 2017

78. The most redeeming feature of The History of Henry Esmons, according to discerning critics.... (a) How the life of an individual could stand in for the social history of a period. (b) Thackeray’s attempt to capture and depict the flavor of the earlier Augustan age English of that period is noteworthy. (c) Thackeray could not decide whether the narrator should condemn or vindicate hi. . . . (d) Thackeray attempts to depict a world which is totally imaginary, including its mo (e) Thackeray arbitrates over social mores and ________manners, even while portraying the fo ..... Ans: (b) ’The History of Henry Esmond’ is a historical novel by ’William Makepeace Thackeray’. The book tells the story of the early life of Henry Esmond, a colonel in the service of Queen Anne of England. Thackeray’s attempt to capture and depict the flavor of the earlier Augustan age is noteworthy. _____________ 79. What is the common theme - not Wordsworth’s desire or theory - that runs through the poem (a) Rustic, simple, unaffected nature (b) Emotions recollected in tranquility (c) Memory of communion with nature (d) Feeling, instinct, and pleasure more significant than formality and mannerism (e) The need to invent a special language for _______poetry ________________________________ Ans: (c) Romantic literature, like other genres, shares comparable literary elements that merge a particular design of poetry. William Wordsworth a romantic poet, used pictures of nature in addition of themes of idealism expressed with feeling of his poetry. These components that Wordsworth utilized were very normal of other romantic works, themes and images, without Wordsworth’s use of them, his poetry would have an entirely different impact. One aspect in romantic literature that is very common is image of nature. In Romantic literature a major theme that is used by a lot of the authors is idealism or living a life much better than the one that people are living now. 80. Stephen Crane in The Red Badge of Courage ’’stages the drama of war, so to speak, within the mind of one man and then admits you as to a theatre” and mostly relies on to help his readers feel a experience of being a soldier. (a) metaphors and symbols (b) dialogues and verbal exchanges (c) images and evocative language (d) actions on the war-front (e) epic-like enumeration of details ____________ Ans: (c) ’The Red Badge of Courage’ is a war novel by American author Stephen Crane. This novel taking place during the American Civil War, the story is about a young private of the Union Army, ’Henry Fleming’. The novel is known for its distinctive style, which includes realistic battle sequences as well as the repeated use of color imagery and ironic tones, so Stephen Crane mostly relies on images and evocative language to help his readers feel a experience of being a soldiers. Hence option (c) is correct. CGPSC Asst. Prof. Exam. 2017

81. Complete the statement by choosing the appropriate option In his novel Tess of the d'Urbervilles, Thomas Hardy (1840-1928) deals with issues of moral ways; one is the relativity of moral values - their variation according to time and place -the (a) how man-made laws and Nature synchronize in the lives of men (b) the opposition between man-made laws and Nature (c) the absoluteness of faith and God’s laws (d) the conflicts between personal guilt, sin and social excommunication (e) the invariable Natural laws that ignore time ________and place ______________________________ Ans: (c) ’Tess of the d’Urbervilles (A pure woman faithfully presented) is a novel by Thomas Hardy. It initially appeared in a censored and serialized version published in 1891. It is considered a major 19th Century English Novel, even Hardy’s fictional master piece. ________________________________________ 82. Explain this remark in Tess of the d'Urbervilles: 'Ah, she makes herself unhappy. ' (a) Hardy looks very closely at the feeling of guilt in Tess about her liaison with Alec and sug... the guilt was necessary. (b) Hardy feels that Tess’s guilt over her liaison with Alec has no bearing on her life in the co Tess herself assumed. (c) Any community, when the guilty is sad and remorseful, would exonerate themselves by sa unhappy’. (d) Tess is unhappy as she suspects that the members of her community mock her and this un.... misery. (e) Hardy here traces the sensitivities of Tess who, though an individual, could not be oblivion Ans: (b) ’Tess of the d’Urbervilles’, novel by Thomas Hardy, first published serially in bowdlerized form in the Grofhic - 1891 and in its entirety in book form (three volumes) the same year. It was subtitled - ’A Pure Woman faithfully presented.’ By this remark - ’Ah, she makes herself unhappy’. Hardy wants to explain - that - Tess’s guilt over her liaison with Alec has no bearing on her life in the Tess herself assumed. Hence option (b) is correct. ______ 83. Which one of the following is not strictly a ’modernist’ writer? (a) Sylvia Plath (b) Ezra Pound (c) E.E. Cummings (d) Somerset Maugham (e) F. Scott Fitzgerald ____________________ Ans: (d) William Somerset Maugham was an English novelist playwright and short story writer of the twentieth century (1900-1945). He is not strictly a modernist writer. Maugham’s plays are largely treatments of middle class attitudes to love and money. The Circle (1921) and The Constant Wife (1926) in which the main female character asserts her independence in a choice of partner.

175

YCT

84.

Sonnet # 130 of Shakespeare begins with this line: My mistress' eyes are nothing like the sun Often dubbed a spoof, the sonnet (a) repudiates all Shakespeare has said of his Dark Lady earlier. (b) parodies the ’conventions’ of Elizabethan love sonnets, both in content and structure (c) is an example of the non-idyllic love felt and expressed by common folk. (d) describes the cooling aspect of his lady love’s eyes, bereft of lust. (e) is Shakespeare’s pragmatic tribute to ordinary _______women in their day everyday exist. _________ Ans: (b) Sonnet no-130 was written by the English poet and playwright William Shakespeare. Though most likely written in the 1590. Like many other sonnets from the same period he parodies the ’Conventions’ of Elizabethan love sonnet, both in content and structure. His poem wrestle with beauty, love and desire. ________________________________ 85. In Oliver Twist, on the night that Nancy has her secret meeting with Rose, a familiar couple i the north. Who are they? (a) Mr. Brownlow and his sister (b) Mr. and Mrs. Bumble (c) Fagin and the Artful Dodger (d) Fagin and Noah Claypole (e) Noah Claypole and Charlotte ______________ Ans: (e) Oliver Twist, or the Parish Boy’s Progress is Charles Dickens’s second novel and was published as a serial from 1837 to 1839 and released as a three volume book in 1838. The story of the novel centre on Orphon Oliver Twist. Nancy is one of Fagin’s gang, now living with Bill Sikes, on the night that Nancy has her secret meeting with rose, a family couple Noah Claypole and Charlotte are there. Hence option (e) is correct. __________________________________ 86. Oliver Twist can be read as a textbook of Victorian child abuse and also as . (a) a journalistic model for reporting such incidents in media (b) a literary model for writing about crimes and oppression in real life in novels (c) a political pamphlet about absence of governance in Victorian England (d) a dispassionate account of the social conditions by a reporter (e) a social document about early Victorian slum ________life ___________________________________ Ans: (e) ’Oliver Twist’ or ’The Parish Boy’s Progress’ is Charles Dickens second novel and was published as a serial form 1837 to 1839. Oliver Twist is notable for its unromantic portrayal of criminals and their sordid lives, as well as for exposing the cruel treatment of the many orphans in London in the mid 19th century. Oliver Twist can be read as a text book of Victorian child abuse and also as a social document about early Victorian slum life. CGPSC Asst. Prof. Exam. 2017

87. What happens in these lines from Arnold’s Dover Beach*. Only, from the long line of spray Where the sea meets the moon-blanched land, Listen! you hear the grating roar Of pebbles which the waves draw back, and fling (a) Arnold shares the visual beauty of the scene with the addressee and then he calls (b) Arnold’s attention shifts from the sight of dark sea and ’blanched’ land to the hea auditory image. (c) Human beings, like pebbles, are flung into life and off it by the ebb and flow of (d) Moon-blanched land as an image captures the temporal and temporary nature of poet to hear the sound of waves. (e) The shift to the auditory images facilitates the ________lyricist to reflect on the sea during __________ Ans: (b) ’’Dover Beach” is a lyric by the English poet Matthew Arnold. It was first published in 1867 in the collection New Poems, the poem expresses a Crisis of faith with the speaker acknowledging the diminished standing of Christianity. Given lines are taken from the second stanza of the poem. In these lines Arnold’s attention shifts from the sight of dark sea and ’blanched’ land to the sea and represent auditory image. Hence option (b) is correct. 88. Identify from the options given below, the feature that makes Keats’s Ode on a Grecian Urn unique among the other odes. (a) the tone and thematic elements of the classical ode (b) no formal structure (c) the use of personifications (d) based on experiments with sonnets (e) full of allusions _________________________ Ans: (a) ’Ode on a Grecian Um’ was written by the influential English poet John Keats in 1819. It is a complex and mysterious poem with a disarmingly simple set up an undefined speak looks at a Grecian Um, which is decorated with evocative images of rustic and rural life in ancient Greece. This ode is rich in tone and thematic elements of the classical ode. 89. What do these lines by Auden refer to? In the nightmare of the dark All the dogs of Europe bark (a) For poetry makes nothing happen, as it is as futile as dogs barking in a nightmare. (b) A world leaning towards World War II — a world without poetry. (c) A world where both thinking and pitying are ’frozen’, unmoving. (d) Elegies always turned toward tomorrow, leaving the dead; but here Auden turns towards t (e) A developing situation in which Yeat’s poetry would have made ’a vineyard of the curse’ 176 YCT

Ans: (b) These lines- ”In the nightmare of the 93. Which One of the differences between ’tragedy’ and ’melodrama’ is not true? Keep in view more dark/Alll the dogs of Europe bark” appears in Wystan related to theater than ’literature’: Hugh Auden’s poem ’In memory of W.B. Yeats’. Here (a) Tragedy ends in and as ’tragedy’; early in these lines Auden refers the ’nightmare of the dark’ melodrama had the hero restored to his hi and European nations that are ’sequestered’ in its hate. (b) Tragic heroes are static; melodramatic ones are Thus he describes accurate the situation before World archetypes. War II where Nazis dominated the Western part of (c) In tragedy the characters determine and control Europe and fuelled nut against other nation and other the plot; in melodrama the plot d nationalities. (d) Tragic heroes are a mixture of good and bad, 90. How does Dover Beach treat the ’sea’? dark and white; the melodramatic h (a) The sea shifts from being an object to an (e) Tragedy inheres and develops hamartia', image, from a visual object to an aural s melodrama depends solely on fate to tri........ (b) The sea develops into a metaphor from being Ans: (b) Tragedy is a very sad event or situation, an image. especially one that involves death but Melodrama is a (c) The sea is used as an image and a metaphor, story, play or film in which a lot of exciting things happen and in which people’s emotions are stronger representing the eternal note of sadness. (d) The sea, an inanimate object, is personified than in real life. Both heroes are static. Hence option (b) is correct. and sadness, a typical human feeling (e) Since the poet is talking about ’Dover Beach’ 94. Eliot’s criticism of Milton revolves around his use of language. Specific details regarding thi the sea is present and felt throughout the details listed below was not expressed Ans: (c) ’’Dover Beach” is the most celebrated poem by Eliot. Identify it: by Matthew Arnold, a writer and educator of the (a) Milton’s images give the sense of particularity, Victorian era. The poem expresses a crisis of faith, the separate words develop in sig with the speaker acknowledging the diminished (b) Milton’s blindness and devotion to music make standing of Christianity, which the speaker sees as his images more aural in natural being unable to withstand the rising tide of scientific (c) Milton was the first to commit violence to discovery. ’Sea’ is symbol of change, ’land’ is symbol language by using distorted construed of continuity. In ’Dover Beach’ Matthew Arnold foreign way or with meaning other than introduces the dominant image in the first line of the accepted in English. poem ’’The sea is calm to night”. The sea is both a (d) ’’Milton’s poetry could only be an influence for symbol and metaphor, representing the eternal note the worse upon any poet whatever (e) Even in his most mature work, Milton does not of sadness. infuse new life into the word, as ....................... 91. Which One of the following specification of Ans: (a) According to Eliot’s criticism on Milton, hamartia in the tragic heroes of Shakespeare is Milton’s images do not give this sense of particularly, (a) Lear -Madness nor are the separate words developed in significance. (b) Othello - Jealousy His language is, if one may use the term without (c) Antony - Infatuation disparagement, artificial and conventional. __________ (d) Hamlet - Procrastination What kind of poem is ’Ulysses’ and what is its (e) Macbeth - Ambition purpose? Ans: (a) The term ’hamartia’ derives from the Greek (a) Lyric; describe the speech of Ulysses before word - hamartanein, which means ”to miss the mark” setting out on his last voyage. or ’to err”. It is most often associated with Greek (b) Dramatic monologue; capture the mood and tragedy. Madness is a specification of hamartia in reflect the attitude, personality and his King Lear, his hamartia stems from his inability to final voyage. strike a balance between his volatile temperament and (c) Ode; Ode to Celebrate a final adventure where Ulysses would strive, seek and find his arrogant manner. Hence option (a) is correct. (d) Elegy; to mourn the death of the Ulysses, the 92. Which One is not a fool in Shakespeare's plays? adventurer/explorer. (a) Jacques |L4s You Like It} (e) Soliloquy; Ulysses introspecting on his nature, (b) Trinculo [The Tempest} life, achievement and future aim. ___________ (c) The Gravediggers [Hamlet} Ans: (b) ’Ulysses’ was written in 1833 by Alfred Lord (d) Touchstone |L4s You Like It} Tennyson. The poem takes the form of a dramatic (e) Feste [Twelfth Night} monologue spoken by Ulysses, a character who also Ans: (a) Jacques is one of the main character in appears in Homer’s Greek epic ’Thee Odyssey’ and Shakespeare’s ’As You Like It’. He is known as ’The Dante’s Italian epic ’The Inferno’ (Ulysses is the melancholy Jacques’. He is not a fool. A faithful lord Latinized name of Odysseus). In the Ulysses struggles to Duke senior, Jacques has an exaggeratedly to return home but in Tennyson’s ’Ulysses’ an aged melancholy disposition and is a devoted though Ulysses is frustrated with domestic life and yearns to cynical observer of the world that surrounds him. set sail again and continue exploring the world. CGPSC Asst. Prof. Exam. 2017

177

YCT

96.

(a) due to negative capability, the poet disappears from the work (b) the poet becomes melded to and indistinguishable from the object being describe (c) there is an invisible nexus between the sensorium and ’art’ (d) one must ’leave’ the world if one has to explore a transcendent, mythical, or aesthetic (e) the immortal in us helps us relate to ’beauty’ in art and sense it as eternal and tru

What does Tennyson, in these lines, achieve? It may be that the gulfs will wash us down: It may be we shall touch the Happy Isles, And see the great Achilles, whom we knew, (a) Unlike Homer, Tennyson makes his speaker assume that he shall meet Achilles in Troy (b) Unlike Homer, Tennyson is making the quest of Ulysses for new lands mythical as Elysium (c) Unlike Homer’s hero, Tennyson’s hero is left unsatisfied and incomplete. (d) Unlike Homer, Tennyson keeps the followers of Ulysses alive in Ithaca and so he ’quest’. (e) Unlike Homer’s Tennyson’s Ulysses shall strive to seek and find, as he shares his

Ans: (b) In these lines Tennyson achieves they might sink down, end up on the happy isles (Island of the blessed in Greek mythology, Elysium located beyond the Western horizon) where they will meet Achilles (greatest of Greek warrior, killer of Hector at Troy and lead figure in Homer’s Iliad).

Ans: (d) The statement ’’One must leave the world if one has to explore a transcendent, mythical or aesthetic” is false in context of the recurring motifs that appear throughout the six odes of Keats. 99.

Identify the poet and the poem: And there shall be for thee all soft delight That shadowy thought can win, (a) Wordsworth : Tintern Abbey (b) Coleridge : Ode to Dejection (c) Keats : Ode to Psyche (d) Yeats : Sailing to Byzantium (e) Milton : Il Penserosos

97. The options below, except One, link the ’Ode to a Nightingale’ to ’Ode on a Grecian Urn’ ......... among them: (a) In the ’Nightingale’, the ineffable principle being alluded to was ’heard, but unse.... Ans: (c) The following lines ’’And there shall be for unheard’. thee all soft delight/ That shadowy thought can win” (b) In the ’Nightingale’, Keats ’cuts to the chase’ appear in ’Ode to Psyche’, a poem by John Keats immediately; in the ’Grecian Um’, are sweeter. written in spring 1819. ’Ode to Psyche’ is a romantic (c) In the ’Nightingale’ death and its lesser cousin ode, a dignified but highly lyrical poem in which the intoxication, he assumes, could h... ’Grecian poet speaks to a person or thing absent or present. Um’, loss of beauty through mortality is 100. Identify the one element that does not belong to precluded. Pindaric Odes in the options given below: (d) Both in the ’Nightingale’ and the ’Grecian Urn’, (a) They are normally serene, private, Keats personifies the bird and t status. contemplative in their content and expression. (e) In both odes, when we are suddenly reminded (b) They follow a very regular 3 -part structure of of our mortal identity and that an strophe, antistrophe, and epode. world far above,” we are stuck back in the (c) They are full of digressions and leaping same condition we began the odes wit transitions from topic to topic. Ans: (c) Ode to a Nightingale and Ode on a Grecian (d) They deal with political and/or religious Um are poems by John Keats. Ode to a Nightingale subjects and are panegyrical. composed in one day, Ode On a Grecian Um is one of (e) They are full of allusions or direct references the ’’Great Odes of 1819”, which also include ”Ode to myths, and thus akin to epics. On Indolence”, ”Ode On Melancholy”, ”Ode to a Nightingale” and ”Ode to Psyche”. Both poems poets admiration and yearning for the world of art. Hence option (c) is correct. 98. One of the options given below is False, identify it: The recurring motifs that appear throughout the six odes of Keats are: 178 CGPSC Asst. Prof. Exam. 2017

Ans: (a) The regular or Pindaric Ode in English is a close imitation of Pindar’s form, with all the strophe and antistrophes written in one stanza pattern and all the epode in another. Pindar’s ode are encomiastic, that they were written to praise and glorify someone. They are not normally serene, private, contemplative in their content and expression. YCT

Chhattisgarh Public Service Commission Asst. Prof. Exam. 2016

ENGLISH (Solved Paper) 1.

In ancient Greece, Lyric Poetry was also known as : (a) Pindaric Poetry (b) Sapphic Poetry (c) Hellenistic Poetry (d) Melic Poetry _____(e) Horatian Poetry________________________ Ans. (d) : In ancient Greece, Lyric Poetry was also known Melic Poetry. It is elegiy, in which the epic hexameter, or verse line of six metrical feet, alternated with a shorter line were traditionally associated with lamentation and an autos accompaniment. 2. Pyrrhic meter involves : (a) two unstressed syllables (b) one stressed syllable, one unstressed syllable (c) two stressed syllables, one unstressed syllable (d) one stressed syllable, two unstressed syllables (e) two stressed syllables, two unstressed ________syllables _____________________________ Ans. (a) : Pyrrhic meter also known as quantitative meter. It is a metrical unit consisting of two unstressed syllables in accentual syllabic verse, or two short syllables. The meter is common in classical Greek poetry but most modem scholars do not use the term, but most modem scholars do not use the term. Rather than identify the pyrrhic as a separate meter. They prefer to attach the unaccented syllables to adjacent feet. 3. A classic ode is structured in three major parts : (a) The antistrophe, the strophe, the epode (b) The strophe, the epode, the antistrophe (c) The strophe, the homostrophe, the epode (d) The strophe, the monostrophe, the epode _____(e) The strophe, the antistrophe, the epode _____ Ans. (e) : Ode was originating in Ancient Greece, this types of poems were originally performed publicity to celebrate athletic victories. Later this poetic form was favored among english romantic poets, who used odes to express emotions using rich, descriptive language. It is a short lyric poem. It is traditionally divided into three sections(1) The strophe In a Greek Ode the strophe usually consists of two or more lines repeated as a unit. (2) The antistrophe It is structured the same way as the strophe but typically offers a thematic counterbalance. (3) The epode This section or stanza typically has a distinct meter and length from the strophe and antistrophe and serves to summarize conclude the ideas of the ode. CGPSC Asst. Prof. Exam. 2016

[Exam. Date : 28 Sep. 2016]

4.

"Ode on Intimations of Immortality from Recollections of Early Childhood" is in the form of : (a) Irregular Ode (b) Pindaric Ode (c) Horatian Ode (d) Regular Ode _____(e) Sapphic Ode __________________________ Ans. (b) : Pindaric ode consists of a strophe an antistrophe that is melodically harmonious and an epode. Pindaric poems are also characterized by irregular line lengths and rhyme schemes ode on intimations of immortality from recollections of early childhood ”is in the form of Pindaric Ode.” Imitation Pindaric Odes were written in england by Thomas Gray in 1757, ’’The progress of poesy” and ’’The Bard”. Abraham Cowley Pindarique Odes (1656) introduced a looser version known as Pindarics. 5. Who among the following is given credit for the invention of the Sonnet? (a) Giacomo Da Lentini (b) Emperor Frederick II (c) Dante Alighieri (d) Emperor Fredirick I _____(e) Petrarca ______________________________ Ans. (a) : Giacomo Da Lentini also known as Jaopo da Lentini was invented ’Sonnet’. He was an Italian Poet of the 13th century. He was a senior poet of the Sicilian school and was also a notary at the Hely Roman Emperor Frederick-II court. 6. Sonnets are usually written in : (a) iambic hexameter (b) iambic trimester (c) iambic pentameter (d) iambic tetrameter _____(e) iambic heptameter _____________________ Ans. (c) : Sonnets are usually written in 14 lines each usually containing 10 syllables and a fixed pattern of rhyme. It was originated in the Italian poetry. The main types of sonnet are the petrarchan, the Shakespearean and the Spenserian. The word ’’Sonnet” is derived from the Italian word ’Sonetto’. meaning ”a sound”. It is a form of lyric and of all its forms. It is most carefully ordered and bound by definite, rigid rules. 7. "In Memory of WB Yeats” is an elegy written by: (a) T S Eliot (b) W H Auden (c) Ezra Pound (d) Lady Gregory (e) George Moore

179

YCT

Ans. (b) : "In memory of W.B. Yeats" is an elegy written by W.H Auden in 1939. It is a 63 lines poem that is divided into three sections. In this poem W.H. Auden tries to transform the "curse" of Yeats physical death into an occasion for rejoining by reminding readers of the paradox of death not being final for a poet. Auden implies that one should not be sad at Yeats passing. The poet lines for even in and through his poetry. W.H. Auden was an Anglo - American poet exerted a major influence on the poetry of the 20th century. 8. ”O Captain! My Captain!” by Walt Whitman is an elegy written for : (a) President Thomas Jefferson (b) President George Washington (c) President Theodore Roosevelt (d) President John F Kennedy _____(e) President Abraham Lincoln ______________ Ans. (e) : "O Captain! My Captain!" by Walt Whitman is an elegy written for the President Abraham Lincoln after his assassination in 1865. The captain in the poem refers to Abraham Lincoln and the ship that arrives represents America. The first line show the metaphor of the victory of the civil war. The poem is one of the best extended allegorical poems. 9. Who among the following is NOT a Roman satirist? (a) Quintilian (b) Horace (c) Juvenal (d) Aristophanes _____(e) Persius _______________________________ Ans. (d) : Aristophanes, often referred to as the father of comedy curate the worlds earliest surviving comic dramas. Aristophanes was a comic playcuright of ancient Athens and a poet of old Attic comedy. He also known as "The prince of Ancient Comedy". 10. In which year did the Archbishop of Canterbury John Whitgift and the Bishop of London Richard Bancroft issue a decree banning verse satire? (a) 1599 (b) 1699 (c) 1598 (d) 1698 (e) 1655 ________________________________ Ans. (a) : In 1599 the Archbishop of Canterbury John Whitgift and the Bishop of London Richard Bancroft issue a decree banning verse satire. 11. Which of the following is NOT an Ancient Epic? (b) Kumarasambhava (a) Aeneid (d) Metamorphoses (c) Orlando Furioso _____(e) Epic of Gilgamesh Ans. (c) : Urdovico Ariosto Italian poet remembered for his Epic poem Orland Furioso (1516). Which is generally regarded as the finest expression of the literacy tendencies and spiritual attitudes of the Italian Renaissance. It is not an Ancient Epic. CGPSC Asst. Prof. Exam. 2016

12.

Epyllion is also referred to as : (a) Trivial Epic (b) Little Epic (c) Major Epic (d) Grand Epic _____(e) Middling Epic _________________________ Ans. (b) : Epyllion is also referred to as, 'Little Epic". It is brief narrative poem in dactylic hexameter of Ancient Greece. Usually dealing with mythological and romantic themes. It is characterized by lively description miniaturist attitude, scholarly allusion and an elevated tone similar to that of the elegy. 13. Athenian tragedies were performed at a festival in honour of : (a) Apollo (b) Dionysus (c) Zeus (d) Poseidon _____(e) Hermes ______________________________ Ans. (b) : Athenian tragedies were performed at a festival in honour of Dionysus. Dionysus was the Ancient Greek God of wine, winemaking grape, cultivation, fertility, ritual madness, theater and religious ecstasy. His Roman name was Bacchus. He may have been worshiped as early as 1500 - 11000 BCE by Mycenaean Greeks. ______________________ 14. Nietzsche’s The Birth of Tragedy deals with the origins of : (a) Roman Tragedy (b) British Tragedy (c) Shakespearean Tragedy (d) German Tragedy _____(e) Greek Tragedy ________________________ Ans. (e) : Friedrich Wilhelm Nietzsche's the birth of Tragedy deals with the origin of Greek Tragedy. He was a German philosopher, cultural critic, composer, poet writer and philologist whose work has exerted a profound influence on modem intellectual history. Nietzsche claimed the exemplary human being must craft his/her own identity through self-realization and do so without relying on anything transcending that life-such as God or a soul. 15. The Muse of Comedy is : (a) Thalia (b) Clio (c) Calliope (d) Urania _____(e) Erato ________________________________ Ans. (a) : In Greek Mythology, Thalia was one of the muses. The goddess who president over comedy and idyllic poetry. In this context her name means "flourishing", because the praises in her songs flourish through time. Thalia was the goddess of festivity and rich banquets and was associated with Aphrodite as party of her retinue. _____________________________ 16. Who said, ’’When one abandons himself to violent laughter, his condition provokes a violent reaction”? (a) Aristotle (b) Aristophanes (c) Dante (d) Plato (e) Meredith

180

YCT

Ans. (d) : Plato said "when one abandons himself to violent laughter his condition provokes a violent reactions." In his work 'Republic' (388) that the guardians of the state should avoid laughter. 17. The oldest written French Farce is thought to be : (a) The Boy and the Blind Man (b) The Miser (c) Tartuffe (d) Le Dindon _____(e) A Flea in Her Ear ______________________ Ans. (a) : The oldest written French Farce is thought to be "the boy and the blind man." "The boy and the blind man is the name of a 13th century French .... It is an anonymous work. In the play there are two scoundrels, a "blind" beggar" and his servant boy. The blind beggar has a secret hoard of coins which the boy tricks away from him. 18. Who said, ’’History repeats itself, first as tragedy, second as farce”? (a) Arthur Rimbaud (b) Mary Wollstonecraft (c) Karl Marx (d) Peter Shaffer _____(e) Francois Rabelais ______________________ Ans. (c) : Karl Marx said, "History repeats itself first as tragedy second as force. The given statement is taken from essay of Karl Marx ’The Eighteenth premiere of Louis Bonaparte published in 1852 in Die Revolution. 19. Which of the following is NOT a nineteenth century melodrama? (a) The Miller and his Men (b) Pygmalion (c) The Broken Sword (d) The Woodsman's Hut (e) A Tale of Mystery Ans. (b) : Pygmalion is a play by G. B. Shaw, named after a Greek Mythological figure. It was written in 16 October 1913. In ancient Greek Mythology. Pygmalion fell in love with one of his sculptures, which them came to life.___________________________________ 20. Who said, ”It is the custom on the stage in all good, murderous melodramas, to present the tragic and the comic scenes in as regular alternation as the layers of red and white in a side of streaky, well-cured becon.”? (a) Ian Gregor (b) E. M. Forster (c) Henry James (d) Thomas Hardy _____(e) Charles Dickens _______________________ Ans. (e) : The given statement is said by Charles Dickens in his novel ’Oliver Twist’ or the parish boy's progress. It was published as a serial from 1837 to 1839 and released as a three volume book in 1838. The story centres on orphan Oliver Twist bom in a workhouse and into apprenticeship with an undertaker. CGPSC Asst. Prof. Exam. 2016

The theme is the age old battle between good and evil means the principle of good surviving through every adverse circumstance and triumphing at last. 21. Which of the following is a one-act play by Arthur Miller? (a) The Bald Soprano (b) A Marriage Proposal (c) A Memory of two Mondays (d) Line _____(e) Pariah _______________________________ Ans. (c) : A memory of two Mondays is one-act play by Arthur Miller. He began writing the play in 1952. While working on the crucible and completed it in 1955. It was finally published in 1976. The novel based on Miller's own experiences the play focuses on a group of desperate workers earning their livings. 22. Which among the following is NOT a one-act play by Tennessee Williams? (a) The Magic Tower (b) At Liberty (c) Something Unspoken (d) Insider Outsider _____(e) The Dark Room _______________________ Ans. (d) : In the given option 'Insider Outsider' is not a one-act play by Tennessee Williams. Thomas laier Williams III (1911- 1983) known by his pen name Tennessee Williams. He is widely regarded as one of the greatest play wrights in American history. Between the mid- 1940s and the early 1960s, he wrote several award - winning plays including the glass menagerie', ’A street car named desire', 'cat on a hat Tin Roof. 23. The masque of the seven deadly sins appears in : (a) The Faerie Queene (b) Hamlet (c) Macbeth (d) The Spanish Tragedy _____(e) The Tempest __________________________ Ans. (a) : 'The Faerie Queen' on allegorical epic written by the 16th century poet Edmund Spenser. Spenser creates an allegory : The characters of his faroff fanciful 'Faerie land' are meant to have a symbolic meaning in the real world. She earns to incorporate chaste resistance with active love, which is what Spenser sees as true Christian love. The masque of the seven deadly sins appears in the Faerie Queene. 24. Ben Jonson wrote a number of masques with stage design by : (a) Dane Crosby (b) Hugo Mark (c) Chris Sadder (d) Mark Faber _____(e) Inigo Jones ___________________________ Ans. (e) : Ben Jonson wrote a number of masques with stage design by Inigo. Jones worked as a producer and architect for masques from 1605 to 1640, but his most known work in this field came from his collaboration with poet and playwright Ben Jenson. Jone's work on masques with Jonson is credited to be one of the first instances of scenery introduced in theater.

181

YCT

25.

At what age did Shakespeare get married to Anne Hathaway? (a) 14 (b) 24 (c) 28 (d) 18 (e) 16 __________________________________ Ans. (d) : At the age 18, Shakespeare got married to Anne Hathaway. William Shakespeare (1564-1616) was an English playwright, poet and actor widely regarded as the greatest writer in the English language and the world's greatest dramatist. He is often called England's Nation Poet and the 'Bard of Avon.' ________ 26. The First Folio of Shakespeare’s works had a poem as preface by : (a) Shakespeare (b) JohnHeminges (c) Ben Jonson (d) John Hall _____(e) Henry Condell ________________________ Ans. (c) : The First Folio of Shakespeare's works had a poem as preface by Ben Benson. The First Folio famously includes 36 Shakespeare plays - 18 of which had never been published before. But there's even more to the first folio. The full title of the first folio includes the three categories into which the first folio divided the plays : Mr. William. 27. Fortinbras in Hamlet is : (a) Prince of Denmark (b) Prince of Norway (c) Prince of Holland (d) Prince of Portugal _____(e) Prince of Sweden ______________________ Ans. (b) : Fortinbras in Hamlet is Prince of Norway whose father the king Calso named Fortinbras was killed by Hamlet's father (also named Hamlet). Now Fortinbras wishes to attack Denmark to avenge his father's honour making him another fail for prince Hamlet. 28. ’’There is nothing either good or bad, but makes it so.” [from Hamlet] (a) telling (b) saying (c) imagining (d) speaking _____(e) thinking ______________________________ Ans. (e) : The given statement "there is nothing either good or bad, but thinking makes it so." from Hamlet. Act 2 Scene-2. It's mean that Hamlet is a prisoner of his own thinking and of his knowledge that his stepfather is a fratricide and his mother incestuous. Hamlet is not indulging in ethical relativism as much as wishing for blissful ignorance. 29. Who is Trinculo in the play the Tempest? (a) King's Butler (b) King's Jester (c) King's Slave (d) King's Son _____(e) King's Brother ________________________ Ans. (b) : The Tempest is a play by English playwright william Shakespeare probably written in 1610 - 11. It is a play about magic betrayal love and forgiveness. It is set on on Island somewhere near Italy where Gospero the onetime Duke of Milan and his beautiful daughterMiranda live with a ???? called Ariel and a strange wildman called caliban 'Trinculo' in the play 'the tempest' is Alonso's (King of Naples) Jester. CGPSC Asst. Prof. Exam. 2016

30.

Who says, ’’Good wombs have borne bad sons.” From Tempest? (a) Prospero (b) Caliban (c) Ferdinand (d) Miranda _____(e) Ariel ________________________________ Ans. (d) : Miranda says, "Good wombs have bom bad sons." in Tempest. Miranda says it and expresses surprise that a lonely woman like her grandmother could give birth to an evil man like her uncle Antonio. The play 'Tempest' is a play by English playwright in 1610-1611 and thought to be one of the last plays of Shakespeare. 31. Which text of Milton does the following line appear in, ’’Thou canst not touch the freedom of my mind”? (a) Paradise Lost (b) Comus (c) Paradise Regained (d) Areopagitica _____(e) Eikonoklastes _________________________ Ans. (b) : Comus is a masque in honour of chastity written by John Milton. The basic phenomenon of the text in honour of the earl becoming lord President of wales. The given sentence has been taken from his book 'Comus'. It was originally published in 1634. 32. Which text of Milton ends with, ’’They also serve who only stand and wait”? (a) Lycidas (b) Paradise Lost (c) Paradise Regained (d) Samson Agonistes _____(e) On His Blindness ______________________ Ans. (e) : 'On His Blindness' is written by John Milton originally published in 1673. It is one of the best known sonnet of John Milton. The given lines has been taken from his poem 'On his Blindness.' 33. John Milton studied at : (a) Cambridge (b) Oxford (c) York (d) Dublin _____(e) Brighton _____________________________ Ans. (a) : John Milton (1608 - 1674) was an English poet and intellectual who served as a civil servant for the common wealth of England. In his study Milton attended St. Paul's school in London for the study of Latin and Greek. In 1625, Milton Begam attending Christ's college Cambridge. He graduated with a B.A. in 1629. ______________________________________ 34. An Essay of Man is often said to ------- the ways of God to Man. (a) justify (b) defend (c) vindicate (d) explain _____(e) rationalize ____________________________ Ans. (c) : 'An Essay of Man' is poem published by Alexander Pope in 1733-34. It is an effort to rationalize or rather "indicate the ways of God to man, a vindicate variation of John Milton's claim in the opening lines of Paradise lost, that he will "justify the ways of God to men."

182

YCT

35.

Who among the following was NOT a member of the Scriblerus Club? (a) Jonathan Swift (b) John Heywood (c) John Gay (d) Thomas Parnell _____(e) John Arbuthnot ________________________ Ans. (b) : Scriblerus Club, 18 th century British literary club whose founding members were the brilliant tory wits Alexander Pope. Jonathan Swift John Gay. Thomas Parnell and John Arbuthnot. Scriblerus Club purpose was to ridicule pretentious erudition and scholarly Jargon through the person of fictitious literary hack, Martinus Scriblerus. 36. How many epistles are there in An Essay of Man? (a) Two (b) Three (c) Four (d) Five (e) Six Ans. (c) : An Essay of Man is a poem by Alexander Pope in 1733-34. It was dedicated to Henry St. John. There are four Epistle in An essay on man. A central theme of the poem is that the universe has an order to it created by God. As part of the order all God’s creatures are put on earth for a purpose. 37. The Vanity of Human Wishes is an imitation of satire by : (a) Horace (b) Juvenal (c) Menippus (d) Aristophanes _____(e) Lucilius ______________________________ Ans. (b) : The Vanity of Human Wishes : The Tenth Satire of Juvenal Imitated is a poem by Samuel Johnson. It was written in late 1748 and published in 1749. As the subtitle suggests, it is an imitation of satire x by the Latin poet Juvenal. 38. The vanity of Human Wishes is written in : (a) villanelle (b) ottavarima (c) rhyme royal (d) terzarima _____(e) heroic couplets ________________________ Ans. (e) : ’’The vanity of Human Wishes” is a poem of 368 lines written in closed heroic completes. The opening lines of this poem announce the universal scope of the poem as well as its central theme that "the antidote to vain human wishes is non-vain spiritual wishes." It is an imitation of satire x by the Latin poet juvenal. 39. In which year was Johnson’s A Dictionary of the English Language published? (a) 1755 (b) 1855 (c) 1759 (d) 1859 (e) 1795 ________________________________ Ans. (a) : ’A Dictionary of the English Language’ is written by Samuel Johnson and published on 15 April 1755. It is among the most influential dictionaries in the history of the English Language. The purpose of this dictionary to write "a dictionary by which the pronunciation of our language may be fixed and its attainment facilitated by which its purity may be preserved. Its use ascertained and its duration lengthened." CGPSC Asst. Prof. Exam. 2016

40.

Wordsworth’s The Borderers is a verse tragedy set during the reign of : (a) King Henry I (b) King Henry IV (c) King Henry V (d) King Henry III _____(e) King Henry II _________________________ Ans. (d) : ’The borderers’ book by William Wordsworth is a verse tragedy set during the reign of king Henry III. It is part of the tradition classics. It contains classical literature works from even two thousand years. ________________________________ 41. Which river figures in the ’Tintern Abbey’? (a) Wye (b) Rye (c) Trent (d) Ouse _____(e) Severn _______________________________ Ans. (a) : ’Wye’ river figures in the "Tintern Abbey". It is the young words worth’s first great statement of his principle them that the memory of pure communion with nature in childhood works. The title lines written (or composed) a Few Miles above Tintern Abbey, July 13, 1798. _________________________________ 42. Which poem of Wordsworth do these lines appear in : ’’Wisdom is oft-times nearer when we stoop Than when we soar”? (a) The Prelude (b) Lyrical Ballads (c) The White Doe of Rylstone (d) Peter Bell _____(e) The excursion _________________________ Ans. (e) : The excursion being a portion of the Recluse is a long poem by romantic poet William Wordsworth and given lines have been taken from the poem ’excursion’.____________________________________ 43. Wordsworth’s Immortality Ode contains stanzas. (a) 7 (b) 9 (c) 11 (d) 14 (e) 5 __________ Ans. (c) : "Ode : Intimations of immortality from recollections of early childhood is a poem by William Wordsworth completed in 1804 and published in poems in volumes (1807). The poem was completed in two parts. The poem contains 11 stanzas. ____________ 44. Keats’s tombstone reads : (a) "Here lies One Whose Name was writ in Poetry." (b) "Here lies One Whose Name was writ in Words." (c) "Here lies One Whose Name was writ in the Hearts." (d) "Here lies One Whose Name was writ in the Minds." (e) "Here lies One Whose Name was writ in ________Water." ______________________________ Ans. (e) : John Keats (1795-1821) was an English poet and prominent figure in the second generation of romantic poets with Lord Byron and P.B. Shelley. The written words on Keats tombstone "here lies one whose name was writ in water."

183

YCT

45.

’’Heard melodies are sweet, but those unheard, are sweeter” appears in : (a) Ode to Psyche (b) Ode on a Grecian Um (c) Ode to a Nightingale (d) Ode on Melancholy _____(e) Ode on Indolence ______________________ Ans. (b) : "Ode on a Grecian Um" is a poem written by John Keats in 1819, first published manyneusly in Annals of the Fine Arts for 1819. The central theme of the poem is the transient nature of human existence. The given lines appear in "Ode on a Grecian Um." The tone of "Ode on Grecian Um" is part melancholy and part wonder and praise. 46. ”A thing of beauty is a joy forever” is a line from Keats’s : (a) To Autumn (b) Ode on a Grecian Um (c) Ode on Melancholy (d) Endymion _____(e) Ode to a Nightingale ____________________ Ans. (d) : "A thing of beauty is a joy forever" is a line from Keats’s ’Endymion’. Endymion is a poem by John Keats first published in 1818 by Taylor and Hessey of Elect street in London. The given lines are the starting line of poem. It starts by painting a rustic scene of trees, rivers, shepherds and sheep. 47. Which among the following does not appear in Poems, Chiefly Lyrical? (a) Mariana (b) All Things Will Die (c) The Dying Swan (d) Nothing will Die _____(e) Maud ________________________________ Ans. (e) : ’Poems, Chiefly Lyrical’ book by Alfred Tennyson. It was published in 1830 and poems numbered 1 - XXIV. Many of the poems contain experimental elements such as irregular metres and words employed for their musical on evocative powers rather than for their strict meaning. All three options except of ’mand’ appears in this poem. 48. Tennyson’s ’’Ulysses” was published in : (a) 1842 (b) 1932 (c) 1822 (d) 1852 (e) 1862 ________________________________ Ans. (a) : Alfred, Lord Tennyson’s (1809-1892) "Ulysses" is a poem in blank verse written in 1833 and published in 1842. The poem is written in dramatic monologue form. In this poem Tennyson draws on Hamer’s narrative of Iliad and Odyssey. Tennyson's Ulysses recalls Dandies Ulysses in his Inferno. 49. Who called Tennyson’s ’’Ulysses” a ’’perfect poem”? (a) John Ruskin (b) Thomas Carlyle (c) Ezra Pound (d) T. S. Eliot (e) Basil Wiley

CGPSC Asst. Prof. Exam. 2016

Ans. (d) : T. S. Eliot called Tennyson's "Ulysses" a "Perfect poem." It is a popular example of the dramatic monologue. Facing old age, mythical hero Ulysses describes his discontent and restlessness upon returning to his kingdom. Ithaca after his for-ranging travels. 50. ”Tis better to have loved and lost than never to have loved at all” appears in Tennyson’s : (a) The Charge of the Light Brigade (b) InMemoriam (c) Mariana (d) The Princess _____(e) The Window __________________________ Ans. (b) : The given lines appears in Tennyson’s poem "In memoriam A.H.H" published in 1850. It is a requiem for the poet’s beloved friend Arthur Henry Hallam, who died suddenly of a cerebral Hemorrhage in Vienna in 1833. The original title of the poem was "The way of the soul". 51. The poem ’Andrea del Sarto’ was inspired by a: (a) religious authority (b) philosopher (c) Victorian writer (d) renaissance artist _____(e) warrior from the Middle Ages ____________ Ans. (d) : The poem ’Andrea del Sarto’ also called "The faultless painter" is a poem by Robert Browning (1812-1889). It was published in 1855. The poem was inspired by a Renaissance artist Raphael Leonardo do Virci and Era Bartolommeo. 52. Which among the following was NOT written by Browning? (a) Princess (b) Pauline (c) Abt Vogler (d) The Ring and the Book _____(e) Paracelsus ____________________________ Ans. (a) : The princess is a Serio-comic blank verse narrative poem written by Alfred Tennyson published in 1847. All other options are related to Robert Browning. 53. Rabbi Ben Ezra by Robert Browning was published at : (b) 1861 (a) 1860 (d) 1863 (c) 1862 (e) 1864________ Ans. (e) : Rabbi Ben Ezra is a poem by Robert Browning about Abraham Ibn Ezra, one of the great poets. He wrote on grammar astronomy. The astrolabe etc. It is not a biography of Abraham Ibn Ezra; like all of Brownin’s historical poems. It is a free interpretation of the idea that Ibn Ezra’s life and work suggests to Browning.

184

YCT

54.

’Andrea del Sarto’ appears in a collection of Ans. (b) : Oliver Twist, in full Oliver Twist; or the poems called: Parish Boy’s Progress, novel by Charles Dickens, (a) Bells and Pomegranates published serially under the pseudonym "Boz” from (b) The Ring and the Book 1837 to 1839 in Bentley’s Miscellany and in a Three (c) Men and Women Volume in 1838. (d) Dramatic Idylls 59. Oliver Twist was originally published in ______(e) Dramatis Personae ______________________ monthly installments in the Magazine Bentley's Ans. (c) : ’Men and Women’ is collection of fifty-one Miscellany from: poems in two volumes by Robert Browning, first (a) 1832-33 (b) 1833-35 published in 1855. The poem contains conversations (c) 1835-36 (d) 1837-39 between Bishop Blougram and Gigadibs, a journalist. (e) 1839-40 The two men argue about the nature of reality and the Ans. (d) : Oliver Twist is Charles Dickens's second nature of faith. ’Andrea del Sarto’ appears in this novel and was published as a serial from 1837 to 1839 collection of poems. and released as a three volume book in 1838. Oliver Twist is a story about the battles of good versus evil, 55. Matthew Arnold worked as a: with the evil continually trying to corrupt and exploit (a) pastor (b) school inspector the good. (c) librarian (d) rugby coach ______(e) bookseller ____________________________ 60. Which novel does the following quote of Dickens belong to: ”1 care for no man on earth, Ans. (b) : Matthew Arnold (1822-1888) was an and no man on earth cares for me”? English poet and cultural critic who worked as an (a) Great Expectations inspector of schools for thirty five years, and supported (b) A Tale of Two Cities the concept of state regulated secondary education. He (c) Hard Times is known for his notable works ’Dover Beach’, ’The (d) David Copperfield Scholar Gipsy', ’Thyrsis' and ’Culture and Anarchy’. ______(e) Our Mutual Friend ______________________ 56. Which of the following does not mention Ans. (b) : ’A Tale of Two Cities' is an 1859 historical ’Dover Beach’? novel by Charles Dickens. The main idea of ’A Tale of (a) Fahrenheit 451 (b) Catch 22 Two Cities’ is the concept of resurrection. Characters (c) I Capture the Castle (d) Things Fall are brought metaphorically brought back to live ______(e) Saturday ______________________________ thought the novel. The portrayal of the French Ans. (d) : All the given options words are included in Revolution and the effect it had on families and the ’Dover Beach’ poem. It is a lyrical poem by the English people is something that was done very well by poet Matthew Arnold. It was published in 1867 in the Dickens ’Collection New Poems'. Arnold begins the poem with Who called Dickens ’’the greatest of superficial a naturalistic and detailed nightscape of the Beach at 61. novelists”? Dover in which auditory imagery plays a significant (a) Leo Tolstoy role. (b) Anton Chekhov 57. Who said, ’’Arnold is, at his best, a very good (c) William Wordsworth but highly derivative poet”? (d) Virginia Woolf (a) John Cowper Powys (b) Lionel Trilling ______(e) Henry James __________________________ (c) David Lodge (d) Northrop Frye ______(e) Harold Bloom _________________________ Ans. (e) : Henry James called Charles Dickens, "the greatest of superficial novelists "in a review of our Ans. (e) : Harold Bloom said, ’’Arnold is, at his best, a Mutual Friend, in The Nation, December 21, 1865.” If very good but highly derivative poet.” Matthew we might hazard a definition of his literary character, Arnold's most famous piece of literary criticism in his we should accordingly call him the greatest of essay ’’The Study of Poetry”. In this work, Arnold is superficial novelists. fundamentally concerned with poetry’s ’’high destiny”, What is the subtitle of the novel Tess of the he believes that "mankind will discover that we have to 62. d'Urbervillesl turn to poetry to interpret life for us to console us, to (a) A Pure Woman Faithfully Represented sustain us" as science and philosophy will eventually (b) An Impure Woman Faithfully Presented prove filensy and unstable. _______________________ (c) A Pure Woman Faithfully Presented 58. Oliver Twist is also known as: (d) A Pure Woman Unfaithfully Presented (a) The Undertaker (e) An Impure Woman Faithfully Represented (b) The Parish Boy’s Progress Ans. (c) : ’Tess of the d'Urbervilles' is a novel by (c) The Trials and Tribulations of Oliver Thomas Hardy. The subtitle of the novel is "A Pure (d) Twists of Fate Woman Faithfully Presented.” It initially appeared in a (e) A Suitable Fate CGPSC Asst. Prof. Exam. 2016

185

YCT

censored and serialized version, published by the British illustrated newspaper 'The Graphic' in 1891, then in book form in three volumes in 1891 and as a single volume in 1892. 63. Tess of the d'Urbervilles initially appeared in a serialized version in: (a) The Graphic (b) The TLS (c) The Magazine (d) The Three Decker (e) The Literary Ans. (a) : 'Tess of d'Urbervilles' initially appeared in a censored and serialized version, published by the British illustrated newspaper 'The Graphic' in 1891, then in book form in three volumes in 1891 and as a single volume in 1892. In it Thomas Hardy was breaking the sexual norms of his society and challenging the sexual double standard that existed. So it was initially banned. 64. In which novel does Hardy say, ’’Love is a possible strength in an actual weakness”? (a) A Pair of Blue Eyes (b) The Woodlanders (c) The Return of the Native (d) Far From the Madding Crowd (e) Desperate Remedies Ans. (d) : 'Far From the Madding Crowd' is Thomas Hardy's fourth novel. It originally appeared anonymously as a monthly serial in Comhill Magazine, where it gained a wide readership. The novel is the first to be set in Thomas Hardy's Wessex in rural Southwest England. It was published in 1874. 65. Which of the following is NOT a short story by Hardy? (a) Enter a Dragon (b) The Dynasts (c) The Lady Penelope (d) A Mere Interlude (e) The Three Strangers Ans. (b) : 'The Dynasts' is an English language closet drama in verse by Thomas Hardy. Hardy himself described this work as "an epic-drama of the war with Napoleon, in three parts, nineteen acts and one hundred and thirty scenes." Not counting the Forescene and the Afterscene, the exact total number of scenes is 131. It was originally published in 1984. All other options are short stories. 66. William Makepeace Thackeray was born in: (a) Kolkata (b) Delhi (c) London (d) Dublin (e) Lisbon Ans. (a) : William Makepeace Thackeray was a British novelist and author. He was bom in Kolkata in 1811 and died in 1863. He is known for his satirical works 'Vanity Fair' (1848). CGPSC Asst. Prof. Exam. 2016

67.

The History of Henry Esmond tells the story of the early life of Henry Esmond, a colonel in the service of: (a) Queen Anne (b) Queen Elizabeth (c) Queen Victoria (d) Queen Mary (e) Queen Matilda Ans. (a) : 'The History of Henry Esmond' is a historical novel by William Makepeace Thackeray, originally published in 1852. The book tells the story of the early life of Henry Esmond, a colonel in the service of Queen Anne of England. 68. In the novel, The History of Henry Esmond, who of the following does not figure? (a) Joseph Addison (b) Richard Steele (c) Jonathan Swift (d) Duke of Marlborough (e) Duke of Neckar Ans. (e) : 'The History of Henry Esmond' is a historical novel by W. M. Thackeray, published in 1852. It is a typical example of Victorian historical novels. In the novel, 'The History of Henry Esmond', Duke of Neckar was not mentioned. 69. Which of the following cannot be attributed to William Makepeace Thackeray? (a) The Irish Sketchbook (b) The Book of Snobs (c) Men's Wives (d) Wives of Great Men (e) The Adventures of Philip Ans. (d) : 'Wives of Great Men' is not be attributed to William Makepeace Thackeray. He was a British novelist and author. He is best known for his novel 'Vanity Fair' in 1848, for his satirical work. All other options are attributed to W.M. Thackeray. 70. Which among the following is NOT an essential element of tragedy, according to Aristotle? (a) Thought (b) Oracle (c) Style (d) Spectacle (e) Character Ans. (b) : According to Aristotle "Tragedy is an imitation of an action that is serious, complete and of a certain magnitude through pity and fear affecting the proper purgation (Catharsis) of these emotions". So the main components of Tragedy is 'Thought', 'Style', 'Spectacle' and 'Character'. Oracle is not important for Tragedy. 71. What does Anagnorisis in Aristotle’s Poetics refer to? (a) Reversal (b) Purgation (c) Recognition (d) Imitation (e) Tragic Flaw Ans. (c) : Anagnorisis in Aristotle's 'Poetics' refer in a literary work, the starting discovery that produces a change from ignorance to knowledge. It is discussed by Aristotle as an essential part of the plot of a tragedy, although anagnorisis occurs in comedy, epic and at a later date the novel as well.

186

YCT

72.

The treatise On the Sublime is dedicated to: (a) Posthumius Terentianus (b) Plotinus (c) Queen of Palmyra (d) Cassius (e) Homer Ans. (a) : In the Sublime is a Roman-era Greek work of literary criticism dated to the 1st century AD. Its author is unknown, but is conventionally referred to as Longinus or Pseudo-Longinus. It is both a treatise on aesthetics. The treatise is dedicated to Posthumius Terentianus, a cultured Roman and public figure. 73. Which poem of Sappho does Longinus refer to as an example of the sublime? (a) Ode to Aphrodite (b) Ode to Athena (c) Ode to Hope (d) Ode to Jealousy (e) Ode to Love Ans. (d) : Sappho’s poem of 'Ode to Jealousy’ survives only because the ancient critic Longinus quoted it as a supreme example of poetic intensity, "Are you not amazed at how she evokes soul body, hearing, tongue, sight, skin, as though they were external and belonged to someone else? 74. In which year was John Dryden made a Poet Laureate? (a) 1628 (b) 1648 (c) 1668 (d) 1688 (e) 1698 Ans. (c) : John Dryden (1631-1700) was an English poet, literary critic, translator and playwright who was appointed England’s first poet Laureate in 1668. A Romanticist writer Sir Walter Scott called him ’’Glorious John". 75. Which, among the following, does not make an appearance in Essay of Dramatic Poetry*! (a) Longinus (b) Aristotle (c) Plato (d) Polonius (e) Menos Ans. (d) : ’Essay of Dramatic Poetry’ is a work by John Dryden. It was published in 1668. In this drama the dialogues between four speakers Eugenics, Crites, Lisideius and Neakder. In this drama John Dryden mentioned 'Longinus, Aristotle, Plato, and Menos, not ’Polonius’. 76. Which among the following cannot be ascribed to Dryden? (a) The Indian Emperour (b) The Conquest of Granada (c) The Conquest of Indiana (d) Religio Laici (e) King Arthur Ans. (c) : 'The Conquest of Indiana’ cannot be ascribed to Dryden. All other options are ascribed to Dryden. 77. Biographia Literaria was originally intended to be: (a) a poem (b) a preface (c) an epilogue (d) a biography (e) a chronicle CGPSC Asst. Prof. Exam. 2016

Ans. (b) : 'Biographia Literaria’ is an autobiography in discourse by Samuel Taylor Coleridge, which he published in 1817, in two volumes of twenty volumes of twenty three chapters. It was originally intended to be a preface 78. In which year was Biographia Literaria published? (a) 1802 (b) 1807 (c) 1817 (d) 1719 (e) 1811 Ans. (c) : The Romantic poet Samuel Taylor Coleridge published ’Biographia Literaria’ in 1817. It is his semiautobiographical work on aesthetic theory, opens with the recollection of Coleridge's education at Christ’s Hospital grammar school. 79. Who according to Arnold, lacks the ’’high seriousness” of classic poets? (a) Chaucer (b) Milton (c) Spencer (d) Shakespeare (e) Wordsworth Ans. (a) : Matthew Arnold’s Comparative Study shows that Chaucer does not have the quality of high seriousness like - Dante, Homer, Shakespeare and Milton. Chaucer fails to represent high seriousness in his writing. This lacking excludes him from the group of the classics ’High Seriousness’ - is the product of poetic truth and poetic beauty. 80. ’’Tradition and the Individual Talent” was first published in: (a) The Matrix (b) The Grand Review (c) The Little Magazine (d) The Egoist (e) Great Writers Ans. (d) : ’Tradition and the Individual Talent’ by T.S. Eliot is an early critical essay by T.S. Eliot. The essay was published across two issues of ’The Egoist’, a magazine for which Eliot had become the assistant editor in 1917. 81. In which poem of T. S. Eliot do these lines appear, ”1 have heard the mermaids singing, each to each. I do not think that they will sing to me”? (a) The Hollow Men (b) The Love Song of J. Alfred Prufrock (c) The Waste Land (d) Four Quarter (e) Ash Wednesday Ans. (b) : The given line 'I have heard the mermaids singing, each. I do not think that they will sing to me’ is from the poem of T.S. Eliot's 'The Love Song of J. Alfred Prufrock’, commonly known as Prufrock is the first professionally published poem by American bom British poet T.S. Eliot. Eliot began writing ’Prufrock’ in Feb. 1910 and it was first published in the June 1915 issue of poetry, a magazine of verse at the instigation of Ezra Pound.

187

YCT

82.

Which poet made illustrations for "II Penseroso"? (a) T S Eliot (b) Milton (c) Blake (d) Keats (e) Spencer Ans. (c) : During the 18th century both ”11 Penseroso” and "L' Allegro" were popular and were widely imitated. The poet, W. Blake who was deeply influenced by Milton's poetry and personality, made illustrations to both - L' Allegro and II Penseroso. 83. "Only connect ..." appears as an epigraph in: (a) A Passage to India (b) A Room with a View (c) Maurice (d) The Longest Journey (e) Howards End Ans. (e) : 'The Howards End' of E.M. Forster's 1910 novel is a country house where the action begins and ends. The novel's epigraph is 'Only connect' and its narrative tension comes from the contrast between the Schlegel and the Wilcox families. 84. The title "A Passage to India” in the novel A Passage to India is borrowed from a poem by: (a) E E Cummings (b) W H Auden (c) Elizabeth Bishop (d) Walt Whitman (e) Wallace Stevens Ans. (d) : The novel 'A Passage to India' is based on Forster's experience in India, during the title from 'Walt Whitman's 1870 poem - 'Leaves and Grass'. The story revolves around four characters; Dr. Aziz, his British friend, Mr. Cyril, Fielding. 85. "Major Callendar” in the novel A Passage to India is a: (a) Doctor (b) Barrister (c) Superintendent of Police (d) Collector (e) Headmaster Ans. (a) : 'Major Callendar' in the novel 'A Passage to India' is the Civil surgeon at Chandrapore, Dr. Aziz's superior. 'Major Callendar' is a boastful, cruel, intolerant and ridiculous man. 86. The novel Sons and Lovers is dedicated to: (a) Miriam (b) Lucy (c) Garnett (d) Mary (e) Lily Ans. (c) : The novel 'Sons and Lovers', a famous novel by D. H. Lawrence. It contains a dedication to Edward Garnett. David Herbert Lawrence is a great and controversial English writer of 20th century. This novel was first published in - 1913. 87. Which among the following is NOT written by D H Lawrence? (a) The Rainbow (b) The Lost Girl (c) The Escaped Cock (d) The Plumed Serpent (e) The Hind and the Panther CGPSC Asst. Prof. Exam. 2016

Ans. (e) : In the given question, all the works of D. H. Lawrence except 'Hind and the Panther'. It is a poem in three parts and an allegory in heroic couplets by John Dryden. It is much the longest of Dryden's poems, written in 1687. The poet argued the case for his adopted Church. 88. Who among the following does NOT appear in the play Saint Joan*! (a) Canon de Chantilly (b) Canon de Courcelles (c) Duchess de la Tremouille (d) Archbishop of Rheims (e) Canon John D'Estivet Ans. (a) : 'Saint Joan' is a play by George Bernard Shaw about 15th century French military figure Joan of Arc in 1923 three year after her canonization by the Roman Catholic Church, the play reflects Shaw's belief that the people involved in Joan's trial acted according to what they thought was right. There 'Canon de Chantilly' does not appear in the play. 89. The lines, "Turning and turning in the widening gyre/The falcon cannot hear the falconer” appear in: (a) Sailing to Byzantium (b) The Second Coming (c) A Prayer for my Daughter (d) A Man Young and Old (e) Lake isle of Innisfree Ans. (b) : The above given line appear in the poem 'The Second Coming' by William Butler Yeats (1919) 'Turning and turning in the widening gyre'. According Yeats it will be a day when nature is disturbed, when good people are apathetic and when evil comes home to roost. 90. ”A Prayer for my Daughter” is written to: (a) Kim (b) Lily (c) Anne (d) Mindy (e) Lucy Ans. (c) : 'A Prayer for my Daughter' is written to 'Anne', his daughter with Georgic Hyde Lees, whom Yeats married after his last marriage proposal to Maud Gonne was rejected in 1916. 91. ’’Sailing to Byzantium” uses the metaphor of journey to: (a) Egypt (b) Constantinople (c) India (d) England (e) Spain Ans. (b) : 'Sailing to Byzantium' is a poem by William Butler Yeats, first published in the 1928 collection 'The Tower'. It comprises four stanzas in Ottava Rima, each made up of eight lines of iambic pentameter. It uses a journey to Byzantium (Constantinople) 92. In which year was the poem "Gerontion" published? (a) 1919 (b) 1920 (c) 1921 (d) 1922 (e) 1923

188

YCT

Ans. (b) : 'Gerontion' is a poem by T.S. Eliot that was first published in 1920. The poem is dramatic monologue relating the opinions and impressions of an elderly man which describes Europe after World War I. 93. Which of the following is NOT a section of The Waste Land?. (a) Death by Water (b) The Fire Sermon (c) What the Thunder Said (d) A Game of Thrones (e) The Burial of the Dead Ans. (d) : 'The Waste Land’ is a poem by T.S. Eliot, widely regarded as one of the most important poems of the 20 th century and a central work of modernist poetry which has five sections and there is not 'A Game of Thrones' in it. The five sections of the 'Waste Land' are entitled as - The Burial of the Dead, A Game of Chess, What the Thunder Said, and Death by Water. 94. ’’The Shield of Achilles” refers to Achilles who was a/an: (a) Chinese Emperor (b) Egyptian Warrior (c) Roman King (d) Greek Hero (e) Macedonian Prince Ans. (d) : 'The Shield of Achilles’ is the shield that Achilles uses in his fight with Hector. It is the poem by W. H. Auden first published in 1952 and the title work of a collection of poem by Auden, published in 1955. It symbolizes the world beyond the battle field and implies that war constitutes only one aspect of existence. Life is as a whole. The Shield reminds us, includes feasts and dances and marketplaces and crops being harvested. In Greek mythology, Achilles was the stronger warrior and 'Hero' in the Greek army during the Trojan War. 95. ’’The American Scholar” was a speech given by Ralph Waldo Emerson in: (a) Cambridge, Massachusetts (b) Chicago, Illinois (c) Houston, Texas (d) New York City, New York (e) Los Angeles, California Ans. (a) : 'The American Scholar' was a speech given by Ralph Waldo Emerson on August 31, 1837, to the Phi Beta Kappa Society of Harvard College at the First Parish in Cambridge, Massachusetts. This essay was delivered at Cambridge in 1837. In his speech, Emerson called for the young country to develop a national intellectual life distinct from lingering colonial influences. 96. Thoreau’s essay ’’Civil Disobedience” is motivated by his disgust for: (a) poverty (b) gender discrimination (c) race issues (d) rural crisis (e) slavery CGPSC Asst. Prof. Exam. 2016

Ans. (e) : 'Civil Disobedience' is called resistance to Civil Government. It is essay by American transcendentalist 'Henry David Thoreau' that was first published in 1849. 'Thoreau' was motivated in part by his disgust with slavery and the Mexican-American War (1846-1848). Thoreau's Civil Disobedience espouses the need to prioritize one's conscience over the dictates of laws. It criticizes American social institutions and policies most prominently slavery and the Mexican-American War. 97. The Scarlet Letter is set in the: (a) 16th century (b) 19th century th (c) 18 century (d) 17th century th (e) 20 century Ans. (d) : 'The Scarlet Letter' is set in Boston in the 17th century, prior to American Independence. The theme of Scarlet Letter is a universal idea presented in a literary piece of work. 'The Scarlet Letter' by Nathanial Hawthorne include Criticism of Puritan beliefs regarding sin individual and society, social norms and sense of guilt. It tells the story of Hester, Prynne who conceives a daughter out of Wedlock and struggle to create a new life. 98. The Red Badge of Courage is a: (a) Romance (b) Science fiction (c) War novel (d) Thriller (e) One Act Play Ans. (c) : 'The Red Badge of Courage' is a war novel by American author 'Stephen Crane' (1871-1900) taking place during the American Civil War, the story is about a young private of the union Army, Henry Fleming, who flees from the field of battle. Stephen Crane was a 19th century American writer best known for his novelsThe Red Badge of Courage' and 'Maggie'. 99. Yank undergoes an identity crisis in The Hairy Ape when he is called: (a) Filthy pig (b) Filthy beast (c) Dirt bag (d) Scum of the earth (e) Thief Ans. (b) : 'The Hairy Ape' (1922) is an expressionist play by American playwright 'Eugene O'Neill'. It is about a beastly, un thinking Laborer known as 'Yank', the protagonist of ......... business refers to him as a 'filthy beast' protagonist of the play as he searches for a sense of belonging in a world controlled by the rich. 100. Which of the following is NOT by Eugene O'Neill? (a) Shell Shock (b) Welded (c) Mourning Becomes Electra (d) Days Without End (e) The Birthday Party Ans. (e) : 'Birth Day Party' is the full length play by Harold Pinter, first published in London in 1950. It is one of the best known and most frequently performed play. The rest are the works of 'Eugene O'Neill' such as - Shell Shock (1918), Welded (1923), Mourning Becomes Electra (1931), Days Without End (1933).

189

YCT

Chhattisgarh Public Service Commission Asst. Prof. Exam. 2014

ENGLISH (Solved Paper) 1.

The opening line of ’Women’s Declaration of Independence’ - ’men and women are created equal’ - was crafted by: (a) Gloria Steinem (b) Harriet Beecher Stowe (c) Susan B. Anthony (d) Elizabeth Cady Stanton (e) None of the other options _________________ Ans: (d) Elizabeth Cady Stanton crafted the opening line of ’Women’s Declaration of Independence’ in 1848 United States as "We hold these truths to be selfevident, that all men and women are created equal". 2. The term ’Renaissance’ points to ’rebirth’ of: (a) Greek civilization and its associated values (b) Roman civilization and its associated values (c) Greco-Roman values and ethics (d) Greco-Roman civilization and its associated values (e) None of the other options _________________ Ans: (d) The Renaissance typically refers to a period in European history approximately between 14001600. The Renaissance is most closely associated with Italy, where it began in the 14th Century. The term renaissance points to rebirth of Greco-Roman civilization and its associated values. 3. Thomas Hardy’s literary agent was: (a) A. P. Watt (b) J. B. Pinker (c) Curtis Brown (d) John Farquharson (e) None of the other options _________________ Ans: (a) A. P. Watt, Thomas Hardy’s literary agent, who handled the publication of ’On the Western Circuit’ both in England (English Illustrated Magazine, December 1891) and in America (Harper's Weekly 28 November 1891). 4. Names associated with Cavalier poetry are: (a) Chaucer, Langland and Gower (b) Wyatt Surrey and Sidney (c) Webster, Middleton, Ben Jonson (d) Lovelace, Suckling and Carew (e) None of the other options _________________ Ans: (d) Cavalier poet, any of a group of English gentleman poets called Cavaliers because of their loyalty to Charles I during the English Civil Wars as opposed to Roundheads who supported parliament. The term embraces Richard Lovelace, Thomas Carew, Sir John Suckling, Edmund Waller and Robert Herrick. CGPSC Asst. Prof. Exam. 2014

[Exam. Date : 21 May 2016]

5.

The phrase ’’dissociation of sensibility” was first used by T. S. Eliot in: (a) ’The Frontiers of Criticism’ (b) ’What is a Classic?' (c) ’Hamlet and His Problems' (d) The Metaphysical Poets’ (e) None of the other options _________________ Ans: (d) Dissociation of Sensibility is a literary term first used by T. S. Eliot in his essay "The Metaphysical Poets’." It refers to way in which intellectual thought was separated from experience of feeling. ______________________________________ 6. The term ’’collective unconscious” was first defined by: (a) Sigmund Freud (b) Carl G. Jung (c) Simone de Beauvoir (d) Jacques Lacan (e) None of the other options _________________ Ans: (b) The 'Collective Unconscious' is a concept originally defined by psychoanalyst Carl Jung. Sometimes referred to as the "Objective Psyche", it refers to the idea that a segment of the deepest unconscious mind is genetically inherited and is not shaped by personal experience. ___________________ 7. A powerful and enduring influence on theatre in Bombay in the 1950s was by: (a) Ebrahim Alkazi (b) Barry John (c) Jennifer Kapoor (d) GirishKarnad (e) None of the other options _________________ Ans: (a) In the early decades of the republic, as India was building institutions, the National School of Drama in Delhi was headed by Ebrahim Alkazi. Ebrahim Alkazi, doyen of contemporary theatre in India and one of the country’s leading post independence theatre director. ____________________ 8. The concept of ’interpretive communities’ as being ’composed of members who share a particular reading ’’strategy’” was propounded by: (a) Hannah Arendt (b) Stanley Fish (c) Susan Sontag (d) Wolfgang Iser (e) None of the other options _________________ Ans: (b) ’Interpretive Communities' are a theoretical concept stemming from reader response criticism and publicized by Stanley Fish. He propounded the concept of ’Interpretive Communities' as being composed of members who share a particular reading "strategy".

190

YCT

9.

Any single meaningful utterance, spoken or written is: (a) the ’langue' (b) a 'parole' (c) the 'deep structure' (d) a 'morpheme' (e) None of the other options __________________ Ans: (b) The concept of Langue and Parole in Linguistics was introduced by the Swiss Structural Linguist Ferdinand de Saussure in his seminal book "Course in General Linguistic, which was published posthumously in 1916 by Saussure's students. A parole is any single meaningful utterance, spoken or written. 10. The protagonist of Henry James* novel The American, is: (a) Christopher Newman (b) Daisy Miller (c) Isabel Archer (d) Verena Tarrant (e) None of the other options __________________ Ans: (a) The American, novel by Henry James, published in 1876 in the Atlantic Monthly and in book form a year later and produced as a four act play in 1891. 'The American' is the story of a self made American Millionaire Christopher Newman, who is the protagonist of the novel. 11. A name associated with Protestant humanism [humanism under reformist agenda] is: (a) Ben Jonson (b) Edmund Spenser (c) John Milton (d) William Congreve (e) None of the other options __________________ Ans: (c) John Milton was an English poet, polemicist, man of letters and a civil servant for the commonwealth of England under Oliver Cromwell. In his 1641 treatise of Reformation, Milton expressed his dislike for Catholicism and episcopacy. His name is associated with protestant humanism. 12. The term ’affective fallacy’ was coined by: (a) Ezra Pound (b) Wimsatt and Beardsley (c) Raymond Williams (d) I A Richards (e) None of the other options __________________ Ans: (b) The term "affective fallacy" was coined by Wimsatt and Beardsley. In an essay published in 1946, W. K. Wimsatt and Beardsley defined the affective fallacy as the error of evaluating a poem by its effects, especially emotional effects upon the reader. * CGPSC Option (d) ____________________________ 13. A ’recrudescence of interest in culture, myth, legend and literature of Celtic people’ first took place in the: (a) first half of the 1 8 th century (b) second half of the 18th century (c) first half of the 19 th century (d) second half of the 19th century (e) None of the other options

CGPSC Asst. Prof. Exam. 2014

Ans: (b) A recrudescence of interest in culture, myth, legend and literature of Celtic people first took place in - second half of the 18th century. 14. The ’dream vision’ convention is NOT used by Chaucer in: (a) The Book of the Duchess (b) The House of Fame (c) ParlementofFoules (d) Troilus and Criseyde (e) None of the other options __________________ Ans: (d) The 'dream vision' convention is not used by Chaucer in Troilus and Criseyde. Troilus and Criseyde is an epic poem, which retells in Middle English the tragic story of the lover Troilus and Criseyde set against a backdrop of war during the siege of Troy. ___________________________________ 15. A precursor of modern cultural studies is: (a) Friedrich Nietzsche (b) Jacques Derrida (c) Roland Barthes (d) Aime Cesaire (e) None of the other options __________________ Ans: (c) Roland Barthes was French essayist and social literary critic whose writing on semiotics, the formal study of symbol and sign pioneered by Ferdinand de Saussure, helped establish structuralism and the new criticism as leading intellectual movements. He is the precursor of modem cultural studies. 16. Bertolt Brecht wrote Mother Courage and her Children to: (a) celebrate motherhood (b) foreground economic inequity (c) counter the rise of fascism (d) highlight the plight of mothers (e) None of the other options __________________ Ans: (c) 'Mother Courage and her Children' is a play written in 1939 by the German dramatist and poet Bertolt Brecht with significant contributions from Margarete Steffin. 17. The Asiatic Society of Bengal was founded by: (a) Thomas Babington Macaulay (b) Sir William Jones (c) Sri Aurobindo (d) Surendranath Banerji (e) None of the other options __________________ Ans: (b) Asiatic society of Bengal, scholarly society founded on Jan 15, 1784 by Sir William Jones, a British lawyer and orientalist, to encourage oriental studies. 18. Phonology is the study of the: (a) way elementary speech sounds are produced and understood (b) ordering of speech sounds into the smallest meaningful groups (c) way that sequences of words are ordered into phrases, clauses and sentences

191

YCT

(d) combination of words in phrases, sentences and larger linguistic units (e) None of the other options Ans: (b) Phonology is the study of the patterns of sounds in a language and across language. It is also the study of ordering of speech sounds into the smallest meaningful groups. 19. John Donne was said to ’affect the metaphysic* in his poetry by: (a) T. S. Eliot (b) Helen Gardner (c) Samuel Johnson (d) John Dryden (e) None of the other options Ans: (d) John Donne was said 'affect the metaphysics' in his poetry by John Dryden. The Metaphysical poets is a term coined by the poet and critic Samuel Johnson. 20. On the Road is a novel by: (a) Allen Ginsberg (b) Jack Kerouac (c) William S. Burroughs (d) Neal Cassady (e) None of the other options Ans: (b) 'On the Road’ (1957) novel by American writer Jack Kerouac, based on the travels of Kerouac and his friends across the United States. It protagonists living life against a backdrop of Jazz poetry and drug use. 21. ’Cyborg’, a conflation of the words ’cybernetic’ and ’organism*, refers to: (a) an alien creature from outer space (b) a creature of science fiction (c) a biological hybrid, possibly controlled by man (d) an inhabitant of cyber space (e) None of the other options Ans: (c) 'Cyborg'-a conflation of the words 'cybernetic' and 'organism' refers to a biological hybrid, possibly controlled by man. 22. The name of Clifford Odets is associated with: (a) aleatory writing (b) agitprop drama (c) alliterative revival (d) alternative theatre (e) None of the other options Ans: (b) The name of Clifford Odets is associated with agitprop drama. Russian agitprop theater was noted for its cardboard characters of perfect virtue and complete evil, and its coarse ridicule. 23. In T. S. Eliot’s play The Family Reunion, the chorus comprises the: (a) Thersites (b) Furies (c) Uncles and aunts (d) Eumenides (e) None of the other options Ans: (c) T. S. Eliot's play 'The Family Reunion' the chorus comprises the Uncles and aunts, actually play a real role in the action of the drama. 24. Coleridge spoke of the ’willing suspension of disbelief in Biographia Literaria’s Chapter (a) XI (b) XII (c) XIII (d) XIV (e) None of the other options CGPSC Asst. Prof. Exam. 2014

Ans: (d) Coleridge spoke of the 'willing suspension of disbelief in 'Biographia Literaria's' chapter XIV. Suspension of disbelief, sometimes called willing suspension of disbelief, is the intentional avoidance of critical thinking. 25. Mikhail Bakhtin explains the differences between the ’’monologic” and the ’dialogic’ form of the novel in: (a) The Dialogic Imagination (b) Rabelais and His World (c) 'Discourse in the Novel' (d) Problems of Dostoevsky's Poetics (e) None of the other options Ans: (a) Mikhail Bakhtin explain the differences between the 'monologic' and 'dialogic' form of the novel in The Dialogic Imagination. Bakhtin's theory is sometimes called dialogic means the process which meaning is evolved out of interactions. Dialogic refers to the use of conversation shared dialogue to explore the meaning of something. Monologism indicates turning off the process of dialogue as well as its potentials. 26. The play Waiting for Godot was originally written in: (a) English (b) French (c) German (d) Italian (e) None of the other options Ans: (b) The play 'Waiting for Godot' was originally written in French. 'Waiting for Godot' is a play by Samuel Beckett in which two characters, Vladimir (Didi) and Estragon (Gogo), engage in a variety of discussion and encounters while awaiting Godot, who never arrives. 27. A name not associated with the Progressive Writers Movement is: (a) SajjadZaheer (b) Rasheed Jahan (c) Mehmood-uz-Zafar (d) Kishwar Naheed (e) None of the other options Ans: (d) Kishwar Naheed is not associated with the progressive writers movement. Progressive movement was a period of widespread social activism and political reform across the united states of America. 28. Final Solutions is a play by: (a) Alyque Padamsee (b) Cyrus Mistry (c) Mahesh Dattani (d) Nissim Ezekiel (e) None of the other options Ans: (c) 'Final Solutions' is a play by Mahesh Dattani. In final solution, Dattani represents the female character like Hardika, Smita and Aruna. They make realization that women are not a shadow of male. Today woman is making her space. 29. Using the term ’ten hands’ for ’ten workers’ is an example of: (a) Metaonymy (b) Synecdoche (c) Personification (d) Mixed metaphor (e) None of the other options

192

YCT

Ans: (b) The term ’ten hands' for 'ten workers' is an example of Synecdoche. It is figure of speech in which a part is made to represent the whole or vice versa. __________________________________________ 30. The Bellwether Prize, acknowledging 'the literature of social change', is associated with: (a) Carrie Fisher (b) Rita Mae Brown (c) Barbara Kingsolver (d) Toni Morrison (e) None of the other options __________________ Ans: (c) The Bellwether Prize, acknowledging 'the literature of social change' is associated with Barbara Kingsolver. She is an American novelist, essayist and poet. 31. 'Nothing but an ideology of hes is Sartre s comment on: (a) dystopia (b) humanism (c) hellensim (d) historicism (e) None of the other options Ans: (b) 'Nothing but an ideology of lies’ is Sartre's comment on - humanism. Humanism, system of education and mode of inquiry that originated in northern Italy during the 13th and 14th centuries, and later spread through continental Europe and England. 32. When two or more alternative meaning are resolved into one, it becomes an example of: (a) amblysia (b) amplification (c) amphigory (d) ambiguity (e) None of the other options __________________ Ans: (d) When two or more alternative meaning are resolved into one, it becomes an example of ambiguity. The definition of ambiguity is a word or sentence that is not clear about the intention or meaning. Or we can say - something of doubtful meaning. 33. Arthur Miller's play, The Crucible, is an example of: (a) a chronicle play (b) a morality play (c) an interlude (d) a dramatic lyric (e) None of the other options __________________ Ans: (a) Arthur Miller’s play, ’The Crucible’ is an example of a chronicle play. The Crucible (1953) is a play by American playwright Arthur Miller. The play was first performed at the Martin Beck Theatre on January 22, 1953. The Crucible is set in a theocratic society, in which the Church and the State are one and the religion is a strict. _______________ 34. The magazine, Blast, is associated with: (a) Vorticism (b) Imagism (c) Expressionism (d) Dadaism (e) None of the other options __________________ Ans: (a) Vorticism is associated with the magazine, ’Blast’. It was a London-based modernist art movement formed in 1914 by the writer and artist Wyndham Lewis. CGPSC Asst. Prof. Exam. 2014

The movement was partially inspired by Cubism and was introduced to the public by means of the publication of the vorticist manifesto in Blast magazine. 35. Browning's poem, 'Andrea del Sarto', is an example of: (a) soliloquy (b) elegy (c) the dramatic monologue (d) fin de siecle (e) None of the other options __________________ Ans: (c) Browning's poem 'Andrea del Sarto’ is an example of the dramatic monologue. In Dramatic monologue, the single person who is patently not the poet, utters the speech that makes up the whole of the poem, in a specific situation at a critical moment. 36. The protagonist of Manjula Padmanabhan's play, Harvest, is: (a) Ma (b) Jai Prakash (c) Om Prakash (d) Ginni (e) None of the other options __________________ Ans: (c) Om Prakash is the protagonist of Manjula Padmanabhan's play ’Harvest'. ’Harvest’ is a futuristic play by Manjula Padmanabhan about organ-setting in India. It was first published in 1997 by Kali for woman. 37. Shakespeare 'was the man who of all modern and perhaps ancient, poets, had the largest and most comprehensive soul' wrote: (a) Dryden in Essay of Dramatic Poesy (b) Pope in The Works of Shakespeare (c) Lamb in On the Tragedies of Shakespeare (d) Wilson Knight in The Wheel of Fire (e) None of the other options __________________ Ans: (a) Dryden wrote in 'Essay of Dramatic Poesy' (1688) about Shakespeare 'was the man who of all modem and perhaps ancient poets, had the largest and most comprehensive soul'. In this work Neander is mouthpiece of Dryden. 38. The 'Harlem Renaissance' takes its name from an area in New York city's: (a) Manhattan (b) The Bronx (c) Brooklyn (d) Queens (e) None of the other options __________________ Ans: (a) The 'Harlem Renaissance' takes its name from an area in New York city is Manhattan. 'Harlem Renaissance' was an intellectual and cultural revival of African American music, dance, art, fashion, literature. It was known as 'The New Negro Movement' a 1925 anthalogy edited by Alain Locke. 39. The poem 'The Blessed Damozel', was composed by: (a) Christina Rossetti (b) D. G. Rossetti (c) Holman Hunt (d) John Millais (e) None of the other options __________________ Ans: (b) D.G. Rossetti was a poet as well as an artist. His poem, 'The Blessed Damozel' was first composed in 1847.

193

YCT

40.

The title of Steinbeck’s novella, Of Mice and Men, has been taken from a poem by: (a) Robert Bums (b) Thomas Gray (c) Robert Southey (d) William Cowper (e) None of the other options _________________ Ans: (a) The title of Steinbeck is novella, "of Mice and Men" has been taken from Robert Bums poem "To a Mouse". 41. The ’bringing together into a single discourse of elements from different traditions and belief systems’ is termed: (a) empiricism (b) utilitarianism (c) transhumanism (d) syncretism (e) None of the other options _________________ Ans: (d) The bringing together into a single discourse of elements from different traditions and belief systems is termed syncretism. It is the combing of different beliefs. 42. Action, scene or character placed where it does not belong is called: (a) 'anachorism' (b) ’anamnesis (c) 'anachronism' (d) 'anaphora' (e) None of the other options _________________ Ans: (a) Action scene or character placed where it does not belong is called anachorism. 43. Shakespearean plays considered ’unfit to be read by a gentleman in the company of ladies’, did not find a place in The Family Shakespeare (1818) produced by: (a) Ben Jonson (b) John Milton (c) Thomas Bowdler (d) Samuel Johnson (e) None of the other options _________________ Ans: (c) Shakespearean plays considered 'unfit to be read by a gentleman in the company of ladies' did not find a place in "The Family Shakespeare" (1818) produced by Thomas Bowdler. 44. A group of lines (often in blank verse) which forms a unit is called? (a) a verse paragraph (b) a verset (c) a versicle (d) a verso sciolto (e) None of the other options _________________ Ans: (a) A group of lines (often in blank verse) which forms a unit is called a verse paragraph. ____________ 45. The Subjection of Women (1869) was authored by (a) John Stuart Mill (b) Margaret Fuller (c) Margaret Mead (d) Jeremy Bentham (e) None of the other options _________________ Ans: (a) 'The Subjection of Women' (1869) is an essay by English philosopher, political economist and civil servant John Stuart Mill, published in 1869 with ideas he developed jointly with his wife Harriet Taylor Mill. CGPSC Asst. Prof. Exam. 2014

46.

The novel, Rajmohan's Wife, was written by: (a) O. Chandu Menon (b) Nandshankar Mehta (c) Bankimchandra Chattopadhyaya (d) Mulk Raj Anand (e) None of the other options _________________ Ans: (c) 'Rajmohan's Wife' (1864) by Bankimchandra Chattopadhyay, is generally regarded as the first Indian novel in English. Its author was the greatest Bengali novelist of the 19th century. 47. The word ’blackout’ is an example of: (a) 'loanword' (b) 'affixation' (c) 'derivation' (d) 'compounding' (e) None of the other options _________________ Ans: (d) The word 'blackout' is an example of compounding. Compounding is the process whereby interest is credited to an existing principal as well as to interest. 48. Writers associated with the American Renaissance are: (a) Washington Irving and James Fenimore Cooper (b) Ralph Waldo Emerson and Edgar Allan Poe (c) Mark Twain and Henry James (d) Robert Frost and Wallace Stevens (e) None of the other options _________________ Ans: (b) The American Renaissance period in American literature ran about 1830 to around the civil war. It was closely associated with Transcendentalism. Writers associated with the American Renaissance areRalph Waldo Emerson, Edgar Allan Poe, Nathaniel Hawthorne, Herman Melvilles and Henry David Thoreau. 49. The Autobiography of an Unknown Indian was written by: (a) Amit Chaudhuri (b) Nirad C. Chaudhuri (c) Torn Dutt (d) Henry Derozio (e) None of the other options _________________ Ans: (b) 'The Autobiography of an Unknown Indian' is the 1951 autobiography of Nirad C. Chaudhuri , an Indian writer, written when he was around 50. The subject of comparative historical, cultural and sociological of early 20th century India and the British colonial encounter in India. 50. The native Indian name for ’America’ translates as: (a) 'little coot' (b) 'turtle island' (c) 'snow goose' (d) 'the loon' (e) None of the other options _________________ Ans: (b) The native Indian name for 'America' translate as 'turtle island'. Turtle Island is a name for the Earth or for North America, used by some Indigenous people in the United States.

194

YCT

51. A text associated with the beginning of 'Enlightenment* is: (a) Some Thoughts Concerning Education (b) Essay Concerning Human Understanding (c) A Letter Concerning Toleration (d) Two Treatises of Government (e) None of the other options _________________ Ans: (b) A text associated with beginning of ’Enlightenment’ is ’An Essay Concerning Human Understanding’ is a work by John Locke concerning the foundation of human knowledge and understanding. 52. Jimmy Porter is a character in: (a) Look Back in Anger (b) The Homecoming (c) The Jealous God (d) The Merchant (e) None of the other options _________________ Ans: (a) Jimmy Porter is a character in ’Look Back in Anger’ (1956) is a realist play written by John Osborne. It focuses on the life and marital struggles of an intelligent and educated but disaffected young man of working class origin. 53. The famous quote ’Shakespeare is the Canon. He sets the standard and the limits of literature* is ascribed to: (a) T.S. Eliot (b) A. C. Bradley (c) D. H. Lawrence (d) Harold Bloom (e) None of the other options _________________ Ans: (d) The famous quote ’Shakespeare is the Canon. He sets the standard and the limits of literature' is ascribed to Harold Bloom encapsulated the essence of Shakespearean literature as the core of classical writing throughout history. 54. The names of Nashe, Greene, Lyly and Peele are associated with the: (a) Bloomsbury Group (b) Celtic Renaissance (c) University Wits (d) Imagists (e) None of the other options _________________ Ans: (c) The name of Nashe, Greene, Lyly and Peele are associated with the University Wits. The University Wits is a phrase used to name of group of 16th century. __________________________________ 55. A literary theory that ’views literature primarily as a specialized use of language, and proposes a fundamental opposition between the literary (or poetical) use of language and the ordinary, ’’practical” use of language* is: (a) Structuralism (b) Poststructuralism (c) Deconstruction (d) Formalism (e) None of the other options _________________ Ans: (d) A literary theory that ’views literature primarily as a specialized use of language and proposes a fundamental opposition between the literary use of language and the ordinary "practical" use of language is Formalism. CGPSC Asst. Prof. Exam. 2014

56.

Agha Shahid Ali’s translations of Pakistani Urdu poet Faiz Ahmed Faiz’s poems is entitled: (a) Ravishing Dis Unities (b) The Country Without A Post Office (c) Call Me Ishmael Tonight (d) The Rebel's Silhouette (e) None of the other options _________________ Ans: (d) Agha Shahid Ali’s translations of Pakistani Urdu poet Faiz Ahmed Faiz’s poems is entitled The Rebel’s Silhouette'. Faiz Ahmed Faiz was a two time Nobel nominee and writer of the 1962 Lenin Peace Prize. 57. When words carry ’a whole range of shades of meaning’, they are: (a) metaphoric (b) polysemic (c) metonymic (d) inflectional (e) None of the other options _________________ Ans: (b) When words carry 'a whole range of shades of meaning' they are Polysemic. 58. The comment ’The two strata of Kipling’s appreciation of India, the stratum of the child and that of the young man’ was made by: (a) Edward Said (b) Salman Rushdie (c) Allan Sealy (d) T.S. Eliot (e) None of the other options _________________ Ans: (d) The comment 'The two strata of Kipling’s appreciation of India, the stratum of the child and that of the young man' was made by T. S. Eliot. 59. Ahmed Ali’s second novel was: (a) Twilight in Delhi (b) The Policeman and the Rose (c) Ocean of Night (d) On the Ganga Ghat (e) None of the other options _________________ Ans: (c) Ahmed Ali’s second novel was 'Ocean of Night'. Ahmed Ali was a Pakistani novelist, poet, critic, translator, diplomat and scholar. His urdu short story collections Angarey (1932), Hamari Gali (1940). 60. The sentence ’the temptation of meaning is upon us all the time’ appears in: (a) Linda Hutcheon's The Politics of Postmodernism (b) Robert Kroestch’s Labyrinths of Voice (c) F. R. Leavis’ Revaluation (d) Robert Lecker’s Making It Real (e) None of the other options _________________ Ans: (b) The sentence 'the temptation of meaning is upon us all the time’ appears in Robert Kroestch’s 'Labyrinths of Voice’. Robert Kroestch is a Canadian novelist, poet and nonfiction writer. 61. Rights of Man was authored by: (a) Thomas Jefferson (b) Ralph Waldo Emerson (c) Henry David Thoreau (d) Thomas Paine (e) None of the other options

195

YCT

Ans: (d) 'Rights of Man' a book by Thomas Paine, including 31 articles, posits that popular political revolution is permissible when a government does not Safeguard the natural rights of its people. ___________ 62. An example of a 'campus novel' is: (a) Kingsley Amis' Lucky Jime (b) Charlotte Bronte's Jane Eyre (c) Charles Dickens' David Copperfield (d) Joseph Conrad's Lord Jim (e) None of the other options _________________ Ans: (a) An example of a 'campus novel' is Kingsley Amis’ 'Lucky Jime’. A campus novel also known as an academic novel, is a novel whose main action is set in and around the campus of a university. _____________ 63. The 'buskin* or 'thick-soled half boot* is associated with: (a) Athenian tragedy (b) miracle plays (c) Comedy of Manners (d) drama of sensibility (e) None of the other options _________________ Ans: (a) The ’buskin' or 'thick-soled half boot' is associated with Athenian tragedy. It is also known as ’Greek Tragedy'. It is a form of theatre from Ancient Greece and Anatolia. ___________________________ 64. The novel, which is a satire on Russian totalitarianism under Stalin, is: (a) George Orwell's Animal Farm (b) William Golding's Lord of the Flies (c) Franz Kafka's The Castle (d) Leo Tolstoy's War and Peace (e) None of the other options _________________ Ans: (a) George Orwell's 'Animal Farm' (1945) is a novel, which is a satire on Russian totalitarianism under Stalin. The book tells the story of a group of farm animal who rebel against their human farmer. 65. A significant work in lesbian and gay studies is History of Sexuality by: (a) Mary Mackintosh (b) Michel Foucault (c) Judith Butler (d) Jurgen Habermas (e) None of the other options _________________ Ans: (b) A significant work in Lesbian and gay studies is 'History of Sexuality' by Michel Foucault. The work was in four volume study of sexuality in the Western World. 66. Cornelia Sorabji's first published work was: (a) Sun Babies (b) Love and Life behind the Purdah (c) Between the Twilights (d) Indian tales of the Great Ones (e) None of the other options _________________ Ans: (b) 'Love and Life behind the Purdah' was the first published work of Cornelia Sorabji. Cornelia Sorabji was an Indian lawyer, social reformer and writer. She was a pioneer in the tradition of Indian-Parsee women's literature in English. CGPSC Asst. Prof. Exam. 2014

67. The 'generalization of morpheme or relation which already exists in the language into new situations of forms* is termed: (a) 'morphological extension' (b) 'analogical extension' (c) 'syntactic change' (d) 'loanword' (e) None of the other options _________________ Ans: (b) The 'generalization of a morpheme or relation which already exists in the language into new situations of forms' is termed 'Analogical extension'. Analogical extension, a productive formation pattern is extended to a new word which did not use it before. 68. Robert Lowell's Life Studies foregrounds the idea of: (a) contextual criticism (b) mimetic criticism (c) confessional poetry (d) performance poetry (e) None of the other options _________________ Ans: (c) Robert Lowell's Life Studies foregrounds the idea of confessional poetry. Confessional poetry or Confessionalism is a style of poetry that emerged in the Unites State during the late 1950s and early 1960s. 69. The 'representation of inanimate natural objects that ascribes to them human capabilities, sensations and emotions' defines: (a) affective fallacy (b) pathetic fallacy (c) personification (d) parthos (e) None of the other options _________________ Ans: (b) The 'representation of inanimate natural objects that ascribes to them human capabilities, sensations and emotions' defines - pathetic fallacy. The term 'pathetic fallacy' was coined by a British writer named John Ruskin, who defined it as 'emotional falseness'. 70. 'Snowman' is the protagonist of Margaret Atwood's novel: (a) The Edible Woman (b) The Handmaid's Tale (c) The Blind Assassin (d) Oryx and Crake (e) None of the other options _________________ Ans: (c) 'Snowman' is the protagonist of Margaret Atwood's novel - 'Oryx and Crake'. She has described the novel as speculative fiction and adventure romance, rather than pure science fiction. 71. 'Altruism' is the antonym of: (a) 'egoism' (b) 'schizophrenia' (c) 'empathy' (d) 'sadism' (e) None of the other options _________________ Ans: (a) Altruism is the antonym of egoism. Altruism- When you care about or help other people, even though this bring no advantage to yourself. Egoism-The belief that you are much better or more important than other people.

196

YCT

72. Crashaw, Vaughan and Herrick are poets ’Rich Like Us’ (1985) is a historical and political fiction novel by Nayantara Sehgal. Sehgal novels as associated with the: ’Rich Like Us’, which confronts civil disorder, (a) Elizabethan age corruption and oppression while detailing the internal (b) Caroline period (c) Puritan Interregnum conflicts in a businessman’s family. (d) Restoration 77. A novel that does not foreground the myth of (e) None of the other options _________________ the nation is: (a) Salman Rushdie’s Midnight's Children Ans: (b) Crashaw, Vaughan and Herrick are poets (b) Mukul Kesavan’s Looking through Glass associated with the Caroline period. (c) Amitav Ghosh’s The Shadow Lines The Caroline era refers to the period in English and Scottish history named for the 24 year reign of (d) Shashi Deshpande’s The Binding Vine (e) None of the other options _________________ Charles 1 (1625-1649). The Caroline era followed the Jacobean era, the reign Ans: (d) A novel that does not foreground the myth of of Charles father James I and VI. the nation is- Shashi Deshpande’s ’The Binding Vine’. 73. ’A break or pause in line of poetry* is referred to Shashi Deshpande is an Indian novelist. She is a recipient of the Sahitya Akademi Award. as: ’The Binding Vine’ is a feminist novel that explicitly (a) stasis (b) caesura talks about sex. It is about marital rape and the rape of (c) strophe (d) cancion a young girl. (e) None of the other options _________________ Ans: (b) A break or pause in line of poetry’ is referred 78. The term ’cultural materialism* was first to as - Caesura. It is metrical pause or break in a verse employed by: (a) Karl Marx where one phrase ends and another phrase begins. (b) Raymond Williams 74. A seminal work that initiated ’archetypal (c) Louis Althusser criticism* is: (d) Stephen Greenblatt (a) Anatomy of Criticism (e) None of the other options _________________ (b) The Starlit Dome (c) The Golden Bough Ans: (b) The term ’cultural materialism’ was first (d) After the New Criticism employed by - Raymond Williams. (e) None of the other options _________________ Cultural materialism as a historical materialist theory of the production of culture is attributed to the British Ans: (c) A seminal work that initiated ’archetypal Marxist Raymond Williams. criticism’ is - The Golden Bough. The Golden Bough : A study in Comparative Religion 79. Donne*$ ’The Good Morrow’ is an example of: (a) the aubade (b) carpe diem is a wide-ranging, comparative study of mythology (c) the palinode (d) a prospect poem and religion, written by the Scottish anthropologist Sir (e) None of the other options _________________ James George Frazer. ___________________________ 75. The statement - ’What is important in a work is Ans: (a) The Good Morrow’ is an aubade - a morning love poem, written by English poet John Donne, likely what it does not say* - appears in: in the 1590. In it the speaker describes love as a (a) Terry Eagleton’s Literary Theory (b) Pierre Macherey’s A Theory of Literary profound experience that almost like a religious Production epiphany._____________________________________ (c) Jonathan Culler’s On Deconstruction 80. Desirable Daughters is a novel by: (d) Martin Heidegger’s An Introduction to (a) Bharati Mukherjee Metaphysics (b) ShaniMootoo (e) None of the other options _________________ (c) Shyam Selvadurai (d) Anjana Appachana Ans: (b) The statement - ’What is important in a work (e) None of the other options _________________ is what it does not say’ appears in - Pierre Macherey’s ’A Theory of Literary Production’. Ans: (a) Desirable Daughters, a 2002 novel by Macherey’s first and most famous work, A Theory of American Indian writer Bharti Mukherjee, tells the Literary Production. ____________________________ story of three sisters as they find their very different 76. Nayantara Sehgal’s novel set during the days of paths in life. It uses autobiographical elements to explore the South Asian immigrant experience. the Emergency in India is: (a) A Time to be Happy 81. Nicholas Udall’s comedy, written for the boys of (b) Rich Like Us Westminster School, is entitled: (c) Storm in Chandigarh (a) The Four P’s (d) Mistaken Identity (b) The Girl of Andros (e) None of the other options _________________ (c) Ralph Roister Doister (d) Jack Juggler Ans: (b) Nayantara Sehgal’s novel set during the days (e) None of the other options of the Emergency in India is - ’Rich Like us’. 197 YCT CGPSC Asst. Prof. Exam. 2014

Ans: (c) Ralph Roister Doister is a 16th century play by Nicholas Udall, which was once regarded as the first comedy to be written in the English language for the boys of Westminster School. 82. The phrase ’carpe diem* first appeared in: (a) Horace (b) Homer (c) Virgil (d) Sophocles (e) None of the other options Ans: (a) Carpe diem, latin - pluck the day or seize the day phrase used by the Roman poet Horace to express the idea that one should enjoy life while one can. Carpe diem is part of Horace’s injunction ’’Carpe diem quam minimum Credula Pastero” which appears in his odes published in 23 BC. 83. Mikhail Bakhtin coined the term *carnivalization* in: (a) The Dialogic Imagination (b) Questions of Literature and Aesthetics (c) The Aesthetics of Verbal Art (d) Art and Answer ability (e) None of the other options Ans: (a) ’Camivalization’ is the term used by Mikhail Bakhtin to describe the shaping effect on literary genres. The idea of camivalism is the discourse of structuralism. Mikhail Bakhtin coined the term ’carmivalization’ in - The Dialogic Imagination. 84. In Swift’s Gulliver’s Travels, the last voyage undertaken by the protagonist is to: (a) Lilliput (b) Brobdingnag (c) Laputa (d) Houyhnhnyms (e) None of the other options Ans: (d) Gulliver’s Travels, the last voyage undertaken by the protagonist is to Houyhnhnyms. The Houyhnhnm concludes that the people of England are not more reasonable than the Yahoos. Gulliver’s Travels is four part satirical work by Anglo Irish author Jonathan Swift published monymously in 1726. 85. The line ’Fair is foul, and foul is fair’ occurs in the play: (a) Volpone (b) The Spanish Tragedy (c) The Duchess of Malfi (d) Macbeth (e) None of the other options Ans: (d) The line ’Fair is foul and foul is fair’ occurs in the play Macbeth in Act I Scene I. Macbeth is a tragedy by William Shakespeare. 86. V. S. Naipaul’s first novel was: (a) A House for Mr. Biswas (b) Miguel Street (c) The Mystic Masseur (d) The Suffrage of Elvira (e) None of the other options CGPSC Asst. Prof. Exam. 2014

Ans: (c) ’The Mystic Masseur’ is a comic novel by V. S. Naipaul. It is set in colonial Trinidad and was published in London in 1957. It was the first novel by V. S. Naipaul. 87. Nine Man-eaters and One Rogue was written by: (a) Kenneth Anderson (b) Jim Corbett (c) Salim Ali (d) Rudyard Kipling (e) None of the other options Ans: (a) Nine Man-eaters and One Rogue was written by Kenneth Anderson. He was and Indian bom British writer and hunter, who wrote books about his adventures in the jungles of South India. 88. The phrase ’a picture made out of word’s is commonly used to refer to: (a) a simile (b) an image (c) a symbol (d) a metaphor (e) None of the other options Ans: (b) The phrase 'a picture made out of word’s is commonly used to refer to an image. 89. Imtiaz Dhaker’s first volume of poetry is entitled(a) Purdah (b) Postcards from God (c) A Spelling Guide to Woman (d) The Golden Threshold (e) None of the other options Ans: (a) Imtiaz Dhaker is a British poet, artist and video film maker. She started sensationally with ’Purdah’ (1989) her first volume : Purdah is a kind of safety. The body finds a place to hide. 90. In the novel The Old Man and the Sea, Santiago thinks of the sea as(a) ’lamer’ (b) ’dasmeen’ (c) ’la man’ (d) ’la mare’ (e) None of the other options Ans: (c) ’The Old Man and the Sea’ short heroic novel by Ernest Hemingway published in 1952 and awarded the 1953 Pulitzer Prize for fiction. He always thought of the sea as ’la man’ which is what people call her in Spanish when they love her. 91. The prominence or emphasis given to a syllable or word, especially in a line of verse is termed(a) ’acalectic’ (b) ’accidence’ (c) ’accent’ (d) ’acroama’ (e) None of the other options Ans: (c) The prominence or emphasis given to a syllable or word, especially in a line of verse is termed as - accents or stresses.

198

YCT

92. Aristotle termed the therapeutic effect of Ans: (a) TSAR Publications began as the literary tragedy on the spectator as: journal The Toronto Review of Contemporary writing (a) ’hamartia’ (b) ’catharsis’ Abroad (which was previously called - The Toronto (c) ’hubris’ (d) ’parados’ South Asian Review) in 1981 by M. G. Vassanji and (e) None of the other options Nurjehan Aziz, among others. Ans: (b) Catharsis, which in Greek signifies 97. The comment - ’O thou weed/Who art so lovely purgation, or purification as both as much disputed. fair and smell’st so sweet/That the sense aches at Aristotle termed the therapeutic effect of tragedy on thee, would thou hadst ne’er been!’ - was made the spectator as - Catharsis. by: 93. The ’Big Four’ of the English detective novel (a) Othello (b) Macbeth are: (c) Lear (d) Hamlet (a) Arthur Conan Doyle, Gaston Leroux, Maurice (e) None of the other options Leblanc, and Nicholas Blake Ans: (a) The comment ”0 thou weed/who art so (b) Elizabeth Ferrars, P. D. James, Margaret lovely fair and smell’st so sweet/That the sense aches Yorke, and June Thomson at thee, would thou hadst never been bom” was made (c) Agatha Christie, Dorothy L. Sayers, Ngaio by Othello. ’Othello’ is a significant tragedy of Marsh and Margery Allingham William Shakespeare. These lines utters Othello to (d) Edgar Allen Poe, Mario Puzo, Erie Stanley Desdemona in (Act IV, Scene II). Gardner, and Stephen King 98. The phrase ’anxiety of influence’ was coined by: (e) None of the other options (a) John Crowe Ransom Ans: (c) ’The Big Four’ is a work of detective fiction (b) Harold Bloom by Agatha Christie, first published in the UK by (c) I. A. Richards William Callins and sons in - 1927. It features (d) ReneWellek Hercule Poirot, Arthur Hastings and Inspector Japp. (e) None of the other options 94. Shelley remark, in his Defense of Poetry (1821) Ans: (b) The phrase ’anxiety of influence’ (1973) was the ’best and happiest moments - arising coined by Harold Bloom. unforeseen and departing unbidden’ - can be The Anxiety of influence : A Theory of Poetry. It said to describe: refers to the psychological struggle of aspiring author (a) euphony (b) irony to overcome the posed by the influence of their (c) epiphany (d) interlude literary antecedent. (e) None of the other options 99. Keats used the term ’negative capability’ in the Ans: (b) Shelley remark, in his ’Defense of Poetry’ context of: (1821) - the best and happiest moment of the happiest (a) Chaucer (b) Shakespeare and the best minds - arising unforeseen and departing (c) Milton (d) Congreve unbidden - can be said to describe - Ironical. (e) None of the other options 95. Petruchio is a character in: Ans: (b) Keat used the term ’Negative Capability’ in (a) Timon of Arthens the context of William Shakespeare. Keats used the (b) Cymbeline phrase only briefly in the private letter, letter to his (c) The Taming of the Shrew brothers George and Thomas in 1817. (d) Titus Andronicus 100. The term that implies a psychological (e) None of the other options relationship between the reader (or viewer) and Ans: (c) ’Petruchio’ is the male protagonist in a work of art is: Shakespeare’s ’The Taming of the Shrew’. He is a (a) ’authorial objectivity’ fortune seeker who enters into a marriage with a (b) ’aesthetic distance’ strong willed young women name - Kate and then (c) ’aestheticism’ proceeds too ’tame’ her temperamental spirit. (d) ’afflatus’ 96. A name associated with the Toronto South Asian (e) None of the other options Review is: Ans: (b) The term that implies a psychological (a) M. G. Vassanji relationship between the reader and a work of art is (b) Rohinton Mistry aesthetic distance. When a reader becomes fully (c) Shani Mootoo engrossed in the illusory narrative world of a book, (d) Anjana Appachana the author has achieved a close aesthetic distance. (e) None of the other options 199 YCT CGPSC Asst. Prof. Exam. 2014

Haryana Public Service Commission

Asst Prof. Re-Exam. 2019

ENGLISH (Solved Paper) 1.

Which of the following is correctly matched? Years Works (a) 1815 The Spirit of Age (b) 1675 The Man of Mode (c) 1825 Prometheus Unbound (d) 1819 Ivanhoe Ans: (d) ’The spirit of the Age’, written by William Hazlitt is published in 1825. ’The Man of Mode’, written by George Etherege is published in 1676. ’Prometheus Unbound’ four act lyrical drama by Percy Bysshe Shelley is published in 1845. ’Ivanhoe’ written by Walter Scott is published in 1819. 2. Who composed the following lines: "Somewhere beyond the scorched gable end and burnt but buses There is a poet indulging His wretched rage for order" (a) Derek Mahon (b) Coleridge (c) Southey __________(d) Wordsworth _______ Ans: (a) The given lines were composed by Derek Mahon. Derek Mahon was an Irish poet. He was widely regarded as one of the most talented and innovative Irish poets of the late 20th century. ________ 3. These lines about French Revolution: Where wealth accumulates, and men decay: Princes and lords may flourish, or may fade; Were written by: (a) Southey (b) Godwin (c) Oliver Goldsmith (d) Blake Ans: (c) ’’Where wealth accumulates, and men decay: Princes and lords may flourish, or may fade.” These lines about French Revolution by Oliver Goldsmith’s poem ’’The Deserted Village” (1770). It is a work of social commentary and condemns rural depopulation and the pursuit of excessive wealth. 4. Which of the following is correctly matched? Years Works (a) 1698 Lyrical Ballads (b) 1814 Childe Harold (c) 1817 Emma (d) 1987 The Haw Lantern Ans: (d) ’’The Haw Lantern” (1987) is a collection of poems written by Irish poet Seamus Heaney. Other correct options areLyrical Ballads - 1798 Child Harold - 1 8 1 2 Emma - 1815 (Title page given 1816). Haryana PSC Asst. Prof. Re-Exam. 2019

5.

[Exam. Date : 12 June 2019]

Mary Shelley’s famous work Frankenstein is also known as: (a) Mythical Prometheus (b) Medieval Prometheus (c) Modem Prometheus (d) Prometheus ____________________________ Ans: (c) Mary Shelley’s famous work ’Frankenstein’ is also known as ’Modem Prometheus’ (1818). It tells the story of Victor Frankenstein, a young scientist who creates a sapient creature in an unorthodox scientific experiment. - ’Frankenstein’ book preface is dedicated to William Godwin. 6. Tennyson’s collection of poems Chiefly Lyrical was published in: (a) 1830 (b) 1850 (c) 1855 _____________(d) 1862 _____________ Ans: (a) Tennyson’s collection of poems ’Chiefly Lyrical’ was published in 1830. It includes the poems1. The Kraken 2. Ode to Memory 3. ’Mariana’ - It is based on character Mariana in ’Measure for Measure’. - Theme a woman waiting for her lovers return. 7. Stones of Venice (1853) was written by: (a) Schiller (b) Alfred Tennyson (c) John Ruskin _______(d) Robert Browning Ans: (c) ’Stones of Venice’ (1853) was written by John Ruskin. It was published in three volumes. Ruskin wrote the work in order to apply to the architecture of Venice, the general principles enunciated in his ’The Seven Lamps of Architecture’. 8. Arrange the following in chronological order: 1. InMemoriam 2. Queen Marry 3. Maud 4. Harold (a) 1,3, 2, 4 (b) 3, 2, 4 , 1 (c) 2, 3, 4 , 1 __________(d) 4, 3, 2 , 1 Ans: (a) In Memoriam A. H. H. - 1850 Maud - 1855 Queen Marry - 1875 Harold - 1876 Hence, correct chronological order is 1, 3, 2, 4. 9. Which of the following is correctly paired? (a) Charles Dickens : The Subjection of Women (b) G. B. Shaw : Hard Times (c) John Stuart Mill : North and South (d) George Meredith : The Egoist 200 YCT

Ans: (c) ’Origin of Species’ (1859) was written by Ans: (d) ’The Egoist’ is a tragicomical novel by Charles Darwin. It full title was ’On the Origin of George Meredith published in 1879. Species by means of Natural Selection, or the Other correct options arepreservation of Favoured Race in the Struggle for • Charles Dickens’ novel ’Hard Times’. Life.’ It was considered to be the foundation of • John Stuart Mill’s ’The Subjection of Women’. evolutionary biology Natural Selection and Scientific 10 ................ produced a paper called ’The Theory. The Opium War was started in 1839 and Athenaeum 1807 to 1809’, not to be confused fought between Britain and Qing dynasty of China. with the The Athenaeum, a well-known British 14. Who among the following British artist magazine published from 1828 to 1921. illustrated the novels of Dickens and Scott? (a) Thomas Hardy (b) George Meredith (a) Reynolds White (b) George Cruishank (c) JohnAikin ________(d) George Eliot ______ (c) John Norfolk ______(d) Richard Grey ______ Ans: (c) John Aikin produced a paper called ’The Ans: (b) George Cruishank was a British caricaturist Athenaeum’ from 1807 to 1809, not to be confused and book illustrator, praised as the ’Modem Hogarth’ with ’The Athenaeum’, a well known British Magazine during his life. He illustrated the novels of Charles published from 1828 to 1921. _____________________ Dickens’ (’Sketches by BOZ’ and ’Oliver Twist’) and 11. We live in a special era of crisis and change, Sir Walter Scott. such a belief seems particularly crucial to the - He wrote periodical ’The Scourge’ a monthly contemporary critical environment. This feeling, Expositor of Imposture and Folly. central for decades to modern painting and 15. Madam Merle is a character in: literature, was not dominant in literary criticism (a) Gone with the Wind until recently. (b) The Portrait of a Lady Above ideas are expressed by: (c) Casablanca (a) Frank Kermode in A Sense of Ending (d) The Heart and the Beauty _________________ (b) Lionel Trilling in Freud and Literature Ans: (b) ’The Portrait of a Lady’ is a novel by Henry (c) F. R. Leavis in The Common Pursuit James, first published as a serial in ’The Atlantic (d) John Crowe Ransom in Bells for John Monthly’ and MacMillan’s magazine. In the novel ________Whiteside's Daughter ____________________ character ’Madam Merle’ is an accomplished, graceful Ans: (a) ”We live in a special era of crisis and and manipulative woman. She is a popular lady who change, such a belief seems ................ literary does not have a husband or a fortune. criticism until recently" ideas are expressed by Frank 16. Who among the following women writers Kermode in ’A Sense of Ending : Studies in the famously imagined the plight of Shakespeare's Theory of Fiction’ (1967). The book originated in sister? Mary Flexner lectures, given at Bryn Mawr College in (a) Iris Murdoch (b) Doris Lessing 1965 under the title ’The Long Perspectives’. (c) George Eliot ______(d) Virginia Woolf 12. Second Boer War was fought between South Ans: (d) 'Virginia Woolf imagined the plight of Africa and the British colonists, which treaty Shakespeare's sister in her book 'A Room of One's was signed at the end of the war? Own' (1929). It contains Woolfs enduring insight into (a) Treaty of Warsaw the creative advantages of the androgynous mind. (b) Treaty of Waitangi 17. 'Among bewildering appliances (c) Treaty of Marsalis For mastering the arts and science (d) Treaty of Vereeniging ____________________ They stroll or run And nerves that never flinched at slaughter Ans: (d) Second Boer War was fought between South Are shot to pieces by the shorter Africa and the British colonists, the treaty signed at Poems of Donne' the end of the war was ’Treaty of Vereeniging’ Above lines drawn from a poem written by: between representatives of the British and ex(a) Rudyard Kipling (b) W. H. Auden republican Boer government. It ended the (c) W. E. Henley (d) W. B. Yeats _______ independence of South Republic and the Orange Free Ans: (b) 'Among bewildering appliances ................ State. Poems of Donne.' 13. Consider the following statements: 1. Origin of Species was written by Thomas These lines are composed by W. H. Auden in his poem 'Under Which Lyre' (The Age of Anxiety). Grey 2. Origin of Species was first published in 1860. 18. In which of his fictional work Dickens' character Little Nell figures? 3. Opium War started in 1839 (a) Hard Times Which of the statements given above are (b) Bleak House incorrect? (c) Great Expectations (a) l a n d 3 (b) Only 2 (d) The Old Curiosity Shop (c) l a n d 2 (d) Only 3 201 YCT Haryana PSC Asst. Prof. Re-Exam. 2019

Ans: (d) Little Nell, fictional character, a frail child who is a major figure in Charles Dickens’ novel ’The Old Curiosity Shop’ (1840-41). The plot follows the life of Nell Trent (little Nell) and her grandfather, both residents of The Old Curiosity Shop in London. 19. Arrange the following in chronological order: 1. Great Expectations 2. Hard Times 3. Bleak House 4. Tale of Two Cities (a) 3, 2, 4 , 1 (b) 1,3, 2, 4 (c) 2, 1,3, 4 (d) 1,3, 4, 2 Ans: (a)Bleak House - 1852 Hard Times - 1854 Tale of Two Cities - 1859 Great Expectations - 1861 Hence, the correct chronological order is 3, 2, 4, 1 20. Dinah Morris is a character in ............ which was published in ........... (a) Middlemarch, 1857 (b) Silas Marner, 1861 (c) Adam Bede, 1859 (d) Daniel Deronda, 1877 ____________________ Ans: (c) Dinah Morris is a major character in George Eliot's novel ’Adam Bede’ (1859) a Methodist lay preacher. She was modeled on Eliot's aunt Elizabeth Evans. Note- George Eliot pen name was 'Mary Ann Evans'. 21. Sir Fopling is a character in: (a) Congreve's - The Way of the World (b) Dryden's - The Indian Emperor (c) Etherege's - The Man of Mode (d) Wycherley’s - The Plain Dealer ___________ Ans: (c) 'The Man of Mode' or 'Sir Fopling Flutter' is a Restoration comedy by George Etherege, written in 1676. The play is set in Restoration London and follows the Womanizer Dorimant as he tries to win over the young heiress Harriet and to disengage himself from his affair with Mrs. Loveit. Its subtitle is based on a character Sir Fopling. __________________ 22. Mac Flecknoe is an attack on Dryden's literary rival: (a) Richard Flecknoe (b) Thomas Shadwell (c) John Wilmot ______(d) Mathew Mack _____ Ans: (b) ’Mac Flecknoe' (full title : Mac Flecknoe or A Satyr upon the True-Blue-Protestant poet, T.S.) is a verse mock-heroic satire written by John Dryden. It is a direct attack on Thomas Shadwell. Dryden subverts the theme of the defining characteristic by giving Shadwell a negative characteristic as his only virtue. 23. 'Verses on the Death of Dr. Swift' was written by: (a) Jonathan Swift (b) Martin Price (c) John Dryden ______(d) Dr. Johnson _______ Ans: (a) 'Verses on the Death of Dr. Swift' was written by Jonathan Swift. Its theme of friendship, death and the transitory nature of fame suggest the satires of the Roman poet Horace, which both Swift and Pope imitated. Haryana PSC Asst. Prof. Re-Exam. 2019

24. Pope's 'Rape of the Lock' was published in 1712 in: (a) Three Cantos (b) Four Cantos (c) Sic Cantos ________(d) Two Cantos _______ Ans: (d) 'The Rape of the Lock' is a mock heroic narrative poem first published anonymously in Linton's miscellaneous poems in 1712 in 2 cantos. Its final version published in 1714, was expanded to five cantos. - It is also an example of high Burlesque. - The final form of the poem was available in 1717 with the addition of Clarissa's speech on good humour. __________________________________ 25. Margaret Doody's 'The Daring Muse' is critical of Augustanism in which of the following terms? (a) Epicureanism and Humanism (b) Humanism and Secularism (c) Stoicism and Amiability (d) Decorum and Correctness _________________ Ans: (d) Margaret Doody's 'The Daring Muse’ is critical of Augustanism in Decorum and correctness. The Daring muse is a challenging account of the richness and complexity of Augustan poetry. 26. Goethe's Faust, 1829 (Part I, Scene I) opens in: (a) Bar (b) Hell (c) Brothel (d) Faust's Study Ans: (d) Goethe's 'Faust' 1829 (Part I, Scene I) opens with a monologue by Faust sitting in his study, contemplating all that he has studied throughout his life. 27. "I will show you my heroine as plain and as small as myself' This is : (a) Emily Bronte in Wuthering Heights (b) Charlotte Bronte in Jane Eyre (c) George Eliot in Romola (d) Anne Bronte in Agnes Grey _______________ Ans: (b) Charlotte Bronte in her novel 'Jane Eyre', created "a heroine as plain, and as small as myself, who", she told her sisters, "shall be as interesting as any of yours". Charlotte Bronte's biographer Elizabeth Gaskell has quoted this statement in her book "The Life of Charlotte Bronte". 28 .................... was the son of an exiled Italian painter and scholar, was distinguished both as a painter and a poet. (a) Tennyson (b) Dante (c) Rossetti __________(d) Raphael __________ Ans: (b) Dante was the son of an exiled Italian painter and scholar was distinguished both as a painter and a poet. His famous book 'Divine Comedy' describe Dante's Journey through Hell, purgatory and paradise. 29. Which of the statements about the Culture and Anarchy are true? (a) It was not written by Mathew Arnold (b) It was not published in 1869 (c) The author divides human society into four main classifications (d) The book is famous for the phrase 'Sweetness and Light'

202

YCT

Ans: (d) ’Culture and Anarchy’ (1869) is a philosophical essay by M. Arnold. - Arnold divided human society into three main classes(1) Barbarians (2) Philistines (3) The populace - ’Culture and Anarchy’ is also famous for its popularization of the phrase ’’Sweetness and light.” 30. Here is the list of heroines from the novels of Thomas Hardy who have been ditched by their lovers. Pick the odd one out: (a) Gertrude Morel (b) Bathsheba Everdene (c) Tess of the D’Urberville (d) Fanny Robin ___________________________ Ans: (a) ’Gertrude Morel’ is character in ’Sons and Lovers’ by D. H. Lawrence. - Bathsheba Everdene is character in Hardy’s novel ’Far from the Madding Crowd’ (1874). - ’Tess Durbeyfield’ is character, Hardy’s novel ’Tess of the D’Urberville. - ’Fanny Robin’ is also character in Hardy’s novel ’’Far from the Madding Crowd.” 31. Who among the following wrote - 'The Age of Exuberance: Backgrounds to Eighteenth Century English Literature'*! (a) Donald Greene (b) Edmund Burke (c) William Godwin (d) Edmund Gibbon Ans: (a) ’The Age of Exuberance : Backgrounds to Eighteenth Century English Literature’ (1970) written by Donald Greene. Donald Johnson Greene was a Literary Critic, English Professor and Scholar of 18 th century. ______________________________________ 32. The religious movement Methodism in the eighteenth century was founded by: (a) John Tillotson (b) Bernard Shaw (c) W. B. Butler (d) John Welsey Ans: (d) The religious movement Methodism in the eighteenth century was founded by John Welsey. Methodism, is a group of historically related denomination of protestant Christianity. 33. Who among the following is an Irish writer? (a) Oscar Wilde (b) Thomas Gray (c) Jack Kerouac ______(d) David Malouf _____ Ans: (a) Oscar Wilde is an Irish poet and playwright. He is remembered for his epigrams and play, his novel 'The Picture of Dorian Gray'. - 'Solome' is a one-act tragedy by Oscar Wilde. ______ 34. Periodical Essays in English literature came into prominence in: (a) 18th Century (b) 17th Century th (c) 16 Century _______(d) None of the above Ans: (a) Periodical Essays in English Literature came into prominence in 18th century. A Periodical Essay is a type of writing that is issued on a regular basis as a part of a series in editions such as journals, magazines, newspaper. It was published twice or thrice in a week. Example 'The Spectator’and ’The Tatler’. Haryana PSC Asst. Prof. Re-Exam. 2019

35.

What led to the sharp decline in the popularity of The Restoration Comedy? (a) Excessive wit and humour (b) Bitter satire and cynicism (c) Glorious Revolution (d) Superficial reflection of society ____________ Ans: (b) Bitter satire and cynicism led to sharp decline in the popularity of The Restoration Comedy. Restoration Comedy is English comedy written and performed in Restoration period 1660-1700. 36. The Naked and The Dead are the famous works of: (a) Anita Desai (b) Norman Mailer (c) V. S. Naipaul ______(d) Salman Rushdie Ans: (b) ’The Naked and The Dead’ is a novel by Norman Mailer (1948). When he was 25, it was his debut novel. It depicts the experiences of Platoon during World War II. 37. John Brady received which award for her historical novel 'Theory of War'! (a) Encore Prize (b) Costa Novel Award (c) Booker Prize ______(d) Pulitzer Prize ______ Ans: (b) ’Theory of War’ is a 1992 novel by American-British writer Joan Brady. John Brady received ’Costa Novel Award’ for her historical novel ’Theory of War’. It took her ten years to write but was rejected by her US agent. 38. Anthony Trollope’s novel Barchester Towers revolves around: (a) Narrative Consciousness (b) Narrative Unconsciousness (c) Magical Realism (d) Concrete ideologies______________________ Ans: (c) Anthony Trollope’s novel ’Barchester Towers revolves around Magical Realism. In his autobiography, Trollope observed ’In the writing of Barcherster Towers I took great delight. 39. Which of the following is incorrectly matched? (a) Eugene O'Neill Dramatist Historian (b) Luigi Pirandello (c) Pablo Neruda Poet Autobiography (d) William O' Douglas : Ans: (b) Luigi Pirandello was an Italian dramatist, novelist, poet and short story writer whose greatest contributions were his play. He was awarded the Nobel Prize in 1934. 40. The renowned German poet was born in Prague and died of bone marrow cancer. When young he met Tolstoy and was influenced by him. The titles of his last two works are ’Sonnets’ and ’Elegies’. He is: (a) Herman Hesse (b) Gunter Grass (c) Raine Maria Rilke (d) Heinrich Heine Ans: (c) Raine Maria Rilke was a Bohemian-Austrian poet and novelist, The Renowned German poet was bom in Prague and died of bone marrow cancer. When young he met Tolstoy and was influenced by him. The titles of his last two works are ’Sonnets’ and ’Elegies’.

203

YCT

41. "On His Blindness" is a poem in which Milton: (a) Is frustrated due to lack of sight (b) Is happy that he will not have to serve God (c) Celebrates Epicureanism and Stoicism (d) None of the above _______________________ Ans: (a) ’On His Blindness’ is a poem composed by John Milton. He is frustrated due to lack of sight. - Matthew Arnold called Milton’s style as ’Grand Style’. - ’’Milton’s Grand Style” is a famous book by Christopher Rick. 42. What is common to the following writers? Identify to the correct description below: John Foxe Raphael Holinshed William Camden All of them were: (a) Elizabethan Chronicles of Antiquaries (b) Elizabethan poets (c) University wits (d) Renaissance playwrights __________________ Ans: (a) The Elizabethan society of Antiquaries has traditionally been regarded as a scholarly group which dissolved due to attrition and perhaps the suspicion of the ruling administration. The some writers areJohn Foxe, Raphael Holinshed and William Camden. 43. The classical influence on Ben Jonson's 'Volpone' is: (a) Ovid (b) Aristophanes (c) Euripides _________(d) Terence __________ Ans: (b) The classical influence on Ben Jonson’s ’Volpone’ is Aristophanes. It full title ’Volpone : The Fox’ (1605). A satire about greed and lust. - Volpone is a wealthy, childless con artist. The play begins with him worshipping his gold in a soliloquy. 44. 'The Peace of the Augustans' was written by and in which year? (a) Pat Rogers in 1931 (b) Andrew Marvel in 1678 (c) George Saintsbury in 1916 (d) Claude Rawson in 1919 __________________ Ans: (c) ’The Peace of the Augustans’ was written by English writer George Saintsbury. It was published in 1916. ________________________________________ 45. The Restoration Comedy 'The Double Dealer' is written by: (a) Wycherley (b) Etherege (c) Fletcher __________(d) William Congreve Ans: (d) The Restoration Comedy ’The Double Dealer (1694) is written by William Congreve. He was member of Kit-Kat club. Note- ’The Double Dealer’ is also a magazine published from 1920. Haryana PSC Asst. Prof. Re-Exam. 2019

46.

Which of the following pairs are incorrectly matched? Author Country of Birth Britain (a) H.G. Wells (b) Rudyard Kipling India : Ukraine (c) Joseph Conrad (d) Harold Pinter : USA Ans: (d) Harold Pinter was a British playwright, screenwriter, director and actor. He received Nobel Prize in literature in 2005. - He is also known for ’Comedy of Menace’. 47. Which of the following movements are in sequential order? (a) Imagism, Dadaism, Surrealism (b) Surrealism, Dadaism, Imagism (c) Dadaism, Surrealism, Imagism (d) Imagism, Surrealism, Dadaism _____________ Ans: (a) Imagism- It was a movement in early 20th century Anglo-American poetry that favored precision of imagery and clear, sharp language. A poetic movement established in 1912 by American and English poets Ezra Pound, Hilda Doolittle, Richard Aldington and F. S. Flint. Dadaism (1916-1920)- It was the first conceptual art movement where the focus was not on crafting aesthetically pleasing art but challenged traditional arts. Surrealism (1920-1950)- It is a movement that focuses on telling a story, readers aren’t sure they understand the points. 48. The Age of Anxiety is written by: (a) Ian Donaldson (b) D. H. Lawrence (c) W. H. Auden (d) Philip Larkin Ans: (c) ’The Age of Anxiety’ (1947) : A Baroque Ecloque is a long poem in six parts by W. H. Auden, written mostly in a modem version of Anglo-Saxon alliterative verse. 49. In Virginia Woolfs To the Lighthouse the lighthouse does not symbolize: (a) Permanence of feelings (b) Change in the unchanging world (c) Celebration of life (d) Celebration of order in spite of chaos ________ Ans: (d) In Virginia Woolfs ’To The Lighthouse’ the light house does not symbolize celebration of order inspite of chaos. 50. Which of the following is the homosexual fiction? (a) Orlando (b) The Wellness of Being (c) The Ballad of the Reading Goal (d) Madam Bovary _________________________ Ans: (a) ’Orlando : A Biography’ is a novel by Virginia Woolf, first published on 11 Oct. 1928. It is a homosexual fiction. The novel spans almost 400 years in the lifetime of its protagonist Orlando.

204

YCT

51. Who among the following has won the Nobel Prize along with Pulitzer Prize for his/her work? (a) Toni Morrison (b) George Orwell (c) Salman Rushdie (d) Elizabeth Bowen Ans: (a) Chloe Anthony Wofford Morrison known as Toni Morrison, was an American novelist, essayist, book editor and college professor. In 1988 Morrison won the Pulitzer Prize for ’Beloved’ (1987); She gained worldwide recognition award the Nobel Prize in Literature in 1993. ___________________________ 52. Identify the work that does not belong to John Osborne: (a) Collected Poems (b) Waiting for Godot (c) The Lord of the Rings (d) All of the above_________________________ Ans: (d) ’Collected Poems’ (1930) - Robert Frost 'Waiting for Godot’ - Samuel Beckett 'The Lord of the Rings’ (1954) - J. R. Tolkien 53. Arrange the following events in the chronological order: 1. Fall or Margret Thatcher Government 2. Return of Hong Kong to China 3. Revolution of Eastern Europe to topple Communist Regimes 4. Election of Margret Thatcher’s Conservative Government (a) 1,2, 3, 4 (b) 4, 3, 1 , 2 (c) 4 , 3 , 2 , 1 __________(d) 1,2, 4, 3 Ans: (b) Chronological orderElection of Margret Thatcher’s In 1979 1. Conservative Government Revolution of Eastern Europe to In 1989 2. topple Communist Regimes In 1990 3. Fall of Margret Thatcher Government Return of Hong Kong to China 4. In 1997 54. Baumgartner's Bombay is a novel about a German Jew who remains an outsider all his life, in his country because he is a Jew and in India, where he is a firangi. Who wrote this moving novel? (a) Shashi Deshpande (b) Vikram Seth (c) Githa Hariharan (d) Anita Desai _______ Ans: (d) Anita Desai’s novel 'Baumgartner’s Bombay’ is about a German Jew who remains an outsider all his life, in his country because he is a Jew and in India, where he is a firangi. 55. Which of the following is written by Peter Ackroyd? (a) Hawksmoor (b) Chatterton (c) Both (a) & (b) _____(d) Why Come to Slaka Ans: (d) 'Hawksmoor' (1985) and ’Chatterton’ (1987) both novels are written by Peter Ackroyd. - ’Hawksmoor’ novel won best novel at the 1985 Whitbread Award and the Guardian Fiction Prize. - ’Why come to Slaka’ has written by Malcolm Bradbury. Haryana PSC Asst. Prof. Re-Exam. 2019

56. ’Magic Realism’ is closely associated with: (a) Italo Calvino (b) Vikram Seth (c) Rohinton Mistry (d) Gabriel Garcia Marquez __________________ Ans: (d) ’Magic Realism’ is closely associated with Gabriel Garcia Marquez. The aim of magic realism to capture the fantastic, mysterious nature of reality. 57. Who amongst the following is not the Jewish American writer? (a) J. D. Salinger (b) Henry Greene (c) William Faulkner (d) Philip Roth Ans: (b) Henry Greene is not the Jewish American writer. He was an English author best remembered for the novels 'Party Going, living and Loving'. 58. U. R. Ananthamurthy’s Samskara was translated by(a) Himself (b) H. S. Shivaprakash (c) A. K. Ramanujan (d) GirishKamad _____ Ans: (c) U. R. Ananthamurthy’s 'Samskara' was translated by A. K. Ramanujan. - Ananthamurthy is an Indian Contemporary writer and critic in the Kannada Language. - He is known for his association with 'Navya' movement. 59. Which among the following is correctly matched? (a) Amitav Ghosh : All about H. Hatterr (b) Anita Desai : The Inheritance of Loss (c) Shashi Deshpande : A Bend in the Ganges (d) Salman Rushdie : The Enchantress of _________________________ Florence_____________ Ans: (d) ’The Enchantress of Florence’ is the ninth novel by Salman Rushdie, published in 2008. Other correct options are'All about H. Hatterr’ (n) - G. V. Desani ’The Inheritance of Loss’ - Kiran Desai ’A Bend in the Ganges’ - Manohar Malgonkar 60. Khuswant Singh served as the editor in Illustrated Weekly during: (a) 1959- 1965 (b) 1969- 1978 (c) 1979- 1985 _______(d) 1965 - 1972 _______ Ans: (b) Khuswant Singh served as the editor in illustrated weekly during 1969-1978. He was an Indian author, journalist and politician. His novel is ’Train to Pakistan’. 61. ’’Poetry : A Magazine of Verse” was founded by Harriet Munroe in the year: (a) 1922 (b) 1931 (c) 1912 _____________(d) 1946 _____________ Ans: (c) ’’Poetry : A Magazine of Verse” was founded by Harriet Munroe in the year 1912. She was an American poet. 62. Doris Lessing’s interest in ................ is widely recognized. (a) Hinduism (b) Sufism (c) Zen Buddhism (d) Judaism

205

YCT

(a) Virginia Woolf (b) Monica Wittig Ans: (b) Doris Lessing’s interest in Sufism is widely (c) Simone de Beauvoir (d) Terry Eagleton recognized. She was awarded the Nobel Prize for Literature in 2007. Her notable work ’The Golden Ans: (c) ’Simone de Beauvoir' (1908-1986) argued Notebook’. ’’Woman’s art would become more political and more 63. ’A trivial comedy for serious people’ was the poetic if liberated from the claustrophobic intensity sub-title for: and self-absorption of a female subjectivity bred and (a) Everyman in His Humour confined in the domestic ghetto’. (b) Area of Darkness Simone de Beauvoir was one of the most prominent (c) The Way in the World French existentialist philosophers and writers. (d) The Importance of Being Earnest ___________ She is known primarily for her 'Treatise Le Deuxieme Ans: (d) ’A trivial comedy for serious people' was the Sexe,' 2 vol. (The Second Sex) and his philosophical sub-title for 'The Importance of Being Earnest' is play works. by Oscar Wilde. It is a farcical comedy in which the protagonists maintain fictitious personae to escape 68. Who among the following is the Canadian critic? burdensome social obligation. (a) I. A. Richards (b) F. R. Leavis 64. Wallace Stevens, ’The Man with the Blue (c) Cleanth Brooks (d) Northrop Frye _____ Guitar’ may be linked to the work of: Ans: (d) Herman Northrop Frye (14 July 1 9 1 2 - 2 3 (a) Modigliani (b) M. F. Husain (c) Picasso ___________(d) Cezanne __________ Jan 1991) was a Canadian literary critic and literary theorist, considered one of the most influential of the Ans: (c) ’The Man with the Blue Guitar’ is a poem published in 1937 by Wallace Stevens. This book 20th century. Frye gained international fame with his linked to the work of Pablo Picasso’s painting ’The first book ’Fearful Symmetry’ which led to the Old Guitarist’. reinterpretation of the poetry of William Blake. 65. The line 'monuments of unageing intellect' 69. The term ’Cultural Materialism’ is associated occurs in which of the poems by W. B. Yeats? with: (a) Byzantium (a) Stephen Greenblatt (b) Raymond Williams (b) Sailing to Byzantium (c) Mathew Arnold (d) Richard Hoggart (c) Among School Children Ans: (b) 'Cultural Materialism' in literary theory and (d) Prayer for My Daughter __________________ cultural studies traces its origin to the work of the leftAns: (b) The line 'Monuments of unageing intellect' wing literary critic 'Raymond Williams'. Cultural has been taken from 'Sailing to Byzantium' poem by Materialism makes analysis based in critical theory, in W. B. Yeats. Except Byzantium, there is no right the tradition of the 'Frankfurt School'. place for an old man who is otherwise a petty thing with his physical powers decaying continuously. 70. The author of 'Gender Trouble' is: Note- Byzantium is called Istanbul now. (a) Elaine Showalter (b) Michelle Barett (c) Julia Kristeva ______(d) Judith Butler ______ 66. The emergence of the concept of World Literature is associated with: Ans: (d) ’Gender Trouble': Feminism and the 1. Friedrich Schiller subversion of Identity (1990; second edition 1999) is a 2. Johann Wolfgang Von Goethe book by the philosopher Judith Butler, in which the 3. Johann Goltfried Herder author argues that gender is a kind of improvised 4. Theodore W. Adorno performance. The work is influential in feminism, Select the correct answer using the code given women's studies, and lesbian and gay studies. Butler’s below: ideas about gender came to be seen as foundational to (a) l a n d 2 (b) 3 and 1 queer theory. __________________________________ (c) 2 and 3 ___________(d) l a n d 4 ___________ 71. The statement ’There is nothing outside the text’ Ans: (b) The term 'World Literature' was first used by is given by: the German writer and statesman 'Johann Wolfgang (a) Jacques Derrida (b) Roland Barthes Von Goethe’, referring to the dissemination of (c) M.M, Bakhtin (d) M. H. Abram literature from and to countries across the Globe. As Ans: (a) The above statement is by Jacques Derrida in literature moves across boundaries of culture and an essay 'On Rousseau in of Grammatology' - which language, it is, in a way, transformed into a unique has been of great interest to his opponents. cultural artifact. It is the assertion that, ’’There is no outside text” (il n’y 67. Who among the following argued? ’Woman’s art would become more political and a pas de hors - texte), which is often mistranslated as more poetic if liberated from the claustrophobic ’There is nothing outside of the text'. intensity and self-absorption of a female 72. In Moby Dick Captain Ahab falls for his: (a) Ignorance (b) Pride subjectivity bred and confined in the domestic (c) Courage (d) Drunkenness ghetto’. 206 YCT Haryana PSC Asst. Prof. Re-Exam. 2019

Ans: (c) In ’Moby Dick’, Captain Ahab falls for his ’Courage’. From the time that his leg is bitten off by the huge white whole called Moby Dick, Captain Ahab monomaniacally pursues his elusive nemesis. Ahab’s obsession with killing Moby Dick brings about his own death as well as those of all, but one of his crew members - ’Ishmael’, the novels narrator. 73. Patrick White’s novel Voss is about: (a) The sea (b) The capital market (c) The landscape _____(d) The judicial system Ans: (c) Patrick White’s novel ’Voss’ is about the life of the nineteenth century Prussian explorer and naturalist Ludwig Leichhurdt, who disappeared while on an expedition into the Australian Outback. The novel is about the landscape, the farm house and the meadow. 74. Which of the following plays is not written by Wole Soyinka? (a) The Lion and the Jewel (b) Master Harold and the Boys (c) Kongi's Harvest (d) The Dance of the Forests _________________ Ans: (b) ’Master Harold and the Boys’ is a play by ’Athol Fugard’, set in 1950. It was first produced at the Yale Repertory Theatre in March 1982 and made its Premiere on Broadway on 4 May at the Lyceum Theatre, where it ran for 344 performances. The play takes place in South Africa during apartheid era, and depicts how institutionalized racism, bigotry or hatred can become absorbed by those who live under it. 75. David Maloufs An Imaginary Life is a retelling of the story: (a) Aristotle (b) Juvenal (c) Horace ___________(d) Ovid _____________ Ans: (d) With ’An Imaginary Life’ (1978), ’David Malouf already a promising poet, emerged as a major novelist. Nominally a story about Ovid in exile, the novel is really about the transforming power of the imagination. Maloufs writing is spare, delicate, meticulous. ’An Imaginary Life’ is considered as the retelling of the story of Ovid. 76. Who among the following does not belong to the ’’Great Tradition” enunciated by F. R. Leavis? (a) Joseph Conrad (b) James Joyce (c) E. M. Forster (d) Virginia Woolf Ans: (b) F. R. Leavis’ ’The Great Tradition’ (1948), an up-compromising critical and polemical survey of English fiction, controversially begins thus; ’’The Great English novelist are Jane Austen, George Eliot, Henry James and Joseph Conrad”. He regards these writers as the best because they not only ’’change the possibilities of art for practitioners and readers”, but also promote an ’’awareness of the possibilities of life. Thus ’James Joyce’ does not belong to the ’Great Tradition’. Haryana PSC Asst. Prof. Re-Exam. 2019

77. The phrase ’’Only Connect” is associated with: (a) D. H. Lawrence (b) James Joyce (c) E. M. Forster ______(d) Virginia Woolf Ans: (c) The phrase ’’Only Connect”, is from E. M. Forster’s novel ’Howards End’, and has taken on new meaning in the past eight weeks. The full quote is perhaps even more poignant; ’’Only Connect” that was the whole of the sermon. Only connect the Prose and the Passion, and both will be exalted, and human love will be seen at its height. 78. The term ’Comedy of Menace’ is associated with: (a) Arnold Wesker (b) Harold Pinter (c) John Arden _______(d) David Hare _______ Ans: (b) ’Comedy of Menace’ is the body of plays written by David Campton, Nigel Dennis, N. F. Simpson and Harold Pinter. The term was coined by drama critic Irving Wardle, who borrowed it from the subtitle of Campton’s play ’The Lunatic View’ A Comedy of Menace, in reviewing Pinter’s and Compton’s plays in Encore in 1958. 79. ’The texture of a poem is the heterogeneous character of its detail, which either fill in its logical outline or else overflows it a little. And it may be said to be imperatively a character to be looked for anything living, such as ’living force’ even if it is not human force; life is such a prodigal kind’ Who is the critic? (a) Mathew Gregory Lewis (b) JohnPoldon (c) John Crowe Ransom (d) Bram Stoker Ans: (c) The above criticism is given by John Crowe Randsom who was an American literary critic, poet and editor. He is considered to be a founder of New Criticism School of Literary Criticism. 80. Which of the following novels has more than one ending? (a) Lucky Jim (b) The Prime of Jean Brodie (c) The French Lieutenant’s Woman (d) Academia Nuts _________________________ Ans: (c) ’The French Lieutenant’s Woman’ is a 1969 postmodern historical fiction novel by John Fowles, after ’The Collector’ (1963) and ’The Magus’ (1965). The novel explores the fraught relationship of gentleman and amateur naturalist ’Charles Smithson’ and ’Sarah Woodruff, the former governess and independent woman with whom he falls love. Often critics will comment on the novel’s multiple endings. The novel has three endings. It is not enough to suggest that the novel with its multiple endings is a mere experiment with the narrative form. There is something more in it. 81. Identify the poet: Life is first boredom then fear Whether or not we use it, it goes And leaves what something hidden from us chose And age, and then the only end of age

207

YCT

(a) Virgil (b) Philip Larkin (c) Seneca ___________(d) Ovid _____________ Ans: (b) Life is first boredom then fear Whether or not we use it, it goes And leaves what something hidden from us chose And age, and then the only end of age Above lines are by Philip Larkin. These lines are taken from the poem ’’Dockery and Son.” ________________________________________ 82. The lines quoted in Question No. 81 are taken from which poem? (a) Hate Again (b) Love Again (c) Mockery and Cousin (d) Dockery and Son Ans: (d) The lines are taken from Philip Larkin’s poem ’’Dockery and Son”. The theme of the poem is youth and age, the fleeting nature of time, relationships, isolation, parenthood. The title ’’Dockery and Son” implies a business which is both familial and mundane whilst also acting as a status symbol. It is inevitable that the middle-aged Protagonist should compare their life to that of others as we all do. 83. Which book of John Ruskin influenced Mahatma Gandhi? (a) Seven Types of Ambiguity (b) The Seven lamps of Architecture (c) Unto this Last (d) Fors Clavigera __________________________ Ans: (c) During a long train journey in South Africa, Gandhiji was given a book of John Ruskin ’Unto this Last’, by one of his friends. About this book he has written that it brought an instantaneous change in his life. The concept of ’Sarvodya’ and ’Antyodaya’ were the products of this influence of Ruskin on Gandhi. 84. R. K. Narayan’s novels are marked by: (a) Regionalism (b) Protestantism (c) Paganism _________(d) Buddhism ________ Ans: (a) Narayan’s Regionalism : Narayan is an eminent regional novelist. His novels are known as Malgudi novels. In fact, the deals with the physical features, people, life, customs, habits, manners, traditions, languages and life style of Malgudi. Thus Narayan’s novels have regional essence. 85. The title of the poem ’’The Second Coming” is taken from: (a) The Bible (b) The Irish mythology (c) The Scottish mythology (d) The Greek mythology ____________________ Ans: (a) William Butler Yeats, ’The Second Coming’, the title is derived from the Christian belief in the second coming of Christ. However, the new nativity Yeats suggests is about to be bom in the Second Coming is a beast, but not a beast like that in the biblical book of Revelation, for it will not be subdued. The title is taken from ’The Bible’. Haryana PSC Asst. Prof. Re-Exam. 2019

86.

Which of the following critic claimed that ’’British Poetry was retracting behind the privet hedges of suburbia”? (a) Toynbee (b) Alvarez (c) Charles Tomlinson (d) Emily ____________ Ans: (c) The above criticism is claimed by Charles Tomlinson, who was English poet and he expressed his best work with clarity and sensitivity. He created a body of lyric poetry remarkable for its Craftsmanship, passion and perception which challenges us to see the world. 87. Twins in the Lord of the Flies are: (a) Ralph and Jack (b) Simon and David (c) Jack and Eric ______(d) Simon and Eric Ans: (d) The twins in ’The Lord of the Flies’ are : ’Simon and Eric’. ’Lord of the Flies’ is a 1954 novel by Nobel Prize winning British author ’William Golding’. The book focuses on a group of British boys stranded on an uninhabited island and their disastrous attempt to govern themselves. 88. Who is Mr. Trench in Graham Greene’s novel ’’The Power and the Glory”? (a) A Teacher (b) A Professor (c) A Dentist _________(d) A Physician _______ Ans: (c) In Graham Greene’s novel ’’The Power and the Glory”, Mr. Trench is a dissatisfied English dentist who longs to return from Mexico to England. He befriends the priest, whom he meets at the quayside, and later witnesses his death. 89. In ’’Leda and Swan” who woos Leda in the form of a swan? (a) Apollo (b) Saturn (c) Zeus _____________ (d) Hercules __________ Ans: (c) In ’Leda and Swan’, The God ’Zeus’, in the form of a swan suddenly attacks ’Leda’, striking her with his enormous wings. The Swan Zeus ejaculates into Leda’s womb and conceives the child, ’Helen’, who will grow up to shape mythological history by causing the fall of Troy and the death of Greek king Agamemnon. ___________ 90. Who invented the term ’Sprung Rhythm’? (a) Coleridge (b) Hopkins (c) Browning _________(d) Wordsworth _______ Ans: (b) ’Sprung Rhythm’ is a poetic rhythm designed to imitate the rhythm of natural speech. It is constructed from feet in which the first syllable is stressed and may be followed by a variable number of unstressed syllables. The British poet Gerard Manley Hopkins said he discovered this previously unnamed poetic rhythm in the natural patterns of English in folk songs, spoken poetry, Shakespeare, Milton, etc. 91. Who is the author of the medieval text 'Sir Gawain And The Green Knight'*! (a) Chaucer (b) Langland (c) Anonymous (d) Rosemary Sutcliff

208

YCT

Ans: (c) Sir Gawain and the Green Knight is a late 14th century middle English Chivalric romance. The author is unknown; the title was given centuries later. It is one of the best known Arthurian stories, with its plot combining two types of folk motifs, the beheading game and the exchange of winnings. 92. William Langland's the first version of 'The Vision of Pier Plowman ' is a satire written in: (a) 1350 (b) 1363 (c) 1368 _____________(d) 1362 _____________ Ans: (c) William Langland’s the first version of ’The Vision of Pier Plowman’ is a satire. 1367-70 is the earliest version. It breaks off, apparently unfinished, at book 11, and book 12 are written by another author or interpolator. The poem runs to about 2,500 lines. 93. Emperor Constantine in 312 AD placed which of the following Latin expressions on his helmet during the war and conquered his enemies? (a) ’hoc signo vinces’ (b) 'justitia' (c) ’libertas et justitia omnibus’ (d) 'aequalitas amnimium librorum’ ____________ Ans: (a) Constantine was a Pagan Monotheist, a devotee of the sun, the unconquered sun. However, before the Milvian Bridge Battle he and his army saw a cross of light in the sky above the sun with words in Greek that are generally translated into Latin as ’In her signo vinces' means sign of conquer or you will win. 94. 'Legends of Holy Women' was written by: (a) Thomas Moore (b) George Ripley (c) Osbem Bokerham (d) ShielsDelany _ _ _ _ _ Ans: (c) ’Legends of Holy Women’ was written by Osbem Bokerham. Osbem Bokerham, also spelled Bokenham, (13931447), English poet and friar best known as the author of a verse collection entitled ’Legends of Holy Women'. Before him, Geoffrey Chaucer had composed ’The Legend of Good Women’. 95. Which of the following pairs is correctly matched? : Peblis to Play (a) James Stuart (b) Sir David Lyndsay Orpheus and Eurydice : The Golden Targe (c) Robert Henryson : Dance of the Seven (d) William Dunbar Deadly Sins Ans: (d) ’Dance of the Seven Deadly Sins’ is written by William Dunbar (1459 or 1460 - 1530), a Scottish maker poet active in the late fifteenth century and the early Sixteenth century. He was closely associated with the court of King James IV. The other works are paired correctly asPeblis to Play - King James I of Scotland. Orpheus and Eurydice - Myth has no first author but retold by Hugh Lupton. The Golden Targe - William Dunbar. Haryana PSC Asst. Prof. Re-Exam. 2019

96.

Who among the following is not associated with the translation of the Bible? (a) Miles Coverdale (b) William Cooper (c) John Wycliffe (d) Thomas Paine Ans: (b) William Cooper is not associated with the translation of the Bible. William Cooper was an American conspiracy theorist, radio broadcaster and author known for his 1991 book behold a Pale Horse, in which he warned of Multiple global conspiracies. 97. 'The Civilization of Renaissance in Italy' is written by whom? (a) Sir Thomas Malory (b) John Gower (c) John Barbour ______(d) Jacob Burckhardt Ans: (d) 'The Civilization of the Renaissance in Italy’ is an 1860 work on the Italian Renaissance by Swiss historian Jacob Burckhardt. Together with his History of the Renaissance in Italy it is counted among the classics of Renaissance historiography. An English translation was produced by S. G. C. Middle more. 98. "Love is not Time's fool, though rosy lips and cheeks Within his bending sickle compass come" In the given lines his stands for: (a) Duration of Love (b) Lover (c) Time ____________(d) Inamorato_________ Ans: (c) These lines have been drawn from Shakespeare’s sonnet 116. This sonnet has been dedicated to the imperishable beauty of the young patron and the indestructible love. Here Shakespeare has launched an eternal poetic war between love and time. He tells that time is very powerful and potential. It destroys everything on its forward march. 99. Which of the following writer does not belong to Renaissance? (a) Andrew Marvel (b) Christopher Marlowe (c) John Webster (d) Thomas Dekker Ans: (a) Andrew Marvel was an English metaphysical poet, satirist and politician, who sat in the house of commons at various times between 1659 and 1678. During the commonwealth period he was a colleague and friend of John Milton. 100. Identify the writer of the book 'Milton's Grand Style': (a) Dr. Johnson (b) Christopher Ricks (c) Mathew Arnold (d) T. S. Eliot _____________________________ Ans: (b) ’Milton’s Grand Style’ is written by ’Christopher Ricks’. This book shows how Milton's Grand Style creates what 'Walter Bagehot’ called a haunting atmosphere of enhancing suggestions.

209

YCT

Haryana Public Service Commission Asst. Prof. Exam. 2019

ENGLISH (Solved Paper) 1.

In the well-known poem "To His Coy Mistress", the word coy means: (a) shy (b) timid (c) voluptuous (d) sensuous Ans: (a) ’To his coy mistress’ is a poem by Metaphysical poet Andrew MarwelL The word ’Coy’ is synonym of ’shy’. Other options ’timid’ means ’coward, ’voluptuous’ and ’sensuous’ means ’erotic’. 2. Can one imagine any private soldier, in the nineties or now, reading Barrack-Room Ballads and feeling that here was a writer who spoke for him? It is very hard to do so. [....] When he is writing not of British but of "loyal" Indians he carries the 'Salaam, Sahib' motif to sometimes disgusting lengths. Yet it remains true that he has far more interest in the common soldier, far more anxiety that he shall get a fair deal, than most of the "liberals" of his day and our own. He sees that the soldier is neglected, meanly underpaid and hypocritically despised by the people whose incomes he safeguards". (a) This is E. M. Forster’s ’’India, Again”. (b) This is Malcolm Muggcridge on E. M. Forster’s India (c) This is T. S. Eliot on Rudyard Kipling (d) This is George Orwell on Rudyard Kipling Ans: (d) The given passage is emanated from George Orwell’s essay ’Rudyard Kipling’. Kipling was bom in India. He has been variously labeled a colonialist and racist. His ’Barrack-Room Ballads’ concern the British enlisted man, the soldier who defends the British Empire but is scorned because of his low birth. 3. In Virginia Woolfs To the Lighthouse the lighthouse does not symbolize: (a) permanence at the heart of change (b) change in the unchanging world (c) celebration of life in the heart of death (d) celebration of order in the heart of chaos _____ Ans: (b) In Virginia Woolfs ’To the Lighthouse’ (1927) the lighthouse does not symbolize change in the unchanging world. It symbolizes the other three statements. Haryana PSC Asst. Prof. Exam. 2019

[Exam. Date : 23 May 2019]

4.

The basic concept that creation was ordered, that every species exists in a hierarchy of status, from God to the lowest creature, was prevalent in the Renaissance. In this hierarchical continuum, man occupies the middle position between the animal kinds and the angels. This world view is known as: (a) Humanism (b) The Enlightenment (c) The Great Chain of Being (d) Calvinism Ans: (c) The Great Chain of Being is a hierarchical structure of all matter and life, thought by medieval Christianity to have been decreed by God. The chain begins with God and descends through angels, humans, animals and plants to minerals. 5. William Shakespeare's Julius Caesar, Antony and Cleopatra and Coriolanus are based on

(a) Holinshed’s Chronicles (b) Folk-tales and legends (c) Older Roman Plays (d) Plutarch’s Lives _________________________ Ans: (d) Shakespeare history plays are mainly relied on Holinshed’s ’Chronicles’ and Roman historian Plutarch’s ’Lives’. His plays on English kings are inspired by ’Chronicles’ whereas ’Julius Caesar’, ’Antony and Cleopatra’, ’Coriolanus’ and ’Timon of Athens’ are based on ’Lives’ translated in English by Thomas North. 6. The following is an exchange between two characters, husband and wife, in a famous play. The lines appear at the very end of an emotionally-charged sequence of the last scene: ”... I’ve stopped believing in miracles.” ’’But I’ll believe. Tell me! Transform ourselves to the point that . . ..?” ’’That our living together could be a true marriage.” (She goes out down the hall.) Which play? Name the characters. (a) Othello. Othello, Desdemona (b) Sure Thing. Bill, Betty (c) A Doll’s House. Helmer, Nora (d) Death of a Salesman. Willy, Linda

210

YCT

history of Australian bushranger and outlaw Ned Ans: (c) The given conversation happens between Kelly. Along with Ned Kelly other characters in the Torvald Helmer and Nora, the husband and the wife in novel are John ’Red’ Kelly, Ellen Kelly, Harry Power, Norwegian playwright Henrik Ibsen’s ’A Doll’s House’ Dan, Stere Hart, Mary Hearn etc. Ned Kelly dedicates (1879). Nora decides to desert her husband when she his narrative to his unborn daughter. For this novel, finds that he does not love her and uses just as a doll. Peter Carey was awarded Booker Prize in 2001. 7. Which of the following statements is not true of 10. By ’language transfer’ is meant: Foucault’s position in History of Sexuality? (a) Knowledge generated in the development of a (a) Modem sexuality is produced through and as learner on account of other domains of discourse. knowledge. (b) The proliferation of modem discourses of (b) The carryover of rules of the mother tongue sexuality is more striking than their suppression. syntax, phonology, or semantic system to the (c) To write historically about sexuality involves Second language in question. increasingly direct, immediate knowledge or (c) The carryover of rules of the Second language understanding of an unchanging sexual syntax, phonology, or semantic system to the essence. mother tongue in question. (d) Modem sexuality is intimately entangled with (d) The vocabulary and sentence structure the historically distinctive contexts and transferred haphazardly during second structures now called ’knowledge’ language acquisition from any other language Ans: (c) To write historically about sexuality involves accessed by the learner. increasingly direct, immediate knowledge or Ans: (b) Language transfer refers the carryover of understanding of an unchanging sexual essence is not rules of the mother tongue syntax, phonology, or a true statement of Foucault’s position in ’History of semantic system to the second language in question. Sexuality’ whereas remaining three statements are 11. More’s Utopia displays strong influence of: true. I. The Arthurian legends 8. Identify the poem that opens with the lines: II. Plato’s Republic I walk through the long schoolroom questioning; III. Amerigo Vespucci’s account of the travels A kind old nun in a white hood replies; IV. The teachings of John Wycliffe The children learn to cipher and to sing. . . The correct combination according to the code (a) ’’Among the Schoolchildren” is: (b) ’’Among School Children” (a) I and III are correct (c) ”A Man Young and Old” (b) II and III are correct (d) ’’The Man Young and Old” (c) II and IV are correct Ans: (b) William Butler Yeats published ’Among the (d) I and IV are correct ______________________ Schoolchildren’ in his famous 1928 collection of Ans: (b) Sir Thomas More's ’Utopia’ was published in poems ’The Tower’. The poem is about a visit made by 1516 in Latin language. The book is highly influenced the ageing Yeats to a convent school in Waterford, by Plato’s ’Republic’ for its concepts of Communism, Ireland. Opening lines of the poem areJustice and equality. At the same time More himself ”1 walk through the long schoolroom questioning; associated Utopia with Amerigo Vespucci's travel A kind old nun in a white hood replies; accounts. The book was translated into English in The children learn to cipher and to sing. ...” 9. Which of the following descriptions is not true of 1551. Peter Carey’s The True History of the Kelly 12. During the colonial era, the British used to call Gang? the Indian Languages vernaculars. We do not (a) It is an epistolary novel use this word for our bhashas because: (b) It has such characters as Edward Kelly, his I. We consider English to be equally mother, and his wife vernacular. (c) It is also about the Bush and the frontier II. Verna is, literally a home-born slave. (d) The novel is dedicated to Edward Kelly’s III. Not all Indian languages are languages of father the Indo-European family, and therefore not Ans: (d) 'True History of the Kelly Gang’ is an all vernacular. (a) IV (b) II and IV epistolary novel by Australian writer Peter Carey, (c) III (d) I and III published in 2000. The novel is based on the life 211 YCT Haryana PSC Asst. Prof. Exam. 2019

Ans: (b) A slave bom of a slave mother in a family and raised in that household was known as a ’Verna’ and was thus distinguished from these slaves who were purchased or inherited. Since, the natives of India were never slaves we do not use this derogatory term ’Vernacular’ as used by the Britishers to our languages. 13. I have known three generations of John Smiths. The type breeds true. John Smith II and III went to the same school, university and learned profession as John Smith I. Yet John Smith I wrote pseudo-Swinburne; John Smith II wrote pseudo-Brooke; and John Smith III is now writing pseudo-Eliot. But unless John Smith can write John Smith, however unfashionable the result, why does he bother to write at all? Surely one Swinburne; one Brooke, and one Eliot are enough in any age? (Robert Graves, ’’The Poet and his Public”) 1. Graves is critical of blind adulation and imitation of successful poets. 2. Graves is critical of blind conformity to standards set by Swinburne, Brooke, and Eliot. 3. Swinburne, Brooke, and Eliot represent the movements : Decadence, the Georgian, and Modernist respectively. 4. The poets in question are Algernon Charles Swinburne, Stopford Brooke, and Thomas Stearns Eliot. (a) Only 1 and 2 are correct (b) Only 4 is incorrect (c) Only 3 and 4 are correct (d) Only 3 is incorrect Ans: (b) In the given excerpt Graves is critical of blind adulation and imitation of great and successful poets like Swinburne, Brooke and Eliot. He asserts that even though there are poets who represented their age like Swinburne was pioneer of decadence moment, Brooke represented the Georgian poetry and T.S. Eliot manifested the Modernist tendencies, one must not imitate them blindly. It is clear in the excerpt that the writer is not attacking here Swinburne, Brooke and T.S. Eliot, but he is critical to those who blindly copy them. 14. Select from among the following plays, the one that best suits the description below: I. Alyque Padamsee invited its author to write it. II. The play had communalism as its theme. III. This play was banned from the Deccan Herald Theatre Festival for dealing with a sensitive issue. Haryana PSC Asst. Prof. Exam. 2019

IV. The play, however, was produced by Playpen in Bangalore on July 1993. The play is ........... (a) Dance like a Man (b) Where there’s a Will (c) Final Solutions (d) The Wisest Fool on Earth Ans: (c) ’Final Solutions’ play written by Mahesh Dattani was first performed in Bombay under Alyque Padamsee’s direction. It evolved through a workshop after the Ahmadabad riots, and gained relevance post the Mumbai riots. The play was banned from the Deccan Herald Theatre Festival for dealing with a sensitive issue of communalism but was produced by Playpen in Bangalore in July 1993 and by Padamsee in Bombay in December. 15. Which of the following arrangements is in the correct chronological sequence? (a) The Castle of Otranto - Melmoth the Wanderer - The Monk - The Mysteries of Udolpho (b) The Castle of Otranto - The Mysteries of Udolpho - The Monk - Melmoth the Wanderer (c) The Mysteries of Udolpho - The Castle of Otranto - The Monk - Melmoth the Wanderer (d) Melmoth the Wanderer - The Castle of Otranto - The Mysteries of Udolpho - The Monk Ans: (b) The given works are examples of gothic novelWriter Publication Novel Year 1764 The Castle of Horace Walpole Otranto 1794 The Mysteries of Ann Radcliffe Udolpho Matthew Gregory 1796 The Monk Lewis Melmoth the Charles Maturin 1820 Wanderer 16. I. A. Richards’ Practical Criticism (1929) inaugurated a new phase in the history of English critical thought. What was this book’s subtitle? (a) Studies in Poetry (b) A Study in Literary Judgement (c) Essays and Studies (d) A Theoretical Guide Ans: (b) I. A. Richard’s ’Practical Criticism’ published in 1929 with a subtitle ’A Study in Literary Judgement’. His ’Practical Criticism’ is revolutionary in the sense that it does not base criticism on writers but rather on texts.

212

YCT

17. In his delivered on the theatre in a short view of Ans: (c) ’Saturday’ (2005) is a novel by Ian McEwan. the Immorality and Profaneness of the English It is set in Fitzrovia, London, on Saturday, 15 Stage, Jeremy Collier specially arraigned February 2003, as a large demonstration is taking ........... and ........... place against the United States 2003 invasion of Iraq. (a) Congreve and Vanbrugh The protagonist of the novel Henry Prowne is a (b) Farquhar and Vanbrugh neurosurgeon. (c) Wycherley and Farquhar 21. This periodical had the avowed intention ”to (d) Congreve and Etherege ___________________ enliven morality with wit and to temper wit with Ans: (a) Jeremy Collier was an English theatre critic, morality ... to bring philosophy out of the bishop and theologian. In his ’A Short View of the closets and libraries, schools and colleges, to Immorality and Profaneness of the English Stage’, he dwell in clubs and assemblies, at tea-tables and attacked a number of Playwrights William coffee houses”. It also promoted family, Wycherly, John Dryden, William Congreve, John marriage and courtesy. Vanbrugh and Thomas D’Urfey. He accused the The periodical under reference is: playwrights of profanity, blasphemy, indecency and (a) TheTatler undermining public morality through the sympathetic (b) The Spectator depiction of vice. (c) The Gentleman’s Magazine (d) The London Magazine ___________________ 18. The author of the book observes, "I have Ans: (b) ’The Spectator’ was a daily publication attempted, through the medium of biography, to founded by Joseph Addison and Richard Steele in present some Victorian visions to the modern England, lasting from 1711 to 1712. In Number 10 of eye”. The four main characters in this book are the periodical Mr. Spectator states that ’The Spectator’ Cardinal Manning, Florence Nightingale, Dr. will aim ”to enliven morality with wit, and to temper Arnold and General Gordon. wit with morality”. He hopes it will be said he has Who is this author? ’’brought philosophy out of closets and libraries, (a) Mathew Arnold schools and colleges, dwell in clubs and assembles at (b) Robert Browning tea-tables and coffee-houses”. The fictional members (c) Lytton Strachey of the Spectator Club are Sir Roger de Coverley, Sir (d) Oscar Wilde Andrew Freeport, Captain Sentry, Will Honeycomb Ans: (c) Lytton Strachey was one of the founding and two unnamed gentlemen, the Templer and the members of the Bloomsbury Group. He is famous for Clergyman. book ’Eminent Victorians’ published in 1918 and consisting of biographies of four leading figures from 22. The opening sentence of Tolstoy’s Anna Karenina, ’’Happy families are all alike, every the Victorian era - Cardinal Manning, Florence unhappy family is unhappy in its own way.” The Nightingale, Dr. Thomas Arnold (father of Matthew specific cause of the unhappiness in Oblonsky’s Arnold) and General Charles Gordon. house was the husband’s affair with: 19. ”Open Forum” as applied to poetry, is the same (a) a kitchen - maid as .................. It is poetry that is not written (b) an English governess according to traditional fixed patterns. (Fill up) (c) a French governess (a) Blank verse (d) a socialite _____________________________ (b) Concrete poetry Ans: (c) Mademoisselle Roland was a French (c) L = A = N = G = U = A = G = Epoetry governess in Dolly and Stiva Oblonsky household (d) Free verse _____________________________ whose affair with Stiva engendered unhappiness in Ans: (d) Free verse is sometimes referred to as open Oblonsky’s house in Leo Tolstoy’s famous novel form verse, or by the French term verse libre. It differs ’Anna Karenina’ (1878). The novel explores the from traditional verse by the fact that its rhythmic themes of hypocrisy, jealousy, faith, fidelity, family, pattern is not organized into a regular metrical form society and the agrarian connection to lend in contrast that is, into feet, or recurrent units of weak and strongto the lifestyles of the city. stressed syllables. 23. Identify the novel with the wrong subtitle listed below: 20. Ian McEwan’s Saturday spans one day in the (a) Middlemarch, a Study of Provincial Life life of: (b) Tess of the D’Urbervilles, A Pure Woman (a) A divorce lawyer (c) The Mayor of Casterbridge, A Man of (b) An ageing pianist Character (c) A London neurosurgeon (d) A famous poet (d) Felix Holt, the Socialist Haryana PSC Asst. Prof. Exam. 2019

213

YCT

(c) (A) is true, but (R) is false Ans: (d) The correct subtitle of George Eliot’s 1866 (d) (A) is false, but (R) is true novel ’Felix Holt’ is - ’the Radical’. ’Felix Holt, the Radical’ is a social novel about political disputes in a Ans: (a) Both (A) and (R) are true and (R) is the small English town at the time of the First Reform Act correct explanation of (A). of 1832. 27. In Blake’s poem ”A Poison Tree” the speaker’s ’Middlemarch, a study of Provincial Life’ is authored anger grows and become ................ by George Eliot, published in 1871. (a) A cherry ’The Mayor of Casterbridge : The Life and Death of a (b) An apple Man of Character’ and ’Tess of the d’Urbervilles : A (c) Anorange Pure Woman Faithfully Presented’ are novels by (d) Arose ________________________________ Thomas Hardy, published in 1886 and 1891 Ans: (b) ’A Poison Tree’ poem by William Blake was respectively. first published in ’Songs of Experience’ (1794). The 24. Why is ’’Universal grammar” so called? (a) It is a set of basic grammatical principles poem is about the power of anger to become corrupted universally followed and easily recognized by into something far more deadly and devious if it is not people. aired honestly. (b) It is a set of basic grammatical principles 28. In Marvell’s ”A Dialogue between Soul and assumed to be fundamental to all natural Body”, who/which of the following has the last languages. word? (c) It is a set of advanced grammatical principles (a) Body assumed to be fundamental to all natural (b) God languages. (c) Soul (d) It is a set of universally respected practices that (d) Satan _________________________________ have come, in time, to be known as Ans: (a) The poem ’A Dialogue Between the Soul and ________"grammar". ____________________________ Body’ by Andrew Marvell describes the conflict Ans: (b) Universal grammar is usually defined as the between the human body and the human soul, each system of categories, mechanisms and constraints attributing its troubles and sufferings to the other. The shared by all human languages and considered to be Soul starts the discussion and the Body finishes it. innate. Linguist Noam Chomsky is credited for evolving the concept of Universal Grammar. 29. Listed below are some English journals widely read by professionals : Screen, Critical 25. In English literature, the trope of the vampire Quarterly, Review of English, Wasafiri. One of was used for the first time by: the above founded by C. B. Cox, and now being (a) Matthew Gregory Lewis edited by Colin MacCabe, carries not only (b) JohnPolidori critical and scholarly essays in English Studies (c) John Stagg but reviews film, culture, language and (d) Bram Stoker ___________________________ contemporary political issues. Identify the Ans: (c) The trope of the vampire was used for the journal: first time in English literature by John Stagg. He (a) Wasafiri composed the poem ’The Vampire’ in 1810. (b) Screen 26. Given below are two statements, one labeled as (c) Critical Quarterly Assertion (A) and the other as Reason (R): (d) Review of English Studies Assertion (A): For deconstructive critics how Ans: (c) Critical Quarterly is a journal currently human beings read and interpret signs they receive edited by Colin MacCabe. It was established in 1958 will determine their modes of knowing and being, by its first editors C. B. Cox and A. E. Dyson. whether those signs come in the form of literary Wasafiri magazine was established in 1984 by texts or bank statements. Susheila Nasta. Its current editor is Malachi McIntosh. Reason (R): The fact of the matter is that human Screen was launched in 1952 as ’The Film Teacher’, beings use signs to function in the world and are its name was changed in 1969. Currently it is always likely to do so. In the context of the two statements, which one published under Oxford University Press. The Review of English Studies was founded in 1925 of the following is correct? (a) Both (A) and (R) are true and (R) is the correct to publish literary-historical research in all areas of English literature and the English language. explanation of (A) (b) Both (A) and (R) are true and (R) is not the At present it is published under Oxford University Press. correct explanation of (A) Haryana PSC Asst. Prof. Exam. 2019

214

YCT

30.

In his Anatomy of Melancholy Robert Burton proposes the following two principal kinds: I. Love II. Death III. Spiritual IV. Religious The correct combination according to the code is: (a) I and II are correct (b) I and III are correct (c) I and IV are correct (d) II and IV are correct _____________________ Ans: (c) ’The Anatomy of Melancholy’, published in 1621 is a book by Robert Burton. It is divided in three parts - first part defines the inbred malady of melancholy, the second part is devoted to its cure and the third part deals with love and religious melancholy. 31. Of the following characters, which one does not belong to A House for Mr. Biswas? (a) Raghu (b) Ralph Singh (c) Dehuti (d) Tara Ans: (b) Ralph Singh is a character in V.S. Naipaul’s ’The Mimic Men’ while the other given characters are linked to his ’A House for Mr. Biswas’. 32. Which of the following novels is not a Partition novel? (a) Azadi (b) Tamas (c) Clear Light of the Day (d) That Long Silence _______________________ Ans: (d) ’That Long Silence’ novel by Shashi Deshpande is based on man-woman relations. The novel won Sahitya Akademi Award in 1989. ’Azadi’ (2001) by Chaman Nahal, ’Tamas’ (1973) by Bhishma Sahani and ’Clear Light of the Day’ (1980) by Anita Desai are novels based on India-Pakistan partition. 33. For, though, I’ve no idea. What this accoutered frowsty ............ is worth, It pleases me to stand in silence here. (Fill in the blank) (a) bar (b) bam (c) attic _____________(d) alcove ____________ Ans: (b) ’’For, though, I’ve no idea. What this accoutered frowsty bam is worth, It pleases me to stand in silence here. ...” These lines have been taken from Philip Larkin’s poem ’Church Going’. The poem was published in Larkin’s collection ’The Less Deceived’. 34. It blurs distinctions among literary, non-literary and cultural texts, showing how all three intercirculate, share in, and mutually constitute each other.” What does it in this statement stand for? Haryana PSC Asst. Prof. Exam. 2019

(a) Marxism (b) Structuralism (c) Formalism (d) New Historicism ________________________ Ans: (d) New Historicism blurs distinctions among literary, non literary and cultural texts, showing how all three intercirculate, share in, and mutually constitute each other. Stephen Greenblatt was the exponent of this theory. 35. From the following list, identify ’’backformation”: Sulk, bulk, stoke, poke, swindle, bundle. (a) Sulk, bulk, stoke, poke (b) Stoke, poke, swindle, bundle (c) Sulk, stoke, bundle (d) Bulk, poke, bundle ______________________ Ans: (d) Backformation is the process of farming a new word by removing actual or supposed affixes from another word. Example of backformationSulk < Sulky; bulk < bulky Stoke < Stoker; poke < poker Swindle < Swindler; bundle < bundler 36. ’’Late capitalism”, by which is meant accelerated technological development and the massive extension of intellectually qualified labour, was first popularized by .............. (a) Terry Eagleton (b) Ernst Mandel (c) Raymond Williams (d) Stanley Fish ___________________________ Ans: (b) The term ’Late capitalism’ was popularized by a German theorist Ernest Mandel. For Mandel late capitalism denoted the economic period that started with the end of World War II and ended in the early 1970s, a time that saw the rise of multinational corporations, mass communication and international finance. 37. G.M. Hopkin’s ’’Windhover” is dedicated: (a) To Christ, our Lord (b) To Christ our lord (c) To no one (d) To Christ, the Lord ______________________ Ans: (b) ’The Windhover’ is a sonnet written in 1887 by the English poet Gerard Marley Hopkins, dedicated to ’’Christ our Lord”. 38. ’’Exorcism” is the title of Act III of who’s Afraid of Virginia Woolf? What is the significance of ’exorcism’ in the context of the play? (a) The casting out of evil spirits (b) Deconstructing of myths involving marriage, fertility and sons (c) Facing life without illusions (d) Exposing all attempts at illusionmaking

215

YCT

Ans: (d) ’Who’s Afraid of Virginia Woolf?’ is a play Ans: (c) Dryden’s ’An Essay on Dramatic Poesy’ is written in the dialogue among four gentlemen : Crites, by Edward Albee published in 1962. George, Martha, Nick and Honey are characters in the play. The play is Eugenius, Lisideius and Neander. Crites favours the divided into three acts. The third act ’Exorcism’ ancient drama, Eugenius favours the modem drama exposes and wipes out illusions of the characters. and attacks the ancient drama, Lisideius favours the French drama and Neander establishes the importance 39. A Subaltern perspective is one where: of English drama. (a) Power-structures define and determine your command of language and language of 42. Jeremy Collier’s short view of the Immorality command in an uneven world. and Profaneness of the English Stage (1698) (b) The politically dispossessed could be voiceless, attacked .................. (a) the practice of mixing tragic and comic themes written out of the historical record and ignore in Shakespeare’s plays. because their activities do not count for (b) the bawdiness of ’’low” characters in ’’Cultural” or ’’Structured” Shakespeare’s plays. (c) You don’t know what your ’story’ is, how to (c) the coarseness and ugliness of Restoration deal with a ’story’ and therefore you are forced Theatre. to put stereotyped situations in it to please your (d) irreligious themes and irreverent attitudes in listeners. the plays of the seventeenth century. (d) You begin to see how we live, how we have been living, how we have been led to imagine Ans: (c) Jeremy Collier in his ’A Short View of the ourselves, how our language has trapped as Immorality and Profaneness of the English Stage’ well as liberated us. attacked the coarseness and ugliness of Restoration Theatre. He condemned the characters of Restoration Ans: (b) The term ’Subaltern’, meaning ’of inferior comedies as impious and wicked and he condemned rank’ was first adopted by Antonio Gramsci. It points their creators. out that the politically dispossessed could be voiceless, written out of the historical record and 43. What is register? ignored because their activities do not count for (a) The way in which a language registers in the ’cultural’ or ’structured’. minds of its users. (b) The way users of a language register the 40. Ralph Ellison enjoys subverting myths about nuances of that language. white purity through characters like: (A) Norton (B) Bledsoe (c) A variety of language used in social situations (C) Rhinehart (D) All of the above or one specially designed for the subject it (a) (A)and(B) deals with. (b) (A), (B) and (C) (d) A variety of language used in non-professional (c) (B)and(C) or informal situations by professionals. (d) (A) and (C) ____________________________ Ans: (c) In linguistics, the register is a variety of Ans: (a) In Ralph Ellison’s ’The Invisible Man’ (1952) language used in social situations or one specially the myth of white purity has been subverted through designed for the subject it deals with. It shows how a characters like Mr. Nortan and Bledsoe. Norton speaker uses language differently in different appears to be a sincere, generous man, but he is circumstances. It contains the words, the tone of voice simply a new breed of racist who perpetuates the and even body language. tradition of exploiting and humiliating blacks. Bledsoe 44. Alexander Pope’s An Essay in Criticism: also perpetuates the myths of white supremacy. (A) Purports to define ”wit” and ’’nature” as 41. One of the most important themes the speakers they apply to the literature of his age. debate in Dryden’s An Essay on Dramatic Poesy (B) Claims no originality in the thought that is ..................... governs this work. (a) European and non-European perceptions of (C) is a prose essay that gives us such quotes as reality. ”A little learning is a dangerous thing !” (b) English and non-English perceptions of reality. (D) Appeared in 1701. (c) The relative merits of French and English (a) (C) and (D) are incorrect theatre. (b) (A) and (B) are incorrect (d) The relative merits of French and English (c) (A) to (D) are correct poetry. (d) Only (A) and (D) are correct Haryana PSC Asst. Prof. Exam. 2019

216

YCT

Ans: (a) ’An Essay on Criticism’ published in 1711 is Ans: (d) The term ’Pathetic Fallacy’ was invented by a three-part poem in which Alexander Pope shares his John Ruskin in 1856 to signify any non-human natural thoughts on the proper rules and etiquette for critics. thing that ascribes human traits. This term was applied The poem has many famous quotes: to descriptions that are not true but imaginary and ”To err is human, to forgive, divine” fanciful. ”A little learning is a dangerous thing” For example’’Words are like leaves”. ”The one red leaf, the last of its clan. Pope purports to define ’wit’ and ’nature’ as they apply That dances as often as dance it can.” to the literature of his age. Ruskin used this term with disapproval because nature 45. Assertion (A): ’’Tarn O’ Shanter” by John Clare is cannot be equated with the human in respect of about the experience of an ordinary human being emotions and responses. and became quite popular during that time. 48. Which of the following statements on John Reason (R): John Clare, having suffered bouts of Dryden is incorrect? madness, could really feel for the misery of (A) John Milton and John Dryden were common man. contemporaries. In the context of the two statements, which of (B) Dryden was a Royalist, while Milton fiercely the following is correct? opposed monarchy. (a) Both (A) and (R) are true and (R) explains (A). (C) Dryden wrote a play on the Mughal (b) Both (A) and (R) are true, but (R) does not Emperor Humayun. explain (A) (D) Dryden was appointed the Poet Laureate of (c) (A) is true but (R) is false England in 1668. (d) (A) is false but (R) is true (a) (A) is incorrect Ans: (b) John Clare was a peasant poet during the (b) (D) is incorrect Romantic period. In the given poem both sentences (c) (C) is incorrect are true but the second statement does not substantiate (d) (B) and (C) are incorrect the first one. 46. Who among the poets in England during the Ans: (c) John Dryden (1631-1700) was an English poet during the Restoration Age. He was 1930s had left-leaning tendencies? contemporary to John Milton. From 1668 to 1688 he (a) T. S. Eliot, Ezra Pound, Richard Aldington (b) Wilfred Owen, Siegfried Sassoon, Rupert was Poet Laureate of the United Kingdom. He is Brooke famous for his works - ’An Essay of Dramatic Poesie’ (c) W. H. Auden, Louis MacNeice, Cecil Day (criticism); Annus Mirabilis, Absalom and Lewis Achitophel, Mac Flecknoe (poems); Aurang-Zebe, All (d) J. Fleckner, W. H. Davies, Edward Marsh for Love, The Indian Emperor (plays). Ans: (c) W.H. Auden and his companions formed a 49. The book was for many years banned for group ’Oxford group’ or ’Auden group’ which included obscenity in Britain and the United States. The Stephen Spender, C. Day Lewis and Louis MacNeice. central character is a Catholic Jew in Ireland. The group adhered to various Marxist and anti-fascist The author claimed that the book is meant to doctrines and addressed social, political and economic make you laugh. Which is this book? concerns in their writings. (a) The Picture of Dorian Grey 47. Which of the following statements on Pathetic (b) Herzog Fallacy is not true? (c) Portnoy’s Complaint (a) This term applies to descriptions that are not (d) Ulysses true but imaginary and fanciful. Ans: (d) James Joyce’s novel ’Ulysses’ before its (b) Pathetic Fallacy is generally understood as publication in book form in 1922, serialized in parts in human traits being applied or attributed to nonthe American journal ’The Little Review’ from 1918 to human things in nature. 1920. Its 13th episode ’The Nassica’ led to a (c) In its first use, the term was used with disapproval because nature cannot be equated prosecution for obscenity in the USA. The novel was with the human in respect of emotions and banned in the United Kingdom until 1936 for obscenity. The USA lifted the ban in 1934. The responses. central character of the novel is a Catholic Jew in (d) The term was originally used by Alexander Ireland Stephen Dedalus. Pope in his Pastorals (1709). Haryana PSC Asst. Prof. Exam. 2019

217

YCT

50.

Which of the following statements about The Ans: (c) ’Lord of the Flies’ (1954) is a novel by Lyrical Ballads is not true? William Golding. Jack is the representative of the (a) It carried only one ballad proper, which was instinct of savagery, violence and the desire for power while Ralph represents rationality. Piggy reflects Coleridge’s The Rime of the Ancient Mariner. pragmatism and Simon represents innocence. (b) It also carried pastoral and other poems. 54. E.M. Forster’s Passage to India begins with a (c) It carried a "Preface” which Wordsworth added description of the city of Chandrapore. It has an in 1800. old Indian part and a new part consisting of the (d) It also printed from Gray’s Elegy Written in a British civil station. Which of the following Country Churchyard. descriptions of the city is not found in the text? Ans: (d) ’Lyrical Ballads’ published in 1798 with four (a) The streets are mean, the temples ineffective. poems by S.T. Coleridge and nineteen poems by (b) It is a city of gardens. William Wordsworth. Interestingly it has only one (c) It is a tropical pleasaunce washed by a noble proper ballad which is Coleridge’s ’The Rime of the river. Ancient Mariner’. Other poems are chiefly pastoral (d) The new civil station is not sensibly planned lyrics which connect the relationship between human and not modem. and nature. The second edition of ’Lyrical Ballads’ Ans: (d) E. M. Forster’s ’A Passage to India’ (1822) was published in 1800 with a ’Preface’ by William begins with a description of the city of Chandrapore. Wordsworth. Its third and fourth editions were The writer gives the accounts of the city in the first published in 1802 and 1805 respectively. Para as ’the streets are mean, the temples ineffective’, Wordsworth was critical of Thomas Gray’s language in the second Para he states ’it is a city of gardens’ and and took an example of Gray’s ’Sonnet on the Death of ’it is a tropical pleasaunce washed by a noble river’. It Richard West’ in his ’Preface’ to show that Gray, does not mention the statement ’the new civil station is governed by a false idea of poetic diction, spoke in the not sensibly planned and not modem’. wrong language. 55. Which one of Brecht’s works was intended to 51. Kafka’s Trial has all the following lampoon the conventional sentimental musical characteristics except : but the public lapped up the work’s sentiment (a) Vivid yet surreal and missed the humour? (b) Dystopian (a) Man is Man (c) The use of historical details of setting (b) Three Penny Opera (d) The depiction of totalitarian society (c) The Mother (d) Life of Galileo Ans: (c) Franz Kafka’s novel ’The Trial’ was published in 1925. It contains a surreal story of a Ans: (b) ’The Three Penny Opera’ by Bertold Brecht, young man Joseph K. who finds himself caught up in first performed in 1928, is a play with music. The the mindless bureaucracy which depicts the dystopian source for the work was John Gay’s ’The Beggar’s condition of the state. It is often considered to be an Opera’. Although the writer, Brecht intended to lampoon the conventional sentimental music but it imaginative anticipation of totalitarianism. 52. While writing or printing, scholarly use prefers became popular for its music numbers. titles in italics. Which of the following is the 56. The novel tells the story of twin brothers, Waldo, the man of reason and intellect, and correct way of writing/printing? Arthur, the innocent half-wit, the way their lives (a) Charles Dicken’s Tale of Two Cities are inextricably intertwined. Which is the (b) Charles Dickens’ Tale of Two Cities novel? (c) Charles Dickens’ A Tale of Two Cities (a) The Tree of Man (d) Charles Dicken’s A Tale of Two Cities (b) Voss Ans: (c) Charles Dickens’ A Tale of Two Cities’ - is (c) The Solid Mandala the correct way of writing/printing. (d) The Vivisector 53. Identify the mismatched pair in the following Ans: (c) ’The Solid Mandala’ (1966) is a novel by where characters in Golding’s Lord on the Flies Australian author Patrick White. It tells the story of twin brothers Waldo, the man of reason and intellect, fit the allegorized pattern of virtues and vices. and Arthur, the innocent half-wit, and the way their (a) Ralph - rationality lives are inextricably intertwined. (b) Piggy - pragmatism ’The Tree of Man’, ’Voss’ and ’The Vivisector’ are (c) Jack -pity other novels by Patrick White. (d) Simon - innocence Haryana PSC Asst. Prof. Exam. 2019

218

YCT

57. This renowned German poet was born in Ans: (b) Prague and died of Leukemia. When young he • Berthe Antoinetta Mason is the mad woman in met Tolstoy and was influenced by him. The Charlotte Bronte’s 1847 novel ’Jane Eyre’. titles of his last two works contain the words • ’Wide Sargasso Sea’ is a 1966 novel by Jean Rhys. ’’sonnets” and ’’elegies”. He is: It is a response to Charlotte Bronte’s novel ’Jane (a) Herman Hesse Eyre’ and presents the point of view of mad (b) Heinrich Heine woman Antoinette Cosway. (c) Joseph Freiherr Von Eichendorff • ’The Yellow Wallpaper’ (1892) is a short story by (d) Raine Marie Rilke Charlotte Perkins Gilman. The narrator of the Ans: (d) Raine Marie Rilke (1875-1926) was a story is gone mad. famous German language poet. He was bom in Prague • ’The Madwoman in the Attic’ is a book by Sandra and died of Leukemia in Valmont, Switzerland in Gilbert and Susan Gubar, in which they examine 1926. Some of his famous works are - ’The Book of Victorian literature from a feminist perspective. Hours’, ’The Notebooks of Malte Laurids Brigge’, The title of the book is drawn from Charlotte ’Duino Elegies’, ’Sonnets to Orpheus’ and ’Letter to a Bronte’s ’Jane Eyre’. Young Poet’. 61. Which of these authors is not a writer of African 58. Based on the following description, identify the American slave narratives? text in reference: (a) Solomon Northrop This is a play in which no one comes, no one (b) Frederick Douglass goes, nothing happens. In its opening scene a (c) Phillis Wheatley man struggles hard to remove his boot. The play (d) Sojourner Truth _________________________ was originally written in French, later translated into English. It was first performed in Ans: (c) Solomon Northrop (’Twelve Years a Slave’), Frederick Douglass (’Narratives of the Life 1953. of Fredrick Douglass, an American Slave’) and (a) Look Back in Anger Sojourner Truth (’Narrative of Sojourner Truth : A (b) Waiting for Godot Northern Slave) are writers of African- American (c) The Zoo Story slave narratives while Phillis Wheatley, although a (d) The Birthday Party ______________________ slave, never wrote on that subject. Phillis was the Ans: (b) ’Waiting for Godot’ is a play by Samuel first African-American author of a published book of Beckelt originally written in French in 1949 and later poetry. translated into English. It was first performed in 1953. In this play no one comes, no one goes, nothing 62. In a classic statement that inaugurated Feminist thought in English, we read: happens. Vladimir and Estrogen are central characters ”A woman writing thinks back through her in this play. This play is put in ’Theatre of Absurd’ mothers”. Where does this occur? category. (a) Virginia Woolfs A Room of One’s Own 59. The tramp in Pinter’s first big hit, (b) Kate Millet’s Sexual Politics The Caretaker, often travels under an assumed (c) Gertrude Stein’s Three Lives name. It is: (d) Mary Hiatt’s The Way Women Write (a) Bernard Jenkins Ans: (a) ’A Room of One’s Own’ is an essay by (b) Roly Jenkins Virginia Woolf published in 1929. She writes in the (c) Jack Jenkins book ”a woman writing thinks back through her (d) Peter Jenkins mother’s”. This statement has a great impact on Ans: (a) ’The Caretaker’ is a play by Harold Pinter feminist writings. first performed in 1960. Mac Davies in an old homeless, tramp man often travels under an assumed 63. Ostensibly a musical treatise, The Anatomy of Melancholy is a reflection on human learning name - Bernard Jenkins. Two brothers Aston and and endeavour published under the pseudonym: Mick are other characters in the play. It is a (a) VoxPopuli tragicomedy play. (b) Epicurus Senior 60. Which of the following works does not have a (c) Democritus Junior mad woman as a character in it? (d) Jesting Pilate ___________________________ (a) The Yellow Wallpaper Ans: (c) Robert Burton published his ’The Anatomy (b) The Mad Woman in the Attic of Melancholy’ in 1621. The ’Anatomy’ uses (c) Jane Eyre melancholy as the lens through which all human (d) Wide Sargasso Sea Haryana PSC Asst. Prof. Exam. 2019

219

YCT

emotions and thought may be scrutinized. Burton (a) Marquez - ’’The Solitude of Latin America” adopts a pseudonym ’’Democritus Junior” in the (b) Borges - ’’Chinese Encyclopaedia” preface of the book. (c) Juan Rulfo - ’’Pedro Paramo” (d) Alejo Carpentier - ”On the Marvelous in 64. The period of Queen Victoria’s reign is: America” (a) 1830-1900 Ans: (b) ’The Order of Things’ (1966) is about the (b) 1837-1901 cognitive status of modem human sciences in the (c) 1830-1901 (d) 1837-1900 _____________________________ production of knowledge. The book is written by Michael Foucault. Foucault writes in the preface of Ans: (b) Queen Victoria was throned in 1837 in the the book - ’’This book first arouse out of a passage in age of 18. She reigned till her death in 1901. Her Borges.... This passage quotes a certain Chinese tenure is known as Victorian Era. During this era, Encyclopaedia.” Britain was transformed from a predominantly rural, 69. In the closing paragraph of The Trial two men agricultural society into an urban, industrial one. accompany Joseph K to a part of the city to 65. Which of the following statements best describes eventually execute him. The place is: the term ’deconstruction’? (a) A Public Park (a) It seeks to expose the problematic nature of (b) A Church ’centered’ discourses. (c) A Quarry (b) It advocates ’subjective’ or ’free’ interpretation. (d) An Abandoned Factory ___________________ (c) It emphasizes the importance of historical Ans: (c) Joseph K is the protagonist of the novel ’The context. Trial’ (1925) by Franz Kafka. A rather ordinary bank (d) It is a method of critical analysis. employee, he is arrested for unspecified crimes. In the Ans: (a) The theory of ’deconstruction’ proposed by end of the novel, two men come to his home and take Jacques Derrida seeks to expose the problematic him to an abandoned quarry where one of them stabs nature of ’centered’ discourses like signs and structures him. The novel portrays the mindless bureaucracy and and binary opposition. totalitarianism. 66. One of the following Canterbury Tales is in 70. Where does Act I Scene 1 of William Congreve’s prose, identify. Way of the World open? (a) The Pardoner’s Tale (a) A Chocolate-House (b) The Parson’s Tale (b) A Pub (c) The Monk’s Tale (c) A Carrefour (d) The Knight’s Tale (d) The drawing room of Sir Willfull's mansion Ans: (b) Geoffrey Chaucer’s ’The Canterbury Tales’ is Ans: (a) William Congreve’s ’The Way of the World’ a collection of 24 stories written between 1387 to (1700) is a Restoration comedy play. It is divided into 1400. The Knight’s Tale is the first and The Parson’s five acts. Act 1 is set in a chocolate house where Tale is the last tale. Where all tales are in poetic form, Mirabell and Fainall have just finished playing cards. the Parson’s Tale is in prose form. Act 2 is set in St. James Park. Acts 3, 4 and 5 are all 67. Here is a list of Partition novels which have set in the home of lady Wishfort. ’violence on the woman’s body’ as a significant 71. Julia Kristeva’s ’Intertextuality’ derives from: theme. Pick the odd one out: (A) Saussure’s signs (a) The Pakistani Bride (B) Chomsky’s deep structure (b) What the Body Remembers (C) Bakhtin’s dialogism (c) Train to Pakistan (D) Derrida’s difference (d) The Ice-Candy Man (a) (A)and(D) (b) (A)and(C) Ans: (c) ’Train to Pakistan’ is a novel by Khushwant (c) (C) and (D) _______(d) (A)and(B) _______ Singh, published in 1956. It is based on the events of Ans: (b) Intertexuality refers to the interdependence India-Pakistan partition. The story is set in a village of texts in relation to one another. Julia Kristeva was Mano Majra. Religious hate, violence on women, the first to coin the term ’intertexuality’ in an attempt power and corruption are major themes of the novel. to synthesize Ferdinand de Saussure’s study of signs with Bakhtin’s dialogism. 68. In his preface to The Order of Things, Foucault mentions being influenced by a Latin American 72. ’’Womanist is to feminist as purple is to lavender”. This is an important statement writer and his work. defining the womanist perspective advanced by: Choose the correct answer: 220 YCT Haryana PSC Asst. Prof. Exam. 2019

(a) Toni Morrison Byron’s ’Manfred’, the protagonist seeks redemption of his sins. (b) Zora Neale Hurston Coleridge’s ’Christabel’ is a supernatural narrative and (c) Alice Walker not a quest narrative. (d) Bell Hooks ____________________________ Ans: (c) Womanism is a form of feminism focused 76. Which, among the following, is a place through which John Bunyan's Christian does not pass? especially on the experiences, conditions and concerns (a) The Slough of Despond of women of color, especially Black women. (b) Mount Helicon American author Alice Walker first used the word (c) The Valley of Humiliation ’Womanist’ in her writings. She stated, ’’Womanist is (d) Vanity Fair to feminist as purple is to lavender”. According to Ans: (b) ’The Pilgrim’s Progress’ is a 1678 Christian womanists while feminism focuses strictly on gender discrimination, womanism opposes discrimination allegory written by John Bunyan. The allegory’s against women in the areas of race, class and gender. protagonist Christian starts his journey from his 73. Who among the following was not a member of hometown, the ’City of Destruction’ to the ’Celestial City’. During his journey, he passes through various the Scriblerus Club? places - Slough of Despond, Mount Sinai, House (a) Thomas Parnell Beautiful, Valley of Humiliation, Vanity Fair, River (b) Alexander Pope of God, Doubting Castle, The River of Death etc. (c) Joseph Addison 77. Along the shore of silver streaming Thames; (d) John Gay Whose rutty bank, the which his river hems, Ans: (c) During the Augustan period, there were two Was painted all with variable flowers,. . . famous clubs in London - Scriblerus Club and Kitcat Fit to deck maidens’ bowers Club. Jonathan Swift, Alexander Pope, John Gay, And crown their paramours John Arbuthnot, Thomas Parnell were associated with Against their bridal day, which is not long; Scriblerus club, while Joseph Addison, William Sweet Thames! run softly till I end my song. Congreve and John Vanbrugh were members of Kit(Spenser’s Prothalamion) Cat club. Another poet fondly recalls these lines but 74. His cooks with long disuse their trade forgot; cannot conceal their heavily ironic tone in: Cool was his kitchen, though his brains were (a) Marianne Moore’s ’’Spenser’s Ireland” hot. (b) Sylvia Plath’s ’’Morning Song” Who is this character whose stinginess passed (c) W.H. Auden’s ”In Praise of Limestone” into a proverb? (d) T.S. Eliot’s Waste Land __________________ (a) Corah Ans: (d) T.S. Eliot in the third part ’The Fire Sermon’ (b) Shimei of his poem ’The Waste Land’ periodically narrates the (c) Zimri scene of river Thames(d) Achitophel ”The river’s tent is broken : the last fingers of leaf Ans: (b) Dryden’s political satire ’Absalom and Clutch and sink into the wet bank. The wind crosses Achitophel’ published in 1681. It was based on Popish the brown land, unheard. The nymphs are departed. Plot. The lines ”His cooks ............... were hot” depict Sweet Thames, run softly, till I end my song.” the characteristics of Shimei, a dishonest crook who steals and cheats the Jews. In Dryden’s poem he 78. Identify the work below that does not belong to the literature of the eighteenth century: represents Slingsby Bethel, the sheriff of London. (a) Advancement of Learning 75. Which of the following is not a quest narrative? (b) Gulliver’s Travels (a) Shelley’s Alastor (c) The Spectator (b) Byron’s Manfred (d) An Epistle to Dr. Arbuthnot (c) Coleridge’s Christabel Ans: (a) ’The Advancement of Learning’ is a book by (d) Keats’s Endymion Francis Bacon, published in the seventeenth century in Ans: (c) A quest is a journey toward a specific missi 1605. ’The Spectator’ (1711-12) by Joseph Addison or a goal. In P.B. Shelley’s ’Alastor’ the speaker and Richard Steele; ’Gulliver’s Travels’ (1726) by ostensibly recounts the life of a poet who zealously Jonathan Swift and ’An Epistle to Dr. Arbuthnot’ pursues the most part of nature in search of strange (1735) by Alexander Pope are the eighteenth century truths. In John Keats’ ’Endymion’, the protagonist literature. ventures into the underworld in search of his love. In Haryana PSC Asst. Prof. Exam. 2019

221

YCT

79.

One of the following texts was published earlier than 1955. Identify the text: (a) William Golding, the Inheritors (b) Philip Larkin, the Less Deceived (c) William Empson, Collected Poems (d) Samuel Becket, Waiting for Godot Ans: (c) - The Inheritors (1955) William Golding - The Less Deceived (1955) Philip Larkin William Empson - The Collected Poems (1949) - Waiting for Godot Samuel Beckett (French -1952, English 1955) 80. Here is a list of early English plays imitating Greek and Latin plays. Pick the odd one out: (a) Gorboduc (b) Tamburlaine (c) Ralph Roister Doister (d) Gammer Gurton’s Needle __________________ Ans: (b) ’Gorboduc’ the first English tragedy, written by Thomas Norton and Thomas Sackville, takes inspiration from Seneca, a Roman Playwright. The comedies ’Ralph Roister Doister’ and ’Grammer Gurton’s Needle’ imitate the style of Terence, a Greeko-Roman playwright and Plautus. ’Tamburlaine’ written by Christopher Marlowe is based on the life of Timur, a Central Asian emperor during the fourteenth-fifteenth century. This play does not imitate Greek and Latin plays. 81. Which of the following poems does not begin in the first person pronoun? (a) Shelley’s ’’Adonais” (b) Byron’s "Don Juan” (c) Keats’s ’’Lamia” (d) Coleridge’s ’The Aeolian Harp’ Ans: (c) - ’’Upon a time, before Keats’ ’Lamia’ (1820) the faery broods” Shelley’s ’Adonais’ (1821) - ”1 weep for Adonais he is dead” Byron’s ’Don Juan’ (1819) - ”1 want a hero : an uncommon want” - ”My pensive Sara! thy Coleridge’s ’The Aeolian Harp’ (1795) soft check reclined” 82. Identify the text in the following list which offers a fictionalized survey of English Literature from Elizabethan times to 1928: (a) E.M. Forster, the Eternal Moment (b) Virginia Woolf, Orlando (c) Robert Graves, Goodbye to All That (d) David Jones, In Parenthesis _______________ Ans: (b) ’Orlando’ is a novel by Virginia Woolf, published in 1928, spans almost 400 years in the lifetime of its protagonist. The novel presents a history of English literature since Elizabethan period till date. Haryana PSC Asst. Prof. Exam. 2019

83.

In which of the following novels Harikatha is strategically used as a medium of ’consciousness raising’? (a) Waiting for the Mahatma (b) The Serpent and the Rope (c) A Bend in the Ganges (d) Kanthapura Ans: (d) ’Kanthapura’ (1938) is a novel by Raja Rao, dealing with the impact of the Gandhian freedom struggle on a remote South Indian Village of that name. In this novel, Jayaramachar, the Harikatha man, in his Harikathas represents Goddess Parvati’s struggle for Lord Shiva as India’s struggle for freedom. Therefore, Harikatha is strategically used as a medium of consciousness raising. 84. Which of the following statements is not true of Mahesh Dattani’s Final Solutions? (a) The play centres on a middle class Hindu family during a communal riot. (b) It challenges communalism. (c) It is concerned with homosexual relationship. (d) It promotes religious pluralism in South Asia. Ans: (c) Mahesh Dattani’s ’Final Solutions’ is a threeact play which handles the theme of communal riots. It was first performed in Bangalore in 1993. The scenes of the play take place inside and outside Ramnik Gandhi’s house where Ramnik has given two Muslim boys shelter from the violent mob outside. The play challenges communalism and promotes religious pluralism. 85. A.S. Byatt in her famous award winning novel of 1990 contrasts past and present involving a search for a Victorian poet’s past illuminating a contemporary university researcher’s life and times. Which is the novel? (a) The Virgin in the Garden (b) Possession (c) Babel Tower (d) Still Life ________________________________ Ans: (b) ’Possession’ is a 1990 novel by A. S. Byatt, concerns the relationship between two fictional Victorian poets, Randolph Henry Ash and Christabel, as uncovered by present-day academics Roland Michell and Maud. While finding the affairs between the Victorian writers, researchers themselves fall in love. 86. Horace Walpole’s novel The Castle of Otranto tells the story of: (a) A defiant and heartless tyrant who kills his own son mercilessly (b) An usurper and a tyrant who kills his own daughter by mistake

222

YCT

(c) A castle that collapses and crushes the young Ans: (c) ’’Forlorn the very word is like a bell To toll and sickly prince to death me back from thee to mu sole self!” - these lines are (d) A tyrant who retires to a monastery at the end emanated from John Keats’ ’Ode to Nightingale’. ’Forlorn’ meaning lonely appears central word of the and lives happily ever after with his queen poem because the poet is in sad mood from the Ans: (b) Horace Walpole’s ’The Castle of Otranto’ is a beginning of the poem. gothic novel. It was published in 1764. It tells the 91. Arrange the following English literary periods story of a tyrant, Manfred - the lord of the Castle of in the order in which they appeared. Use the Otranto. After his son Conrad is killed by falling codes given below: helment, he wants to marry with his future wife Codes: Isabella. Later, he mistakenly kills his own daughter I. Elizabethan II. Caroline Matilda. III. Anglo Norman IV. Early Tudor 87. Which one of the following plays does not use The correct combination according to the code the device of ’’the play within the play”? is: (a) Hamlet (a) III, II, IV, I (b) III, IV, II, I (b) Women Beware Women (c) II, III, IV, I ________(d) III, IV, I, II ________ (c) The Spanish Tragedy Ans: (d)Anglo-Norman period - 1066-1204 (d) A Midsummer Nights’ Dream Early Tudor period - 1485-1603 Ans: (b) ’The play within the play’ is a technique Elizabethan period - 1558-1603 through which characters of the play try to find Jacobean period - 1603-1625 something that is hidden. It is used in Thomas Kyd’s Early Tudor monarchs were - Henry VIII (1485’The Spanish Tragedy’ and William Shakespeare’s 1509), Henry VIII (1509-1547), Edward VI (1547plays ’Hamlet’, ’A Midsummer Night’s Dream’, 'Love’s 1553), Mary I (1553-1558), Elizabeth I (1558-1603). Labours Lost' and 'The Taming of the Shrew'. 92. Which of the following statements is not true of 88.................... is a theological term brought into Wole Soyinka’s The Swamp Dwellers? (a) It talks about the family, the extended family in literary criticism by ......... the African society. (a) Entelechy, St. Augustine (b) It is a confrontation between the traditional and (b) Ambiguity, William Empson modern society. (c) Adequation, Fr Walter Ong (c) It talks about the migration of people, crossing (d) Epiphany, James Joyce of borders and diasporic anguish. Ans: (d) Epiphany is a theological term brought into (d) It is a comment about the city, urban, modem literary criticism by James Joyce. Epiphany means 'a and the country rural, the swamp, the ancient. manifestation' or 'showing forth' and by Christian Ans: (c) 'The Swamp Dwellers’ is a play by Nigerian thinkers, it was used to signify manifestation of God's writer Wole Soyinka, published in 1958. The play presence within the created world. James Joyce has talks about the family, the extended family in the used this term in his 'A Portrait of the Artist as a African society. It is a confrontation between the Young Man'. traditional and modem society. It is a comment about 89. Fluency’ in language is the same as: the city, urban, modem and the country rural, the (a) The ability to put oneself across comfortably in swamp, the ancient. Alu, Makuri and Igwezu are main speech and/or writing. characters of the play. (b) The ability to command language rather than 93. Which of the following is not true of postlanguage commanding the user. structuralism? (c) Glibness (a) It seeks to undermine the idea that meaning (d) Accuracy pre-exists its linguistic expression. Ans: (a) Fluency in the language is the same as the (b) There can be no meaning which is not formulated and no language formulation ability to command language rather than language reaches anywhere beyond language. commanding the user. (c) There is no a-textual 'origin' of a text. 90.................... the very word is like a bell (d) Every sign refers to every other sign To toll me back from thee to my sole self! adequately. Which word? Ans: (d) 'Every sign refers to every other signs (a) Bird (b) Immortal adequately’ is not related to post-structuralism theory. (c) Forlorn (d) Fancy Haryana PSC Asst. Prof. Exam. 2019

223

YCT

94.

’’Every demon carries within him unknown to Ans: (b) Bakhtin’s Carnival is a manifestation of ’folk himself, a tiny seed of self-destruction and goes laughter’ and ’folk humour’; it embodies a popular, up in the most unexpected moment.” To which folk based culture which is defined by its irreverent of R.K. Narayan’s characters the above antipathy to the official and hierarchical structures of everyday non carnival life. It emphasis on play, statement applies? parody, pleasure and the body. It is a liberation from (a) Raju - The Guide the prevailing truth and established order. ___________ (b) Jagan - The Sweet Vendor 98. Why does Jean Baudrillard adopt Disneyland as (c) Vasu - Man Eater of Malgudi his own sign? (d) Margayya - The Financial Expert (a) Disneyland is by far the most eminently Ans: (c) R.K. Narayan writes in his novel ’Man Eater noticeable cultural sign in the post modem world. of Malgudi’ that every demon carries within him (b) Disneyland captures ’essences’ and ’nonunknown to himself, a tiny seed of self-destruction essences’ of Reality more convincingly than and goes up in thin air at the most unexpected other cultural venues. moment. Vasu a demon-like man who torments his (c) Disneyland is an artifact that so obviously neighbours in the novel, kills himself in an attempt to announces its own fictiveness that it would smash a mosquito sitting on his head and damaging seem to imply some counter balancing reality. one of his nerves with his powerful hand. Therefore, (d) A ____________________________________ he ends up his life like Bhasmasura. Ans: (c) Jean Baudrillard adopts Disneyland as his 95. Conventional scholarship dates ’Early Modern own sign because Disneyland is an artifact that so obviously announces its own fictiveness that it would English’ as beginning around: seem to imply some counter balancing reality. (a) 450 (b) 1066 Disneyland is an example of hyperreality which (c) 1500 _____________(d) 1800 _____________ Baudrillard calls ’a real without origin or reality’. Ans: (c) Early Modem English begins around 1500. 99. Match the following correctly: 96. Which of the following statements is not true of (A) Noam Chomsky I. Langue/Parole Patrick White? (B) C. S. Pierce II. Competence/ (a) He is remembered today for his epic and Performance III. leonic/Indexical (C) Ferdinand de psychological narrative art. Saussure (b) He is the only Australian to receive the Nobel (D) Roland Barthes IV. Readerly/Writerly Prize in literature. I. II. III. IV (c) He pioneered a new fictional landscape and (a) (C) (B) (A) (D) introduced a new continent in literature. (b) (C) (A) (B) (D) (d) His style is noted for lucidity and simplicity. (c) (A) (C) (D) (B) Ans: (d) Patrick White (1912-1990) was an (d) (B) (C) (A) (D) Australian writer. He is the first and only Australian to Ans: (b) Correctly matchedreceive the Nobel Prize in literature. He was awarded I. Langue/Parole - Ferdinand de Saussure II. Competence/ - Noam Chomsky in 1973 for ”an epic and psychological narrative art Performance which has introduced a new continent into literature.” III. leonic/Indexical - C. S. Pierce He is known for his humour, florid prose and stream IV. Readerly/Writerly Roland Barthes of consciousness techniques. 100. Identify the correctly matched pair: His famous novels are - ’Happy Valley’ (1939), ’Voss’ (a) Amitav Ghosh All About H. (1957), ’Rids in the Chariat’ (1961), ’The Solid Halterr Mandala’ (1966) and ’The Vivisector’ (1970). (b) Anita Desai Inheritance of Loss 97. According to Bakhtin the idea of the (c) Shashi Deshpande A Bend in the Carnivalesque represents the following Ganges characteristics except: (d) Salman Rushdie The Enchantress of Florence (a) A liberation from the prevailing truth and established order Ans: (d) Correctly matched(b) A harking back to the past All About H. Halter G. V. Desani (c) Emphasis on play, parody, pleasure and the Inheritance of Loss Kiran Desai body A Bend in the Ganges Manohar Malgonkar (d) The suspension of all hierarchical rank, The Enchantress of Salman Rushdie Florence principles, norms and prohibitions Haryana PSC Asst. Prof. Exam. 2019

224

YCT

Available on GET IT ON

asm

>

Google

Seated QuickLinks

Play

YCT Books a

BUY-PASS

0 E- Book 0 Test Series

0 Quiz 0 Daily Quiz

w

* T k i» wS‘'h5c”B.

0 Current Affairs 0 Exam Notification

© 12 Month Pass © 6 Month Pass © 3 Month Pass

T 499 399 ? 299 ajci..

n/g

YCT

BOOK

yctbooks.com i?l 3n&

. *